Site Loader

Π‘ΠΎΠ΄Π΅Ρ€ΠΆΠ°Π½ΠΈΠ΅

Ρ‡Ρ‚ΠΎ это Ρ‚Π°ΠΊΠΎΠ΅ ΠΈ ΠΊΠ°ΠΊ ΠΎΠ½ Π²ΠΎΠ·Π½ΠΈΠΊΠ°Π΅Ρ‚

Π‘Π΅Π· элСктричСства Π½Π΅Π²ΠΎΠ·ΠΌΠΎΠΆΠ½ΠΎ ΠΏΡ€Π΅Π΄ΡΡ‚Π°Π²ΠΈΡ‚ΡŒ Тизнь соврСмСнного Ρ‡Π΅Π»ΠΎΠ²Π΅ΠΊΠ°. Π’ΠΎΠ»ΡŒΡ‚Ρ‹, АмпСры, Π’Π°Ρ‚Ρ‚Ρ‹ – эти слова Π·Π²ΡƒΡ‡Π°Ρ‚ Π² Ρ€Π°Π·Π³ΠΎΠ²ΠΎΡ€Π΅ ΠΎΠ± устройствах, ΠΊΠΎΡ‚ΠΎΡ€Ρ‹Π΅ Ρ€Π°Π±ΠΎΡ‚Π°ΡŽΡ‚ ΠΎΡ‚ элСктричСства. Но Ρ‡Ρ‚ΠΎ это Ρ‚Π°ΠΊΠΎΠ΅ элСктричСский Ρ‚ΠΎΠΊ ΠΈ ΠΊΠ°ΠΊΠΎΠ²Ρ‹ условия Π΅Π³ΠΎ сущСствования? Об этом ΠΌΡ‹ расскаТСм Π΄Π°Π»Π΅Π΅, прСдоставив ΠΊΡ€Π°Ρ‚ΠΊΠΎΠ΅ объяснСниС для Π½Π°Ρ‡ΠΈΠ½Π°ΡŽΡ‰ΠΈΡ… элСктриков.

ΠžΠΏΡ€Π΅Π΄Π΅Π»Π΅Π½ΠΈΠ΅

ЭлСктричСским Ρ‚ΠΎΠΊΠΎΠΌ являСтся Π½Π°ΠΏΡ€Π°Π²Π»Π΅Π½Π½ΠΎΠ΅ Π΄Π²ΠΈΠΆΠ΅Π½ΠΈΠ΅ носитСлСй зарядов – это стандартная Ρ„ΠΎΡ€ΠΌΡƒΠ»ΠΈΡ€ΠΎΠ²ΠΊΠ° ΠΈΠ· ΡƒΡ‡Π΅Π±Π½ΠΈΠΊΠ° Ρ„ΠΈΠ·ΠΈΠΊΠΈ. Π’ свою ΠΎΡ‡Π΅Ρ€Π΅Π΄ΡŒ носитСлями заряда Π½Π°Π·Ρ‹Π²Π°ΡŽΡ‚ΡΡ ΠΎΠΏΡ€Π΅Π΄Π΅Π»Π΅Π½Π½Ρ‹Π΅ частицы вСщСства. Ими ΠΌΠΎΠ³ΡƒΡ‚ Π±Ρ‹Ρ‚ΡŒ:

  • Π­Π»Π΅ΠΊΡ‚Ρ€ΠΎΠ½Ρ‹ – ΠΎΡ‚Ρ€ΠΈΡ†Π°Ρ‚Π΅Π»ΡŒΠ½Ρ‹Π΅ носитСли заряда.
  • Π˜ΠΎΠ½Ρ‹ – ΠΏΠΎΠ»ΠΎΠΆΠΈΡ‚Π΅Π»ΡŒΠ½Ρ‹Π΅ носитСли заряда.

Но ΠΎΡ‚ΠΊΡƒΠ΄Π° бСрутся носитСли заряда? Для ΠΎΡ‚Π²Π΅Ρ‚Π° Π½Π° этот вопрос Π½ΡƒΠΆΠ½ΠΎ Π²ΡΠΏΠΎΠΌΠ½ΠΈΡ‚ΡŒ Π±Π°Π·ΠΎΠ²Ρ‹Π΅ знания ΠΎ строСнии вСщСства. Всё Ρ‡Ρ‚ΠΎ нас ΠΎΠΊΡ€ΡƒΠΆΠ°Π΅Ρ‚ – вСщСство, ΠΎΠ½ΠΎ состоит ΠΈΠ· ΠΌΠΎΠ»Π΅ΠΊΡƒΠ», ΠΌΠ΅Π»ΡŒΡ‡Π°ΠΉΡˆΠΈΡ… Π΅Π³ΠΎ частиц. ΠœΠΎΠ»Π΅ΠΊΡƒΠ»Ρ‹ состоят ΠΈΠ· Π°Ρ‚ΠΎΠΌΠΎΠ². Атом состоит ΠΈΠ· ядра, Π²ΠΎΠΊΡ€ΡƒΠ³ ΠΊΠΎΡ‚ΠΎΡ€ΠΎΠ³ΠΎ двиТутся элСктроны Π½Π° Π·Π°Π΄Π°Π½Π½Ρ‹Ρ… ΠΎΡ€Π±ΠΈΡ‚Π°Ρ…. ΠœΠΎΠ»Π΅ΠΊΡƒΠ»Ρ‹ Ρ‚Π°ΠΊΠΆΠ΅ Ρ…Π°ΠΎΡ‚ΠΈΡ‡Π½ΠΎ двиТутся. Π”Π²ΠΈΠΆΠ΅Π½ΠΈΠ΅ ΠΈ структура ΠΊΠ°ΠΆΠ΄ΠΎΠΉ ΠΈΠ· этих частиц зависят ΠΎΡ‚ самого вСщСства ΠΈ влияния Π½Π° Π½Π΅Π³ΠΎ ΠΎΠΊΡ€ΡƒΠΆΠ°ΡŽΡ‰Π΅ΠΉ срСды, Π½Π°ΠΏΡ€ΠΈΠΌΠ΅Ρ€ Ρ‚Π΅ΠΌΠΏΠ΅Ρ€Π°Ρ‚ΡƒΡ€Ρ‹, напряТСния ΠΈ ΠΏΡ€ΠΎΡ‡Π΅Π³ΠΎ.

Ионом Π½Π°Π·Ρ‹Π²Π°ΡŽΡ‚ Π°Ρ‚ΠΎΠΌ, Ρƒ ΠΊΠΎΡ‚ΠΎΡ€ΠΎΠ³ΠΎ измСнилось ΡΠΎΠΎΡ‚Π½ΠΎΡˆΠ΅Π½ΠΈΠ΅ элСктронов ΠΈ ΠΏΡ€ΠΎΡ‚ΠΎΠ½ΠΎΠ². Если ΠΈΠ·Π½Π°Ρ‡Π°Π»ΡŒΠ½ΠΎ Π°Ρ‚ΠΎΠΌ Π½Π΅ΠΉΡ‚Ρ€Π°Π»Π΅Π½, Ρ‚ΠΎ ΠΈΠΎΠ½Ρ‹ Π² свою ΠΎΡ‡Π΅Ρ€Π΅Π΄ΡŒ дСлят Π½Π°:

  • Анионы – ΠΏΠΎΠ»ΠΎΠΆΠΈΡ‚Π΅Π»ΡŒΠ½Ρ‹ΠΉ ΠΈΠΎΠ½ Π°Ρ‚ΠΎΠΌΠ°, ΠΏΠΎΡ‚Π΅Ρ€ΡΠ²ΡˆΠ΅Π³ΠΎ элСктроны.
  • ΠšΠ°Ρ‚ΠΈΠΎΠ½Ρ‹ – это Π°Ρ‚ΠΎΠΌ с «лишними» элСктронами, ΠΏΡ€ΠΈΡΠΎΠ΅Π΄ΠΈΠ½ΠΈΠ²ΡˆΠΈΠ΅ΡΡ ΠΊ Π°Ρ‚ΠΎΠΌΡƒ.

Π•Π΄ΠΈΠ½ΠΈΡ†Π° измСрСния Ρ‚ΠΎΠΊΠ° – АмпСр, согласно Π·Π°ΠΊΠΎΠ½Ρƒ Ома ΠΎΠ½ вычисляСтся ΠΏΠΎ Ρ„ΠΎΡ€ΠΌΡƒΠ»Π΅:

I=U/R,

Π³Π΄Π΅ U – напряТСниС, [Π’], Π° R – сопротивлСниС, [Ом].

Или прямопропорционалСн количСству заряда, пСрСнСсСнному Π·Π° Π΅Π΄ΠΈΠ½ΠΈΡ†Ρƒ Π²Ρ€Π΅ΠΌΠ΅Π½ΠΈ:

I=Q/t,

Π³Π΄Π΅ Q – заряд, [Кл], t – врСмя, [с].

Условия сущСствования элСктричСского Ρ‚ΠΎΠΊΠ°

Π§Ρ‚ΠΎ Ρ‚Π°ΠΊΠΎΠ΅ элСктричСский Ρ‚ΠΎΠΊ ΠΌΡ‹ Ρ€Π°Π·ΠΎΠ±Ρ€Π°Π»ΠΈΡΡŒ, Ρ‚Π΅ΠΏΠ΅Ρ€ΡŒ Π΄Π°Π²Π°ΠΉΡ‚Π΅ ΠΏΠΎΠ³ΠΎΠ²ΠΎΡ€ΠΈΠΌ ΠΎ Ρ‚ΠΎΠΌ, ΠΊΠ°ΠΊ ΠΎΠ±Π΅ΡΠΏΠ΅Ρ‡ΠΈΡ‚ΡŒ Π΅Π³ΠΎ ΠΏΡ€ΠΎΡ‚Π΅ΠΊΠ°Π½ΠΈΠ΅. Для протСкания элСктричСского Ρ‚ΠΎΠΊΠ° Π½Π΅ΠΎΠ±Ρ…ΠΎΠ΄ΠΈΠΌΠΎ Π²Ρ‹ΠΏΠΎΠ»Π½Π΅Π½ΠΈΠ΅ Π΄Π²ΡƒΡ… условий:

  1. НаличиС свободных носитСлСй заряда.
  2. ЭлСктричСскоС ΠΏΠΎΠ»Π΅.

ΠŸΠ΅Ρ€Π²ΠΎΠ΅ условиС сущСствования ΠΈ протСкания элСктричСства зависит ΠΎΡ‚ вСщСства, Π² ΠΊΠΎΡ‚ΠΎΡ€ΠΎΠΌ ΠΏΡ€ΠΎΡ‚Π΅ΠΊΠ°Π΅Ρ‚ (ΠΈΠ»ΠΈ Π½Π΅ ΠΏΡ€ΠΎΡ‚Π΅ΠΊΠ°Π΅Ρ‚) Ρ‚ΠΎΠΊ, Π° Ρ‚Π°ΠΊΠΆΠ΅ Π΅Π³ΠΎ состояния. Π’Ρ‚ΠΎΡ€ΠΎΠ΅ условиС Ρ‚Π°ΠΊΠΆΠ΅ Π²Ρ‹ΠΏΠΎΠ»Π½ΠΈΠΌΠΎ: для сущСствования элСктричСского поля ΠΎΠ±ΡΠ·Π°Ρ‚Π΅Π»ΡŒΠ½ΠΎ Π½Π°Π»ΠΈΡ‡ΠΈΠ΅ Ρ€Π°Π·Π½Ρ‹Ρ… ΠΏΠΎΡ‚Π΅Π½Ρ†ΠΈΠ°Π»ΠΎΠ², ΠΌΠ΅ΠΆΠ΄Ρƒ ΠΊΠΎΡ‚ΠΎΡ€Ρ‹ΠΌΠΈ находится срСда, Π² ΠΊΠΎΡ‚ΠΎΡ€ΠΎΠΉ Π±ΡƒΠ΄ΡƒΡ‚ ΠΏΡ€ΠΎΡ‚Π΅ΠΊΠ°Ρ‚ΡŒ носитСли заряда.

Напомним: НапряТСниС, Π­Π”Π‘ – это Ρ€Π°Π·Π½ΠΎΡΡ‚ΡŒ ΠΏΠΎΡ‚Π΅Π½Ρ†ΠΈΠ°Π»ΠΎΠ². ΠžΡ‚ΡΡŽΠ΄Π° слСдуСт, Ρ‡Ρ‚ΠΎ для выполнСния условий сущСствования Ρ‚ΠΎΠΊΠ° – наличия элСктричСского поля ΠΈ элСктричСского Ρ‚ΠΎΠΊΠ°, Π½ΡƒΠΆΠ½ΠΎ напряТСниС. Π­Ρ‚ΠΎ ΠΌΠΎΠ³ΡƒΡ‚ Π±Ρ‹Ρ‚ΡŒ ΠΎΠ±ΠΊΠ»Π°Π΄ΠΊΠΈ заряТСнного кондСнсатора, Π³Π°Π»ΡŒΠ²Π°Π½ΠΈΡ‡Π΅ΡΠΊΠΈΠΉ элСмСнт, Π­Π”Π‘ возникшСС ΠΏΠΎΠ΄ дСйствиСм ΠΌΠ°Π³Π½ΠΈΡ‚Π½ΠΎΠ³ΠΎ поля (Π³Π΅Π½Π΅Ρ€Π°Ρ‚ΠΎΡ€).

Как ΠΎΠ½ Π²ΠΎΠ·Π½ΠΈΠΊΠ°Π΅Ρ‚, ΠΌΡ‹ Ρ€Π°Π·ΠΎΠ±Ρ€Π°Π»ΠΈΡΡŒ, Π΄Π°Π²Π°ΠΉΡ‚Π΅ ΠΏΠΎΠ³ΠΎΠ²ΠΎΡ€ΠΈΠΌ ΠΎ Ρ‚ΠΎΠΌ, ΠΊΡƒΠ΄Π° ΠΎΠ½ Π½Π°ΠΏΡ€Π°Π²Π»Π΅Π½. Π’ΠΎΠΊ, Π² основном, Π² ΠΏΡ€ΠΈΠ²Ρ‹Ρ‡Π½ΠΎΠΌ для нас использовании, двиТСтся Π² ΠΏΡ€ΠΎΠ²ΠΎΠ΄Π½ΠΈΠΊΠ°Ρ… (элСктропроводка Π² ΠΊΠ²Π°Ρ€Ρ‚ΠΈΡ€Π΅, Π»Π°ΠΌΠΏΠΎΡ‡ΠΊΠΈ накаливания) ΠΈΠ»ΠΈ Π² ΠΏΠΎΠ»ΡƒΠΏΡ€ΠΎΠ²ΠΎΠ΄Π½ΠΈΠΊΠ°Ρ… (свСтодиоды, процСссор вашСго смартфона ΠΈ другая элСктроника), Ρ€Π΅ΠΆΠ΅ Π² Π³Π°Π·Π°Ρ… (Π»ΡŽΠΌΠΈΠ½Π΅ΡΡ†Π΅Π½Ρ‚Π½Ρ‹Π΅ Π»Π°ΠΌΠΏΡ‹).

Π’Π°ΠΊ Π²ΠΎΡ‚ основными носитСлями заряда Π² Π±ΠΎΠ»ΡŒΡˆΠΈΠ½ΡΡ‚Π²Π΅ случаСв ΡΠ²Π»ΡΡŽΡ‚ΡΡ элСктроны, ΠΎΠ½ΠΈ двиТутся ΠΎΡ‚ минуса (Ρ‚ΠΎΡ‡ΠΊΠΈ с ΠΎΡ‚Ρ€ΠΈΡ†Π°Ρ‚Π΅Π»ΡŒΠ½Ρ‹ΠΌ ΠΏΠΎΡ‚Π΅Π½Ρ†ΠΈΠ°Π»ΠΎΠΌ) ΠΊ ΠΏΠ»ΡŽΡΡƒ (Ρ‚ΠΎΡ‡ΠΊΠ΅ с ΠΏΠΎΠ»ΠΎΠΆΠΈΡ‚Π΅Π»ΡŒΠ½Ρ‹ΠΌ ΠΏΠΎΡ‚Π΅Π½Ρ†ΠΈΠ°Π»ΠΎΠΌ, ΠΏΠΎΠ΄Ρ€ΠΎΠ±Π½Π΅Π΅ ΠΎΠ± этом Π²Ρ‹ ΡƒΠ·Π½Π°Π΅Ρ‚Π΅ Π½ΠΈΠΆΠ΅).

Но интСрСсСн Ρ‚ΠΎΡ‚ Ρ„Π°ΠΊΡ‚, Ρ‡Ρ‚ΠΎ Π·Π° Π½Π°ΠΏΡ€Π°Π²Π»Π΅Π½ΠΈΠ΅ двиТСния Ρ‚ΠΎΠΊΠ° Π±Ρ‹Π»ΠΎ принято Π΄Π²ΠΈΠΆΠ΅Π½ΠΈΠ΅ ΠΏΠΎΠ»ΠΎΠΆΠΈΡ‚Π΅Π»ΡŒΠ½Ρ‹Ρ… зарядов – ΠΎΡ‚ плюса ΠΊ минусу. Π₯отя фактичСски всё происходит Π½Π°ΠΎΠ±ΠΎΡ€ΠΎΡ‚. Π”Π΅Π»ΠΎ Π² Ρ‚ΠΎΠΌ, Ρ‡Ρ‚ΠΎ Ρ€Π΅ΡˆΠ΅Π½ΠΈΠ΅ ΠΎ Π½Π°ΠΏΡ€Π°Π²Π»Π΅Π½ΠΈΠΈ Ρ‚ΠΎΠΊΠ° Π±Ρ‹Π»ΠΎ принято Π΄ΠΎ изучСния Π΅Π³ΠΎ ΠΏΡ€ΠΈΡ€ΠΎΠ΄Ρ‹, Π° Ρ‚Π°ΠΊΠΆΠ΅ Π΄ΠΎ Ρ‚ΠΎΠ³ΠΎ, ΠΊΠ°ΠΊ Π±Ρ‹Π»ΠΎ ΠΎΠΏΡ€Π΅Π΄Π΅Π»Π΅Π½ΠΎ Π·Π° счСт Ρ‡Π΅Π³ΠΎ ΠΏΡ€ΠΎΡ‚Π΅ΠΊΠ°Π΅Ρ‚ ΠΈ сущСствуСт Ρ‚ΠΎΠΊ.

ЭлСктричСский Ρ‚ΠΎΠΊ Π² Ρ€Π°Π·Π½Ρ‹Ρ… срСдах

ΠœΡ‹ ΡƒΠΆΠ΅ ΡƒΠΏΠΎΠΌΠΈΠ½Π°Π»ΠΈ ΠΎ Ρ‚ΠΎΠΌ, Ρ‡Ρ‚ΠΎ Π² Ρ€Π°Π·Π»ΠΈΡ‡Π½Ρ‹Ρ… срСдах элСктричСский Ρ‚ΠΎΠΊ ΠΌΠΎΠΆΠ΅Ρ‚ Ρ€Π°Π·Π»ΠΈΡ‡Π°Ρ‚ΡŒΡΡ ΠΏΠΎ Ρ‚ΠΈΠΏΡƒ носитСлСй заряда. Π‘Ρ€Π΅Π΄Ρ‹ ΠΌΠΎΠΆΠ½ΠΎ Ρ€Π°Π·Π΄Π΅Π»ΠΈΡ‚ΡŒ ΠΏΠΎ Ρ…Π°Ρ€Π°ΠΊΡ‚Π΅Ρ€Ρƒ проводимости (ΠΏΠΎ ΡƒΠ±Ρ‹Π²Π°Π½ΠΈΡŽ проводимости):

  1. ΠŸΡ€ΠΎΠ²ΠΎΠ΄Π½ΠΈΠΊ (ΠΌΠ΅Ρ‚Π°Π»Π»Ρ‹).
  2. ΠŸΠΎΠ»ΡƒΠΏΡ€ΠΎΠ²ΠΎΠ΄Π½ΠΈΠΊ (ΠΊΡ€Π΅ΠΌΠ½ΠΈΠΉ, Π³Π΅Ρ€ΠΌΠ°Π½ΠΈΠΉ, арсСнид галия ΠΈ ΠΏΡ€).
  3. ДиэлСктрик (Π²Π°ΠΊΡƒΡƒΠΌ, Π²ΠΎΠ·Π΄ΡƒΡ…, дистиллированная Π²ΠΎΠ΄Π°).

Π’ ΠΌΠ΅Ρ‚Π°Π»Π»Π°Ρ…

Π’ ΠΌΠ΅Ρ‚Π°Π»Π»Π°Ρ… Π΅ΡΡ‚ΡŒ свободныС носитСли зарядов, ΠΈΡ… ΠΈΠ½ΠΎΠ³Π΄Π° Π½Π°Π·Ρ‹Π²Π°ΡŽΡ‚ «элСктричСским Π³Π°Π·ΠΎΠΌΒ». ΠžΡ‚ΠΊΡƒΠ΄Π° бСрутся свободныС носитСли зарядов? Π”Π΅Π»ΠΎ Π² Ρ‚ΠΎΠΌ, Ρ‡Ρ‚ΠΎ ΠΌΠ΅Ρ‚Π°Π»Π», ΠΊΠ°ΠΊ ΠΈ любоС вСщСство, состоит ΠΈΠ· Π°Ρ‚ΠΎΠΌΠΎΠ². Атомы, Ρ‚Π°ΠΊ ΠΈΠ»ΠΈ ΠΈΠ½Π°Ρ‡Π΅ двиТутся ΠΈΠ»ΠΈ ΠΊΠΎΠ»Π΅Π±Π»ΡŽΡ‚ΡΡ. Π§Π΅ΠΌ Π²Ρ‹ΡˆΠ΅ Ρ‚Π΅ΠΌΠΏΠ΅Ρ€Π°Ρ‚ΡƒΡ€Π° ΠΌΠ΅Ρ‚Π°Π»Π»Π°, Ρ‚Π΅ΠΌ сильнСС это Π΄Π²ΠΈΠΆΠ΅Π½ΠΈΠ΅. ΠŸΡ€ΠΈ этом сами Π°Ρ‚ΠΎΠΌΡ‹ Π² ΠΎΠ±Ρ‰Π΅ΠΌ Π²ΠΈΠ΄Π΅ ΠΎΡΡ‚Π°ΡŽΡ‚ΡΡ Π½Π° своих мСстах, собствСнно ΠΈ формируя структуру ΠΌΠ΅Ρ‚Π°Π»Π»Π°.

Π’ элСктронных ΠΎΠ±ΠΎΠ»ΠΎΡ‡ΠΊΠ°Ρ… Π°Ρ‚ΠΎΠΌΠ° ΠΎΠ±Ρ‹Ρ‡Π½ΠΎ Π΅ΡΡ‚ΡŒ нСсколько элСктронов, Ρƒ ΠΊΠΎΡ‚ΠΎΡ€Ρ‹Ρ… связь с ядром достаточно слабая. Под воздСйствиСм Ρ‚Π΅ΠΌΠΏΠ΅Ρ€Π°Ρ‚ΡƒΡ€, химичСских Ρ€Π΅Π°ΠΊΡ†ΠΈΠΉ ΠΈ взаимодСйствия примСсСй, ΠΊΠΎΡ‚ΠΎΡ€Ρ‹Π΅ Π² любом случаС находятся Π² ΠΌΠ΅Ρ‚Π°Π»Π»Π΅, элСктроны ΠΎΡ‚Ρ€Ρ‹Π²Π°ΡŽΡ‚ΡΡ ΠΎΡ‚ своих Π°Ρ‚ΠΎΠΌΠΎΠ², ΠΎΠ±Ρ€Π°Π·ΡƒΡŽΡ‚ΡΡ ΠΏΠΎΠ»ΠΎΠΆΠΈΡ‚Π΅Π»ΡŒΠ½ΠΎ заряТСнныС ΠΈΠΎΠ½Ρ‹. ΠžΡ‚ΠΎΡ€Π²Π°Π²ΡˆΠΈΠ΅ΡΡ элСктроны Π½Π°Π·Ρ‹Π²Π°ΡŽΡ‚ΡΡ свободными ΠΈ Π΄Π²ΠΈΠ³Π°ΡŽΡ‚ΡΡ Ρ…Π°ΠΎΡ‚ΠΈΡ‡Π½ΠΎ.

Если Π½Π° Π½ΠΈΡ… Π±ΡƒΠ΄Π΅Ρ‚ Π²ΠΎΠ·Π΄Π΅ΠΉΡΡ‚Π²ΠΎΠ²Π°Ρ‚ΡŒ элСктричСскоС ΠΏΠΎΠ»Π΅, Π½Π°ΠΏΡ€ΠΈΠΌΠ΅Ρ€, Ссли ΠΏΠΎΠ΄ΠΊΠ»ΡŽΡ‡ΠΈΡ‚ΡŒ ΠΊ куску ΠΌΠ΅Ρ‚Π°Π»Π»Π° Π±Π°Ρ‚Π°Ρ€Π΅ΠΉΠΊΡƒ – Ρ…Π°ΠΎΡ‚ΠΈΡ‡Π½ΠΎΠ΅ Π΄Π²ΠΈΠΆΠ΅Π½ΠΈΠ΅ элСктронов станСт упорядочСнным. Π­Π»Π΅ΠΊΡ‚Ρ€ΠΎΠ½Ρ‹ ΠΎΡ‚ Ρ‚ΠΎΡ‡ΠΊΠΈ, Π² ΠΊΠΎΡ‚ΠΎΡ€ΡƒΡŽ ΠΏΠΎΠ΄ΠΊΠ»ΡŽΡ‡Π΅Π½ ΠΎΡ‚Ρ€ΠΈΡ†Π°Ρ‚Π΅Π»ΡŒΠ½Ρ‹ΠΉ ΠΏΠΎΡ‚Π΅Π½Ρ†ΠΈΠ°Π» (ΠΊΠ°Ρ‚ΠΎΠ΄ Π³Π°Π»ΡŒΠ²Π°Π½ΠΈΡ‡Π΅ΡΠΊΠΎΠ³ΠΎ элСмСнта, Π½Π°ΠΏΡ€ΠΈΠΌΠ΅Ρ€), Π½Π°Ρ‡Π½ΡƒΡ‚ Π΄Π²ΠΈΠ³Π°Ρ‚ΡŒΡΡ ΠΊ Ρ‚ΠΎΡ‡ΠΊΠ΅ с ΠΏΠΎΠ»ΠΎΠΆΠΈΡ‚Π΅Π»ΡŒΠ½Ρ‹ΠΌ ΠΏΠΎΡ‚Π΅Π½Ρ†ΠΈΠ°Π»ΠΎΠΌ.

Π’ ΠΏΠΎΠ»ΡƒΠΏΡ€ΠΎΠ²ΠΎΠ΄Π½ΠΈΠΊΠ°Ρ…

ΠŸΠΎΠ»ΡƒΠΏΡ€ΠΎΠ²ΠΎΠ΄Π½ΠΈΠΊΠ°ΠΌΠΈ ΡΠ²Π»ΡΡŽΡ‚ΡΡ Ρ‚Π°ΠΊΠΈΠ΅ ΠΌΠ°Ρ‚Π΅Ρ€ΠΈΠ°Π»Ρ‹, Π² ΠΊΠΎΡ‚ΠΎΡ€Ρ‹Ρ… Π² Π½ΠΎΡ€ΠΌΠ°Π»ΡŒΠ½ΠΎΠΌ состоянии Π½Π΅Ρ‚ свободных носитСлСй заряда. Они находятся Π² Ρ‚Π°ΠΊ Π½Π°Π·Ρ‹Π²Π°Π΅ΠΌΠΎΠΉ Π·Π°ΠΏΡ€Π΅Ρ‰Π΅Π½Π½ΠΎΠΉ Π·ΠΎΠ½Π΅. Но Ссли ΠΏΡ€ΠΈΠ»ΠΎΠΆΠΈΡ‚ΡŒ внСшниС силы, Ρ‚Π°ΠΊΠΈΠ΅ ΠΊΠ°ΠΊ элСктричСскоС ΠΏΠΎΠ»Π΅, Ρ‚Π΅ΠΏΠ»ΠΎ, Ρ€Π°Π·Π»ΠΈΡ‡Π½Ρ‹Π΅ излучСния (свСтовоС, Ρ€Π°Π΄ΠΈΠ°Ρ†ΠΈΠΎΠ½Π½ΠΎΠ΅ ΠΈ ΠΏΡ€.), ΠΎΠ½ΠΈ ΠΏΡ€Π΅ΠΎΠ΄ΠΎΠ»Π΅Π²Π°ΡŽΡ‚ Π·Π°ΠΏΡ€Π΅Ρ‰Π΅Π½Π½ΡƒΡŽ Π·ΠΎΠ½Ρƒ ΠΈ пСрСходят Π² ΡΠ²ΠΎΠ±ΠΎΠ΄Π½ΡƒΡŽ Π·ΠΎΠ½Ρƒ ΠΈΠ»ΠΈ Π·ΠΎΠ½Ρƒ проводимости. Π­Π»Π΅ΠΊΡ‚Ρ€ΠΎΠ½Ρ‹ ΠΎΡ‚Ρ€Ρ‹Π²Π°ΡŽΡ‚ΡΡ ΠΎΡ‚ своих Π°Ρ‚ΠΎΠΌΠΎΠ² ΠΈ становятся свободными, образуя ΠΈΠΎΠ½Ρ‹ – ΠΏΠΎΠ»ΠΎΠΆΠΈΡ‚Π΅Π»ΡŒΠ½Ρ‹Π΅ носитСли зарядов.

ΠŸΠΎΠ»ΠΎΠΆΠΈΡ‚Π΅Π»ΡŒΠ½Ρ‹Π΅ носитСли Π² ΠΏΠΎΠ»ΡƒΠΏΡ€ΠΎΠ²ΠΎΠ΄Π½ΠΈΠΊΠ°Ρ… Π½Π°Π·Ρ‹Π²Π°ΡŽΡ‚ΡΡ Π΄Ρ‹Ρ€ΠΊΠ°ΠΌΠΈ.

Если просто ΠΏΠ΅Ρ€Π΅Π΄Π°Ρ‚ΡŒ ΡΠ½Π΅Ρ€Π³ΠΈΡŽ ΠΏΠΎΠ»ΡƒΠΏΡ€ΠΎΠ²ΠΎΠ΄Π½ΠΈΠΊΡƒ, ΠΊ ΠΏΡ€ΠΈΠΌΠ΅Ρ€Ρƒ Π½Π°Π³Ρ€Π΅Ρ‚ΡŒ, начнСтся Ρ…Π°ΠΎΡ‚ΠΈΡ‡Π½ΠΎΠ΅ Π΄Π²ΠΈΠΆΠ΅Π½ΠΈΠ΅ носитСлСй заряда. Но Ссли Ρ€Π΅Ρ‡ΡŒ ΠΈΠ΄Π΅Ρ‚ ΠΎ ΠΏΠΎΠ»ΡƒΠΏΡ€ΠΎΠ²ΠΎΠ΄Π½ΠΈΠΊΠΎΠ²Ρ‹Ρ… элСмСнтах, Ρ‚ΠΈΠΏΠ° Π΄ΠΈΠΎΠ΄Π° ΠΈΠ»ΠΈ транзистора, Ρ‚ΠΎ Π½Π° ΠΏΡ€ΠΎΡ‚ΠΈΠ²ΠΎΠΏΠΎΠ»ΠΎΠΆΠ½Ρ‹Ρ… ΠΊΠΎΠ½Ρ†Π°Ρ… кристалла (Π½Π° Π½ΠΈΡ… нанСсСн ΠΌΠ΅Ρ‚Π°Π»Π»ΠΈΠ·ΠΈΡ€ΠΎΠ²Π°Π½Π½Ρ‹ΠΉ слой ΠΈ припаяны Π²Ρ‹Π²ΠΎΠ΄Ρ‹) Π²ΠΎΠ·Π½ΠΈΠΊΠ½Π΅Ρ‚ Π­Π”Π‘, Π½ΠΎ это Π½Π΅ относится ΠΊ Ρ‚Π΅ΠΌΠ΅ сСгодняшнСй ΡΡ‚Π°Ρ‚ΡŒΠΈ.

Если ΠΏΡ€ΠΈΠ»ΠΎΠΆΠΈΡ‚ΡŒ источник Π­Π”Π‘ ΠΊ ΠΏΠΎΠ»ΡƒΠΏΡ€ΠΎΠ²ΠΎΠ΄Π½ΠΈΠΊΡƒ, Ρ‚ΠΎ носитСли заряда Ρ‚Π°ΠΊΠΆΠ΅ ΠΏΠ΅Ρ€Π΅ΠΉΠ΄ΡƒΡ‚ Π² Π·ΠΎΠ½Ρƒ проводимости, Π° Ρ‚Π°ΠΊΠΆΠ΅ начнСтся ΠΈΡ… Π½Π°ΠΏΡ€Π°Π²Π»Π΅Π½Π½ΠΎΠ΅ Π΄Π²ΠΈΠΆΠ΅Π½ΠΈΠ΅ – Π΄Ρ‹Ρ€ΠΊΠΈ ΠΏΠΎΠΉΠ΄ΡƒΡ‚ Π² сторону с мСньшим элСктричСским ΠΏΠΎΡ‚Π΅Π½Ρ†ΠΈΠ°Π»ΠΎΠΌ, Π° элСктроны – Π² сторону с большим.

Π’ Π²Π°ΠΊΡƒΡƒΠΌΠ΅ ΠΈ Π³Π°Π·Π΅

Π’Π°ΠΊΡƒΡƒΠΌΠΎΠΌ Π½Π°Π·Ρ‹Π²Π°ΡŽΡ‚ срСду с ΠΏΠΎΠ»Π½Ρ‹ΠΌ (ΠΈΠ΄Π΅Π°Π»ΡŒΠ½Ρ‹ΠΉ случай) отсутствиСм Π³Π°Π·ΠΎΠ² ΠΈΠ»ΠΈ ΠΌΠΈΠ½ΠΈΠΌΠΈΠ·ΠΈΡ€ΠΎΠ²Π°Π½Π½Ρ‹ΠΌ (Π² Ρ€Π΅Π°Π»ΡŒΠ½ΠΎΡΡ‚ΠΈ) Π΅Π³ΠΎ количСством. Π’Π°ΠΊ ΠΊΠ°ΠΊ Π² Π²Π°ΠΊΡƒΡƒΠΌΠ΅ Π½Π΅Ρ‚ Π½ΠΈΠΊΠ°ΠΊΠΎΠ³ΠΎ вСщСства, Ρ‚ΠΎ ΠΈ носитСлям заряда Π±Ρ€Π°Ρ‚ΡŒΡΡ Π½Π΅ ΠΎΡ‚ΠΊΡƒΠ΄Π°. Однако ΠΏΡ€ΠΎΡ‚Π΅ΠΊΠ°Π½ΠΈΠ΅ Ρ‚ΠΎΠΊΠ° Π² Π²Π°ΠΊΡƒΡƒΠΌΠ΅ ΠΏΠΎΠ»ΠΎΠΆΠΈΠ»ΠΎ Π½Π°Ρ‡Π°Π»ΠΎ элСктроникС ΠΈ Ρ†Π΅Π»ΠΎΠΉ эпохС элСктронных элСмСнтов – элСктровакуумных Π»Π°ΠΌΠΏ. Π˜Ρ… использовали Π² ΠΏΠ΅Ρ€Π²ΠΎΠΉ ΠΏΠΎΠ»ΠΎΠ²ΠΈΠ½Π΅ ΠΏΡ€ΠΎΡˆΠ»ΠΎΠ³ΠΎ Π²Π΅ΠΊΠ°, Π° Π² 50-Ρ… Π³ΠΎΠ΄Π°Ρ… ΠΎΠ½ΠΈ Π½Π°Ρ‡Π°Π»ΠΈ постСпСнно ΡƒΡΡ‚ΡƒΠΏΠ°Ρ‚ΡŒ мСсту транзисторам (Π² зависимости ΠΎΡ‚ ΠΊΠΎΠ½ΠΊΡ€Π΅Ρ‚Π½ΠΎΠΉ сфСры элСктроники).

Допустим, Ρ‡Ρ‚ΠΎ Ρƒ нас Π΅ΡΡ‚ΡŒ сосуд, ΠΈΠ· ΠΊΠΎΡ‚ΠΎΡ€ΠΎΠ³ΠΎ ΠΎΡ‚ΠΊΠ°Ρ‡Π°Π»ΠΈ вСсь Π³Π°Π·, Ρ‚.Π΅. Π² Π½Ρ‘ΠΌ ΠΏΠΎΠ»Π½Ρ‹ΠΉ Π²Π°ΠΊΡƒΡƒΠΌ. Π’ сосуд ΠΏΠΎΠΌΠ΅Ρ‰Π΅Π½ΠΎ Π΄Π²Π° элСктрода, Π½Π°Π·ΠΎΠ²Π΅ΠΌ ΠΈΡ… Π°Π½ΠΎΠ΄ ΠΈ ΠΊΠ°Ρ‚ΠΎΠ΄. Если ΠΌΡ‹ ΠΏΠΎΠ΄ΠΊΠ»ΡŽΡ‡ΠΈΠΌ ΠΊ ΠΊΠ°Ρ‚ΠΎΠ΄Ρƒ ΠΎΡ‚Ρ€ΠΈΡ†Π°Ρ‚Π΅Π»ΡŒΠ½Ρ‹ΠΉ ΠΏΠΎΡ‚Π΅Π½Ρ†ΠΈΠ°Π» источника Π­Π”Π‘, Π° ΠΊ Π°Π½ΠΎΠ΄Ρƒ ΠΏΠΎΠ»ΠΎΠΆΠΈΡ‚Π΅Π»ΡŒΠ½Ρ‹ΠΉ – Π½ΠΈΡ‡Π΅Π³ΠΎ Π½Π΅ ΠΏΡ€ΠΎΠΈΠ·ΠΎΠΉΠ΄Π΅Ρ‚ ΠΈ Ρ‚ΠΎΠΊ ΠΏΡ€ΠΎΡ‚Π΅ΠΊΠ°Ρ‚ΡŒ Π½Π΅ Π±ΡƒΠ΄Π΅Ρ‚. Но Ссли ΠΌΡ‹ Π½Π°Ρ‡Π½Π΅ΠΌ Π½Π°Π³Ρ€Π΅Π²Π°Ρ‚ΡŒ ΠΊΠ°Ρ‚ΠΎΠ΄ – Ρ‚ΠΎΠΊ Π½Π°Ρ‡Π½Π΅Ρ‚ ΠΏΡ€ΠΎΡ‚Π΅ΠΊΠ°Ρ‚ΡŒ. Π­Ρ‚ΠΎΡ‚ процСсс называСтся тСрмоэлСктронной эмиссиСй – испусканиС элСктронов с Π½Π°Π³Ρ€Π΅Ρ‚ΠΎΠΉ повСрхности элСктрона.

На рисункС ΠΈΠ·ΠΎΠ±Ρ€Π°ΠΆΠ΅Π½ процСсс протСкания Ρ‚ΠΎΠΊΠ° Π² Π²Π°ΠΊΡƒΡƒΠΌΠ½ΠΎΠΉ Π»Π°ΠΌΠΏΠ΅. Π’ Π²Π°ΠΊΡƒΡƒΠΌΠ½Ρ‹Ρ… Π»Π°ΠΌΠΏΠ°Ρ… ΠΊΠ°Ρ‚ΠΎΠ΄ Π½Π°Π³Ρ€Π΅Π²Π°ΡŽΡ‚ располоТСнной рядом Π½ΠΈΡ‚ΡŒΡŽ Π½Π°ΠΊΠ°Π»Π° Π½Π° рис (Н), Ρ‚ΠΈΠΏΠ° Ρ‚Π°ΠΊΠΎΠΉ, ΠΊΠ°ΠΊ Π² ΠΎΡΠ²Π΅Ρ‚ΠΈΡ‚Π΅Π»ΡŒΠ½ΠΎΠΉ Π»Π°ΠΌΠΏΠ΅.

ΠŸΡ€ΠΈ этом, Ссли ΠΈΠ·ΠΌΠ΅Π½ΠΈΡ‚ΡŒ ΠΏΠΎΠ»ΡΡ€Π½ΠΎΡΡ‚ΡŒ питания – Π½Π° Π°Π½ΠΎΠ΄ ΠΏΠΎΠ΄Π°Ρ‚ΡŒ минус, Π° Π½Π° ΠΊΠ°Ρ‚ΠΎΠ΄ ΠΏΠΎΠ΄Π°Ρ‚ΡŒ плюс – Ρ‚ΠΎΠΊ ΠΏΡ€ΠΎΡ‚Π΅ΠΊΠ°Ρ‚ΡŒ Π½Π΅ Π±ΡƒΠ΄Π΅Ρ‚. Π­Ρ‚ΠΎ Π΄ΠΎΠΊΠ°ΠΆΠ΅Ρ‚, Ρ‡Ρ‚ΠΎ Ρ‚ΠΎΠΊ Π² Π²Π°ΠΊΡƒΡƒΠΌΠ΅ ΠΏΡ€ΠΎΡ‚Π΅ΠΊΠ°Π΅Ρ‚ Π·Π° счСт двиТСния элСктронов ΠΎΡ‚ ΠšΠΠ’ΠžΠ”Π ΠΊ ΠΠΠžΠ”Π£.

Π“Π°Π· Ρ‚Π°ΠΊΠΆΠ΅ ΠΊΠ°ΠΊ ΠΈ любоС вСщСство состоит ΠΈΠ· ΠΌΠΎΠ»Π΅ΠΊΡƒΠ» ΠΈ Π°Ρ‚ΠΎΠΌΠΎΠ², это Π·Π½Π°Ρ‡ΠΈΡ‚, Ρ‡Ρ‚ΠΎ Ссли Π³Π°Π· Π±ΡƒΠ΄Π΅Ρ‚ Π½Π°Ρ…ΠΎΠ΄ΠΈΡ‚ΡŒΡΡ ΠΏΠΎΠ΄ воздСйствиСм элСктричСского поля, Ρ‚ΠΎ ΠΏΡ€ΠΈ ΠΎΠΏΡ€Π΅Π΄Π΅Π»Π΅Π½Π½ΠΎΠΉ Π΅Π³ΠΎ силС (напряТСниС ΠΈΠΎΠ½ΠΈΠ·Π°Ρ†ΠΈΠΈ) элСктроны оторвутся ΠΎΡ‚ Π°Ρ‚ΠΎΠΌΠ°, Ρ‚ΠΎΠ³Π΄Π° Π±ΡƒΠ΄ΡƒΡ‚ Π²Ρ‹ΠΏΠΎΠ»Π½Π΅Π½Ρ‹ ΠΎΠ±Π° условия протСкания элСктричСского Ρ‚ΠΎΠΊΠ° – ΠΏΠΎΠ»Π΅ ΠΈ свободныС носитСли.

Как ΡƒΠΆΠ΅ Π±Ρ‹Π»ΠΎ сказано, этот процСсс называСтся ΠΈΠΎΠ½ΠΈΠ·Π°Ρ†ΠΈΠ΅ΠΉ. Она ΠΌΠΎΠΆΠ΅Ρ‚ ΠΏΡ€ΠΎΠΈΡΡ…ΠΎΠ΄ΠΈΡ‚ΡŒ Π½Π΅ Ρ‚ΠΎΠ»ΡŒΠΊΠΎ ΠΎΡ‚ ΠΏΡ€ΠΈΠ»ΠΎΠΆΠ΅Π½Π½ΠΎΠ³ΠΎ напряТСния, Π½ΠΎ ΠΈ ΠΏΡ€ΠΈ Π½Π°Π³Ρ€Π΅Π²Π΅ Π³Π°Π·Π°, рСнтгСновском ΠΈΠ·Π»ΡƒΡ‡Π΅Π½ΠΈΠΈ, ΠΏΠΎΠ΄ воздСйствиСм ΡƒΠ»ΡŒΡ‚Ρ€Π°Ρ„ΠΈΠΎΠ»Π΅Ρ‚Π° ΠΈ ΠΏΡ€ΠΎΡ‡Π΅Π³ΠΎ.

Π’ΠΎΠΊ Ρ‡Π΅Ρ€Π΅Π· Π²ΠΎΠ·Π΄ΡƒΡ… ΠΏΠΎΡ‚Π΅Ρ‡Π΅Ρ‚, Π΄Π°ΠΆΠ΅ Ссли ΠΌΠ΅ΠΆΠ΄Ρƒ элСктродами ΡƒΡΡ‚Π°Π½ΠΎΠ²ΠΈΡ‚ΡŒ Π³ΠΎΡ€Π΅Π»ΠΊΡƒ.

ΠŸΡ€ΠΎΡ‚Π΅ΠΊΠ°Π½ΠΈΠ΅ Ρ‚ΠΎΠΊΠ° Π² ΠΈΠ½Π΅Ρ€Ρ‚Π½Ρ‹Ρ… Π³Π°Π·Π°Ρ… сопровоТдаСтся Π»ΡŽΠΌΠΈΠ½Π΅ΡΡ†Π΅Π½Ρ†ΠΈΠ΅ΠΉ Π³Π°Π·Π°, это явлСниС Π°ΠΊΡ‚ΠΈΠ²Π½ΠΎ ΠΈΡΠΏΠΎΠ»ΡŒΠ·ΡƒΠ΅Ρ‚ΡΡ Π² Π»ΡŽΠΌΠΈΠ½Π΅ΡΡ†Π΅Π½Ρ‚Π½Ρ‹Ρ… Π»Π°ΠΌΠΏΠ°Ρ…. ΠŸΡ€ΠΎΡ‚Π΅ΠΊΠ°Π½ΠΈΠ΅ элСктричСского Ρ‚ΠΎΠΊΠ° Π² Π³Π°Π·ΠΎΠ²ΠΎΠΉ срСдС называСтся Π³Π°Π·ΠΎΠ²Ρ‹ΠΌ разрядом.

Π’ Тидкости

Допустим, Ρ‡Ρ‚ΠΎ Ρƒ нас Π΅ΡΡ‚ΡŒ сосуд с Π²ΠΎΠ΄ΠΎΠΉ Π² ΠΊΠΎΡ‚ΠΎΡ€Ρ‹ΠΉ ΠΏΠΎΠΌΠ΅Ρ‰Π΅Π½Ρ‹ Π΄Π²Π° элСктрода, ΠΊ ΠΊΠΎΡ‚ΠΎΡ€Ρ‹ΠΌ ΠΏΠΎΠ΄ΠΊΠ»ΡŽΡ‡Π΅Π½ источник питания. Если Π²ΠΎΠ΄Π° дистиллированная, Ρ‚ΠΎ Π΅ΡΡ‚ΡŒ чистая ΠΈ Π½Π΅ содСрТит примСсСй, Ρ‚ΠΎ ΠΎΠ½Π° являСтся диэлСктриком. Но Ссли ΠΌΡ‹ Π΄ΠΎΠ±Π°Π²ΠΈΠΌ Π² Π²ΠΎΠ΄Ρƒ Π½Π΅ΠΌΠ½ΠΎΠ³ΠΎ соли, сСрной кислоты ΠΈΠ»ΠΈ любого Π΄Ρ€ΡƒΠ³ΠΎΠ³ΠΎ вСщСства, образуСтся элСктролит ΠΈ Ρ‡Π΅Ρ€Π΅Π· Π½Π΅Π³ΠΎ Π½Π°Ρ‡Π½Π΅Ρ‚ ΠΏΡ€ΠΎΡ‚Π΅ΠΊΠ°Ρ‚ΡŒ Ρ‚ΠΎΠΊ.

Π­Π»Π΅ΠΊΡ‚Ρ€ΠΎΠ»ΠΈΡ‚ – вСщСство, ΠΊΠΎΡ‚ΠΎΡ€ΠΎΠ΅ ΠΏΡ€ΠΎΠ²ΠΎΠ΄ΠΈΡ‚ элСктричСский Ρ‚ΠΎΠΊ вслСдствиС диссоциации Π½Π° ΠΈΠΎΠ½Ρ‹.

Если Π² Π²ΠΎΠ΄Ρƒ Π΄ΠΎΠ±Π°Π²ΠΈΡ‚ΡŒ ΠΌΠ΅Π΄Π½Ρ‹ΠΉ купорос, Ρ‚ΠΎ Π½Π° ΠΎΠ΄Π½ΠΎΠΌ ΠΈΠ· элСктродов (ΠΊΠ°Ρ‚ΠΎΠ΄Π΅) осядСт слой ΠΌΠ΅Π΄ΠΈ – это называСтся элСктролиз, Ρ‡Ρ‚ΠΎ Π΄ΠΎΠΊΠ°Π·Ρ‹Π²Π°Π΅Ρ‚ Ρ‡Ρ‚ΠΎ элСктричСский Ρ‚ΠΎΠΊ Π² Тидкости осущСствляСтся Π·Π° счСт двиТСния ΠΈΠΎΠ½ΠΎΠ² – ΠΏΠΎΠ»ΠΎΠΆΠΈΡ‚Π΅Π»ΡŒΠ½Ρ‹Ρ… ΠΈ ΠΎΡ‚Ρ€ΠΈΡ†Π°Ρ‚Π΅Π»ΡŒΠ½Ρ‹Ρ… носитСлСй заряда.

Π­Π»Π΅ΠΊΡ‚Ρ€ΠΎΠ»ΠΈΠ· – Ρ„ΠΈΠ·ΠΈΠΊΠΎ-химичСский процСсс, ΠΊΠΎΡ‚ΠΎΡ€Ρ‹ΠΉ Π·Π°ΠΊΠ»ΡŽΡ‡Π°Π΅Ρ‚ΡΡ Π² Π²Ρ‹Π΄Π΅Π»Π΅Π½ΠΈΠΈ Π½Π° элСктродах ΠΊΠΎΠΌΠΏΠΎΠ½Π΅Π½Ρ‚ΠΎΠ² ΡΠΎΡΡ‚Π°Π²Π»ΡΡŽΡ‰ΠΈΡ… элСктролит.

Π’Π°ΠΊΠΈΠΌ ΠΎΠ±Ρ€Π°Π·ΠΎΠΌ происходит ΠΎΠΌΠ΅Π΄Π½Π΅Π½ΠΈΠ΅, золочСния ΠΈ ΠΏΠΎΠΊΡ€Ρ‹Ρ‚ΠΈΠ΅ Π΄Ρ€ΡƒΠ³ΠΈΠΌΠΈ ΠΌΠ΅Ρ‚Π°Π»Π»Π°ΠΌΠΈ.

Π—Π°ΠΊΠ»ΡŽΡ‡Π΅Π½ΠΈΠ΅

ПодвСдСм ΠΈΡ‚ΠΎΠ³ΠΈ, для протСкания элСктричСского Ρ‚ΠΎΠΊΠ° Π½ΡƒΠΆΠ½Ρ‹ свободныС носитСли зарядов:

  • элСктроны Π² ΠΏΡ€ΠΎΠ²ΠΎΠ΄Π½ΠΈΠΊΠ°Ρ… (ΠΌΠ΅Ρ‚Π°Π»Π»Ρ‹) ΠΈ Π²Π°ΠΊΡƒΡƒΠΌΠ΅;
  • элСктроны ΠΈ Π΄Ρ‹Ρ€ΠΊΠΈ Π² ΠΏΠΎΠ»ΡƒΠΏΡ€ΠΎΠ²ΠΎΠ΄Π½ΠΈΠΊΠ°Ρ…;
  • ΠΈΠΎΠ½Ρ‹ (Π°Π½ΠΈΠΎΠ½Ρ‹ ΠΈ ΠΊΠ°Ρ‚ΠΈΠΎΠ½Ρ‹) Π² Тидкости ΠΈ Π³Π°Π·Π°Ρ….

Для Ρ‚ΠΎΠ³ΠΎ, Ρ‡Ρ‚ΠΎΠ±Ρ‹ Π΄Π²ΠΈΠΆΠ΅Π½ΠΈΠ΅ этих носитСлСй стало упорядочСнны, Π½ΡƒΠΆΠ½ΠΎ элСктричСскоС ΠΏΠΎΠ»Π΅. ΠŸΡ€ΠΎΡΡ‚Ρ‹ΠΌΠΈ словами – ΠΏΡ€ΠΈΠ»ΠΎΠΆΠΈΡ‚ΡŒ напряТСниС Π½Π° ΠΊΠΎΠ½Ρ†Π°Ρ… Ρ‚Π΅Π»Π° ΠΈΠ»ΠΈ ΡƒΡΡ‚Π°Π½ΠΎΠ²ΠΈΡ‚ΡŒ Π΄Π²Π° элСктрода Π² срСдС, Π³Π΄Π΅ прСдполагаСтся ΠΏΡ€ΠΎΡ‚Π΅ΠΊΠ°Π½ΠΈΠ΅ элСктричСского Ρ‚ΠΎΠΊΠ°.

Π’Π°ΠΊΠΆΠ΅ стоит ΠΎΡ‚ΠΌΠ΅Ρ‚ΠΈΡ‚ΡŒ, Ρ‡Ρ‚ΠΎ Ρ‚ΠΎΠΊ ΠΎΠΏΡ€Π΅Π΄Π΅Π»Π΅Π½Π½Ρ‹ΠΌ ΠΎΠ±Ρ€Π°Π·ΠΎΠΌ воздСйствуСт Π½Π° вСщСство, Ρ€Π°Π·Π»ΠΈΡ‡Π°ΡŽΡ‚ Ρ‚Ρ€ΠΈ Ρ‚ΠΈΠΏΠ° воздСйствия:

  • Ρ‚Π΅ΠΏΠ»ΠΎΠ²ΠΎΠ΅;
  • химичСскоС;
  • физичСскоС.

НапослСдок Ρ€Π΅ΠΊΠΎΠΌΠ΅Π½Π΄ΡƒΠ΅ΠΌ ΠΏΡ€ΠΎΡΠΌΠΎΡ‚Ρ€Π΅Ρ‚ΡŒ ΠΏΠΎΠ»Π΅Π·Π½ΠΎ Π²ΠΈΠ΄Π΅ΠΎ, Π² ΠΊΠΎΡ‚ΠΎΡ€ΠΎΠΌ Π±ΠΎΠ»Π΅Π΅ ΠΏΠΎΠ΄Ρ€ΠΎΠ±Π½ΠΎ Ρ€Π°ΡΡΠΌΠ°Ρ‚Ρ€ΠΈΠ²Π°ΡŽΡ‚ΡΡ условия сущСствования ΠΈ протСкания элСктричСского Ρ‚ΠΎΠΊΠ°:

ПолСзноС ΠΏΠΎ Ρ‚Π΅ΠΌΠ΅:

ЭлСктричСский Ρ‚ΠΎΠΊ, ΠΎΡ‚ΠΊΡƒΠ΄Π° ΠΎΠ½ бСрСтся ΠΈ ΠΊΠ°ΠΊ добираСтся Π΄ΠΎ Π½Π°ΡˆΠΈΡ… Π΄ΠΎΠΌΠΎΠ²? ⋆ Geoenergetics.ru

Π£Π²Π°ΠΆΠ°Π΅ΠΌΡ‹Π΅ Ρ‡ΠΈΡ‚Π°Ρ‚Π΅Π»ΠΈ ΠΈ просто посСтитСли нашСго ΠΆΡƒΡ€Π½Π°Π»Π°! ΠœΡ‹ достаточно ΠΌΠ½ΠΎΠ³ΠΎ ΠΈ довольно ΠΏΠΎΠ΄Ρ€ΠΎΠ±Π½ΠΎ пишСм ΠΎ Ρ‚ΠΎΠΌ, ΠΊΠ°ΠΊΠΈΠΌΠΈ способами, ΠΏΡ€ΠΈ ΠΏΠΎΠΌΠΎΡ‰ΠΈ ΠΊΠ°ΠΊΠΈΡ… ΠΈΠΌΠ΅Π½Π½ΠΎ энСргСтичСских рСсурсов, производится элСктроэнСргия Π½Π° элСктростанциях. Атом, Π³Π°Π·, Π²ΠΎΠ΄Π° – Π±Ρ‹Π»ΠΈ нашими с Π²Π°ΠΌΠΈ «гСроями», Ρ€Π°Π·Π²Π΅ Ρ‡Ρ‚ΠΎ Π΄ΠΎ Π°Π»ΡŒΡ‚Π΅Ρ€Π½Π°Ρ‚ΠΈΠ²Π½Ρ‹Ρ… , Β«Π·Π΅Π»Π΅Π½Ρ‹Ρ…Β» Π²Π°Ρ€ΠΈΠ°Π½Ρ‚ΠΎΠ² Π΅Ρ‰Π΅ Π½Π΅ успСли Π΄ΠΎΠ±Ρ€Π°Ρ‚ΡŒΡΡ. Но, Ссли ΠΏΡ€ΠΈΡΠΌΠΎΡ‚Ρ€Π΅Ρ‚ΡŒΡΡ Π²Π½ΠΈΠΌΠ°Ρ‚Π΅Π»ΡŒΠ½ΠΎ, рассказы Π±Ρ‹Π»ΠΈ Π΄Π°Π»Π΅ΠΊΠΎ Π½Π΅ ΠΏΠΎΠ»Π½Ρ‹ΠΌΠΈ. Π•Ρ‰Π΅ Π½ΠΈ Ρ€Π°Π·Ρƒ ΠΌΡ‹ Π½Π΅ ΠΏΡ€ΠΎΠ±ΠΎΠ²Π°Π»ΠΈ ΠΎΡ‚ΡΠ»Π΅Π΄ΠΈΡ‚ΡŒ Π΄Π΅Ρ‚Π°Π»ΡŒΠ½ΠΎ ΠΏΡƒΡ‚ΡŒ элСктроэнСргии ΠΎΡ‚ Ρ‚ΡƒΡ€Π±ΠΈΠ½Ρ‹ Π΄ΠΎ Π½Π°ΡˆΠΈΡ… с Π²Π°ΠΌΠΈ Ρ€ΠΎΠ·Π΅Ρ‚ΠΎΠΊ, с Ρ‚Ρ€ΠΎΠΏΠΈΠ½ΠΊΠ°ΠΌΠΈ Π½Π° освСщСниС Π½Π°ΡˆΠΈΡ… насСлСнных ΠΏΡƒΠ½ΠΊΡ‚ΠΎΠ² ΠΈ Π΄ΠΎΡ€ΠΎΠ³, Π½Π° обСспСчСниС Ρ€Π°Π±ΠΎΡ‚Ρ‹ многочислСнных насосов, ΠΎΠ±Π΅ΡΠΏΠ΅Ρ‡ΠΈΠ²Π°ΡŽΡ‰ΠΈΡ… ΠΊΠΎΠΌΡ„ΠΎΡ€Ρ‚ Π½Π°ΡˆΠΈΡ… с Π²Π°ΠΌΠΈ ΠΆΠΈΠ»ΠΈΡ‰.

Π”ΠΎΡ€ΠΎΠ³ΠΈ ΠΈ Ρ‚Ρ€ΠΎΠΏΠΈΠ½ΠΊΠΈ эти ΠΎΡ‚Π½ΡŽΠ΄ΡŒ Π½Π΅ просты, ΠΏΠΎΡ€ΠΎΠΉ извилисты ΠΈ ΠΌΠ½ΠΎΠ³ΠΎΠΊΡ€Π°Ρ‚Π½ΠΎ ΠΌΠ΅Π½ΡΡŽΡ‚ Π½Π°ΠΏΡ€Π°Π²Π»Π΅Π½ΠΈΠ΅, Π½ΠΎ Π·Π½Π°Ρ‚ΡŒ, ΠΊΠ°ΠΊ ΠΎΠ½ΠΈ выглядят – ΠΎΠ±ΡΠ·Π°Π½Π½ΠΎΡΡ‚ΡŒ ΠΊΠ°ΠΆΠ΄ΠΎΠ³ΠΎ ΠΊΡƒΠ»ΡŒΡ‚ΡƒΡ€Π½ΠΎΠ³ΠΎ Ρ‡Π΅Π»ΠΎΠ²Π΅ΠΊΠ° XXI Π²Π΅ΠΊΠ°. Π’Π΅ΠΊΠ°, ΠΎΠ±Π»ΠΈΠΊ ΠΊΠΎΡ‚ΠΎΡ€ΠΎΠ³ΠΎ Π²ΠΎ ΠΌΠ½ΠΎΠ³ΠΎΠΌ опрСдСляСт ΠΏΠΎΠΊΠΎΡ€ΠΈΠ²ΡˆΠ°ΡΡΡ Π½Π°ΠΌ элСктроэнСргия, ΠΊΠΎΡ‚ΠΎΡ€ΡƒΡŽ ΠΌΡ‹ Π½Π°ΡƒΡ‡ΠΈΠ»ΠΈΡΡŒ ΠΏΡ€Π΅ΠΎΠ±Ρ€Π°Π·ΠΎΠ²Ρ‹Π²Π°Ρ‚ΡŒ Ρ‚Π°ΠΊ, Ρ‡Ρ‚ΠΎΠ±Ρ‹ Π±Ρ‹Π»ΠΈ ΡƒΠ΄ΠΎΠ²Π»Π΅Ρ‚Π²ΠΎΡ€Π΅Π½Ρ‹ всС наши потрСбности – ΠΊΠ°ΠΊ Π² ΠΏΡ€ΠΎΠΌΡ‹ΡˆΠ»Π΅Π½Π½ΠΎΡΡ‚ΠΈ, Ρ‚Π°ΠΊ ΠΈ Π² частном пользовании. Π’ΠΎΠΊ Π² ΠΏΡ€ΠΎΠ²ΠΎΠ΄Π°Ρ… Π»ΠΈΠ½ΠΈΠΉ элСктропСрСдач ΠΈ Ρ‚ΠΎΠΊ Π² Π±Π°Ρ‚Π°Ρ€Π΅ΠΉΠΊΠ°Ρ… Π½Π°ΡˆΠΈΡ… Π³Π°Π΄ΠΆΠ΅Ρ‚ΠΎΠ² – ΠΎΡ‡Π΅Π½ΡŒ Ρ€Π°Π·Π½Ρ‹Π΅ Ρ‚ΠΎΠΊΠΈ, Π½ΠΎ ΠΎΠ½ΠΈ ΠΎΡΡ‚Π°ΡŽΡ‚ΡΡ всС Ρ‚Π΅ΠΌ ΠΆΠ΅ элСктричСством. КакиС усилия приходится ΠΏΡ€ΠΈΠ»Π°Π³Π°Ρ‚ΡŒ элСктроэнСргСтикам, ΠΈΠ½ΠΆΠ΅Π½Π΅Ρ€Π°ΠΌ, Ρ‡Ρ‚ΠΎΠ±Ρ‹ ΠΎΠ±Π΅ΡΠΏΠ΅Ρ‡ΠΈΡ‚ΡŒ ΠΌΠΎΡ‰Π½Π΅ΠΉΡˆΠΈΠ΅ Ρ‚ΠΎΠΊΠΈ ΡΡ‚Π°Π»Π΅ΠΏΠ»Π°Π²ΠΈΠ»ΡŒΠ½Ρ‹Ρ… Π·Π°Π²ΠΎΠ΄ΠΎΠ² ΠΈ малСнькиС, ΠΊΡ€ΠΎΡˆΠ΅Ρ‡Π½Ρ‹Π΅ Ρ‚ΠΎΠΊΠΈ, допустим, Π½Π°Ρ€ΡƒΡ‡Π½Ρ‹Ρ… часов? Бколько Ρ€Π°Π±ΠΎΡ‚Ρ‹ приходится ΠΏΡ€ΠΎΠ΄Π΅Π»Ρ‹Π²Π°Ρ‚ΡŒ всСм Ρ‚Π΅ΠΌ, ΠΊΡ‚ΠΎ ΠΏΠΎΠ΄Π΄Π΅Ρ€ΠΆΠΈΠ²Π°Π΅Ρ‚ систСму ΠΏΡ€Π΅ΠΎΠ±Ρ€Π°Π·ΠΎΠ²Π°Π½ΠΈΠΉ, ΠΏΠ΅Ρ€Π΅Π΄Π°Ρ‡ΠΈ ΠΈ распрСдСлСния элСктроэнСргии, ΠΊΠ°ΠΊΠΈΠΌΠΈ Ρ‚Π°ΠΊΠΈΠΌΠΈ ΠΌΠ΅Ρ‚ΠΎΠ΄Π°ΠΌΠΈ обСспСчСна ΡΡ‚Π°Π±ΠΈΠ»ΡŒΠ½ΠΎΡΡ‚ΡŒ этой систСмы? Π§Π΅ΠΌ «БистСмный ΠžΠΏΠ΅Ρ€Π°Ρ‚ΠΎΡ€Β» отличаСтся ΠΎΡ‚ Β«Π€Π΅Π΄Π΅Ρ€Π°Π»ΡŒΠ½ΠΎΠΉ Π‘Π΅Ρ‚Π΅Π²ΠΎΠΉ Компании», ΠΏΠΎΡ‡Π΅ΠΌΡƒ ΠΎΠ±Π΅ этих ΠΊΠΎΠΌΠΏΠ°Π½ΠΈΠΈ Π±Ρ‹Π»ΠΈ, Π΅ΡΡ‚ΡŒ ΠΈ Π±ΡƒΠ΄ΡƒΡ‚ Π² России Π½Π΅ частными Π° государствСнными?

Вопросов ΠΎΡ‡Π΅Π½ΡŒ ΠΌΠ½ΠΎΠ³ΠΎ, ΠΎΡ‚Π²Π΅Ρ‚Ρ‹ Π½Π° Π½ΠΈΡ… Π½Π°Π΄ΠΎ Π·Π½Π°Ρ‚ΡŒ, Ρ‡Ρ‚ΠΎΠ±Ρ‹ Π±ΠΎΠ»Π΅Π΅ ΠΌΠ΅Π½Π΅Π΅ ΠΏΡ€Π΅Π΄ΡΡ‚Π°Π²Π»ΡΡ‚ΡŒ, Π·Π°Ρ‡Π΅ΠΌ Π½Π°ΠΌ Ρ‚Π°ΠΊ ΠΌΠ½ΠΎΠ³ΠΎ энСргСтиков ΠΈ Ρ‡Π΅ΠΌ ΠΆΠ΅ ΠΎΠ½ΠΈ, Π³Ρ€ΡƒΠ±ΠΎ говоря, Π·Π°Π½ΠΈΠΌΠ°ΡŽΡ‚ΡΡ? ΠœΡ‹ вСдь Π½Π°ΡΡ‚ΠΎΠ»ΡŒΠΊΠΎ ΠΏΡ€ΠΈΠ²Ρ‹ΠΊΠ»ΠΈ, Ρ‡Ρ‚ΠΎ с элСктричСством Π² Π΄ΠΎΠΌΠ°Ρ… ΠΈ Π² Π³ΠΎΡ€ΠΎΠ΄Π°Ρ… всС Π² ΠΏΠΎΠ»Π½ΠΎΠΌ порядкС, Ρ‡Ρ‚ΠΎ ΠΏΡ€ΠΎ элСктроинТСнСров вспоминаСм Ρ‚ΠΎΠ»ΡŒΠΊΠΎ Ρ‚ΠΎΠ³Π΄Π°, ΠΊΠΎΠ³Π΄Π° Ρ‡Ρ‚ΠΎ-Ρ‚ΠΎ Π²Π΄Ρ€ΡƒΠ³ пСрСстаСт Ρ€Π°Π±ΠΎΡ‚Π°Ρ‚ΡŒ, ΠΊΠΎΠ³Π΄Π° ΠΌΡ‹ Π²Ρ‹ΠΏΠ°Π΄Π°Π΅ΠΌ ΠΈΠ· Π·ΠΎΠ½Ρ‹ ΠΏΡ€ΠΈΠ²Ρ‹Ρ‡Π½ΠΎΠ³ΠΎ уровня ΠΊΠΎΠΌΡ„ΠΎΡ€Ρ‚Π°. Π’Π΅ΠΌΠ½ΠΎ ΠΈ Ρ…ΠΎΠ»ΠΎΠ΄Π½ΠΎ – Π²ΠΎΡ‚ Ρ‚ΠΎΠ»ΡŒΠΊΠΎ Ρ‚ΠΎΠ³Π΄Π° ΠΌΡ‹ с Π²Π°ΠΌΠΈ ΠΈ Π³ΠΎΠ²ΠΎΡ€ΠΈΠΌ ΠΎΠ± энСргСтиках, ΠΏΡ€ΠΈΡ‡Π΅ΠΌ Π³ΠΎΠ²ΠΎΡ€ΠΈΠΌ Ρ‚Π°ΠΊΠΈΠ΅ слова, ΠΊΠΎΡ‚ΠΎΡ€Ρ‹Π΅ ΠΌΡ‹ ΠΏΠ΅Ρ‡Π°Ρ‚Π°Ρ‚ΡŒ Ρ‚ΠΎΡ‡Π½ΠΎ Π½Π΅ Π±ΡƒΠ΄Π΅ΠΌ.

ΠœΡ‹ ΡƒΠ²Π΅Ρ€Π΅Π½Ρ‹, Ρ‡Ρ‚ΠΎ Π½Π°ΠΌ ΠΎΡ‚ΠΊΡ€ΠΎΠ²Π΅Π½Π½ΠΎ ΠΏΠΎΠ²Π΅Π·Π»ΠΎ – Π²Π·ΡΡ‚ΡŒΡΡ Π·Π° эту Π½Π΅ ΠΏΡ€ΠΎΡΡ‚ΡƒΡŽ, Π½ΡƒΠΆΠ½ΡƒΡŽ, Π΄Π° Π΅Ρ‰Π΅ ΠΈ ΠΎΠ³Ρ€ΠΎΠΌΠ½ΡƒΡŽ Ρ‚Π΅ΠΌΡƒ согласился настоящий профСссионал. ΠŸΡ€ΠΎΡΠΈΠΌ Π»ΡŽΠ±ΠΈΡ‚ΡŒ ΠΈ ΠΆΠ°Π»ΠΎΠ²Π°Ρ‚ΡŒ – Π”ΠΌΠΈΡ‚Ρ€ΠΈΠΉ Π’Π°Π»Π°Π½ΠΎΠ², Π˜Π½ΠΆΠ΅Π½Π΅Ρ€ с большой Π±ΡƒΠΊΠ²Ρ‹. Π—Π½Π°Π΅Ρ‚Π΅, Π΅ΡΡ‚ΡŒ такая страна – Ѐинляндия, Π² ΠΊΠΎΡ‚ΠΎΡ€ΠΎΠΉ Π·Π²Π°Π½ΠΈΠ΅ ΠΈΠ½ΠΆΠ΅Π½Π΅Ρ€Π° Π½Π°ΡΡ‚ΠΎΠ»ΡŒΠΊΠΎ Π·Π½Π°Ρ‡ΠΈΠΌΠΎ, Ρ‡Ρ‚ΠΎ Π² своС врСмя Π΅ΠΆΠ΅Π³ΠΎΠ΄Π½ΠΎ издавался ΠΊΠ°Ρ‚Π°Π»ΠΎΠ³ с ΠΏΠ΅Ρ€Π΅Ρ‡Π½Π΅ΠΌ спСциалистов, Π΅Π³ΠΎ ΠΈΠΌΠ΅ΡŽΡ‰ΠΈΡ…. Π₯ΠΎΡ‚Π΅Π»ΠΎΡΡŒ Π±Ρ‹, Ρ‡Ρ‚ΠΎΠ±Ρ‹ ΠΈ Π² России ΠΊΠΎΠ³Π΄Π°-Π½ΠΈΠ±ΡƒΠ΄ΡŒ появилась такая славная традиция, Π±Π»Π°Π³ΠΎ Π² наш элСктронно-ΠΈΠ½Ρ‚Π΅Ρ€Π½Π΅Ρ‚Π½Ρ‹ΠΉ Π²Π΅ΠΊ завСсти Ρ‚Π°ΠΊΠΎΠΉ Π΅ΠΆΠ΅Π³ΠΎΠ΄Π½ΠΎ обновляСмый ΠΊΠ°Ρ‚Π°Π»ΠΎΠ³ Π½Π°ΠΌΠ½ΠΎΠ³ΠΎ ΠΏΡ€ΠΎΡ‰Π΅.

Π‘Ρ‚Π°Ρ‚ΡŒΡ, ΠΊΠΎΡ‚ΠΎΡ€ΡƒΡŽ ΠΌΡ‹ ΠΏΡ€Π΅Π΄Π»Π°Π³Π°Π΅ΠΌ Π²Π°ΡˆΠ΅ΠΌΡƒ вниманию ΠΏΠΎ ΠΈΠ½ΠΆΠ΅Π½Π΅Ρ€Π½ΠΎΠΌΡƒ ΠΊΠΎΡ€ΠΎΡ‚ΠΊΠ°, Ρ‚ΠΎΡ‡Π½Π° ΠΈ Π΅ΠΌΠΊΠ°. ΠšΠΎΠ½Π΅Ρ‡Π½ΠΎ, ΠΎΠ±ΠΎ всСм, Ρ‡Ρ‚ΠΎ написал Π”ΠΌΠΈΡ‚Ρ€ΠΈΠΉ, ΠΌΠΎΠΆΠ½ΠΎ Ρ€Π°ΡΡΠΊΠ°Π·Π°Ρ‚ΡŒ Π½Π°ΠΌΠ½ΠΎΠ³ΠΎ ΠΏΠΎΠ΄Ρ€ΠΎΠ±Π½Π΅Π΅, ΠΈ Π² своС врСмя наш ΠΆΡƒΡ€Π½Π°Π» Π½Π°Ρ‡Π°Π» Ρ†ΠΈΠΊΠ» статСй ΠΎ Ρ‚ΠΎΠΌ, ΠΊΠ°ΠΊ Π² XIX Π²Π΅ΠΊΠ΅ происходило ΠΏΠΎΠΊΠΎΡ€Π΅Π½ΠΈΠ΅ элСктричСства.

Π“Π΅ΠΎΡ€Π³ Ом, Π“Π΅Π½Ρ€ΠΈΡ… Π“Π΅Ρ€Ρ†, АндрС-ΠœΠ°Ρ€ΠΈ АмпСр, АлСссандро Π’ΠΎΠ»ΡŒΡ‚, ДТСймс Π’Π°Ρ‚Ρ‚, Π€Π°Ρ€Π°Π΄Π΅ΠΉ, Π―ΠΊΠΎΠ±ΠΈ, Π›Π΅Π½Ρ†, Π“Ρ€Π°ΠΌΠΌ, Π€ΠΎΠ½Ρ‚Π΅Π½, Π›ΠΎΠ΄Ρ‹Π³ΠΈΠ½, Π”ΠΎΠ»ΠΈΠ²ΠΎ-Π”ΠΎΠ±Ρ€ΠΎΠ²ΠΎΠ»ΡŒΡΠΊΠΈΠΉ, ВСсла, Π―Π±Π»ΠΎΡ‡ΠΊΠΎΠ², Π”Π΅ΠΏΡ€Ρ‘, Эдисон, МаксвСлл, ΠšΠΈΡ€Ρ…Π³ΠΎΡ„, Π±Ρ€Π°Ρ‚ΡŒΡ БимСнсы ΠΈ Π±Ρ€Π°Ρ‚ΡŒΡ ВСстингаузы – Π² истории элСктричСства ΠΌΠ½ΠΎΠ³ΠΎ славных ΠΈΠΌΠ΅Π½, достойных Ρ‚ΠΎΠ³ΠΎ, Ρ‡Ρ‚ΠΎΠ±Ρ‹ ΠΌΡ‹ ΠΎ Π½ΠΈΡ… ΠΏΠΎΠΌΠ½ΠΈΠ»ΠΈ. Π’ ΠΎΠ±Ρ‰Π΅ΠΌ, Ссли ΠΊΠΎΠΌΡƒ-Ρ‚ΠΎ хочСтся ΠΏΡ€ΠΈΠΏΠΎΠΌΠ½ΠΈΡ‚ΡŒ подробности Ρ‚ΠΎΠ³ΠΎ, ΠΊΠ°ΠΊ всС Π½Π°Ρ‡ΠΈΠ½Π°Π»ΠΎΡΡŒ, милости просим, Π° ΡΡ‚Π°Ρ‚ΡŒΡ Дмитрия – Π½Π°Ρ‡Π°Π»ΠΎ совсСм Π΄Ρ€ΡƒΠ³ΠΎΠΉ истории. ΠžΡ‡Π΅Π½ΡŒ надССмся, Ρ‡Ρ‚ΠΎ ΠΎΠ½Π° Π²Π°ΠΌ понравится, Π° ΠΏΡ€ΠΎΠ΄ΠΎΠ»ΠΆΠ΅Π½ΠΈΠ΅ статСй Дмитрия Π’Π°Π»Π°Π½ΠΎΠ²Π° ΠΌΡ‹ ΡƒΠ²ΠΈΠ΄ΠΈΠΌ Π² самоС блиТайшСС врСмя.

Π£Π²Π°ΠΆΠ°Π΅ΠΌΠΎΠ³ΠΎ Дмитрия ΠΎΡ‚ сСбя Π»ΠΈΡ‡Π½ΠΎ – с Π΄Π΅Π±ΡŽΡ‚ΠΎΠΌ, ΠΊΠΎ всСм читатСлям ΠΏΡ€ΠΎΡΡŒΠ±Π° – Π½Π΅ ΡΠΊΡƒΠΏΠΈΡ‚Π΅ΡΡŒ Π½Π° ΠΊΠΎΠΌΠΌΠ΅Π½Ρ‚Π°Ρ€ΠΈΠΈ!

Β 

Π§Ρ‚ΠΎ Ρ‚Π°ΠΊΠΎΠ΅ элСктричСский Ρ‚ΠΎΠΊ, ΠΎΡ‚ΠΊΡƒΠ΄Π° ΠΎΠ½ бСрСтся ΠΈ ΠΊΠ°ΠΊ добираСтся Π΄ΠΎ Π½Π°ΡˆΠΈΡ… Π΄ΠΎΠΌΠΎΠ²?

Для Ρ‡Π΅Π³ΠΎ Π½Π°ΠΌ элСктроэнСргия ΠΈ насколько ΠΎΠ½Π° ΠΏΠΎΠΌΠΎΠ³Π°Π΅Ρ‚ Π½Π°ΠΌ ΠΆΠΈΡ‚ΡŒ, ΠΌΠΎΠΆΠ΅Ρ‚ ΡƒΠ·Π½Π°Ρ‚ΡŒ ΠΊΠ°ΠΆΠ΄Ρ‹ΠΉ, обвСдя критичСским взглядом своС ΠΆΠΈΠ»ΠΈΡ‰Π΅ ΠΈ мСсто Ρ€Π°Π±ΠΎΡ‚Ρ‹.

ΠŸΠ΅Ρ€Π²ΠΎΠ΅, Ρ‡Ρ‚ΠΎ бросаСтся Π² Π³Π»Π°Π·Π°, это освСщСниС. И Π²Π΅Ρ€Π½ΠΎ, Π±Π΅Π· Π½Π΅Π³ΠΎ Π΄Π°ΠΆΠ΅ 8-часовой Ρ€Π°Π±ΠΎΡ‡ΠΈΠΉ дСнь прСвратился Π±Ρ‹ Π² ΠΌΡƒΠΊΡƒ. Π”ΠΎΠ±ΠΈΡ€Π°Ρ‚ΡŒΡΡ Π΄ΠΎ Ρ€Π°Π±ΠΎΡ‚Ρ‹ Π²ΠΎ ΠΌΠ½ΠΎΠ³ΠΈΡ… мСгаполисах ΠΈ Ρ‚Π°ΠΊ нСбольшоС ΡΡ‡Π°ΡΡ‚ΡŒΠ΅, Π° Ссли придСтся это Π΄Π΅Π»Π°Ρ‚ΡŒ Π² Ρ‚Π΅ΠΌΠ½ΠΎΡ‚Π΅? А Π·ΠΈΠΌΠΎΠΉ Ρ‚Π°ΠΊ ΠΈ Π² ΠΎΠ±Π° ΠΊΠΎΠ½Ρ†Π°! Π“Π°Π·ΠΎΠ²Ρ‹Π΅ Ρ„ΠΎΠ½Π°Ρ€ΠΈ ΠΏΠΎΠΌΠΎΠ³ΡƒΡ‚ Π½Π° Π³Π»Π°Π²Π½Ρ‹Ρ… магистралях, Π½ΠΎ Ρ‡ΡƒΡ‚ΡŒ свСрнул Π² сторону, ΠΈ Π½Π΅ Π²ΠΈΠ΄Π½ΠΎ Π½ΠΈ Π·Π³ΠΈ. МоТно Π»Π΅Π³ΠΊΠΎ ΠΏΡ€ΠΎΠ²Π°Π»ΠΈΡ‚ΡŒΡΡ Π² ΠΏΠΎΠ΄Π²Π°Π» ΠΈΠ»ΠΈ яму. А Π·Π° Π³ΠΎΡ€ΠΎΠ΄ΠΎΠΌ Π½Π° ΠΏΡ€ΠΈΡ€ΠΎΠ΄Π΅, освСщаСмой Ρ‚ΠΎΠ»ΡŒΠΊΠΎ свСтом Π·Π²Π΅Π·Π΄?

НочноС освСщСниС ΡƒΠ»ΠΈΡ†Ρ‹, Π€ΠΎΡ‚ΠΎ:Β pixabay.com

Π£Π΄Π°Π»ΡΡ‚ΡŒ ΠΆΠ°Ρ€Ρƒ ΠΈΠ· офисов, ΠΊΡƒΠ΄Π° с Ρ‚Ρ€ΡƒΠ΄ΠΎΠΌ добрался, Π±Π΅Π· элСктричСства Ρ‚ΠΎΠΆΠ΅ Π½Π΅Ρ‡Π΅ΠΌ. МоТно, ΠΊΠΎΠ½Π΅Ρ‡Π½ΠΎ, ΠΎΡ‚ΠΊΡ€Ρ‹Ρ‚ΡŒ ΠΎΠΊΠ½Π° ΠΈ ΠΎΠ±Π²ΡΠ·Π°Ρ‚ΡŒ Π³ΠΎΠ»ΠΎΠ²Ρƒ ΠΌΠΎΠΊΡ€Ρ‹ΠΌ ΠΏΠΎΠ»ΠΎΡ‚Π΅Π½Ρ†Π΅ΠΌ, Π½ΠΎ Π½Π°Π΄ΠΎΠ»Π³ΠΎ Π»ΠΈ это ΠΏΠΎΠΌΠΎΠΆΠ΅Ρ‚. ΠšΠ°Ρ‡Π°ΡŽΡ‰ΠΈΠΌ Π²ΠΎΠ΄Ρƒ насосам Ρ‚ΠΎΠΆΠ΅ Π½ΡƒΠΆΠ½ΠΎ элСктричСство, ΠΈΠ»ΠΈ придСтся рСгулярно Ρ…ΠΎΠ΄ΠΈΡ‚ΡŒ с Π²Π΅Π΄Ρ€ΠΎΠΌ Π½Π° Ρ€ΡƒΡ‡Π½ΡƒΡŽ ΠΊΠΎΠ»ΠΎΠ½ΠΊΡƒ.

ΠšΠΎΡ„Π΅ Π² офисС? Π—Π°Π±ΡƒΠ΄ΡŒΡ‚Π΅! Волько Ссли всСм сразу ΠΈ Π½Π΅ часто, Ρ‡Ρ‚ΠΎΠ±Ρ‹ Π΄Ρ‹ΠΌ ΠΎΡ‚ ΡΠ³ΠΎΡ€Π°ΡŽΡ‰Π΅Π³ΠΎ угля Π½Π΅ ΠΎΡ‚Ρ€Π°Π²ΠΈΠ» Ρ€Π°Π±ΠΎΡ‡ΡƒΡŽ атмосфСру. Или Π·Π° Π΄ΠΎΠΏΠΎΠ»Π½ΠΈΡ‚Π΅Π»ΡŒΠ½ΡƒΡŽ Π΄Π΅Π½Π΅ΠΆΠΊΡƒ ΠΏΠΎΠ»ΡƒΡ‡Π°Ρ‚ΡŒ ΠΈΠ· сосСднСго Ρ‚Ρ€Π°ΠΊΡ‚ΠΈΡ€Π°.

ΠžΡ‚ΠΏΡ€Π°Π²ΠΈΡ‚ΡŒ письмо Π² сосСдний офис? Надо Π²Π·ΡΡ‚ΡŒ Π±ΡƒΠΌΠ°Π³Ρƒ, Π½Π°ΠΏΠΈΡΠ°Ρ‚ΡŒ письмо ΠΎΡ‚ Ρ€ΡƒΠΊΠΈ, Π·Π°Ρ‚Π΅ΠΌ Π½ΠΎΠΆΠΊΠ°ΠΌΠΈ отнСсти Π΅Π³ΠΎ. На Π΄Ρ€ΡƒΠ³ΠΎΠΉ ΠΊΠΎΠ½Π΅Ρ† Π³ΠΎΡ€ΠΎΠ΄Π°? Π’Ρ‹Π·Ρ‹Π²Π°Π΅ΠΌ ΠΊΡƒΡ€ΡŒΠ΅Ρ€Π°. Π’ Π΄Ρ€ΡƒΠ³ΡƒΡŽ страну? А Π²Ρ‹ Π·Π½Π°Π΅Ρ‚Π΅, сколько это Π±ΡƒΠ΄Π΅Ρ‚ ΡΡ‚ΠΎΠΈΡ‚ΡŒ? К Ρ‚ΠΎΠΌΡƒ ΠΆΠ΅ ΠΎΡ‚Π²Π΅Ρ‚Π° Π½Π΅ ΠΆΠ΄ΠΈΡ‚Π΅ Ρ€Π°Π½Π΅Π΅ ΠΏΠΎΠ»ΡƒΠ³ΠΎΠ΄Π° ΠΈΠ· сосСдних стран ΠΈ ΠΎΡ‚ Π³ΠΎΠ΄Π° Π΄ΠΎ пяти ΠΈΠ·-Π·Π° ΠΎΠΊΠ΅Π°Π½Π°.

Π’Π΅Ρ€Π½ΡƒΠ»ΠΈΡΡŒ Π΄ΠΎΠΌΠΎΠΉ, Π½Π°Π΄ΠΎ Π·Π°ΠΆΠ΅Ρ‡ΡŒ свСчи. Π§ΠΈΡ‚Π°Ρ‚ΡŒ ΠΏΡ€ΠΈ Π½ΠΈΡ… – ΠΌΡƒΡ‡Π΅Π½ΠΈΠ΅ для Π³Π»Π°Π·, поэтому придСтся Π·Π°Π½ΡΡ‚ΡŒΡΡ Ρ‡Π΅ΠΌ-Ρ‚ΠΎ Π΄Ρ€ΡƒΠ³ΠΈΠΌ. А Ρ‡Π΅ΠΌ? Π’Π’ Π½Π΅Ρ‚, ΠΊΠΎΠΌΠΏΡŒΡŽΡ‚Π΅Ρ€ΠΎΠ² Π½Π΅Ρ‚, смартфонов – ΠΈ Ρ‚Π΅Ρ… Π½Π΅Ρ‚, ΠΈΠ±ΠΎ Π½Π΅Ρ‡Π΅ΠΌ ΠΈΡ… Π·Π°ΠΏΠΈΡ‚Π°Ρ‚ΡŒ. Π›Π΅ΠΆΠΈ Π½Π° Π»Π°Π²ΠΊΠ΅ ΠΈ гляди Π² ΠΏΠΎΡ‚ΠΎΠ»ΠΎΠΊ! Π₯отя Ρ€ΠΎΠΆΠ΄Π°Π΅ΠΌΠΎΡΡ‚ΡŒ Ρ‚ΠΎΡ‡Π½ΠΎ повысится.

К этому слСдуСт Π΄ΠΎΠ±Π°Π²ΠΈΡ‚ΡŒ, Ρ‡Ρ‚ΠΎ всС пластмассы ΠΈ удобрСния сСйчас ΠΏΠΎΠ»ΡƒΡ‡Π°ΡŽΡ‚ ΠΈΠ· ΠΏΡ€ΠΈΡ€ΠΎΠ΄Π½ΠΎΠ³ΠΎ Π³Π°Π·Π° Π½Π° Π·Π°Π²ΠΎΠ΄Π°Ρ…, Π³Π΄Π΅ крутятся тысячи ΠΌΠΎΡ‚ΠΎΡ€ΠΎΠ², ΠΏΡ€ΠΈΠ²ΠΎΠ΄ΠΈΠΌΡ‹Ρ… Π² Π΄Π²ΠΈΠΆΠ΅Π½ΠΈΠ΅ всё Ρ‚Π΅ΠΌ ΠΆΠ΅ элСктричСством. ΠžΡ‚ΡΡŽΠ΄Π° список доступных ΡƒΠ΄ΠΎΠ±Ρ€Π΅Π½ΠΈΠΉ сильно укорачиваСтся Π΄ΠΎ Ρ‚Π΅Ρ…, ΠΊΠΎΡ‚ΠΎΡ€Ρ‹Π΅ ΠΌΠΎΠΆΠ½ΠΎ ΠΏΡ€ΠΈΠ³ΠΎΡ‚ΠΎΠ²ΠΈΡ‚ΡŒ ΠΈΠ· ΠΏΡ€ΠΈΡ€ΠΎΠ΄Π½ΠΎΠ³ΠΎ ΡΡ‹Ρ€ΡŒΡ Π² Ρ‡Π°Π½Π°Ρ…, Ρ€Π°Π·ΠΌΠ΅ΡˆΠΈΠ²Π°Ρ Π² Π½ΠΈΡ… ΡΠ΄ΠΎΠ²ΠΈΡ‚ΡƒΡŽ ΠΆΠΈΠΆΡƒ Π»ΠΎΠΏΠ°Ρ‚ΠΊΠ°ΠΌΠΈ с Ρ€ΡƒΡ‡Π½Ρ‹ΠΌ, водяным ΠΈΠ»ΠΈ ΠΏΠ°Ρ€ΠΎΠ²Ρ‹ΠΌ ΠΏΡ€ΠΈΠ²ΠΎΠ΄ΠΎΠΌ. Как Ρ€Π΅Π·ΡƒΠ»ΡŒΡ‚Π°Ρ‚, сильно сТимаСтся объСм ΠΏΡ€ΠΎΠΈΠ·Π²ΠΎΠ΄ΠΈΠΌΡ‹Ρ… ΠΏΡ€ΠΎΠ΄ΡƒΠΊΡ‚ΠΎΠ².

О пластмассах – Π·Π°Π±ΡƒΠ΄ΡŒΡ‚Π΅! Π­Π±ΠΎΠ½ΠΈΡ‚ – нашС Π²Ρ‹ΡΡˆΠ΅Π΅ ΡΡ‡Π°ΡΡ‚ΡŒΠ΅ ΠΈΠ· Π΄Π»ΠΈΠ½Π½ΠΎΠ³ΠΎ списка. А ΠΈΠ· ΠΌΠ΅Ρ‚Π°Π»Π»ΠΎΠ² самым доступным становится Ρ‡ΡƒΠ³ΡƒΠ½. Из ΠΌΠ΅Π΄ΠΈΡ†ΠΈΠ½Ρ‹ Π½Π° сцСну Π² качСствС Π³Π»Π°Π²Π½ΠΎΠ³ΠΎ орудия снова Π²Ρ‹ΡΡ‚ΡƒΠΏΠ°ΡŽΡ‚ стСтоскоп ΠΈ быстро Ρ€ΠΆΠ°Π²Π΅ΡŽΡ‰ΠΈΠΉ скальпСль. ΠžΡΡ‚Π°Π»ΡŒΠ½ΠΎΠ΅ ΠΊΠ°Π½Π΅Ρ‚ Π² Π›Π΅Ρ‚Ρƒ.

ΠŸΡ€ΠΎΠ΄ΠΎΠ»ΠΆΠ°Ρ‚ΡŒ ΠΌΠΎΠΆΠ½ΠΎ Π΄ΠΎΠ»Π³ΠΎ, Π½ΠΎ идСя Π΄ΠΎΠ»ΠΆΠ½Π° Π±Ρ‹Ρ‚ΡŒ ΡƒΠΆΠ΅ понятна. Нам Π½ΡƒΠΆΠ½ΠΎ элСктричСство. ΠœΡ‹ ΠΌΠΎΠΆΠ΅ΠΌ Π²Ρ‹ΠΆΠΈΡ‚ΡŒ Π±Π΅Π· Π½Π΅Π³ΠΎ, Π½ΠΎ Ρ‡Ρ‚ΠΎ это Π±ΡƒΠ΄Π΅Ρ‚ Π·Π° Тизнь! Π’Π°ΠΊ ΠΎΡ‚ΠΊΡƒΠ΄Π° ΠΆΠ΅ появилось это волшСбноС элСктричСство?

ΠžΡ‚ΠΊΡ€Ρ‹Ρ‚ΠΈΠ΅ элСктричСства

ВсС ΠΌΡ‹ Π·Π½Π°Π΅ΠΌ Ρ„ΠΈΠ·ΠΈΡ‡Π΅ΡΠΊΡƒΡŽ истину, Ρ‡Ρ‚ΠΎ Π½ΠΈΡ‡Ρ‚ΠΎ Π½ΠΈΠΊΡƒΠ΄Π° бСсслСдно Π½Π΅ исчСзаСт, Π° Ρ‚ΠΎΠ»ΡŒΠΊΠΎ ΠΏΠ΅Ρ€Π΅Ρ…ΠΎΠ΄ΠΈΡ‚ ΠΈΠ· ΠΎΠ΄Π½ΠΎΠ³ΠΎ состояния Π² Π΄Ρ€ΡƒΠ³ΠΎΠ΅. Π‘ этой истиной столкнулся грСчСский философ ЀалСс ΠœΠΈΠ»Π΅Ρ‚ΡΠΊΠΈΠΉ Π² VII Π²Π΅ΠΊΠ΅ Π΄ΠΎ Π½. э. ΠΎΠ±Π½Π°Ρ€ΡƒΠΆΠΈΠ² элСктричСство ΠΊΠ°ΠΊ Π²ΠΈΠ΄ энСргии, натирая кусок янтаря ΡˆΠ΅Ρ€ΡΡ‚ΡŒΡŽ. Π§Π°ΡΡ‚ΡŒ мСханичСской энСргии ΠΏΡ€ΠΈ этом ΠΏΠ΅Ρ€Π΅ΡˆΠ»Π° Π² ΡΠ»Π΅ΠΊΡ‚Ρ€ΠΈΡ‡Π΅ΡΠΊΡƒΡŽ ΠΈ ΡΠ½Ρ‚Π°Ρ€ΡŒ (Π½Π° дрСвнСгрСчСском «элСктрон») элСктризовался, Ρ‚ΠΎ Π΅ΡΡ‚ΡŒ ΠΏΡ€ΠΈΠΎΠ±Ρ€Π΅Π» свойства ΠΏΡ€ΠΈΡ‚ΡΠ³ΠΈΠ²Π°Ρ‚ΡŒ Π»Π΅Π³ΠΊΠΈΠ΅ ΠΏΡ€Π΅Π΄ΠΌΠ΅Ρ‚Ρ‹.

Π­Ρ‚ΠΎΡ‚ Π²ΠΈΠ΄ элСктричСства сСйчас Π½Π°Π·Ρ‹Π²Π°ΡŽΡ‚ статичСским, ΠΈ ΠΎΠ½ нашСл сСбС ΡˆΠΈΡ€ΠΎΠΊΠΎΠ΅ ΠΏΡ€ΠΈΠΌΠ΅Π½Π΅Π½ΠΈΠ΅, Π² Ρ‚ΠΎΠΌ числС Π² систСмах очистки Π³Π°Π·ΠΎΠ² Π½Π° элСктростанциях. Но Π² Π”Ρ€Π΅Π²Π½Π΅ΠΉ Π“Ρ€Π΅Ρ†ΠΈΠΈ Π΅ΠΌΡƒ Π½Π΅ нашлось примСнСния ΠΈ, Ссли Π±Ρ‹ ЀалСс ΠœΠΈΠ»Π΅Ρ‚ΡΠΊΠΈΠΉ Π½Π΅ оставил послС сСбя записСй ΠΎ своих экспСримСнтах, ΠΌΡ‹ Π±Ρ‹ Π½ΠΈΠΊΠΎΠ³Π΄Π° Π½Π΅ ΡƒΠ·Π½Π°Π»ΠΈ, ΠΊΡ‚ΠΎ Π±Ρ‹Π» Ρ‚ΠΎΡ‚ ΠΏΠ΅Ρ€Π²Ρ‹ΠΉ ΠΌΡ‹ΡΠ»ΠΈΡ‚Π΅Π»ΡŒ, Π·Π°ΠΎΡΡ‚Ρ€ΠΈΠ²ΡˆΠΈΠΉ своС Π²Π½ΠΈΠΌΠ°Π½ΠΈΠ΅ Π½Π° Π²ΠΈΠ΄Π΅ энСргии, ΡΠ²Π»ΡΡŽΡ‰Π΅ΠΉΡΡ Π΅Π΄Π²Π° Π»ΠΈ Π½Π΅ самой чистой срСди всСх, с ΠΊΠΎΡ‚ΠΎΡ€Ρ‹ΠΌΠΈ ΠΌΡ‹ Π·Π½Π°ΠΊΠΎΠΌΡ‹ ΠΏΠΎ настоящий дСнь. Π•ΡŽ Ρ‚Π°ΠΊΠΆΠ΅ Π½Π°ΠΈΠ±ΠΎΠ»Π΅Π΅ ΡƒΠ΄ΠΎΠ±Π½ΠΎ ΡƒΠΏΡ€Π°Π²Π»ΡΡ‚ΡŒ.

Π‘Π°ΠΌ Ρ‚Π΅Ρ€ΠΌΠΈΠ½ «элСктричСство» – Ρ‚ΠΎ Π΅ΡΡ‚ΡŒ Β«ΡΠ½Ρ‚Π°Ρ€Π½ΠΎΡΡ‚ΡŒΒ» – Π²Π²Π΅Π» Π² ΡƒΠΏΠΎΡ‚Ρ€Π΅Π±Π»Π΅Π½ΠΈΠ΅ Уильям Π“ΠΈΠ»Π±Π΅Ρ€Ρ‚ Π² 1600 Π³ΠΎΠ΄Ρƒ. Π‘ этого Π²Ρ€Π΅ΠΌΠ΅Π½ΠΈ с элСктричСством Π½Π°Ρ‡ΠΈΠ½Π°ΡŽΡ‚ ΡˆΠΈΡ€ΠΎΠΊΠΎ ΡΠΊΡΠΏΠ΅Ρ€ΠΈΠΌΠ΅Π½Ρ‚ΠΈΡ€ΠΎΠ²Π°Ρ‚ΡŒ, ΠΏΡ‹Ρ‚Π°ΡΡΡŒ Ρ€Π°Π·Π³Π°Π΄Π°Ρ‚ΡŒ Π΅Π³ΠΎ ΠΏΡ€ΠΈΡ€ΠΎΠ΄Ρƒ.

Как Ρ€Π΅Π·ΡƒΠ»ΡŒΡ‚Π°Ρ‚, с 1600 ΠΏΠΎ 1747 Π³ΠΎΠ΄Ρ‹ послСдовала Ρ‡Π΅Ρ€Π΅Π΄Π° ΡƒΠ²Π»Π΅ΠΊΠ°Ρ‚Π΅Π»ΡŒΠ½Ρ‹Ρ… ΠΎΡ‚ΠΊΡ€Ρ‹Ρ‚ΠΈΠΉ ΠΈ появилась пСрвая тСория элСктричСства, созданная Π°ΠΌΠ΅Ρ€ΠΈΠΊΠ°Π½Ρ†Π΅ΠΌ Π‘Π΅Π½Π΄ΠΆΠ°ΠΌΠΈΠ½ΠΎΠΌ Π€Ρ€Π°Π½ΠΊΠ»ΠΈΠ½ΠΎΠΌ. Он Π²Π²Π΅Π» понятиС ΠΏΠΎΠ»ΠΎΠΆΠΈΡ‚Π΅Π»ΡŒΠ½ΠΎΠ³ΠΎ ΠΈ ΠΎΡ‚Ρ€ΠΈΡ†Π°Ρ‚Π΅Π»ΡŒΠ½ΠΎΠ³ΠΎ заряда, ΠΈΠ·ΠΎΠ±Ρ€Π΅Π» ΠΌΠΎΠ»Π½ΠΈΠ΅ΠΎΡ‚Π²ΠΎΠ΄ ΠΈ с Π΅Π³ΠΎ ΠΏΠΎΠΌΠΎΡ‰ΡŒΡŽ Π΄ΠΎΠΊΠ°Π·Π°Π» ΡΠ»Π΅ΠΊΡ‚Ρ€ΠΈΡ‡Π΅ΡΠΊΡƒΡŽ ΠΏΡ€ΠΈΡ€ΠΎΠ΄Ρƒ ΠΌΠΎΠ»Π½ΠΈΠΉ.

Π”Π°Π»Π΅Π΅ Π² 1785 происходит ΠΎΡ‚ΠΊΡ€Ρ‹Ρ‚ΠΈΠ΅ Π·Π°ΠΊΠΎΠ½Π° ΠšΡƒΠ»ΠΎΠ½Π°, Π° Π² 1800 Π³ΠΎΠ΄Ρƒ ΠΈΡ‚Π°Π»ΡŒΡΠ½Π΅Ρ† Π’ΠΎΠ»ΡŒΡ‚Π° ΠΈΠ·ΠΎΠ±Ρ€Π΅Ρ‚Π°Π΅Ρ‚ Π³Π°Π»ΡŒΠ²Π°Π½ΠΈΡ‡Π΅ΡΠΊΠΈΠΉ элСмСнт (ΠΏΠ΅Ρ€Π²Ρ‹ΠΉ источник постоянного Ρ‚ΠΎΠΊΠ°, ΠΏΡ€Π΅Π΄ΡˆΠ΅ΡΡ‚Π²Π΅Π½Π½ΠΈΠΊ Π½Ρ‹Π½Π΅ΡˆΠ½ΠΈΡ… Π±Π°Ρ‚Π°Ρ€Π΅ΠΉ ΠΈ аккумуляторов), ΠΏΡ€Π΅Π΄ΡΡ‚Π°Π²Π»ΡΠ²ΡˆΠΈΠΉ собой столб ΠΈΠ· Ρ†ΠΈΠ½ΠΊΠΎΠ²Ρ‹Ρ… ΠΈ сСрСбряных ΠΊΡ€ΡƒΠΆΠΎΡ‡ΠΊΠΎΠ², Ρ€Π°Π·Π΄Π΅Π»Ρ‘Π½Π½Ρ‹Ρ… смочСнной Π² подсолСнной Π²ΠΎΠ΄Π΅ Π±ΡƒΠΌΠ°Π³ΠΎΠΉ. Π‘ появлСниСм этого, ΡΡ‚Π°Π±ΠΈΠ»ΡŒΠ½ΠΎΠ³ΠΎ ΠΏΠΎ Ρ‚Π΅ΠΌ Π²Ρ€Π΅ΠΌΠ΅Π½Π°ΠΌ, источника элСктричСства Π½ΠΎΠ²Ρ‹Π΅ ΠΈ ваТнСйшиС открытия быстро ΡΠ»Π΅Π΄ΡƒΡŽΡ‚ ΠΎΠ΄Π½ΠΎ Π·Π° Π΄Ρ€ΡƒΠ³ΠΈΠΌ.

Майкл Π€Π°Ρ€Π°Π΄Π΅ΠΉ, Ρ‡ΠΈΡ‚Π°ΡŽΡ‰ΠΈΠΉ Ρ€ΠΎΠΆΠ΄Π΅ΡΡ‚Π²Π΅Π½ΡΠΊΡƒΡŽ Π»Π΅ΠΊΡ†ΠΈΡŽ Π² ΠšΠΎΡ€ΠΎΠ»Π΅Π²ΡΠΊΠΎΠΌ институтС. Π€Ρ€Π°Π³ΠΌΠ΅Π½Ρ‚ Π»ΠΈΡ‚ΠΎΠ³Ρ€Π°Ρ„ΠΈΠΈ, Π€ΠΎΡ‚ΠΎ:Β republic.ru

Π’ 1820 Π³ΠΎΠ΄Ρƒ датский Ρ„ΠΈΠ·ΠΈΠΊ ЭрстСд ΠΎΠ±Π½Π°Ρ€ΡƒΠΆΠΈΠ» элСктромагнитноС взаимодСйствиС: замыкая ΠΈ размыкая Ρ†Π΅ΠΏΡŒ с постоянным Ρ‚ΠΎΠΊΠΎΠΌ, ΠΎΠ½ Π·Π°ΠΌΠ΅Ρ‚ΠΈΠ» Ρ†ΠΈΠΊΠ»ΠΈΡ‡Π½Ρ‹Π΅ колСбания стрСлки компаса, располоТСнной Π²Π±Π»ΠΈΠ·ΠΈ ΠΏΡ€ΠΎΠ²ΠΎΠ΄Π½ΠΈΠΊΠ°. А Π² 1821 Π³ΠΎΠ΄Ρƒ французский Ρ„ΠΈΠ·ΠΈΠΊ АмпСр ΠΎΡ‚ΠΊΡ€Ρ‹Π», Ρ‡Ρ‚ΠΎ Π²ΠΎΠΊΡ€ΡƒΠ³ ΠΏΡ€ΠΎΠ²ΠΎΠ΄Π½ΠΈΠΊΠ° с ΠΏΠ΅Ρ€Π΅ΠΌΠ΅Π½Π½Ρ‹ΠΌ элСктричСским Ρ‚ΠΎΠΊΠΎΠΌ образуСтся ΠΏΠ΅Ρ€Π΅ΠΌΠ΅Π½Π½ΠΎΠ΅ элСктромагнитноС ΠΏΠΎΠ»Π΅. Π­Ρ‚ΠΎ ΠΏΠΎΠ·Π²ΠΎΠ»ΠΈΠ»ΠΎ ΡƒΠΆΠ΅ ΠœΠ°ΠΉΠΊΠ»Ρƒ Π€Π°Ρ€Π°Π΄Π΅ΡŽ Π² 1831 Π³ΠΎΠ΄Ρƒ ΠΎΡ‚ΠΊΡ€Ρ‹Ρ‚ΡŒ ΡΠ»Π΅ΠΊΡ‚Ρ€ΠΎΠΌΠ°Π³Π½ΠΈΡ‚Π½ΡƒΡŽ ΠΈΠ½Π΄ΡƒΠΊΡ†ΠΈΡŽ, ΠΎΠΏΠΈΡΠ°Ρ‚ΡŒ уравнСниями элСктричСскоС ΠΈ ΠΌΠ°Π³Π½ΠΈΡ‚Π½ΠΎΠ΅ ΠΏΠΎΠ»Π΅ ΠΈ ΡΠΎΠ·Π΄Π°Ρ‚ΡŒ ΠΏΠ΅Ρ€Π²Ρ‹ΠΉ элСктрогСнСратор ΠΏΠ΅Ρ€Π΅ΠΌΠ΅Π½Π½ΠΎΠ³ΠΎ Ρ‚ΠΎΠΊΠ°. Π€Π°Ρ€Π°Π΄Π΅ΠΉ Π²Π΄Π²ΠΈΠ³Π°Π» ΠΊΠ°Ρ‚ΡƒΡˆΠΊΡƒ с ΠΏΡ€ΠΎΠ²ΠΎΠ΄ΠΎΠΌ Π² Π½Π°ΠΌΠ°Π³Π½ΠΈΡ‡Π΅Π½Π½Ρ‹ΠΉ сСрдСчник ΠΈ Π² Ρ€Π΅Π·ΡƒΠ»ΡŒΡ‚Π°Ρ‚Π΅ Π² ΠΎΠ±ΠΌΠΎΡ‚ΠΊΠ΅ ΠΊΠ°Ρ‚ΡƒΡˆΠΊΠΈ появлялся элСктричСский Ρ‚ΠΎΠΊ. Π€Π°Ρ€Π°Π΄Π΅ΠΉ Ρ‚Π°ΠΊΠΆΠ΅ ΠΏΡ€ΠΈΠ΄ΡƒΠΌΠ°Π» ΠΏΠ΅Ρ€Π²Ρ‹ΠΉ ΡΠ»Π΅ΠΊΡ‚Ρ€ΠΎΠ΄Π²ΠΈΠ³Π°Ρ‚Π΅Π»ΡŒ – ΠΏΡ€ΠΎΠ²ΠΎΠ΄Π½ΠΈΠΊ с элСктричСским Ρ‚ΠΎΠΊΠΎΠΌ, Π²Ρ€Π°Ρ‰Π°ΡŽΡ‰ΠΈΠΉΡΡ Π²ΠΎΠΊΡ€ΡƒΠ³ постоянного ΠΌΠ°Π³Π½ΠΈΡ‚Π°.

ВсСх участников Β«Π³ΠΎΠ½ΠΊΠΈ Π·Π° элСктричСством» Π½Π΅Π²ΠΎΠ·ΠΌΠΎΠΆΠ½ΠΎ ΡƒΠΏΠΎΠΌΡΠ½ΡƒΡ‚ΡŒ Π² этой ΡΡ‚Π°Ρ‚ΡŒΠ΅, Π½ΠΎ Ρ€Π΅Π·ΡƒΠ»ΡŒΡ‚Π°Ρ‚ΠΎΠΌ ΠΈΡ… усилий явилась доказуСмая экспСримСнтом тСория, Π΄Π΅Ρ‚Π°Π»ΡŒΠ½ΠΎ ΠΎΠΏΠΈΡΡ‹Π²Π°ΡŽΡ‰Π°Ρ элСктричСство ΠΈ ΠΌΠ°Π³Π½Π΅Ρ‚ΠΈΠ·ΠΌ, Π² соотвСтствии с ΠΊΠΎΡ‚ΠΎΡ€ΠΎΠΉ ΠΌΡ‹ ΠΏΡ€ΠΎΠΈΠ·Π²ΠΎΠ΄ΠΈΠΌ сСйчас всё, Ρ‡Ρ‚ΠΎ Ρ‚Ρ€Π΅Π±ΡƒΠ΅Ρ‚ элСктричСства для своСго функционирования.

ΠŸΠΎΡΡ‚ΠΎΡΠ½Π½Ρ‹ΠΉ ΠΈΠ»ΠΈ ΠΏΠ΅Ρ€Π΅ΠΌΠ΅Π½Π½Ρ‹ΠΉ Ρ‚ΠΎΠΊ?

Π’ ΠΊΠΎΠ½Ρ†Π΅ 1880-Ρ… Π³ΠΎΠ΄ΠΎΠ², Π΅Ρ‰Π΅ Π΄ΠΎ появлСния ΠΌΠΈΡ€ΠΎΠ²Ρ‹Ρ… стандартов Π½Π° производство, распрСдСлСниС ΠΈ ΠΏΠΎΡ‚Ρ€Π΅Π±Π»Π΅Π½ΠΈΠ΅ ΠΏΡ€ΠΎΠΌΡ‹ΡˆΠ»Π΅Π½Π½ΠΎΠΉ элСктроэнСргии, Ρ€Π°Π·Ρ€Π°Π·ΠΈΠ»Π°ΡΡŒ Π±ΠΈΡ‚Π²Π° ΠΌΠ΅ΠΆΠ΄Ρƒ сторонниками использования постоянного ΠΈ ΠΏΠ΅Ρ€Π΅ΠΌΠ΅Π½Π½ΠΎΠ³ΠΎ Ρ‚ΠΎΠΊΠ°. Π’ΠΎ Π³Π»Π°Π²Π΅ противостоящих Π΄Ρ€ΡƒΠ³ Π΄Ρ€ΡƒΠ³Ρƒ Π°Ρ€ΠΌΠΈΠΉ встали ВСсла ΠΈ Эдисон.

Оба Π±Ρ‹Π»ΠΈ Ρ‚Π°Π»Π°Π½Ρ‚Π»ΠΈΠ²Ρ‹ΠΌΠΈ изобрСтатСлями. Π Π°Π·Π²Π΅ Ρ‡Ρ‚ΠΎ Эдисон ΠΎΠ±Π»Π°Π΄Π°Π» ΠΊΡƒΠ΄Π° Π±ΠΎΠ»Π΅Π΅ Ρ€Π°Π·Π²ΠΈΡ‚Ρ‹ΠΌΠΈ способностями ΠΊ бизнСсу ΠΈ ΠΊ ΠΌΠΎΠΌΠ΅Π½Ρ‚Ρƒ Π½Π°Ρ‡Π°Π»Π° Β«Π²ΠΎΠΉΠ½Ρ‹Β» успСл Π·Π°ΠΏΠ°Ρ‚Π΅Π½Ρ‚ΠΎΠ²Π°Ρ‚ΡŒ мноТСство тСхничСских Ρ€Π΅ΡˆΠ΅Π½ΠΈΠΉ, Π² ΠΊΠΎΡ‚ΠΎΡ€Ρ‹Ρ… использовался постоянный Ρ‚ΠΎΠΊ (Π² Ρ‚ΠΎ врСмя Π² БША постоянный Ρ‚ΠΎΠΊ являлся стандартом ΠΏΠΎ ΡƒΠΌΠΎΠ»Ρ‡Π°Π½ΠΈΡŽ; постоянным называСтся Ρ‚ΠΎΠΊ, Π½Π°ΠΏΡ€Π°Π²Π»Π΅Π½ΠΈΠ΅ ΠΊΠΎΡ‚ΠΎΡ€ΠΎΠ³ΠΎ Π½Π΅ мСняСтся ΠΏΠΎ Π²Ρ€Π΅ΠΌΠ΅Π½ΠΈ).

Но Π±Ρ‹Π»Π° ΠΎΠ΄Π½Π° ΠΏΡ€ΠΎΠ±Π»Π΅ΠΌΠ°: Π² Ρ‚Π΅ Π²Ρ€Π΅ΠΌΠ΅Π½Π° постоянный Ρ‚ΠΎΠΊ Π±Ρ‹Π»ΠΎ ΠΎΡ‡Π΅Π½ΡŒ Ρ‚Ρ€ΡƒΠ΄Π½ΠΎ Ρ‚Ρ€Π°Π½ΡΡ„ΠΎΡ€ΠΌΠΈΡ€ΠΎΠ²Π°Ρ‚ΡŒ Π² Π±ΠΎΠ»Π΅Π΅ высокоС ΠΈΠ»ΠΈ Π½ΠΈΠ·ΠΊΠΎΠ΅ напряТСниС. Π’Π΅Π΄ΡŒ Ссли сСгодня ΠΌΡ‹ ΠΏΠΎΠ»ΡƒΡ‡Π°Π΅ΠΌ ΡΠ»Π΅ΠΊΡ‚Ρ€ΠΎΡΠ½Π΅Ρ€Π³ΠΈΡŽ напряТСниСм 240 Π²ΠΎΠ»ΡŒΡ‚, Π° наш Ρ‚Π΅Π»Π΅Ρ„ΠΎΠ½ Ρ‚Ρ€Π΅Π±ΡƒΠ΅Ρ‚ 5 Π²ΠΎΠ»ΡŒΡ‚, ΠΌΡ‹ Π²Ρ‚Ρ‹ΠΊΠ°Π΅ΠΌ Π² Ρ€ΠΎΠ·Π΅Ρ‚ΠΊΡƒ ΡƒΠ½ΠΈΠ²Π΅Ρ€ΡΠ°Π»ΡŒΠ½ΡƒΡŽ ΠΊΠΎΡ€ΠΎΠ±ΠΎΡ‡ΠΊΡƒ, которая ΠΏΡ€Π΅ΠΎΠ±Ρ€Π°Π·ΡƒΠ΅Ρ‚ Ρ‡Ρ‚ΠΎ ΡƒΠ³ΠΎΠ΄Π½ΠΎ Π²ΠΎ Ρ‡Ρ‚ΠΎ ΡƒΠ³ΠΎΠ΄Π½ΠΎ Π² Π½ΡƒΠΆΠ½ΠΎΠΌ Π½Π°ΠΌ Π΄ΠΈΠ°ΠΏΠ°Π·ΠΎΠ½Π΅, ΠΈΡΠΏΠΎΠ»ΡŒΠ·ΡƒΡ соврСмСнныС транзисторы, управляСмыС ΠΊΡ€ΠΎΡˆΠ΅Ρ‡Π½Ρ‹ΠΌΠΈ логичСскими схСмами с ΠΈΠ·ΠΎΡ‰Ρ€Π΅Π½Π½Ρ‹ΠΌ ΠΏΡ€ΠΎΠ³Ρ€Π°ΠΌΠΌΠ½Ρ‹ΠΌ обСспСчСниСм. А Ρ‡Ρ‚ΠΎ ΠΌΠΎΠΆΠ½ΠΎ Π±Ρ‹Π»ΠΎ ΡΠ΄Π΅Π»Π°Ρ‚ΡŒ Ρ‚ΠΎΠ³Π΄Π°, ΠΊΠΎΠ³Π΄Π° Π΄ΠΎ изобрСтСния самых ΠΏΡ€ΠΈΠΌΠΈΡ‚ΠΈΠ²Π½Ρ‹Ρ… транзисторов ΠΎΡΡ‚Π°Π²Π°Π»ΠΎΡΡŒ Π΅Ρ‰Π΅ 70 Π»Π΅Ρ‚? И Ссли ΠΏΠΎ условиям элСктричСских ΠΏΠΎΡ‚Π΅Ρ€ΡŒ Ρ‚Ρ€Π΅Π±ΠΎΠ²Π°Π»ΠΎΡΡŒ ΠΏΠΎΠ²Ρ‹ΡΠΈΡ‚ΡŒ напряТСниС Π΄ΠΎ 100’000 Π²ΠΎΠ»ΡŒΡ‚, Ρ‡Ρ‚ΠΎΠ±Ρ‹ Π΄ΠΎΡΡ‚Π°Π²ΠΈΡ‚ΡŒ ΡΠ»Π΅ΠΊΡ‚Ρ€ΠΎΡΠ½Π΅Ρ€Π³ΠΈΡŽ Π½Π° расстояниС 100 ΠΈΠ»ΠΈ 200 ΠΊΠΈΠ»ΠΎΠΌΠ΅Ρ‚Ρ€ΠΎΠ², Π»ΡŽΠ±Ρ‹Π΅ столбы Π’ΠΎΠ»ΡŒΡ‚Π° ΠΈ ΠΏΡ€ΠΈΠΌΠΈΡ‚ΠΈΠ²Π½Ρ‹Π΅ Π³Π΅Π½Π΅Ρ€Π°Ρ‚ΠΎΡ€Ρ‹ постоянного Ρ‚ΠΎΠΊΠ° ΠΎΠΊΠ°Π·Ρ‹Π²Π°Π»ΠΈΡΡŒ Π±Π΅ΡΡΠΈΠ»ΡŒΠ½Ρ‹.

Понимая это, ВСсла выступал Π·Π° ΠΏΠ΅Ρ€Π΅ΠΌΠ΅Π½Π½Ρ‹ΠΉ Ρ‚ΠΎΠΊ, трансформация ΠΊΠΎΡ‚ΠΎΡ€ΠΎΠ³ΠΎ Π² Π»ΡŽΠ±Ρ‹Π΅ ΡƒΡ€ΠΎΠ²Π½ΠΈ напряТСния Π½Π΅ прСдставляла Ρ‚Ρ€ΡƒΠ΄Π° ΠΈ Π² Ρ‚Π΅ Π²Ρ€Π΅ΠΌΠ΅Π½Π° (ΠΏΠ΅Ρ€Π΅ΠΌΠ΅Π½Π½Ρ‹ΠΌ считаСтся Ρ‚ΠΎΠΊ, Π²Π΅Π»ΠΈΡ‡ΠΈΠ½Π° ΠΈ Π½Π°ΠΏΡ€Π°Π²Π»Π΅Π½ΠΈΠ΅ ΠΊΠΎΡ‚ΠΎΡ€ΠΎΠ³ΠΎ пСриодичСски ΠΌΠ΅Π½ΡΡŽΡ‚ΡΡ со Π²Ρ€Π΅ΠΌΠ΅Π½Π΅ΠΌ Π΄Π°ΠΆΠ΅ ΠΏΡ€ΠΈ Π½Π΅ΠΈΠ·ΠΌΠ΅Π½Π½ΠΎΠΌ сопротивлСнии этому Ρ‚ΠΎΠΊΡƒ; ΠΏΡ€ΠΈ частотС сСти 50Π“Ρ† это происходит 50 Ρ€Π°Π· Π² сСкунду). Эдисон ΠΆΠ΅, Π½Π΅ ТСлая Ρ‚Π΅Ρ€ΡΡ‚ΡŒ ΠΏΠ°Ρ‚Π΅Π½Ρ‚Π½Ρ‹Π΅ отчислСния сСбС, Ρ€Π°Π·Π²Π΅Ρ€Π½ΡƒΠ» кампанию ΠΏΠΎ дискрСдитации ΠΏΠ΅Ρ€Π΅ΠΌΠ΅Π½Π½ΠΎΠ³ΠΎ Ρ‚ΠΎΠΊΠ°. Он увСрял, Ρ‡Ρ‚ΠΎ этот Π²ΠΈΠ΄ Ρ‚ΠΎΠΊΠ° особо опасСн для всСго ΠΆΠΈΠ²ΠΎΠ³ΠΎ, ΠΈ Π² Π΄ΠΎΠΊΠ°Π·Π°Ρ‚Π΅Π»ΡŒΡΡ‚Π²ΠΎ ΠΏΡƒΠ±Π»ΠΈΡ‡Π½ΠΎ ΡƒΠ±ΠΈΠ²Π°Π» бродячих кошСк ΠΈ собак, прикладывая ΠΊ Π½ΠΈΠΌ элСктроды, соСдинСнныС с источником ΠΏΠ΅Ρ€Π΅ΠΌΠ΅Π½Π½ΠΎΠ³ΠΎ Ρ‚ΠΎΠΊΠ°.

Эдисон ΠΏΡ€ΠΎΠΈΠ³Ρ€Π°Π» Π±ΠΈΡ‚Π²Ρƒ, ΠΊΠΎΠ³Π΄Π° ВСсла ΠΏΡ€Π΅Π΄Π»ΠΎΠΆΠΈΠ» Π·Π° 399’000 Π΄ΠΎΠ»Π»Π°Ρ€ΠΎΠ² ΠΎΡΠ²Π΅Ρ‚ΠΈΡ‚ΡŒ вСсь Π³ΠΎΡ€ΠΎΠ΄ Π‘ΡƒΡ„Ρ„Π°Π»ΠΎ ΠΏΡ€ΠΎΡ‚ΠΈΠ² прСдлоТСния Эдисона ΡΠ΄Π΅Π»Π°Ρ‚ΡŒ Ρ‚ΠΎ ΠΆΠ΅ Π·Π° 554’000 Π΄ΠΎΠ»Π»Π°Ρ€ΠΎΠ². Π’ дСнь, ΠΊΠΎΠ³Π΄Π° Π³ΠΎΡ€ΠΎΠ΄ освСтился элСктричСством, ΠΏΠΎΠ»ΡƒΡ‡Π΅Π½Π½Ρ‹ΠΌ ΠΎΡ‚ станции, располоТСнной Ρƒ Ниагарского Π²ΠΎΠ΄ΠΎΠΏΠ°Π΄Π° ΠΈ Π²Ρ‹Ρ€Π°Π±Π°Ρ‚Ρ‹Π²Π°ΡŽΡ‰Π΅ΠΉ ΠΈΠΌΠ΅Π½Π½ΠΎ ΠΏΠ΅Ρ€Π΅ΠΌΠ΅Π½Π½Ρ‹ΠΉ Ρ‚ΠΎΠΊ, компания General Electric Π²Ρ‹ΠΊΠΈΠ½ΡƒΠ»Π° постоянный Ρ‚ΠΎΠΊ ΠΈΠ· рассмотрСния Π² своих Π±ΡƒΠ΄ΡƒΡ‰ΠΈΡ… бизнСс-ΠΏΡ€ΠΎΠ΅ΠΊΡ‚Π°Ρ…, ΠΏΠΎΠ»Π½ΠΎΡΡ‚ΡŒΡŽ ΠΏΠΎΠ΄Π΄Π΅Ρ€ΠΆΠ°Π² своим влияниСм ΠΈ дСньгами ΠΏΠ΅Ρ€Π΅ΠΌΠ΅Π½Π½Ρ‹ΠΉ Ρ‚ΠΎΠΊ.

Вомас Эдисон (БША), Рис.: cdn.redshift.autodesk.com

ΠœΠΎΠΆΠ΅Ρ‚ ΠΏΠΎΠΊΠ°Π·Π°Ρ‚ΡŒΡΡ, Ρ‡Ρ‚ΠΎ ΠΏΠ΅Ρ€Π΅ΠΌΠ΅Π½Π½Ρ‹ΠΉ Ρ‚ΠΎΠΊ навсСгда Π·Π°Π²ΠΎΠ΅Π²Π°Π» ΠΌΠΈΡ€. Однако Ρƒ Π½Π΅Π³ΠΎ ΠΈΠΌΠ΅ΡŽΡ‚ΡΡ наслСдствСнныС болячки, растущиС ΠΈΠ· самого Ρ„Π°ΠΊΡ‚Π° пСрСмСнности. ΠŸΡ€Π΅ΠΆΠ΄Π΅ всСго это элСктричСскиС ΠΏΠΎΡ‚Π΅Ρ€ΠΈ, связанныС с потСрями Π² ΠΈΠ½Π΄ΡƒΠΊΡ‚ΠΈΠ²Π½ΠΎΠΉ ΡΠΎΡΡ‚Π°Π²Π»ΡΡŽΡ‰Π΅ΠΉ ΠΏΡ€ΠΎΠ²ΠΎΠ΄ΠΎΠ² Π›Π­ΠŸ, ΠΊΠΎΡ‚ΠΎΡ€Ρ‹Π΅ ΠΈΡΠΏΠΎΠ»ΡŒΠ·ΡƒΡŽΡ‚ΡΡ для ΠΏΠ΅Ρ€Π΅Π΄Π°Ρ‡ΠΈ элСктроэнСргии Π½Π° большиС расстояния. Π­Ρ‚ΠΈ ΠΏΠΎΡ‚Π΅Ρ€ΠΈ Π² 10-20 Ρ€Π°Π· ΠΏΡ€Π΅Π²Ρ‹ΡˆΠ°ΡŽΡ‚ Π²ΠΎΠ·ΠΌΠΎΠΆΠ½Ρ‹Π΅ ΠΏΠΎΡ‚Π΅Ρ€ΠΈ Π² Ρ‚Π΅Ρ… ΠΆΠ΅ самых Π›Π­ΠŸ Π² случаС протСкания ΠΏΠΎ Π½ΠΈΠΌ постоянного Ρ‚ΠΎΠΊΠ°. Плюс сказываСтся ΠΏΠΎΠ²Ρ‹ΡˆΠ΅Π½Π½Π°Ρ ΡΠ»ΠΎΠΆΠ½ΠΎΡΡ‚ΡŒ синхронизации ΡƒΠ·Π»ΠΎΠ² энСргосистСмы (для ΠΏΡƒΡ‰Π΅Π³ΠΎ понимания, скаТСм, ΠΎΡ‚Π΄Π΅Π»ΡŒΠ½Ρ‹Ρ… Π³ΠΎΡ€ΠΎΠ΄ΠΎΠ²), вСдь для этого трСбуСтся Π½Π΅ Ρ‚ΠΎΠ»ΡŒΠΊΠΎ Π²Ρ‹Ρ€ΠΎΠ²Π½ΡΡ‚ΡŒ напряТСния ΡƒΠ·Π»ΠΎΠ², Π½ΠΎ ΠΈ ΠΈΡ… Ρ„Π°Π·Ρƒ, ΠΈΠ±ΠΎ ΠΏΠ΅Ρ€Π΅ΠΌΠ΅Π½Π½Ρ‹ΠΉ Ρ‚ΠΎΠΊ прСдставляСт собой Π²ΠΎΠ»Π½Ρƒ синусоиды.

ΠžΡ‚ΡΡŽΠ΄Π° Π²ΠΈΠ΄Π½Π° ΠΈ Π·Π½Π°Ρ‡ΠΈΡ‚Π΅Π»ΡŒΠ½ΠΎ большая ΠΏΡ€ΠΈΠ²Π΅Ρ€ΠΆΠ΅Π½Π½ΠΎΡΡ‚ΡŒ ΠΊ «качаниям» ΡƒΠ·Π»ΠΎΠ² ΠΏΠΎ ΠΎΡ‚Π½ΠΎΡˆΠ΅Π½ΠΈΡŽ ΠΊ Π΄Ρ€ΡƒΠ³ Π΄Ρ€ΡƒΠ³Ρƒ, ΠΊΠΎΠ³Π΄Π° напряТСниС-частота Π½Π°Ρ‡ΠΈΠ½Π°ΡŽΡ‚ ΠΌΠ΅Π½ΡΡ‚ΡŒΡΡ Π²Π²Π΅Ρ€Ρ…-Π²Π½ΠΈΠ·, Π½Π° Ρ‡Ρ‚ΠΎ ΠΎΠ±Ρ‹Ρ‡Π½Ρ‹ΠΉ ΠΏΠΎΡ‚Ρ€Π΅Π±ΠΈΡ‚Π΅Π»ΡŒ ΠΎΠ±Ρ€Π°Ρ‰Π°Π΅Ρ‚ Π²Π½ΠΈΠΌΠ°Π½ΠΈΠ΅, ΠΊΠΎΠ³Π΄Π° Ρƒ Π½Π΅Π³ΠΎ Π² ΠΊΠ²Π°Ρ€Ρ‚ΠΈΡ€Π΅ ΠΌΠΈΠ³Π°Π΅Ρ‚ свСт. ΠžΠ±Ρ‹Ρ‡Π½ΠΎ это прСдвСстник ΠΊΠΎΠ½Ρ†Π° совмСстной Ρ€Π°Π±ΠΎΡ‚Ρ‹ ΡƒΠ·Π»ΠΎΠ²: связи ΠΌΠ΅ΠΆΠ΄Ρƒ Π½ΠΈΠΌΠΈ рвутся ΠΈ ΠΊΠ°ΠΊΠΈΠ΅-Ρ‚ΠΎ ΡƒΠ·Π»Ρ‹ ΠΎΠΊΠ°Π·Ρ‹Π²Π°ΡŽΡ‚ΡΡ с Π΄Π΅Ρ„ΠΈΡ†ΠΈΡ‚ΠΎΠΌ энСргии, Ρ‡Ρ‚ΠΎ Π²Π΅Π΄Π΅Ρ‚ ΠΊ сниТСнию Π² Π½ΠΈΡ… частоты (Ρ‚.Π΅. ΠΊ сниТСнию скорости вращСния Ρ‚Π΅Ρ… ΠΆΠ΅ элСктродвигатСлСй ΠΈ вСнтиляторов), Π° ΠΊΠ°ΠΊΠΈΠ΅-Ρ‚ΠΎ с ΠΈΠ·Π±Ρ‹Ρ‚ΠΊΠΎΠΌ энСргии, приводящСм ΠΊ опасному ΠΏΠΎΠ²Ρ‹ΡˆΠ΅Π½ΠΈΡŽ напряТСния ΠΏΠΎ всСму ΡƒΠ·Π»Ρƒ, Π²ΠΊΠ»ΡŽΡ‡Π°Ρ наши Ρ€ΠΎΠ·Π΅Ρ‚ΠΊΠΈ с ΠΏΠΎΠ΄ΠΊΠ»ΡŽΡ‡Π΅Π½Π½Ρ‹ΠΌΠΈ ΠΊ Π½ΠΈΠΌ устройствам. А ΠΏΡ€ΠΈ достаточно большой Π΄Π»ΠΈΠ½Π΅ Π›Π­ΠŸ, Ρ‡Ρ‚ΠΎ, ΠΊ ΠΏΡ€ΠΈΠΌΠ΅Ρ€Ρƒ, ΠΊΡ€ΠΈΡ‚ΠΈΡ‡Π½ΠΎ для Π Π€, Π½Π°Ρ‡ΠΈΠ½Π°ΡŽΡ‚ ΠΏΡ€ΠΎΡΠ²Π»ΡΡ‚ΡŒΡΡ ΠΈ Π΄Ρ€ΡƒΠ³ΠΈΠ΅ портящиС настроСниС элСктрикам эффСкты. НС вдаваясь Π² Π΄Π΅Ρ‚Π°Π»ΠΈ, ΠΌΠΎΠΆΠ½ΠΎ ΡƒΠΊΠ°Π·Π°Ρ‚ΡŒ, Ρ‡Ρ‚ΠΎ ΠΏΠ΅Ρ€Π΅Π΄Π°Π²Π°Ρ‚ΡŒ ΡΠ»Π΅ΠΊΡ‚Ρ€ΠΎΡΠ½Π΅Ρ€Π³ΠΈΡŽ ΠΏΠ΅Ρ€Π΅ΠΌΠ΅Π½Π½ΠΎΠ³ΠΎ Ρ‚ΠΎΠΊΠ° ΠΏΠΎ ΠΏΡ€ΠΎΠ²ΠΎΠ΄Π°ΠΌ Π½Π° ΡΠ²Π΅Ρ€Ρ…Π΄Π°Π»ΡŒΠ½ΠΈΠ΅ расстояния становится Ρ‚Ρ€ΡƒΠ΄Π½ΠΎ, Π° ΠΈΠ½ΠΎΠ³Π΄Π° ΠΈ Π½Π΅Π²ΠΎΠ·ΠΌΠΎΠΆΠ½ΠΎ. Для свСдСния, Π΄Π»ΠΈΠ½Π° Π²ΠΎΠ»Π½Ρ‹ частотой 50 Π“Ρ† составляСт 6000 ΠΊΠΌ, ΠΈ ΠΏΡ€ΠΈ ΠΏΡ€ΠΈΠ±Π»ΠΈΠΆΠ΅Π½ΠΈΠΈ ΠΊ ΠΏΠΎΠ»ΠΎΠ²ΠΈΠ½Π΅ этой Π΄Π»ΠΈΠ½Ρ‹ – 3000 ΠΊΠΌ – Π½Π°Ρ‡ΠΈΠ½Π°ΡŽΡ‚ ΡΠΊΠ°Π·Ρ‹Π²Π°Ρ‚ΡŒΡΡ эффСкты Π±Π΅Π³ΡƒΡ‰ΠΈΡ… ΠΈ стоячих Π²ΠΎΠ»Π½ плюс эффСкты, связанныС с рСзонансом.

Π­Ρ‚ΠΈ эффСкты ΠΎΡ‚ΡΡƒΡ‚ΡΡ‚Π²ΡƒΡŽΡ‚ ΠΏΡ€ΠΈ использовании постоянного Ρ‚ΠΎΠΊΠ°. А Π·Π½Π°Ρ‡ΠΈΡ‚, ΠΏΠΎΠ²Ρ‹ΡˆΠ°Π΅Ρ‚ΡΡ ΡΡ‚Π°Π±ΠΈΠ»ΡŒΠ½ΠΎΡΡ‚ΡŒ Ρ€Π°Π±ΠΎΡ‚Ρ‹ энСргосистСмы Π² Ρ†Π΅Π»ΠΎΠΌ. ΠŸΡ€ΠΈΠ½ΠΈΠΌΠ°Ρ это Π²ΠΎ Π²Π½ΠΈΠΌΠ°Π½ΠΈΠ΅, Π° Ρ‚Π°ΠΊΠΆΠ΅ Ρ‚ΠΎ, Ρ‡Ρ‚ΠΎ ΠΊΠΎΠΌΠΏΡŒΡŽΡ‚Π΅Ρ€Ρ‹, свСтодиоды, солнСчныС ΠΏΠ°Π½Π΅Π»ΠΈ, аккумуляторы ΠΈ ΠΌΠ½ΠΎΠ³ΠΎΠ΅ Π΄Ρ€ΡƒΠ³ΠΎΠ΅ ΠΈΡΠΏΠΎΠ»ΡŒΠ·ΡƒΡŽΡ‚ для своСй Ρ€Π°Π±ΠΎΡ‚Ρ‹ ΠΈΠΌΠ΅Π½Π½ΠΎ постоянный Ρ‚ΠΎΠΊ, ΠΌΠΎΠΆΠ½ΠΎ Π·Π°ΠΊΠ»ΡŽΡ‡ΠΈΡ‚ΡŒ: Π²ΠΎΠΉΠ½Π° с постоянным Ρ‚ΠΎΠΊΠΎΠΌ Π΅Ρ‰Π΅ Π½Π΅ ΠΏΡ€ΠΎΠΈΠ³Ρ€Π°Π½Π°. Π‘ΠΎΠ²Ρ€Π΅ΠΌΠ΅Π½Π½Ρ‹ΠΌ прСобразоватСлям постоянного Ρ‚ΠΎΠΊΠ° Π½Π° Π»ΡŽΠ±Ρ‹Π΅ ΠΈΡΠΏΠΎΠ»ΡŒΠ·ΡƒΠ΅ΠΌΡ‹Π΅ сСгодня мощности ΠΈ напряТСния ΠΎΡΡ‚Π°Π»ΠΎΡΡŒ совсСм Π½Π΅ΠΌΠ½ΠΎΠ³ΠΎ, Ρ‡Ρ‚ΠΎΠ±Ρ‹ ΡΡ€Π°Π²Π½ΡΡ‚ΡŒΡΡ Π² Ρ†Π΅Π½Π΅ с ΠΏΡ€ΠΈΠ²Ρ‹Ρ‡Π½Ρ‹ΠΌΠΈ чСловСчСству трансформаторами ΠΏΠ΅Ρ€Π΅ΠΌΠ΅Π½Π½ΠΎΠ³ΠΎ Ρ‚ΠΎΠΊΠ°. ПослС Ρ‡Π΅Π³ΠΎ, Π²ΠΈΠ΄ΠΈΠΌΠΎ, начнСтся Ρ‚Ρ€ΠΈΡƒΠΌΡ„Π°Π»ΡŒΠ½ΠΎΠ΅ ΡˆΠ΅ΡΡ‚Π²ΠΈΠ΅ ΠΏΠΎ ΠΏΠ»Π°Π½Π΅Ρ‚Π΅ ΡƒΠΆΠ΅ постоянного Ρ‚ΠΎΠΊΠ°.

Π€ΠΎΡ‚ΠΎ:Β itc.ua

Π§Ρ‚ΠΎ Ρ‚Π°ΠΊΠΎΠ΅ элСктричСский Ρ‚ΠΎΠΊ: ΠΎΠΏΡ€Π΅Π΄Π΅Π»Π΅Π½ΠΈΠ΅, характСристики, Π²ΠΈΠ΄Ρ‹

ΠžΡ‚ΠΊΡ€Ρ‹Ρ‚ΠΈΡ, связанныС с элСктричСством, ΠΊΠ°Ρ€Π΄ΠΈΠ½Π°Π»ΡŒΠ½ΠΎ ΠΈΠ·ΠΌΠ΅Π½ΠΈΠ»ΠΈ Π½Π°ΡˆΡƒ Тизнь. Π˜ΡΠΏΠΎΠ»ΡŒΠ·ΡƒΡ элСктричСский Ρ‚ΠΎΠΊ ΠΊΠ°ΠΊ источник энСргии, чСловСчСство сдСлало ΠΏΡ€ΠΎΡ€Ρ‹Π² Π² тСхнологиях, ΠΊΠΎΡ‚ΠΎΡ€Ρ‹Π΅ ΠΎΠ±Π»Π΅Π³Ρ‡ΠΈΠ»ΠΈ нашС сущСствованиС. БСгодня элСктричСство ΠΏΡ€ΠΈΠ²ΠΎΠ΄ΠΈΡ‚ Π² Π΄Π²ΠΈΠΆΠ΅Π½ΠΈΠ΅ Ρ‚ΠΎΠΊΠ°Ρ€Π½Ρ‹Π΅ станки, Π°Π²Ρ‚ΠΎΠΌΠΎΠ±ΠΈΠ»ΠΈ, управляСт Ρ€ΠΎΠ±ΠΎΡ‚ΠΈΠ·ΠΈΡ€ΠΎΠ²Π°Π½Π½ΠΎΠΉ Ρ‚Π΅Ρ…Π½ΠΈΠΊΠΎΠΉ, обСспСчиваСт связь. Π­Ρ‚ΠΎΡ‚ список ΠΌΠΎΠΆΠ½ΠΎ ΠΏΡ€ΠΎΠ΄ΠΎΠ»ΠΆΠ°Ρ‚ΡŒ ΠΎΡ‡Π΅Π½ΡŒ Π΄ΠΎΠ»Π³ΠΎ. Π”Π°ΠΆΠ΅ Ρ‚Ρ€ΡƒΠ΄Π½ΠΎ Π½Π°Π·Π²Π°Ρ‚ΡŒ ΠΎΡ‚Ρ€Π°ΡΠ»ΡŒ, Π³Π΄Π΅ ΠΌΠΎΠΆΠ½ΠΎ ΠΎΠ±ΠΎΠΉΡ‚ΠΈΡΡŒ Π±Π΅Π· элСктроэнСргии.

Π’ Ρ‡Ρ‘ΠΌ сСкрСт Ρ‚Π°ΠΊΠΎΠ³ΠΎ массового использования элСктричСства? Π’Π΅Π΄ΡŒ Π² ΠΏΡ€ΠΈΡ€ΠΎΠ΄Π΅ ΡΡƒΡ‰Π΅ΡΡ‚Π²ΡƒΡŽΡ‚ ΠΈ Π΄Ρ€ΡƒΠ³ΠΈΠ΅ источники энСргии, Π±ΠΎΠ»Π΅Π΅ Π΄Π΅ΡˆΠ΅Π²Ρ‹Π΅, Ρ‡Π΅ΠΌ элСктричСство. ΠžΠΊΠ°Π·Ρ‹Π²Π°Π΅Ρ‚ΡΡ всё Π΄Π΅Π»ΠΎ Π² транспортировкС.

Π­Π»Π΅ΠΊΡ‚Ρ€ΠΈΡ‡Π΅ΡΠΊΡƒΡŽ ΡΠ½Π΅Ρ€Π³ΠΈΡŽ ΠΌΠΎΠΆΠ½ΠΎ Π΄ΠΎΡΡ‚Π°Π²ΠΈΡ‚ΡŒ практичСски Π²Π΅Π·Π΄Π΅:

  • ΠΊ производствСнному Ρ†Π΅Ρ…Ρƒ;
  • ΠΊΠ²Π°Ρ€Ρ‚ΠΈΡ€Π΅;
  • Π½Π° ΠΏΠΎΠ»Π΅;
  • Π² ΡˆΠ°Ρ…Ρ‚Ρƒ, ΠΏΠΎΠ΄ Π²ΠΎΠ΄Ρƒ ΠΈ Ρ‚. Π΄.

Π­Π»Π΅ΠΊΡ‚Ρ€ΠΎΡΠ½Π΅Ρ€Π³ΠΈΡŽ, Π½Π°ΠΊΠΎΠΏΠ»Π΅Π½Π½ΡƒΡŽ аккумулятором, ΠΌΠΎΠΆΠ½ΠΎ Π½ΠΎΡΠΈΡ‚ΡŒ с собой. ΠœΡ‹ ΠΏΠΎΠ»ΡŒΠ·ΡƒΠ΅ΠΌΡΡ этим Π΅ΠΆΠ΅Π΄Π½Π΅Π²Π½ΠΎ, бСря с собой сотовый Ρ‚Π΅Π»Π΅Ρ„ΠΎΠ½. Ни ΠΎΠ΄ΠΈΠ½ Π΄Ρ€ΡƒΠ³ΠΎΠΉ Π²ΠΈΠ΄ энСргии Π½Π΅ ΠΎΠ±Π»Π°Π΄Π°Π΅Ρ‚ Ρ‚Π°ΠΊΠΈΠΌΠΈ ΡƒΠ½ΠΈΠ²Π΅Ρ€ΡΠ°Π»ΡŒΠ½Ρ‹ΠΌΠΈ свойствами ΠΊΠ°ΠΊ элСктричСство. Π Π°Π·Π²Π΅ это Π½Π΅ являСтся достаточной ΠΏΡ€ΠΈΡ‡ΠΈΠ½ΠΎΠΉ для Ρ‚ΠΎΠ³ΠΎ, Ρ‡Ρ‚ΠΎΠ±Ρ‹ Π³Π»ΡƒΠ±ΠΆΠ΅ ΠΈΠ·ΡƒΡ‡ΠΈΡ‚ΡŒ ΠΏΡ€ΠΈΡ€ΠΎΠ΄Ρƒ ΠΈ свойства элСктричСства?

Π§Ρ‚ΠΎ Ρ‚Π°ΠΊΠΎΠ΅ элСктричСский Ρ‚ΠΎΠΊ?

ЭлСктричСскиС явлСния наблюдались Π΄Π°Π²Π½ΠΎ, Π½ΠΎ ΠΎΠ±ΡŠΡΡΠ½ΠΈΡ‚ΡŒ ΠΈΡ… ΠΏΡ€ΠΈΡ€ΠΎΠ΄Ρƒ Ρ‡Π΅Π»ΠΎΠ²Π΅ΠΊ смог ΠΎΡ‚Π½ΠΎΡΠΈΡ‚Π΅Π»ΡŒΠ½ΠΎ Π½Π΅Π΄Π°Π²Π½ΠΎ. Π£Π΄Π°Ρ€ ΠΌΠΎΠ»Π½ΠΈΠΈ казался Ρ‡Π΅ΠΌ-Ρ‚ΠΎ нССстСствСнным, Π½Π΅ΠΎΠ±ΡŠΡΡΠ½ΠΈΠΌΡ‹ΠΌ. Π‘Ρ‚Ρ€Π°Π½Π½Ρ‹ΠΌ казалось потрСскиваниС Π½Π΅ΠΊΠΎΡ‚ΠΎΡ€Ρ‹Ρ… ΠΏΡ€Π΅Π΄ΠΌΠ΅Ρ‚ΠΎΠ² ΠΏΡ€ΠΈ ΠΈΡ… Ρ‚Ρ€Π΅Π½ΠΈΠΈ. Π˜ΡΠΊΡ€ΡΡ‰Π°ΡΡΡ Π² Ρ‚Π΅ΠΌΠ½ΠΎΡ‚Π΅ расчёска, послС расчёсывания ΡˆΠ΅Ρ€ΡΡ‚ΠΈ ΠΆΠΈΠ²ΠΎΡ‚Π½Ρ‹Ρ… (Π½Π°ΠΏΡ€ΠΈΠΌΠ΅Ρ€, кошки) Π²Ρ‹Π·Π²Π°Π»Π° Π½Π΅Π΄ΠΎΡƒΠΌΠ΅Π½ΠΈΠ΅, Π½ΠΎ ΠΏΠΎΠ΄ΠΎΠ³Ρ€Π΅Π²Π°Π»Π° интСрСс ΠΊ этому явлСнию.

Как всё Π½Π°Ρ‡ΠΈΠ½Π°Π»ΠΎΡΡŒ

Π•Ρ‰Ρ‘ Π΄Ρ€Π΅Π²Π½ΠΈΠΌ Π³Ρ€Π΅ΠΊΠ°ΠΌ Π±Ρ‹Π»ΠΎ извСстно свойство янтаря, ΠΏΠΎΡ‚Ρ‘Ρ€Ρ‚ΠΎΠ³ΠΎ ΠΎ ΡˆΠ΅Ρ€ΡΡ‚ΡŒ, ΠΏΡ€ΠΈΡ‚ΡΠ³ΠΈΠ²Π°Ρ‚ΡŒ Π½Π΅ΠΊΠΎΡ‚ΠΎΡ€Ρ‹Π΅ ΠΌΠ΅Π»ΠΊΠΈΠ΅ ΠΏΡ€Π΅Π΄ΠΌΠ΅Ρ‚Ρ‹. ΠšΡΡ‚Π°Ρ‚ΠΈ, ΠΎΡ‚ грСчСского названия янтаря –«элСктрон» пошло Π½Π°Π·Π²Π°Π½ΠΈΠ΅ «элСктричСство».

Когда Ρ„ΠΈΠ·ΠΈΠΊΠΈ Π²ΠΏΠ»ΠΎΡ‚Π½ΡƒΡŽ занялись исслСдованиСм элСктризации Ρ‚Π΅Π», ΠΎΠ½ΠΈ Π½Π°Ρ‡Π°Π»ΠΈ ΠΏΠΎΠ½ΠΈΠΌΠ°Ρ‚ΡŒ ΠΏΡ€ΠΈΡ€ΠΎΠ΄Ρƒ ΠΏΠΎΠ΄ΠΎΠ±Π½Ρ‹Ρ… явлСний. А ΠΏΠ΅Ρ€Π²Ρ‹ΠΉ ΠΊΡ€Π°Ρ‚ΠΊΠΎΠ²Ρ€Π΅ΠΌΠ΅Π½Π½Ρ‹ΠΉ элСктричСский Ρ‚ΠΎΠΊ, созданный Ρ‡Π΅Π»ΠΎΠ²Π΅ΠΊΠΎΠΌ, появился ΠΏΡ€ΠΈ соСдинСнии ΠΏΡ€ΠΎΠ²ΠΎΠ΄Π½ΠΈΠΊΠΎΠΌ Π΄Π²ΡƒΡ… наэлСктризованных ΠΏΡ€Π΅Π΄ΠΌΠ΅Ρ‚ΠΎΠ² (см. рис. 1). Π’ 1729 Π³ΠΎΠ΄Ρƒ Π°Π½Π³Π»ΠΈΡ‡Π°Π½Π΅ Π“Ρ€Π΅ΠΉ ΠΈ Π£ΠΈΠ»Π»Π΅Ρ€ ΠΎΡ‚ΠΊΡ€Ρ‹Π»ΠΈ ΠΏΡ€ΠΎΠ²ΠΎΠ΄ΠΈΠΌΠΎΡΡ‚ΡŒ зарядов Π½Π΅ΠΊΠΎΡ‚ΠΎΡ€Ρ‹ΠΌΠΈ ΠΌΠ°Ρ‚Π΅Ρ€ΠΈΠ°Π»Π°ΠΌΠΈ. Но опрСдСлСния элСктричСского Ρ‚ΠΎΠΊΠ° ΠΎΠ½ΠΈ Π½Π΅ смогли Π΄Π°Ρ‚ΡŒ, хотя ΠΈ ΠΏΠΎΠ½ΠΈΠΌΠ°Π»ΠΈ, Ρ‡Ρ‚ΠΎ заряды ΠΏΠ΅Ρ€Π΅ΠΌΠ΅Ρ‰Π°ΡŽΡ‚ΡΡ ΠΎΡ‚ ΠΎΠ΄Π½ΠΎΠ³ΠΎ Ρ‚Π΅Π»Π° ΠΊ Π΄Ρ€ΡƒΠ³ΠΎΠΌΡƒ ΠΏΠΎ ΠΏΡ€ΠΎΠ²ΠΎΠ΄Π½ΠΈΠΊΡƒ.

Рис. 1. ΠžΠΏΡ‹Ρ‚ с заряТСнными Ρ‚Π΅Π»Π°ΠΌΠΈ

Об элСктричСском Ρ‚ΠΎΠΊΠ΅, ΠΊΠ°ΠΊ ΠΎ физичСском явлСнии Π·Π°Π³ΠΎΠ²ΠΎΡ€ΠΈΠ»ΠΈ лишь послС Ρ‚ΠΎΠ³ΠΎ, ΠΊΠ°ΠΊ ΠΈΡ‚Π°Π»ΡŒΡΠ½Π΅Ρ† Π’ΠΎΠ»ΡŒΡ‚Π° Π΄Π°Π» объяснСниС ΠΎΠΏΡ‹Ρ‚Π°ΠΌ Π“Π°Π»ΡŒΠ²Π°Π½ΠΈ, Π° Π² 1794 Π³ΠΎΠ΄Ρƒ ΠΈΠ·ΠΎΠ±Ρ€Ρ‘Π» ΠΏΠ΅Ρ€Π²Ρ‹ΠΉ Π² ΠΌΠΈΡ€Π΅ источник элСктричСства – Π³Π°Π»ΡŒΠ²Π°Π½ΠΈΡ‡Π΅ΡΠΊΠΈΠΉ элСмСнт (столб Π’ΠΎΠ»ΡŒΡ‚Π°). Он обосновал упорядочСнноС ΠΏΠ΅Ρ€Π΅ΠΌΠ΅Ρ‰Π΅Π½ΠΈΠ΅ заряТСнных частиц ΠΏΠΎ Π·Π°ΠΌΠΊΠ½ΡƒΡ‚ΠΎΠΉ Ρ†Π΅ΠΏΠΈ.

ΠžΠΏΡ€Π΅Π΄Π΅Π»Π΅Π½ΠΈΠ΅

Π’ соврСмСнной Ρ‚Ρ€Π°ΠΊΡ‚ΠΎΠ²ΠΊΠ΅ элСктричСским Ρ‚ΠΎΠΊΠΎΠΌ Π½Π°Π·Ρ‹Π²Π°ΡŽΡ‚ Π½Π°ΠΏΡ€Π°Π²Π»Π΅Π½Π½ΠΎΠ΅ ΠΏΠ΅Ρ€Π΅ΠΌΠ΅Ρ‰Π΅Π½ΠΈΠ΅ силами элСктричСского поля заряТСнных частиц, НоситСлями зарядов мСталличСских ΠΏΡ€ΠΎΠ²ΠΎΠ΄Π½ΠΈΠΊΠΎΠ² ΡΠ²Π»ΡΡŽΡ‚ΡΡ элСктроны, Π° растворов кислот ΠΈ солСй β€” ΠΎΡ‚Ρ€ΠΈΡ†Π°Ρ‚Π΅Π»ΡŒΠ½Ρ‹Π΅ ΠΈ ΠΏΠΎΠ»ΠΎΠΆΠΈΡ‚Π΅Π»ΡŒΠ½Ρ‹Π΅ ΠΈΠΎΠ½Ρ‹. ΠŸΠΎΠ»ΡƒΠΏΡ€ΠΎΠ²ΠΎΠ΄Π½ΠΈΠΊΠΎΠ²Ρ‹ΠΌΠΈ носитСлями зарядов ΡΠ²Π»ΡΡŽΡ‚ΡΡ элСктроны ΠΈ Β«Π΄Ρ‹Ρ€ΠΊΠΈΒ».

Для Ρ‚ΠΎΠ³ΠΎ Ρ‡Ρ‚ΠΎΠ±Ρ‹ элСктричСский Ρ‚ΠΎΠΊ сущСствовал, Π½Π΅ΠΎΠ±Ρ…ΠΎΠ΄ΠΈΠΌΠΎ всё врСмя ΠΏΠΎΠ΄Π΄Π΅Ρ€ΠΆΠΈΠ²Π°Ρ‚ΡŒ элСктричСскоС ΠΏΠΎΠ»Π΅. Π”ΠΎΠ»ΠΆΠ½Π° ΡΡƒΡ‰Π΅ΡΡ‚Π²ΠΎΠ²Π°Ρ‚ΡŒ Ρ€Π°Π·Π½ΠΈΡ†Π° ΠΏΠΎΡ‚Π΅Π½Ρ†ΠΈΠ°Π»ΠΎΠ², ΠΏΠΎΠ΄Π΄Π΅Ρ€ΠΆΠΈΠ²Π°ΡŽΡ‰Π°Ρ Π½Π°Π»ΠΈΡ‡ΠΈΠ΅ ΠΏΠ΅Ρ€Π²Ρ‹Ρ… Π΄Π²ΡƒΡ… условий. Π”ΠΎ Ρ‚Π΅Ρ… ΠΏΠΎΡ€, ΠΏΠΎΠΊΠ° эти условия ΡΠΎΠ±Π»ΡŽΠ΄Π΅Π½Ρ‹, заряды Π±ΡƒΠ΄ΡƒΡ‚ упорядочСнно ΠΏΠ΅Ρ€Π΅ΠΌΠ΅Ρ‰Π°Ρ‚ΡŒΡΡ ΠΏΠΎ участкам Π·Π°ΠΌΠΊΠ½ΡƒΡ‚ΠΎΠΉ элСктричСской Ρ†Π΅ΠΏΠΈ. Π­Ρ‚Ρƒ Π·Π°Π΄Π°Ρ‡Ρƒ Π²Ρ‹ΠΏΠΎΠ»Π½ΡΡŽΡ‚ источники элСктричСства.

Π’Π°ΠΊΠΈΠ΅ условия ΠΌΠΎΠΆΠ½ΠΎ ΡΠΎΠ·Π΄Π°Ρ‚ΡŒ, Π½Π°ΠΏΡ€ΠΈΠΌΠ΅Ρ€, с ΠΏΠΎΠΌΠΎΡ‰ΡŒΡŽ элСктрофорной ΠΌΠ°ΡˆΠΈΠ½Ρ‹ (рис. 2). Если Π΄Π²Π° диска Π²Ρ€Π°Ρ‰Π°Ρ‚ΡŒ Π² ΠΏΡ€ΠΎΡ‚ΠΈΠ²ΠΎΠΏΠΎΠ»ΠΎΠΆΠ½Ρ‹Ρ… направлСниях, Ρ‚ΠΎ ΠΎΠ½ΠΈ Π±ΡƒΠ΄ΡƒΡ‚ Π·Π°Ρ€ΡΠΆΠ°Ρ‚ΡŒΡΡ Ρ€Π°Π·Π½ΠΎΠΈΠΌΡ‘Π½Π½Ρ‹ΠΌΠΈ зарядами. На Ρ‰Ρ‘Ρ‚ΠΊΠ°Ρ…, ΠΏΡ€ΠΈΠ»Π΅Π³Π°ΡŽΡ‰ΠΈΡ… ΠΊ дискам, появится Ρ€Π°Π·Π½ΠΈΡ†Π° ΠΏΠΎΡ‚Π΅Π½Ρ†ΠΈΠ°Π»ΠΎΠ². Π‘ΠΎΠ΅Π΄ΠΈΠ½ΠΈΠ² ΠΊΠΎΠ½Ρ‚Π°ΠΊΡ‚Ρ‹ ΠΏΡ€ΠΎΠ²ΠΎΠ΄Π½ΠΈΠΊΠΎΠΌ, ΠΌΡ‹ заставим заряТСнныС частицы Π΄Π²ΠΈΠ³Π°Ρ‚ΡŒΡΡ упорядочСнно. Π’ΠΎ Π΅ΡΡ‚ΡŒ элСктрофорная машина являСтся источником элСктричСства.

Рисунок 2. ЭлСктрофорная машина

Π˜ΡΡ‚ΠΎΡ‡Π½ΠΈΠΊΠΈ Ρ‚ΠΎΠΊΠ°

ΠŸΠ΅Ρ€Π²Ρ‹ΠΌΠΈ источниками элСктричСской энСргии, нашСдшими практичСскоС ΠΏΡ€ΠΈΠΌΠ΅Π½Π΅Π½ΠΈΠ΅, Π±Ρ‹Π»ΠΈ упомянутыС Π²Ρ‹ΡˆΠ΅ Π³Π°Π»ΡŒΠ²Π°Π½ΠΈΡ‡Π΅ΡΠΊΠΈΠ΅ элСмСнты. Π£ΡΠΎΠ²Π΅Ρ€ΡˆΠ΅Π½ΡΡ‚Π²ΠΎΠ²Π°Π½Π½Ρ‹Π΅ Π³Π°Π»ΡŒΠ²Π°Π½ΠΈΡ‡Π΅ΡΠΊΠΈΠ΅ элСмСнты (Π½Π°Ρ€ΠΎΠ΄Π½ΠΎΠ΅ Π½Π°Π·Π²Π°Π½ΠΈΠ΅ – Π±Π°Ρ‚Π°Ρ€Π΅ΠΉΠΊΠΈ) ΡˆΠΈΡ€ΠΎΠΊΠΎ ΠΏΡ€ΠΈΠΌΠ΅Π½ΡΡŽΡ‚ΡΡ ΠΏΠΎ сСй дСнь. Они ΠΈΡΠΏΠΎΠ»ΡŒΠ·ΡƒΡŽΡ‚ΡΡ для питания ΠΏΡƒΠ»ΡŒΡ‚ΠΎΠ² управлСния, элСктронных часов, дСтских ΠΈΠ³Ρ€ΡƒΡˆΠ΅ΠΊ ΠΈ ΠΌΠ½ΠΎΠ³ΠΈΡ… Π΄Ρ€ΡƒΠ³ΠΈΡ… Π³Π°Π΄ΠΆΠ΅Ρ‚ΠΎΠ².

Π‘ ΠΈΠ·ΠΎΠ±Ρ€Π΅Ρ‚Π΅Π½ΠΈΠ΅ΠΌ Π³Π΅Π½Π΅Ρ€Π°Ρ‚ΠΎΡ€ΠΎΠ² ΠΏΠ΅Ρ€Π΅ΠΌΠ΅Π½Π½Ρ‹Ρ… Ρ‚ΠΎΠΊΠΎΠ² элСктричСство ΠΏΡ€ΠΈΠΎΠ±Ρ€Π΅Π»ΠΎ Π²Ρ‚ΠΎΡ€ΠΎΠ΅ Π΄Ρ‹Ρ…Π°Π½ΠΈΠ΅. ΠΠ°Ρ‡Π°Π»Π°ΡΡŒ эра элСктрификации Π³ΠΎΡ€ΠΎΠ΄ΠΎΠ², Π° ΠΏΠΎΠ·ΠΆΠ΅ ΠΈ всСх насСлённых ΠΏΡƒΠ½ΠΊΡ‚ΠΎΠ². ЭлСктричСская энСргия стала доступной для всСх Π³Ρ€Π°ΠΆΠ΄Π°Π½ Ρ€Π°Π·Π²ΠΈΡ‚Ρ‹Ρ… стран.

БСгодня чСловСчСство ΠΈΡ‰Π΅Ρ‚ возобновляСмыС источники элСктроэнСргии. Π‘ΠΎΠ»Π½Π΅Ρ‡Π½Ρ‹Π΅ ΠΏΠ°Π½Π΅Π»ΠΈ, вСтряныС элСктростанции ΡƒΠΆΠ΅ Π·Π°Π½ΠΈΠΌΠ°ΡŽΡ‚ свои ниши Π² энСргосистСмах ΠΌΠ½ΠΎΠ³ΠΈΡ… стран, Π²ΠΊΠ»ΡŽΡ‡Π°Ρ Россию.

Π₯арактСристики

ЭлСктричСский Ρ‚ΠΎΠΊ характСризуСтся Π²Π΅Π»ΠΈΡ‡ΠΈΠ½Π°ΠΌΠΈ, ΠΊΠΎΡ‚ΠΎΡ€Ρ‹Π΅ ΠΎΠΏΠΈΡΡ‹Π²Π°ΡŽΡ‚ Π΅Π³ΠΎ свойства.

Π‘ΠΈΠ»Π° ΠΈ ΠΏΠ»ΠΎΡ‚Π½ΠΎΡΡ‚ΡŒ Ρ‚ΠΎΠΊΠ°

Для описания характСристики элСктричСства часто ΠΈΡΠΏΠΎΠ»ΡŒΠ·ΡƒΡŽΡ‚ Ρ‚Π΅Ρ€ΠΌΠΈΠ½ «сила Ρ‚ΠΎΠΊΠ°Β». НазваниС Π½Π΅ совсСм ΡƒΠ΄Π°Ρ‡Π½ΠΎΠ΅, Ρ‚Π°ΠΊ ΠΊΠ°ΠΊ ΠΎΠ½ΠΎ Ρ…Π°Ρ€Π°ΠΊΡ‚Π΅Ρ€ΠΈΠ·ΡƒΠ΅Ρ‚ Ρ‚ΠΎΠ»ΡŒΠΊΠΎ ΠΈΠ½Ρ‚Π΅Π½ΡΠΈΠ²Π½ΠΎΡΡ‚ΡŒ двиТСния элСктричСских зарядов, Π° Π½Π΅ ΠΊΠ°ΠΊΡƒΡŽ-Ρ‚ΠΎ силу Π² Π±ΡƒΠΊΠ²Π°Π»ΡŒΠ½ΠΎΠΌ смыслС. Π’Π΅ΠΌ Π½Π΅ ΠΌΠ΅Π½Π΅Π΅, этим Ρ‚Π΅Ρ€ΠΌΠΈΠ½ΠΎΠΌ ΠΏΠΎΠ»ΡŒΠ·ΡƒΡŽΡ‚ΡΡ, ΠΈ ΠΎΠ½ ΠΎΠ·Π½Π°Ρ‡Π°Π΅Ρ‚ количСство элСктричСства (зарядов) проходящСго Ρ‡Π΅Ρ€Π΅Π· ΠΏΠ»ΠΎΡΠΊΠΎΡΡ‚ΡŒ ΠΏΠΎΠΏΠ΅Ρ€Π΅Ρ‡Π½ΠΎΠ³ΠΎ сСчСния ΠΏΡ€ΠΎΠ²ΠΎΠ΄Π½ΠΈΠΊΠ°. Π•Π΄ΠΈΠ½ΠΈΡ†Π΅ΠΉ измСрСния силы Ρ‚ΠΎΠΊΠ° Π² систСмС БИ являСтся Π°ΠΌΠΏΠ΅Ρ€ (А).

1 А ΠΎΠ·Π½Π°Ρ‡Π°Π΅Ρ‚ Ρ‚ΠΎ, Ρ‡Ρ‚ΠΎ Π·Π° ΠΎΠ΄Π½Ρƒ сСкунду Ρ‡Π΅Ρ€Π΅Π· ΠΏΠΎΠΏΠ΅Ρ€Π΅Ρ‡Π½ΠΎΠ΅ сСчСниС ΠΏΡ€ΠΎΠ²ΠΎΠ΄Π½ΠΈΠΊΠ° ΠΏΡ€ΠΎΡ…ΠΎΠ΄ΠΈΡ‚ элСктричСский заряд 1 Кл. (1А = 1 Кл/с).

ΠŸΠ»ΠΎΡ‚Π½ΠΎΡΡ‚ΡŒ Ρ‚ΠΎΠΊΠ°  –  вСкторная Π²Π΅Π»ΠΈΡ‡ΠΈΠ½Π°. Π’Π΅ΠΊΡ‚ΠΎΡ€ Π½Π°ΠΏΡ€Π°Π²Π»Π΅Π½ Π² сторону двиТСния ΠΏΠΎΠ»ΠΎΠΆΠΈΡ‚Π΅Π»ΡŒΠ½Ρ‹Ρ… зарядов. ΠœΠΎΠ΄ΡƒΠ»ΡŒ этого Π²Π΅ΠΊΡ‚ΠΎΡ€Π° Ρ€Π°Π²Π΅Π½ ΠΎΡ‚Π½ΠΎΡˆΠ΅Π½ΠΈΡŽ силы Ρ‚ΠΎΠΊΠ° Π½Π° Π½Π΅ΠΊΠΎΡ‚ΠΎΡ€ΠΎΠΌ пСрпСндикулярном ΠΊ Π½Π°ΠΏΡ€Π°Π²Π»Π΅Π½ΠΈΡŽ двиТСния зарядов сСчСнии ΠΏΡ€ΠΎΠ²ΠΎΠ΄Π½ΠΈΠΊΠ° ΠΊ ΠΏΠ»ΠΎΡ‰Π°Π΄ΠΈ этого сСчСния. Π’ систСмС БИ измСряСтся Π² А/ΠΌ2. ΠŸΠ»ΠΎΡ‚Π½ΠΎΡΡ‚ΡŒ Π±ΠΎΠ»Π΅Π΅ Ρ‘ΠΌΠΊΠΎ Ρ…Π°Ρ€Π°ΠΊΡ‚Π΅Ρ€ΠΈΠ·ΡƒΠ΅Ρ‚ элСктричСство, ΠΎΠ΄Π½Π°ΠΊΠΎ Π½Π° ΠΏΡ€Π°ΠΊΡ‚ΠΈΠΊΠ΅ Ρ‡Π°Ρ‰Π΅ ΠΈΡΠΏΠΎΠ»ΡŒΠ·ΡƒΠ΅Ρ‚ΡΡ Π²Π΅Π»ΠΈΡ‡ΠΈΠ½Π° «сила Ρ‚ΠΎΠΊΠ°Β».

Π Π°Π·Π½ΠΈΡ†Π° ΠΏΠΎΡ‚Π΅Π½Ρ†ΠΈΠ°Π»ΠΎΠ² (напряТСниС) Π½Π° участкС Ρ†Π΅ΠΏΠΈ выраТаСтся ΡΠΎΠΎΡ‚Π½ΠΎΡˆΠ΅Π½ΠΈΠ΅ΠΌ: U = IΓ—R, Π³Π΄Π΅ U – напряТСниС, I – сила Ρ‚ΠΎΠΊΠ°, Π° R – сопротивлСниС. Π­Ρ‚ΠΎ Π·Π½Π°ΠΌΠ΅Π½ΠΈΡ‚Ρ‹ΠΉ Π·Π°ΠΊΠΎΠ½ Ома.

ΠœΠΎΡ‰Π½ΠΎΡΡ‚ΡŒ

ЭлСктричСскими силами ΡΠΎΠ²Π΅Ρ€ΡˆΠ°Π΅Ρ‚ΡΡ Ρ€Π°Π±ΠΎΡ‚Π° ΠΏΡ€ΠΎΡ‚ΠΈΠ² Π°ΠΊΡ‚ΠΈΠ²Π½ΠΎΠ³ΠΎ ΠΈ Ρ€Π΅Π°ΠΊΡ‚ΠΈΠ²Π½ΠΎΠ³ΠΎ сопротивлСния. На пассивных сопротивлСниях Ρ€Π°Π±ΠΎΡ‚Π° прСобразуСтся Π² Ρ‚Π΅ΠΏΠ»ΠΎΠ²ΡƒΡŽ ΡΠ½Π΅Ρ€Π³ΠΈΡŽ. ΠœΠΎΡ‰Π½ΠΎΡΡ‚ΡŒΡŽ Π½Π°Π·Ρ‹Π²Π°ΡŽΡ‚ Ρ€Π°Π±ΠΎΡ‚Ρƒ, Π²Ρ‹ΠΏΠΎΠ»Π½Π΅Π½Π½ΡƒΡŽ Π·Π° Π΅Π΄ΠΈΠ½ΠΈΡ†Ρƒ Π²Ρ€Π΅ΠΌΠ΅Π½ΠΈ. По ΠΎΡ‚Π½ΠΎΡˆΠ΅Π½ΠΈΡŽ ΠΊ элСктричСству ΠΏΡ€ΠΈΠΌΠ΅Π½ΡΡŽΡ‚ Ρ‚Π΅Ρ€ΠΌΠΈΠ½ Β«ΠΌΠΎΡ‰Π½ΠΎΡΡ‚ΡŒ Ρ‚Π΅ΠΏΠ»ΠΎΠ²Ρ‹Ρ… ΠΏΠΎΡ‚Π΅Ρ€ΡŒΒ». Π€ΠΈΠ·ΠΈΠΊΠΈ Π”ΠΆΠΎΡƒΠ»ΡŒ ΠΈ Π›Π΅Π½Ρ† Π΄ΠΎΠΊΠ°Π·Π°Π»ΠΈ, Ρ‡Ρ‚ΠΎ ΠΌΠΎΡ‰Π½ΠΎΡΡ‚ΡŒ Ρ‚Π΅ΠΏΠ»ΠΎΠ²Ρ‹Ρ… ΠΏΠΎΡ‚Π΅Ρ€ΡŒ ΠΏΡ€ΠΎΠ²ΠΎΠ΄Π½ΠΈΠΊΠ° Ρ€Π°Π²Π½Π° силС Ρ‚ΠΎΠΊΠ° ΡƒΠΌΠ½ΠΎΠΆΠ΅Π½Π½ΠΎΠΉ Π½Π° напряТСниС: P = IΓ— U. Π•Π΄ΠΈΠ½ΠΈΡ†Π° измСрСния мощности – Π²Π°Ρ‚Ρ‚ (Π’Ρ‚).

Частота

ΠŸΠ΅Ρ€Π΅ΠΌΠ΅Π½Π½Ρ‹ΠΉ Ρ‚ΠΎΠΊ характСризуСтся Ρ‚Π°ΠΊΠΆΠ΅ частотой. Данная характСристика ΠΏΠΎΠΊΠ°Π·Ρ‹Π²Π°Π΅Ρ‚, ΠΊΠ°ΠΊ Π·Π° Π΅Π΄ΠΈΠ½ΠΈΡ†Ρƒ Π²Ρ€Π΅ΠΌΠ΅Π½ΠΈ измСняСтся количСство ΠΏΠ΅Ρ€ΠΈΠΎΠ΄ΠΎΠ² (ΠΊΠΎΠ»Π΅Π±Π°Π½ΠΈΠΉ). Π•Π΄ΠΈΠ½ΠΈΡ†Π΅ΠΉ измСрСния частоты являСтся Π³Π΅Ρ€Ρ†. 1 Π“Ρ† = 1 ΠΏΠ΅Ρ€ΠΈΠΎΠ΄Ρƒ Π·Π° сСкунду. Бтандартная частота ΠΏΡ€ΠΎΠΌΡ‹ΡˆΠ»Π΅Π½Π½ΠΎΠ³ΠΎ Ρ‚ΠΎΠΊΠ° составляСт 50 Π“Ρ†.

Π’ΠΎΠΊ смСщСния

ΠŸΠΎΠ½ΡΡ‚ΠΈΠ΅ Β«Ρ‚ΠΎΠΊ смСщСния» Π²Π²Π΅Π»ΠΈ для удобства, хотя Π² классичСском ΠΏΠΎΠ½ΠΈΠΌΠ°Π½ΠΈΠΈ Π΅Π³ΠΎ нСльзя Π½Π°Π·Π²Π°Ρ‚ΡŒ Ρ‚ΠΎΠΊΠΎΠΌ, Ρ‚Π°ΠΊ ΠΊΠ°ΠΊ отсутствуСт пСрСнос заряда. Π‘ Π΄Ρ€ΡƒΠ³ΠΎΠΉ стороны, ΠΈΠ½Ρ‚Π΅Π½ΡΠΈΠ²Π½ΠΎΡΡ‚ΡŒ ΠΌΠ°Π³Π½ΠΈΡ‚Π½ΠΎΠ³ΠΎ поля ΠΏΡ€Π΅Π±Ρ‹Π²Π°Π΅Ρ‚ Π² зависимости ΠΎΡ‚ Ρ‚ΠΎΠΊΠΎΠ² проводимости ΠΈ смСщСния.

Π’ΠΎΠΊΠΈ смСщСния ΠΌΠΎΠΆΠ½ΠΎ Π½Π°Π±Π»ΡŽΠ΄Π°Ρ‚ΡŒ Π² кондСнсаторах. НСсмотря Π½Π° Ρ‚ΠΎ, Ρ‡Ρ‚ΠΎ ΠΏΡ€ΠΈ зарядкС ΠΈ разрядкС ΠΌΠ΅ΠΆΠ΄Ρƒ ΠΎΠ±ΠΊΠ»Π°Π΄ΠΊΠ°ΠΌΠΈ кондСнсатора Π½Π΅ происходит пСрСмСщСния заряда, Ρ‚ΠΎΠΊ смСщСния ΠΏΡ€ΠΎΡ‚Π΅ΠΊΠ°Π΅Ρ‚ Ρ‡Π΅Ρ€Π΅Π· кондСнсатор ΠΈ Π·Π°ΠΌΡ‹ΠΊΠ°Π΅Ρ‚ ΡΠ»Π΅ΠΊΡ‚Ρ€ΠΈΡ‡Π΅ΡΠΊΡƒΡŽ Ρ†Π΅ΠΏΡŒ.

Π’ΠΈΠ΄Ρ‹ Ρ‚ΠΎΠΊΠ°

По способу Π³Π΅Π½Π΅Ρ€Π°Ρ†ΠΈΠΈ ΠΈ свойствам элСктроток Π±Ρ‹Π²Π°Π΅Ρ‚ постоянным ΠΈ ΠΏΠ΅Ρ€Π΅ΠΌΠ΅Π½Π½Ρ‹ΠΌ. ΠŸΠΎΡΡ‚ΠΎΡΠ½Π½Ρ‹ΠΉ – это Ρ‚Π°ΠΊΠΎΠΉ, Ρ‡Ρ‚ΠΎ Π½Π΅ мСняСт своСго направлСния. Он Ρ‚Π΅Ρ‡Ρ‘Ρ‚ всСгда Π² ΠΎΠ΄Π½Ρƒ сторону. ΠŸΠ΅Ρ€Π΅ΠΌΠ΅Π½Π½Ρ‹ΠΉ Ρ‚ΠΎΠΊ пСриодичСски мСняСт Π½Π°ΠΏΡ€Π°Π²Π»Π΅Π½ΠΈΠ΅. Под ΠΏΠ΅Ρ€Π΅ΠΌΠ΅Π½Π½Ρ‹ΠΌ ΠΏΠΎΠ½ΠΈΠΌΠ°ΡŽΡ‚ любой Ρ‚ΠΎΠΊ, ΠΊΡ€ΠΎΠΌΠ΅ постоянного. Если ΠΌΠ³Π½ΠΎΠ²Π΅Π½Π½Ρ‹Π΅ значСния ΠΏΠΎΠ²Ρ‚ΠΎΡ€ΡΡŽΡ‚ΡΡ Π² Π½Π΅ΠΈΠ·ΠΌΠ΅Π½Π½ΠΎΠΉ ΠΏΠΎΡΠ»Π΅Π΄ΠΎΠ²Π°Ρ‚Π΅Π»ΡŒΠ½ΠΎΡΡ‚ΠΈ Ρ‡Π΅Ρ€Π΅Π· Ρ€Π°Π²Π½Ρ‹Π΅ ΠΏΡ€ΠΎΠΌΠ΅ΠΆΡƒΡ‚ΠΊΠΈ Π²Ρ€Π΅ΠΌΠ΅Π½ΠΈ, Ρ‚ΠΎ Ρ‚Π°ΠΊΠΎΠΉ элСктроток Π½Π°Π·Ρ‹Π²Π°ΡŽΡ‚ пСриодичСским.

ΠšΠ»Π°ΡΡΠΈΡ„ΠΈΠΊΠ°Ρ†ΠΈΡ ΠΏΠ΅Ρ€Π΅ΠΌΠ΅Π½Π½ΠΎΠ³ΠΎ Ρ‚ΠΎΠΊΠ°

ΠšΠ»Π°ΡΡΠΈΡ„ΠΈΡ†ΠΈΡ€ΠΎΠ²Π°Ρ‚ΡŒ ΠΈΠ·ΠΌΠ΅Π½ΡΡŽΡ‰ΠΈΠ΅ΡΡ Π²ΠΎ Π²Ρ€Π΅ΠΌΠ΅Π½ΠΈ Ρ‚ΠΎΠΊΠΈ ΠΌΠΎΠΆΠ½ΠΎ ΡΠ»Π΅Π΄ΡƒΡŽΡ‰ΠΈΠΌ ΠΎΠ±Ρ€Π°Π·ΠΎΠΌ:

  1. Π‘ΠΈΠ½ΡƒΡΠΎΠΈΠ΄Π°Π»ΡŒΠ½Ρ‹ΠΉ, ΠΏΠΎΠ΄Ρ‡ΠΈΠ½ΡΡŽΡ‰ΠΈΠΉΡΡ ΡΠΈΠ½ΡƒΡΠΎΠΈΠ΄Π°Π»ΡŒΠ½ΠΎΠΉ Ρ„ΡƒΠ½ΠΊΡ†ΠΈΠΈ Π²ΠΎ Π²Ρ€Π΅ΠΌΠ΅Π½ΠΈ.
  2. квазистационарный – ΠΏΠ΅Ρ€Π΅ΠΌΠ΅Π½Π½Ρ‹ΠΉ, ΠΌΠ΅Π΄Π»Π΅Π½Π½ΠΎ ΠΈΠ·ΠΌΠ΅Π½ΡΡŽΡ‰ΠΈΠΉΡΡ Π²ΠΎ Π²Ρ€Π΅ΠΌΠ΅Π½ΠΈ. ΠžΠ±Ρ‹Ρ‡Π½Ρ‹Π΅ ΠΏΡ€ΠΎΠΌΡ‹ΡˆΠ»Π΅Π½Π½Ρ‹Π΅ Ρ‚ΠΎΠΊΠΈ ΡΠ²Π»ΡΡŽΡ‚ΡΡ квазистационарными.
  3. Высокочастотный – частота ΠΊΠΎΡ‚ΠΎΡ€ΠΎΠ³ΠΎ ΠΏΡ€Π΅Π²Ρ‹ΡˆΠ°Π΅Ρ‚ дСсятки ΠΊΠ“Ρ†.
  4. ΠŸΡƒΠ»ΡŒΡΠΈΡ€ΡƒΡŽΡ‰ΠΈΠΉ – ΠΈΠΌΠΏΡƒΠ»ΡŒΡ ΠΊΠΎΡ‚ΠΎΡ€ΠΎΠ³ΠΎ пСриодичСски измСняСтся.

Π Π°Π·Π»ΠΈΡ‡Π°ΡŽΡ‚ Ρ‚Π°ΠΊΠΆΠ΅ Π²ΠΈΡ…Ρ€Π΅Π²Ρ‹Π΅ Ρ‚ΠΎΠΊΠΈ, ΠΊΠΎΡ‚ΠΎΡ€Ρ‹Π΅ Π²ΠΎΠ·Π½ΠΈΠΊΠ°ΡŽΡ‚ Π² ΠΏΡ€ΠΎΠ²ΠΎΠ΄Π½ΠΈΠΊΠ΅ ΠΏΡ€ΠΈ ΠΈΠ·ΠΌΠ΅Π½Π΅Π½ΠΈΠΈ ΠΌΠ°Π³Π½ΠΈΡ‚Π½ΠΎΠ³ΠΎ ΠΏΠΎΡ‚ΠΎΠΊΠ°. Π‘Π»ΡƒΠΆΠ΄Π°ΡŽΡ‰ΠΈΠ΅ Ρ‚ΠΎΠΊΠΈ Π€ΡƒΠΊΠΎ, ΠΊΠ°ΠΊ ΠΈΡ… Π΅Ρ‰Ρ‘ Π½Π°Π·Ρ‹Π²Π°ΡŽΡ‚, Π½Π΅ Ρ‚Π΅ΠΊΡƒΡ‚ ΠΏΠΎ ΠΏΡ€ΠΎΠ²ΠΎΠ΄Π°ΠΌ, Π° ΠΎΠ±Ρ€Π°Π·ΡƒΡŽΡ‚ Π²ΠΈΡ…Ρ€Π΅Π²Ρ‹Π΅ ΠΊΠΎΠ½Ρ‚ΡƒΡ€Ρ‹. Π˜Π½Π΄ΡƒΠΊΡ†ΠΈΠΎΠ½Π½Ρ‹ΠΉ Ρ‚ΠΎΠΊ ΠΈΠΌΠ΅Π΅Ρ‚ Ρ‚Ρƒ ΠΆΠ΅ ΠΏΡ€ΠΈΡ€ΠΎΠ΄Ρƒ Ρ‡Ρ‚ΠΎ ΠΈ Π²ΠΈΡ…Ρ€Π΅Π²ΠΎΠΉ.

ДрСйфовая ΡΠΊΠΎΡ€ΠΎΡΡ‚ΡŒ элСктронов

ЭлСктричСство ΠΏΠΎ мСталличСскому ΠΏΡ€ΠΎΠ²ΠΎΠ΄Π½ΠΈΠΊΡƒ распространяСтся со ΡΠΊΠΎΡ€ΠΎΡΡ‚ΡŒΡŽ свСта. Но это Π½Π΅ ΠΎΠ·Π½Π°Ρ‡Π°Π΅Ρ‚, Ρ‡Ρ‚ΠΎ заряТСнныС частицы нСсутся ΠΎΡ‚ полюса ΠΊ ΠΏΠΎΠ»ΡŽΡΡƒ с Ρ‚Π°ΠΊΠΎΠΉ ΠΆΠ΅ ΡΠΊΠΎΡ€ΠΎΡΡ‚ΡŒΡŽ. Π­Π»Π΅ΠΊΡ‚Ρ€ΠΎΠ½Ρ‹ Π² мСталличСских ΠΏΡ€ΠΎΠ²ΠΎΠ΄Π½ΠΈΠΊΠ°Ρ… Π²ΡΡ‚Ρ€Π΅Ρ‡Π°ΡŽΡ‚ Π½Π° своём ΠΏΡƒΡ‚ΠΈ сопротивлСниС Π°Ρ‚ΠΎΠΌΠΎΠ², поэтому ΠΈΡ… Ρ€Π΅Π°Π»ΡŒΠ½ΠΎΠ΅ ΠΏΠ΅Ρ€Π΅ΠΌΠ΅Ρ‰Π΅Π½ΠΈΠ΅ составляСт всСго 0,1 ΠΌΠΌ Π·Π° сСкунду. РСальная, упорядочСнная ΡΠΊΠΎΡ€ΠΎΡΡ‚ΡŒ пСрСмСщСния элСктронов Π² ΠΏΡ€ΠΎΠ²ΠΎΠ΄Π½ΠΈΠΊΠ΅ называСтся Π΄Ρ€Π΅ΠΉΡ„ΠΎΠ²ΠΎΠΉ.

Если Π·Π°ΠΌΠΊΠ½ΡƒΡ‚ΡŒ ΠΏΡ€ΠΎΠ²ΠΎΠ΄Π½ΠΈΠΊΠΎΠΌ ΠΏΠΎΠ»ΡŽΡΡ‹ источника питания, Ρ‚ΠΎ Π²ΠΎΠΊΡ€ΡƒΠ³ ΠΏΡ€ΠΎΠ²ΠΎΠ΄Π½ΠΈΠΊΠ° молниСносно образуСтся элСктричСскоС ΠΏΠΎΠ»Π΅. Π§Π΅ΠΌ большС Π­Π”Π‘ источников, Ρ‚Π΅ΠΌ сильнСС проявляСтся Π½Π°ΠΏΡ€ΡΠΆΡ‘Π½Π½ΠΎΡΡ‚ΡŒ элСктричСского поля. РСагируя Π½Π° Π½Π°ΠΏΡ€ΡΠΆΡ‘Π½Π½ΠΎΡΡ‚ΡŒ, заряТСнныС частицы Π²ΠΌΠΈΠ³ ΠΏΡ€ΠΈΠ½ΠΈΠΌΠ°ΡŽΡ‚ упорядочСнноС Π΄Π²ΠΈΠΆΠ΅Π½ΠΈΠ΅ ΠΈ Π½Π°Ρ‡ΠΈΠ½Π°ΡŽΡ‚ Π΄Ρ€Π΅ΠΉΡ„ΠΎΠ²Π°Ρ‚ΡŒ.

НаправлСниС элСктричСского Ρ‚ΠΎΠΊΠ°

Π’Ρ€Π°Π΄ΠΈΡ†ΠΈΠΎΠ½Π½ΠΎ ΡΡ‡ΠΈΡ‚Π°ΡŽΡ‚, Ρ‡Ρ‚ΠΎ Π²Π΅ΠΊΡ‚ΠΎΡ€ элСктричСского Ρ‚ΠΎΠΊΠ° Π½Π°ΠΏΡ€Π°Π²Π»Π΅Π½ ΠΊ ΠΎΡ‚Ρ€ΠΈΡ†Π°Ρ‚Π΅Π»ΡŒΠ½ΠΎΠΌΡƒ ΠΏΠΎΠ»ΡŽΡΡƒ источника. Но Π½Π° самом Π΄Π΅Π»Π΅ элСктроны двиТутся ΠΊ ΠΏΠΎΠ»ΠΎΠΆΠΈΡ‚Π΅Π»ΡŒΠ½ΠΎΠΌΡƒ ΠΏΠΎΠ»ΡŽΡΡƒ. Врадиция Π²ΠΎΠ·Π½ΠΈΠΊΠ»Π° ΠΈΠ·-Π·Π° Ρ‚ΠΎΠ³ΠΎ, Ρ‡Ρ‚ΠΎ Π·Π° Π½Π°ΠΏΡ€Π°Π²Π»Π΅Π½ΠΈΠ΅ Π²Π΅ΠΊΡ‚ΠΎΡ€Π° Π±Ρ‹Π»ΠΎ Π²Ρ‹Π±Ρ€Π°Π½ΠΎ Π΄Π²ΠΈΠΆΠ΅Π½ΠΈΠ΅ ΠΏΠΎΠ»ΠΎΠΆΠΈΡ‚Π΅Π»ΡŒΠ½Ρ‹Ρ… ΠΈΠΎΠ½ΠΎΠ² Π² элСктролитах, ΠΊΠΎΡ‚ΠΎΡ€Ρ‹Π΅ Π΄Π΅ΠΉΡΡ‚Π²ΠΈΡ‚Π΅Π»ΡŒΠ½ΠΎ стрСмятся ΠΊ Π½Π΅Π³Π°Ρ‚ΠΈΠ²Π½ΠΎΠΌΡƒ ΠΏΠΎΠ»ΡŽΡΡƒ.

Π­Π»Π΅ΠΊΡ‚Ρ€ΠΎΠ½Ρ‹ проводимости с ΠΎΡ‚Ρ€ΠΈΡ†Π°Ρ‚Π΅Π»ΡŒΠ½Ρ‹ΠΌ зарядом Π² ΠΌΠ΅Ρ‚Π°Π»Π»Π°Ρ… Π±Ρ‹Π»ΠΈ ΠΎΡ‚ΠΊΡ€Ρ‹Ρ‚Ρ‹ ΠΏΠΎΠ·ΠΆΠ΅, Π½ΠΎ Ρ„ΠΈΠ·ΠΈΠΊΠΈ Π½Π΅ стали ΠΌΠ΅Π½ΡΡ‚ΡŒ ΠΏΠ΅Ρ€Π²ΠΎΠ½Π°Ρ‡Π°Π»ΡŒΠ½Ρ‹Π΅ убСТдСния. Π’Π°ΠΊ ΡƒΠΊΡ€Π΅ΠΏΠΈΠ»ΠΎΡΡŒ ΡƒΡ‚Π²Π΅Ρ€ΠΆΠ΄Π΅Π½ΠΈΠ΅, Ρ‡Ρ‚ΠΎ Ρ‚ΠΎΠΊ Π½Π°ΠΏΡ€Π°Π²Π»Π΅Π½ ΠΎΡ‚ плюса ΠΊ минусу.

ЭлСктричСский Ρ‚ΠΎΠΊ Π² Ρ€Π°Π·Π»ΠΈΡ‡Π½Ρ‹Ρ… срСдах

Π’ ΠΌΠ΅Ρ‚Π°Π»Π»Π°Ρ…

НоситСлями Ρ‚ΠΎΠΊΠ° Π² мСталличСских ΠΏΡ€ΠΎΠ²ΠΎΠ΄Π½ΠΈΠΊΠ°Ρ… ΡΠ²Π»ΡΡŽΡ‚ΡΡ свободныС элСктроны, ΠΊΠΎΡ‚ΠΎΡ€Ρ‹Π΅ ΠΈΠ·-Π·Π° слабых элСктричСских связСй Ρ…Π°ΠΎΡ‚ΠΈΡ‡Π½ΠΎ Π±Π»ΡƒΠΆΠ΄Π°ΡŽΡ‚ Π²Π½ΡƒΡ‚Ρ€ΠΈ кристалличСских Ρ€Π΅ΡˆΡ‘Ρ‚ΠΎΠΊ (рис. 3). Как Ρ‚ΠΎΠ»ΡŒΠΊΠΎ Π² ΠΏΡ€ΠΎΠ²ΠΎΠ΄Π½ΠΈΠΊΠ΅ появляСтся Π­Π”Π‘, элСктроны Π½Π°Ρ‡ΠΈΠ½Π°ΡŽΡ‚ упорядочСно Π΄Ρ€Π΅ΠΉΡ„ΠΎΠ²Π°Ρ‚ΡŒ Π² сторону ΠΏΠΎΠ·ΠΈΡ‚ΠΈΠ²Π½ΠΎΠ³ΠΎ полюса источника питания.

Рис. 3. ЭлСктричСский Ρ‚ΠΎΠΊ Π² ΠΌΠ΅Ρ‚Π°Π»Π»Π°Ρ…

Π’ Ρ€Π΅Π·ΡƒΠ»ΡŒΡ‚Π°Ρ‚Π΅ прохоТдСния Ρ‚ΠΎΠΊΠ° Π²ΠΎΠ·Π½ΠΈΠΊΠ°Π΅Ρ‚ сопротивлСниС ΠΏΡ€ΠΎΠ²ΠΎΠ΄Π½ΠΈΠΊΠΎΠ², ΠΊΠΎΡ‚ΠΎΡ€ΠΎΠ΅ прСпятствуСт ΠΏΠΎΡ‚ΠΎΠΊΡƒ элСктронов ΠΈ ΠΏΡ€ΠΈΠ²ΠΎΠ΄ΠΈΡ‚ Π½Π°Π³Ρ€Π΅Π²Π°Π½ΠΈΡŽ. ΠŸΡ€ΠΈ ΠΊΠΎΡ€ΠΎΡ‚ΠΊΠΎΠΌ Π·Π°ΠΌΡ‹ΠΊΠ°Π½ΠΈΠΈ Π²Ρ‹Π΄Π΅Π»Π΅Π½ΠΈΠ΅ Ρ‚Π΅ΠΏΠ»Π° Π½Π°ΡΡ‚ΠΎΠ»ΡŒΠΊΠΎ сильноС, Ρ€Π°Π·Ρ€ΡƒΡˆΠ°Π΅Ρ‚ ΠΏΡ€ΠΎΠ²ΠΎΠ΄Π½ΠΈΠΊ.

Π’ ΠΏΠΎΠ»ΡƒΠΏΡ€ΠΎΠ²ΠΎΠ΄Π½ΠΈΠΊΠ°Ρ…

Π’ ΠΎΠ±Ρ‹Ρ‡Π½ΠΎΠΌ состоянии Ρƒ ΠΏΠΎΠ»ΡƒΠΏΡ€ΠΎΠ²ΠΎΠ΄Π½ΠΈΠΊΠ° Π½Π΅Ρ‚ свободных носитСлСй зарядов.Β  Но Ссли ΡΠΎΠ΅Π΄ΠΈΠ½ΠΈΡ‚ΡŒ Π΄Π²Π° Ρ€Π°Π·Π½Ρ‹Ρ… Ρ‚ΠΈΠΏΠ° ΠΏΠΎΠ»ΡƒΠΏΡ€ΠΎΠ²ΠΎΠ΄Π½ΠΈΠΊΠΎΠ², Ρ‚ΠΎ ΠΏΡ€ΠΈ прямом ΠΏΠΎΠ΄ΠΊΠ»ΡŽΡ‡Π΅Π½ΠΈΠΈ ΠΎΠ½ΠΈ ΠΏΡ€Π΅Π²Ρ€Π°Ρ‰Π°ΡŽΡ‚ΡΡ Π² ΠΏΡ€ΠΎΠ²ΠΎΠ΄Π½ΠΈΠΊ. ΠŸΡ€ΠΎΠΈΡΡ…ΠΎΠ΄ΠΈΡ‚ это ΠΏΠΎΡ‚ΠΎΠΌΡƒ, Ρ‡Ρ‚ΠΎ Ρƒ ΠΎΠ΄Π½ΠΎΠ³ΠΎ Ρ‚ΠΈΠΏΠ° Π΅ΡΡ‚ΡŒ ΠΏΠΎΠ»ΠΎΠΆΠΈΡ‚Π΅Π»ΡŒΠ½ΠΎ заряТСнныС ΠΈΠΎΠ½Ρ‹ (Π΄Ρ‹Ρ€ΠΊΠΈ), Π° Ρƒ Π΄Ρ€ΡƒΠ³ΠΎΠ³ΠΎ – ΠΎΡ‚Ρ€ΠΈΡ†Π°Ρ‚Π΅Π»ΡŒΠ½Ρ‹Π΅ ΠΈΠΎΠ½Ρ‹ (Π°Ρ‚ΠΎΠΌΡ‹ с лишним элСктроном).

Под напряТСниСм элСктроны ΠΈΠ· ΠΎΠ΄Π½ΠΎΠ³ΠΎ ΠΏΠΎΠ»ΡƒΠΏΡ€ΠΎΠ²ΠΎΠ΄Π½ΠΈΠΊΠ° ΡƒΡΡ‚Ρ€Π΅ΠΌΠ»ΡΡŽΡ‚ΡΡ для замСщСния (Ρ€Π΅ΠΊΠΎΠΌΠ±ΠΈΠ½Π°Ρ†ΠΈΠΈ) Π΄Ρ‹Ρ€ΠΎΠΊ Π² Π΄Ρ€ΡƒΠ³ΠΎΠΌ. Π’ΠΎΠ·Π½ΠΈΠΊΠ°Π΅Ρ‚ упорядочСнноС Π΄Π²ΠΈΠΆΠ΅Π½ΠΈΠ΅ свободных зарядов. Π’Π°ΠΊΡƒΡŽ ΠΏΡ€ΠΎΠ²ΠΎΠ΄ΠΈΠΌΠΎΡΡ‚ΡŒ Π½Π°Π·Ρ‹Π²Π°ΡŽΡ‚ элСктронно-Π΄Ρ‹Ρ€ΠΎΡ‡Π½ΠΎΠΉ.

Π’ Π²Π°ΠΊΡƒΡƒΠΌΠ΅ ΠΈ Π³Π°Π·Π΅

ЭлСктричСский Ρ‚ΠΎΠΊ Π²ΠΎΠ·ΠΌΠΎΠΆΠ΅Π½ ΠΈ Π² ΠΈΠΎΠ½ΠΈΠ·ΠΈΡ€ΠΎΠ²Π°Π½Π½ΠΎΠΌ Π³Π°Π·Π΅. Заряд пСрСносится ΠΏΠΎΠ»ΠΎΠΆΠΈΡ‚Π΅Π»ΡŒΠ½Ρ‹ΠΌΠΈ ΠΈ ΠΎΡ‚Ρ€ΠΈΡ†Π°Ρ‚Π΅Π»ΡŒΠ½Ρ‹ΠΌΠΈ ΠΈΠΎΠ½Π°ΠΌΠΈ. Π˜ΠΎΠ½ΠΈΠ·Π°Ρ†ΠΈΡ Π³Π°Π·ΠΎΠ² Π²ΠΎΠ·ΠΌΠΎΠΆΠ½Π° ΠΏΠΎΠ΄ дСйствиСм излучСния ΠΈΠ»ΠΈ вслСдствиС сильного нагрСвания. Под дСйствиСм этих Ρ„Π°ΠΊΡ‚ΠΎΡ€ΠΎΠ² Π²ΠΎΠ·Π±ΡƒΠΆΠ΄Π°ΡŽΡ‚ΡΡ Π°Ρ‚ΠΎΠΌΡ‹, ΠΊΠΎΡ‚ΠΎΡ€Ρ‹Π΅ ΠΏΡ€Π΅Π²Ρ€Π°Ρ‰Π°ΡŽΡ‚ΡΡ Π² ΠΈΠΎΠ½Ρ‹ (рис. 4).

Рис 4. ЭлСктричСский Ρ‚ΠΎΠΊ Π² Π³Π°Π·Π°Ρ…

Π’ Π²Π°ΠΊΡƒΡƒΠΌΠ΅ элСктричСскиС заряды Π½Π΅ Π²ΡΡ‚Ρ€Π΅Ρ‡Π°ΡŽΡ‚ сопротивлСния, поэтому. заряТСнныС частицы двиТутся с околосвСтовыми скоростями. НоситСлями зарядов ΡΠ²Π»ΡΡŽΡ‚ΡΡ элСктроны. Для возникновСния Ρ‚ΠΎΠΊΠ° Π² Π²Π°ΠΊΡƒΡƒΠΌΠ΅ Π½Π΅ΠΎΠ±Ρ…ΠΎΠ΄ΠΈΠΌΠΎ ΡΠΎΠ·Π΄Π°Ρ‚ΡŒ источник элСктронов ΠΈ достаточно большой ΠΏΠΎΠ»ΠΎΠΆΠΈΡ‚Π΅Π»ΡŒΠ½Ρ‹ΠΉ ΠΏΠΎΡ‚Π΅Π½Ρ†ΠΈΠ°Π» Π½Π° элСктродС.

ΠŸΡ€ΠΈΠΌΠ΅Ρ€ΠΎΠΌ ΠΌΠΎΠΆΠ΅Ρ‚ ΡΠ»ΡƒΠΆΠΈΡ‚ΡŒ Ρ€Π°Π±ΠΎΡ‚Π° Π²Π°ΠΊΡƒΡƒΠΌΠ½ΠΎΠΉ Π»Π°ΠΌΠΏΡ‹ ΠΈΠ»ΠΈ элСктронно-лучСвая Ρ‚Ρ€ΡƒΠ±ΠΊΠ°.

Π’ Тидкостях

ΠžΠ³ΠΎΠ²ΠΎΡ€ΠΈΠΌΡΡ сразу – Π½Π΅ всС Тидкости ΡΠ²Π»ΡΡŽΡ‚ΡΡ ΠΏΡ€ΠΎΠ²ΠΎΠ΄Π½ΠΈΠΊΠ°ΠΌΠΈ. ЭлСктричСский Ρ‚ΠΎΠΊ Π²ΠΎΠ·ΠΌΠΎΠΆΠ΅Π½ Π² кислотных, Ρ‰Ρ‘Π»ΠΎΡ‡Π½Ρ‹Ρ… ΠΈ соляных растворах. Π˜Π½Π°Ρ‡Π΅ говоря – Π² срСдах, Π³Π΄Π΅ ΠΈΠΌΠ΅ΡŽΡ‚ΡΡ заряТСнныС ΠΈΠΎΠ½Ρ‹.

Если ΠΎΠΏΡƒΡΡ‚ΠΈΡ‚ΡŒ Π² раствор Π΄Π²Π° элСктрода ΠΈ ΠΏΠΎΠ΄ΠΊΠ»ΡŽΡ‡ΠΈΡ‚ΡŒ ΠΈΡ… ΠΊ полюсам источника, Ρ‚ΠΎ ΠΌΠ΅ΠΆΠ΄Ρƒ Π½ΠΈΠΌΠΈ Π±ΡƒΠ΄Π΅Ρ‚ ΠΏΡ€ΠΎΡ‚Π΅ΠΊΠ°Ρ‚ΡŒ элСктричСский Ρ‚ΠΎΠΊ (рис. 5). Под дСйствиСм Π­Π”Π‘ ΠΊΠ°Ρ‚ΠΈΠΎΠ½Ρ‹ устрСмятся ΠΊ ΠΊΠ°Ρ‚ΠΎΠ΄Ρƒ (минусу), Π° Π°Π½ΠΈΠΎΠ½Ρ‹ ΠΊ Π°Π½ΠΎΠ΄Ρƒ. ΠŸΡ€ΠΈ этом Π±ΡƒΠ΄Π΅Ρ‚ ΠΏΡ€ΠΎΠΈΡΡ…ΠΎΠ΄ΠΈΡ‚ΡŒ химичСскоС воздСйствиС Π½Π° элСктроды – Π½Π° Π½ΠΈΡ… Π±ΡƒΠ΄ΡƒΡ‚ ΠΎΡΠ΅Π΄Π°Ρ‚ΡŒ Π°Ρ‚ΠΎΠΌΡ‹ растворённых вСщСств. Π’Π°ΠΊΠΎΠ΅ явлСниС Π½Π°Π·Ρ‹Π²Π°ΡŽΡ‚ элСктролизом.

Рис. 5. Π­Π»Π΅ΠΊΡ‚Ρ€ΠΎΡ‚ΠΎΠΊ Π² Тидкостях

Для Π»ΡƒΡ‡ΡˆΠ΅Π³ΠΎ понимания свойств элСктротока Π² Ρ€Π°Π·Π½Ρ‹Ρ… срСдах, ΠΏΡ€Π΅Π΄Π»Π°Π³Π°ΡŽ Ρ€Π°ΡΡΠΌΠΎΡ‚Ρ€Π΅Ρ‚ΡŒ ΠΊΠ°Ρ€Ρ‚ΠΈΠ½ΠΊΡƒ Π½Π° рисункС 6. ΠžΠ±Ρ€Π°Ρ‚ΠΈΡ‚Π΅ Π²Π½ΠΈΠΌΠ°Π½ΠΈΠ΅ Π½Π° Π²ΠΎΠ»ΡŒΡ‚Π°ΠΌΠΏΠ΅Ρ€Π½Ρ‹Π΅ характСристики (4 столбСц).

Рис. 6. ЭлСктричСский Ρ‚ΠΎΠΊ Π² срСдах

ΠŸΡ€ΠΎΠ²ΠΎΠ΄Π½ΠΈΠΊΠΈ элСктричСского Ρ‚ΠΎΠΊΠ°

Π‘Ρ€Π΅Π΄ΠΈ мноТСства вСщСств, лишь Π½Π΅ΠΊΠΎΡ‚ΠΎΡ€Ρ‹Π΅ ΡΠ²Π»ΡΡŽΡ‚ΡΡ ΠΏΡ€ΠΎΠ²ΠΎΠ΄Π½ΠΈΠΊΠ°ΠΌΠΈ. К Ρ…ΠΎΡ€ΠΎΡˆΠΈΠΌ ΠΏΡ€ΠΎΠ²ΠΎΠ΄Π½ΠΈΠΊΠ°ΠΌ относятся ΠΌΠ΅Ρ‚Π°Π»Π»Ρ‹. Π’Π°ΠΆΠ½ΠΎΠΉ характСристикой ΠΏΡ€ΠΎΠ²ΠΎΠ΄Π½ΠΈΠΊΠ° являСтся Π΅Π³ΠΎ ΡƒΠ΄Π΅Π»ΡŒΠ½ΠΎΠ΅ сопротивлСниС.

НСбольшоС сопротивлСниС ΠΈΠΌΠ΅ΡŽΡ‚:

  • всС Π±Π»Π°Π³ΠΎΡ€ΠΎΠ΄Π½Ρ‹Π΅ ΠΌΠ΅Ρ‚Π°Π»Π»Ρ‹;
  • мСдь;
  • алюминий;
  • ΠΎΠ»ΠΎΠ²ΠΎ;
  • свинСц.

На ΠΏΡ€Π°ΠΊΡ‚ΠΈΠΊΠ΅ Π½Π°ΠΈΠ±ΠΎΠ»Π΅Π΅ часто ΠΏΡ€ΠΈΠΌΠ΅Π½ΡΡŽΡ‚ Π°Π»ΡŽΠΌΠΈΠ½ΠΈΠ΅Π²Ρ‹Π΅ ΠΈ ΠΌΠ΅Π΄Π½Ρ‹Π΅ ΠΏΡ€ΠΎΠ²ΠΎΠ΄Π½ΠΈΠΊΠΈ, Ρ‚Π°ΠΊ ΠΊΠ°ΠΊ ΠΎΠ½ΠΈ Π½Π΅ слишком Π΄ΠΎΡ€ΠΎΠ³ΠΈΠ΅.

Π­Π»Π΅ΠΊΡ‚Ρ€ΠΎΠ±Π΅Π·ΠΎΠΏΠ°ΡΠ½ΠΎΡΡ‚ΡŒ

НСсмотря Π½Π° Ρ‚ΠΎ Ρ‡Ρ‚ΠΎ элСктричСство ΠΏΡ€ΠΎΡ‡Π½ΠΎ вошло Π² Π½Π°ΡˆΡƒ Тизнь, Π½Π΅ слСдуСт Π·Π°Π±Ρ‹Π²Π°Ρ‚ΡŒ ΠΎΠ± элСктробСзопасности. ВысокиС напряТСния опасны для ΠΆΠΈΠ·Π½ΠΈ, Π° ΠΊΠΎΡ€ΠΎΡ‚ΠΊΠΈΠ΅ замыкания становятся ΠΏΡ€ΠΈΡ‡ΠΈΠ½ΠΎΠΉ ΠΏΠΎΠΆΠ°Ρ€ΠΎΠ².

ΠŸΡ€ΠΈ Π²Ρ‹ΠΏΠΎΠ»Π½Π΅Π½ΠΈΠΈ Ρ€Π΅ΠΌΠΎΠ½Ρ‚Π½Ρ‹Ρ… Ρ€Π°Π±ΠΎΡ‚ Π½Π΅ΠΎΠ±Ρ…ΠΎΠ΄ΠΈΠΌΠΎ строго ΡΠΎΠ±Π»ΡŽΠ΄Π°Ρ‚ΡŒ ΠΏΡ€Π°Π²ΠΈΠ»Π° бСзопасности: Π½Π΅ Ρ€Π°Π±ΠΎΡ‚Π°Ρ‚ΡŒ ΠΏΠΎΠ΄ высоким напряТСниСм, ΠΈΡΠΏΠΎΠ»ΡŒΠ·ΠΎΠ²Π°Ρ‚ΡŒ Π·Π°Ρ‰ΠΈΡ‚Π½ΡƒΡŽ ΠΎΠ΄Π΅ΠΆΠ΄Ρƒ ΠΈ ΡΠΏΠ΅Ρ†ΠΈΠ°Π»ΡŒΠ½Ρ‹Π΅ инструмСнты, ΠΏΡ€ΠΈΠΌΠ΅Π½ΡΡ‚ΡŒ Π½ΠΎΠΆΠΈ зазСмлСния ΠΈ Ρ‚.ΠΏ.

Π’ Π±Ρ‹Ρ‚Ρƒ ΠΈΡΠΏΠΎΠ»ΡŒΠ·ΡƒΠΉΡ‚Π΅ Ρ‚ΠΎΠ»ΡŒΠΊΠΎ Ρ‚Π°ΠΊΡƒΡŽ элСктротСхнику, которая рассчитана Π½Π° Ρ€Π°Π±ΠΎΡ‚Ρƒ Π² ΡΠΎΠΎΡ‚Π²Π΅Ρ‚ΡΡ‚Π²ΡƒΡŽΡ‰Π΅ΠΉ сСти. Никогда Π½Π΅ ΡΡ‚Π°Π²ΡŒΡ‚Π΅ Β«ΠΆΡƒΡ‡ΠΊΠΈΒ» вмСсто ΠΏΡ€Π΅Π΄ΠΎΡ…Ρ€Π°Π½ΠΈΡ‚Π΅Π»Π΅ΠΉ.

ΠŸΠΎΠΌΠ½ΠΈΡ‚Π΅, Ρ‡Ρ‚ΠΎ ΠΌΠΎΡ‰Π½Ρ‹Π΅ элСктролитичСскиС кондСнсаторы ΠΈΠΌΠ΅ΡŽΡ‚ Π±ΠΎΠ»ΡŒΡˆΡƒΡŽ ΡΠ»Π΅ΠΊΡ‚Ρ€ΠΈΡ‡Π΅ΡΠΊΡƒΡŽ Π΅ΠΌΠΊΠΎΡΡ‚ΡŒ. НакоплСнная Π² Π½ΠΈΡ… энСргия ΠΌΠΎΠΆΠ΅Ρ‚ Π²Ρ‹Π·Π²Π°Ρ‚ΡŒ ΠΏΠΎΡ€Π°ΠΆΠ΅Π½ΠΈΠ΅ Π΄Π°ΠΆΠ΅ спустя нСсколько ΠΌΠΈΠ½ΡƒΡ‚ послС ΠΎΡ‚ΠΊΠ»ΡŽΡ‡Π΅Π½ΠΈΡ ΠΎΡ‚ сСти.

ΠŸΠΎΡΡ‚ΠΎΡΠ½Π½Ρ‹ΠΉ элСктричСский Ρ‚ΠΎΠΊ. НаправлСниС Ρ‚ΠΎΠΊΠ°, Ρ„ΠΎΡ€ΠΌΡƒΠ»Π°

Β 

Автор ΡΡ‚Π°Ρ‚ΡŒΠΈ β€” ΠΏΡ€ΠΎΡ„Π΅ΡΡΠΈΠΎΠ½Π°Π»ΡŒΠ½Ρ‹ΠΉ Ρ€Π΅ΠΏΠ΅Ρ‚ΠΈΡ‚ΠΎΡ€, Π°Π²Ρ‚ΠΎΡ€ ΡƒΡ‡Π΅Π±Π½Ρ‹Ρ… пособий для ΠΏΠΎΠ΄Π³ΠΎΡ‚ΠΎΠ²ΠΊΠΈ ΠΊ Π•Π“Π­ Π˜Π³ΠΎΡ€ΡŒ ВячСславович Π―ΠΊΠΎΠ²Π»Π΅Π²

Π’Π΅ΠΌΡ‹ ΠΊΠΎΠ΄ΠΈΡ„ΠΈΠΊΠ°Ρ‚ΠΎΡ€Π° Π•Π“Π­: постоянный элСктричСский Ρ‚ΠΎΠΊ, сила Ρ‚ΠΎΠΊΠ°, напряТСниС.

ЭлСктричСский Ρ‚ΠΎΠΊ обСспСчиваСт ΠΊΠΎΠΌΡ„ΠΎΡ€Ρ‚ΠΎΠΌ Тизнь соврСмСнного Ρ‡Π΅Π»ΠΎΠ²Π΅ΠΊΠ°. ВСхнологичСскиС достиТСния Ρ†ΠΈΠ²ΠΈΠ»ΠΈΠ·Π°Ρ†ΠΈΠΈ β€” энСргСтика, транспорт, Ρ€Π°Π΄ΠΈΠΎ, Ρ‚Π΅Π»Π΅Π²ΠΈΠ΄Π΅Π½ΠΈΠ΅, ΠΊΠΎΠΌΠΏΡŒΡŽΡ‚Π΅Ρ€Ρ‹, мобильная связь β€” основаны Π½Π° использовании элСктричСского Ρ‚ΠΎΠΊΠ°.

ЭлСктричСский Ρ‚ΠΎΠΊ β€” это Π½Π°ΠΏΡ€Π°Π²Π»Π΅Π½Π½ΠΎΠ΅ Π΄Π²ΠΈΠΆΠ΅Π½ΠΈΠ΅ заряТСнных частиц, ΠΏΡ€ΠΈ ΠΊΠΎΡ‚ΠΎΡ€ΠΎΠΌ происходит пСрСнос заряда ΠΈΠ· ΠΎΠ΄Π½ΠΈΡ… областСй пространства Π² Π΄Ρ€ΡƒΠ³ΠΈΠ΅.

ЭлСктричСский Ρ‚ΠΎΠΊ ΠΌΠΎΠΆΠ΅Ρ‚ Π²ΠΎΠ·Π½ΠΈΠΊΠ°Ρ‚ΡŒ Π² самых Ρ€Π°Π·Π»ΠΈΡ‡Π½Ρ‹Ρ… срСдах: Ρ‚Π²Ρ‘Ρ€Π΄Ρ‹Ρ… Ρ‚Π΅Π»Π°Ρ…, Тидкостях, Π³Π°Π·Π°Ρ…. ΠŸΠΎΡ€ΠΎΠΉ ΠΈ срСды Π½ΠΈΠΊΠ°ΠΊΠΎΠΉ Π½Π΅ Π½ΡƒΠΆΠ½ΠΎ β€” Ρ‚ΠΎΠΊ ΠΌΠΎΠΆΠ΅Ρ‚ ΡΡƒΡ‰Π΅ΡΡ‚Π²ΠΎΠ²Π°Ρ‚ΡŒ Π΄Π°ΠΆΠ΅ Π² Π²Π°ΠΊΡƒΡƒΠΌΠ΅! ΠœΡ‹ ΠΏΠΎΠ³ΠΎΠ²ΠΎΡ€ΠΈΠΌ ΠΎΠ± этом Π² своё врСмя, Π° ΠΏΠΎΠΊΠ° ΠΏΡ€ΠΈΠ²Π΅Π΄Ρ‘ΠΌ лишь Π½Π΅ΠΊΠΎΡ‚ΠΎΡ€Ρ‹Π΅ ΠΏΡ€ΠΈΠΌΠ΅Ρ€Ρ‹.

β€’ Π—Π°ΠΌΠΊΠ½Ρ‘ΠΌ полюса Π±Π°Ρ‚Π°Ρ€Π΅ΠΉΠΊΠΈ мСталличСским ΠΏΡ€ΠΎΠ²ΠΎΠ΄ΠΎΠΌ. Π‘Π²ΠΎΠ±ΠΎΠ΄Π½Ρ‹Π΅ элСктроны ΠΏΡ€ΠΎΠ²ΠΎΠ΄Π° Π½Π°Ρ‡Π½ΡƒΡ‚ Π½Π°ΠΏΡ€Π°Π²Π»Π΅Π½Π½ΠΎΠ΅ Π΄Π²ΠΈΠΆΠ΅Π½ΠΈΠ΅ ΠΎΡ‚ «минуса» Π±Π°Ρ‚Π°Ρ€Π΅ΠΉΠΊΠΈ ΠΊ Β«ΠΏΠ»ΡŽΡΡƒΒ».
Π­Ρ‚ΠΎ β€” ΠΏΡ€ΠΈΠΌΠ΅Ρ€ Ρ‚ΠΎΠΊΠ° Π² ΠΌΠ΅Ρ‚Π°Π»Π»Π°Ρ….

β€’ Бросим Π² стакан Π²ΠΎΠ΄Ρ‹ Ρ‰Π΅ΠΏΠΎΡ‚ΠΊΡƒ ΠΏΠΎΠ²Π°Ρ€Π΅Π½Π½ΠΎΠΉ соли . ΠœΠΎΠ»Π΅ΠΊΡƒΠ»Ρ‹ соли Π΄ΠΈΡΡΠΎΡ†ΠΈΠΈΡ€ΡƒΡŽΡ‚ Π½Π° ΠΈΠΎΠ½Ρ‹, Ρ‚Π°ΠΊ Ρ‡Ρ‚ΠΎ Π² растворС появятся свободныС заряды: ΠΏΠΎΠ»ΠΎΠΆΠΈΡ‚Π΅Π»ΡŒΠ½Ρ‹Π΅ ΠΈΠΎΠ½Ρ‹ ΠΈ ΠΎΡ‚Ρ€ΠΈΡ†Π°Ρ‚Π΅Π»ΡŒΠ½Ρ‹Π΅ ΠΈΠΎΠ½Ρ‹ . Π’Π΅ΠΏΠ΅Ρ€ΡŒ засунСм Π² Π²ΠΎΠ΄Ρƒ Π΄Π²Π° элСктрода, соСдинённыС с полюсами Π±Π°Ρ‚Π°Ρ€Π΅ΠΉΠΊΠΈ. Π˜ΠΎΠ½Ρ‹ Π½Π°Ρ‡Π½ΡƒΡ‚ Π½Π°ΠΏΡ€Π°Π²Π»Π΅Π½Π½ΠΎΠ΅ Π΄Π²ΠΈΠΆΠ΅Π½ΠΈΠ΅ ΠΊ ΠΎΡ‚Ρ€ΠΈΡ†Π°Ρ‚Π΅Π»ΡŒΠ½ΠΎΠΌΡƒ элСктроду, Π° ΠΈΠΎΠ½Ρ‹ β€” ΠΊ ΠΏΠΎΠ»ΠΎΠΆΠΈΡ‚Π΅Π»ΡŒΠ½ΠΎΠΌΡƒ.
Π­Ρ‚ΠΎ β€” ΠΏΡ€ΠΈΠΌΠ΅Ρ€ прохоТдСния Ρ‚ΠΎΠΊΠ° Ρ‡Π΅Ρ€Π΅Π· раствор элСктролита.

β€’ Π“Ρ€ΠΎΠ·ΠΎΠ²Ρ‹Π΅ Ρ‚ΡƒΡ‡ΠΈ ΡΠΎΠ·Π΄Π°ΡŽΡ‚ ΡΡ‚ΠΎΠ»ΡŒ ΠΌΠΎΡ‰Π½Ρ‹Π΅ элСктричСскиС поля, Ρ‡Ρ‚ΠΎ оказываСтся Π²ΠΎΠ·ΠΌΠΎΠΆΠ½Ρ‹ΠΌ ΠΏΡ€ΠΎΠ±ΠΎΠΉ Π²ΠΎΠ·Π΄ΡƒΡˆΠ½ΠΎΠ³ΠΎ ΠΏΡ€ΠΎΠΌΠ΅ΠΆΡƒΡ‚ΠΊΠ° Π΄Π»ΠΈΠ½ΠΎΠΉ Π² нСсколько ΠΊΠΈΠ»ΠΎΠΌΠ΅Ρ‚Ρ€ΠΎΠ². Π’ Ρ€Π΅Π·ΡƒΠ»ΡŒΡ‚Π°Ρ‚Π΅ сквозь Π²ΠΎΠ·Π΄ΡƒΡ… ΠΏΡ€ΠΎΡ…ΠΎΠ΄ΠΈΡ‚ гигантский разряд β€” молния.
Π­Ρ‚ΠΎ β€” ΠΏΡ€ΠΈΠΌΠ΅Ρ€ элСктричСского Ρ‚ΠΎΠΊΠ° Π² Π³Π°Π·Π΅.

Π’ΠΎ всСх Ρ‚Ρ€Ρ‘Ρ… рассмотрСнных ΠΏΡ€ΠΈΠΌΠ΅Ρ€Π°Ρ… элСктричСский Ρ‚ΠΎΠΊ обусловлСн Π΄Π²ΠΈΠΆΠ΅Π½ΠΈΠ΅ΠΌ заряТСнных частиц Π²Π½ΡƒΡ‚Ρ€ΠΈ Ρ‚Π΅Π»Π° ΠΈ называСтся Ρ‚ΠΎΠΊΠΎΠΌ проводимости.

β€’ Π’ΠΎΡ‚ нСсколько ΠΈΠ½ΠΎΠΉ ΠΏΡ€ΠΈΠΌΠ΅Ρ€. Π‘ΡƒΠ΄Π΅ΠΌ ΠΏΠ΅Ρ€Π΅ΠΌΠ΅Ρ‰Π°Ρ‚ΡŒ Π² пространствС заряТСнноС Ρ‚Π΅Π»ΠΎ. Вакая ситуация согласуСтся с ΠΎΠΏΡ€Π΅Π΄Π΅Π»Π΅Π½ΠΈΠ΅ΠΌ Ρ‚ΠΎΠΊΠ°! НаправлСнноС Π΄Π²ΠΈΠΆΠ΅Π½ΠΈΠ΅ зарядов β€” Π΅ΡΡ‚ΡŒ, пСрСнос заряда Π² пространствС β€” присутствуСт. Π’ΠΎΠΊ, созданный Π΄Π²ΠΈΠΆΠ΅Π½ΠΈΠ΅ΠΌ макроскопичСского заряТСнного Ρ‚Π΅Π»Π°, называСтся ΠΊΠΎΠ½Π²Π΅ΠΊΡ†ΠΈΠΎΠ½Π½Ρ‹ΠΌ.

Π—Π°ΠΌΠ΅Ρ‚ΠΈΠΌ, Ρ‡Ρ‚ΠΎ Π½Π΅ всякоС Π΄Π²ΠΈΠΆΠ΅Π½ΠΈΠ΅ заряТСнных частиц ΠΎΠ±Ρ€Π°Π·ΡƒΠ΅Ρ‚ Ρ‚ΠΎΠΊ. НапримСр, хаотичСскоС Ρ‚Π΅ΠΏΠ»ΠΎΠ²ΠΎΠ΅ Π΄Π²ΠΈΠΆΠ΅Π½ΠΈΠ΅ зарядов ΠΏΡ€ΠΎΠ²ΠΎΠ΄Π½ΠΈΠΊΠ° β€” Π½Π΅ Π½Π°ΠΏΡ€Π°Π²Π»Π΅Π½Π½ΠΎΠ΅ (ΠΎΠ½ΠΎ ΡΠΎΠ²Π΅Ρ€ΡˆΠ°Π΅Ρ‚ΡΡ Π² ΠΊΠ°ΠΊΠΈΡ… ΡƒΠ³ΠΎΠ΄Π½ΠΎ направлСниях), ΠΈ ΠΏΠΎΡ‚ΠΎΠΌΡƒ Ρ‚ΠΎΠΊΠΎΠΌ Π½Π΅ являСтся (ΠΏΡ€ΠΈ Π²ΠΎΠ·Π½ΠΈΠΊΠ½ΠΎΠ²Π΅Π½ΠΈΠΈ Ρ‚ΠΎΠΊΠ° свободныС заряды ΠΏΡ€ΠΎΠ΄ΠΎΠ»ΠΆΠ°ΡŽΡ‚ ΡΠΎΠ²Π΅Ρ€ΡˆΠ°Ρ‚ΡŒ Ρ‚Π΅ΠΏΠ»ΠΎΠ²ΠΎΠ΅ Π΄Π²ΠΈΠΆΠ΅Π½ΠΈΠ΅! ΠŸΡ€ΠΎΡΡ‚ΠΎ Π² этом случаС ΠΊ хаотичСским пСрСмСщСниям заряТСнных частиц добавляСтся ΠΈΡ… упорядочСнный Π΄Ρ€Π΅ΠΉΡ„ Π² ΠΎΠΏΡ€Π΅Π΄Π΅Π»Ρ‘Π½Π½ΠΎΠΌ
Π½Π°ΠΏΡ€Π°Π²Π»Π΅Π½ΠΈΠΈ).
НС Π±ΡƒΠ΄Π΅Ρ‚ Ρ‚ΠΎΠΊΠΎΠΌ ΠΈ ΠΏΠΎΡΡ‚ΡƒΠΏΠ°Ρ‚Π΅Π»ΡŒΠ½ΠΎΠ΅ Π΄Π²ΠΈΠΆΠ΅Π½ΠΈΠ΅ элСктричСски Π½Π΅ΠΉΡ‚Ρ€Π°Π»ΡŒΠ½ΠΎΠ³ΠΎ Ρ‚Π΅Π»Π°: хотя заряТСнныС частицы Π² Π΅Π³ΠΎ Π°Ρ‚ΠΎΠΌΠ°Ρ… ΠΈ ΡΠΎΠ²Π΅Ρ€ΡˆΠ°ΡŽΡ‚ Π½Π°ΠΏΡ€Π°Π²Π»Π΅Π½Π½ΠΎΠ΅ Π΄Π²ΠΈΠΆΠ΅Π½ΠΈΠ΅, Π½Π΅ происходит пСрСноса заряда ΠΈΠ· ΠΎΠ΄Π½ΠΈΡ… участков пространства Π² Π΄Ρ€ΡƒΠ³ΠΈΠ΅.

Β 

НаправлСниС элСктричСского Ρ‚ΠΎΠΊΠ°

Β 

НаправлСниС двиТСния заряТСнных частиц, ΠΎΠ±Ρ€Π°Π·ΡƒΡŽΡ‰ΠΈΡ… Ρ‚ΠΎΠΊ, зависит ΠΎΡ‚ Π·Π½Π°ΠΊΠ° ΠΈΡ… заряда. ΠŸΠΎΠ»ΠΎΠΆΠΈΡ‚Π΅Π»ΡŒΠ½ΠΎ заряТСнныС частицы Π±ΡƒΠ΄ΡƒΡ‚ Π΄Π²ΠΈΠ³Π°Ρ‚ΡŒΡΡ ΠΎΡ‚ «плюса» ΠΊ «минусу», Π° ΠΎΡ‚Ρ€ΠΈΡ†Π°Ρ‚Π΅Π»ΡŒΠ½ΠΎ заряТСнныС β€” Π½Π°ΠΎΠ±ΠΎΡ€ΠΎΡ‚, ΠΎΡ‚ «минуса» ΠΊ Β«ΠΏΠ»ΡŽΡΡƒΒ». Π’ элСктролитах ΠΈ Π³Π°Π·Π°Ρ…, Π½Π°ΠΏΡ€ΠΈΠΌΠ΅Ρ€, ΠΏΡ€ΠΈΡΡƒΡ‚ΡΡ‚Π²ΡƒΡŽΡ‚ ΠΊΠ°ΠΊ ΠΏΠΎΠ»ΠΎΠΆΠΈΡ‚Π΅Π»ΡŒΠ½Ρ‹Π΅, Ρ‚Π°ΠΊ ΠΈ ΠΎΡ‚Ρ€ΠΈΡ†Π°Ρ‚Π΅Π»ΡŒΠ½Ρ‹Π΅ свободныС заряды, ΠΈ Ρ‚ΠΎΠΊ создаётся ΠΈΡ… встрСчным Π΄Π²ΠΈΠΆΠ΅Π½ΠΈΠ΅ΠΌ Π² ΠΎΠ±ΠΎΠΈΡ… направлСниях. КакоС ΠΆΠ΅ ΠΈΠ· этих Π½Π°ΠΏΡ€Π°Π²Π»Π΅Π½ΠΈΠΉ ΠΏΡ€ΠΈΠ½ΡΡ‚ΡŒ Π·Π° Π½Π°ΠΏΡ€Π°Π²Π»Π΅Π½ΠΈΠ΅ элСктричСского Ρ‚ΠΎΠΊΠ°?

НаправлСниСм Ρ‚ΠΎΠΊΠ° принято ΡΡ‡ΠΈΡ‚Π°Ρ‚ΡŒ Π½Π°ΠΏΡ€Π°Π²Π»Π΅Π½ΠΈΠ΅ двиТСния ΠΏΠΎΠ»ΠΎΠΆΠΈΡ‚Π΅Π»ΡŒΠ½Ρ‹Ρ… зарядов.

ΠŸΠΎΠΏΡ€ΠΎΡΡ‚Ρƒ говоря, ΠΏΠΎ соглашСнию Ρ‚ΠΎΠΊ Ρ‚Π΅Ρ‡Ρ‘Ρ‚ ΠΎΡ‚ «плюса» ΠΊ «минусу» (рис. 1; ΠΏΠΎΠ»ΠΎΠΆΠΈΡ‚Π΅Π»ΡŒΠ½Π°Ρ ΠΊΠ»Π΅ΠΌΠΌΠ° источника Ρ‚ΠΎΠΊΠ° ΠΈΠ·ΠΎΠ±Ρ€Π°ΠΆΠ΅Π½Π° Π΄Π»ΠΈΠ½Π½ΠΎΠΉ Ρ‡Π΅Ρ€Ρ‚ΠΎΠΉ, ΠΎΡ‚Ρ€ΠΈΡ†Π°Ρ‚Π΅Π»ΡŒΠ½Π°Ρ ΠΊΠ»Π΅ΠΌΠΌΠ° β€” ΠΊΠΎΡ€ΠΎΡ‚ΠΊΠΎΠΉ).

Рис. 1. НаправлСниС Ρ‚ΠΎΠΊΠ°

Π”Π°Π½Π½ΠΎΠ΅ соглашСниС вступаСт Π² Π½Π΅ΠΊΠΎΡ‚ΠΎΡ€ΠΎΠ΅ ΠΏΡ€ΠΎΡ‚ΠΈΠ²ΠΎΡ€Π΅Ρ‡ΠΈΠ΅ с Π½Π°ΠΈΠ±ΠΎΠ»Π΅Π΅ распространённым случаСм мСталличСских ΠΏΡ€ΠΎΠ²ΠΎΠ΄Π½ΠΈΠΊΠΎΠ². Π’ ΠΌΠ΅Ρ‚Π°Π»Π»Π΅ носитСлями заряда ΡΠ²Π»ΡΡŽΡ‚ΡΡ свободныС элСктроны, ΠΈ Π΄Π²ΠΈΠ³Π°ΡŽΡ‚ΡΡ ΠΎΠ½ΠΈ ΠΎΡ‚ «минуса» ΠΊ Β«ΠΏΠ»ΡŽΡΡƒΒ». Но Π² соотвСтствии с соглашСниСм ΠΌΡ‹ Π²Ρ‹Π½ΡƒΠΆΠ΄Π΅Π½Ρ‹ ΡΡ‡ΠΈΡ‚Π°Ρ‚ΡŒ, Ρ‡Ρ‚ΠΎ Π½Π°ΠΏΡ€Π°Π²Π»Π΅Π½ΠΈΠ΅ Ρ‚ΠΎΠΊΠ° Π² мСталличСском ΠΏΡ€ΠΎΠ²ΠΎΠ΄Π½ΠΈΠΊΠ΅ ΠΏΡ€ΠΎΡ‚ΠΈΠ²ΠΎΠΏΠΎΠ»ΠΎΠΆΠ½ΠΎ двиТСнию свободных элСктронов. Π­Ρ‚ΠΎ, ΠΊΠΎΠ½Π΅Ρ‡Π½ΠΎ, Π½Π΅ ΠΎΡ‡Π΅Π½ΡŒ ΡƒΠ΄ΠΎΠ±Π½ΠΎ.

Π’ΡƒΡ‚, ΠΎΠ΄Π½Π°ΠΊΠΎ, Π½ΠΈΡ‡Π΅Π³ΠΎ Π½Π΅ подСлаСшь β€” придётся ΠΏΡ€ΠΈΠ½ΡΡ‚ΡŒ эту ΡΠΈΡ‚ΡƒΠ°Ρ†ΠΈΡŽ ΠΊΠ°ΠΊ Π΄Π°Π½Π½ΠΎΡΡ‚ΡŒ. Π’Π°ΠΊ ΡƒΠΆ историчСски слоТилось. Π’Ρ‹Π±ΠΎΡ€ направлСния Ρ‚ΠΎΠΊΠ° Π±Ρ‹Π» ΠΏΡ€Π΅Π΄Π»ΠΎΠΆΠ΅Π½ АмпСром (Π΄ΠΎΠ³ΠΎΠ²ΠΎΡ€Ρ‘Π½Π½ΠΎΡΡ‚ΡŒ ΠΎ Π½Π°ΠΏΡ€Π°Π²Π»Π΅Π½ΠΈΠΈ Ρ‚ΠΎΠΊΠ° понадобилась АмпСру для Ρ‚ΠΎΠ³ΠΎ, Ρ‡Ρ‚ΠΎΠ±Ρ‹ Π΄Π°Ρ‚ΡŒ Ρ‡Ρ‘Ρ‚ΠΊΠΎΠ΅ ΠΏΡ€Π°Π²ΠΈΠ»ΠΎ опрСдСлСния направлСния силы, Π΄Π΅ΠΉΡΡ‚Π²ΡƒΡŽΡ‰Π΅ΠΉ Π½Π° ΠΏΡ€ΠΎΠ²ΠΎΠ΄Π½ΠΈΠΊ с Ρ‚ΠΎΠΊΠΎΠΌ Π² ΠΌΠ°Π³Π½ΠΈΡ‚Π½ΠΎΠΌ ΠΏΠΎΠ»Π΅. БСгодня эту силу ΠΌΡ‹ Π½Π°Π·Ρ‹Π²Π°Π΅ΠΌ силой АмпСра, Π½Π°ΠΏΡ€Π°Π²Π»Π΅Π½ΠΈΠ΅ ΠΊΠΎΡ‚ΠΎΡ€ΠΎΠΉ опрСдСляСтся ΠΏΠΎ ΠΏΡ€Π°Π²ΠΈΠ»Ρƒ Π»Π΅Π²ΠΎΠΉ Ρ€ΡƒΠΊΠΈ) Π² ΠΏΠ΅Ρ€Π²ΠΎΠΉ ΠΏΠΎΠ»ΠΎΠ²ΠΈΠ½Π΅ XIX Π²Π΅ΠΊΠ°, Π·Π° 70 Π»Π΅Ρ‚ Π΄ΠΎ открытия элСктрона. К этому Π²Ρ‹Π±ΠΎΡ€Ρƒ всС ΠΏΡ€ΠΈΠ²Ρ‹ΠΊΠ»ΠΈ, ΠΈ ΠΊΠΎΠ³Π΄Π° Π² 1916 Π³ΠΎΠ΄Ρƒ Π²Ρ‹ΡΡΠ½ΠΈΠ»ΠΎΡΡŒ, Ρ‡Ρ‚ΠΎ Ρ‚ΠΎΠΊ Π² ΠΌΠ΅Ρ‚Π°Π»Π»Π°Ρ… Π²Ρ‹Π·Π²Π°Π½ Π΄Π²ΠΈΠΆΠ΅Π½ΠΈΠ΅ΠΌ свободных элСктронов, Π½ΠΈΡ‡Π΅Π³ΠΎ ΠΌΠ΅Π½ΡΡ‚ΡŒ ΡƒΠΆΠ΅ Π½Π΅ стали.

Β 

ДСйствия элСктричСского Ρ‚ΠΎΠΊΠ°

Β 

Как ΠΌΡ‹ ΠΌΠΎΠΆΠ΅ΠΌ ΠΎΠΏΡ€Π΅Π΄Π΅Π»ΠΈΡ‚ΡŒ, ΠΏΡ€ΠΎΡ‚Π΅ΠΊΠ°Π΅Ρ‚ элСктричСский Ρ‚ΠΎΠΊ ΠΈΠ»ΠΈ Π½Π΅Ρ‚? О Π²ΠΎΠ·Π½ΠΈΠΊΠ½ΠΎΠ²Π΅Π½ΠΈΠΈ элСктричСского Ρ‚ΠΎΠΊΠ° ΠΌΠΎΠΆΠ½ΠΎ ΡΡƒΠ΄ΠΈΡ‚ΡŒ ΠΏΠΎ ΡΠ»Π΅Π΄ΡƒΡŽΡ‰ΠΈΠΌ Π΅Π³ΠΎ проявлСниям.

1. Π’Π΅ΠΏΠ»ΠΎΠ²ΠΎΠ΅ дСйствиС Ρ‚ΠΎΠΊΠ°. ЭлСктричСский Ρ‚ΠΎΠΊ Π²Ρ‹Π·Ρ‹Π²Π°Π΅Ρ‚ Π½Π°Π³Ρ€Π΅Π²Π°Π½ΠΈΠ΅ вСщСства, Π² ΠΊΠΎΡ‚ΠΎΡ€ΠΎΠΌ ΠΎΠ½ ΠΏΡ€ΠΎΡ‚Π΅ΠΊΠ°Π΅Ρ‚. ИмСнно Ρ‚Π°ΠΊ Π½Π°Π³Ρ€Π΅Π²Π°ΡŽΡ‚ΡΡ спирали Π½Π°Π³Ρ€Π΅Π²Π°Ρ‚Π΅Π»ΡŒΠ½Ρ‹Ρ… ΠΏΡ€ΠΈΠ±ΠΎΡ€ΠΎΠ² ΠΈ Π»Π°ΠΌΠΏ накаливания. ИмСнно поэтому ΠΌΡ‹ Π²ΠΈΠ΄ΠΈΠΌ молнию. Π’ основС дСйствия Ρ‚Π΅ΠΏΠ»ΠΎΠ²Ρ‹Ρ… Π°ΠΌΠΏΠ΅Ρ€ΠΌΠ΅Ρ‚Ρ€ΠΎΠ² Π»Π΅ΠΆΠΈΡ‚ Ρ‚Π΅ΠΏΠ»ΠΎΠ²ΠΎΠ΅ Ρ€Π°ΡΡˆΠΈΡ€Π΅Π½ΠΈΠ΅ ΠΏΡ€ΠΎΠ²ΠΎΠ΄Π½ΠΈΠΊΠ° с Ρ‚ΠΎΠΊΠΎΠΌ, приводящСС ΠΊ ΠΏΠ΅Ρ€Π΅ΠΌΠ΅Ρ‰Π΅Π½ΠΈΡŽ стрСлки ΠΏΡ€ΠΈΠ±ΠΎΡ€Π°.

2. ΠœΠ°Π³Π½ΠΈΡ‚Π½ΠΎΠ΅ дСйствиС Ρ‚ΠΎΠΊΠ°. ЭлСктричСский Ρ‚ΠΎΠΊ создаёт ΠΌΠ°Π³Π½ΠΈΡ‚Π½ΠΎΠ΅ ΠΏΠΎΠ»Π΅: стрСлка компаса, располоТСнная рядом с ΠΏΡ€ΠΎΠ²ΠΎΠ΄ΠΎΠΌ, ΠΏΡ€ΠΈ Π²ΠΊΠ»ΡŽΡ‡Π΅Π½ΠΈΠΈ Ρ‚ΠΎΠΊΠ° поворачиваСтся пСрпСндикулярно ΠΏΡ€ΠΎΠ²ΠΎΠ΄Ρƒ. ΠœΠ°Π³Π½ΠΈΡ‚Π½ΠΎΠ΅ ΠΏΠΎΠ»Π΅ Ρ‚ΠΎΠΊΠ° ΠΌΠΎΠΆΠ½ΠΎ ΠΌΠ½ΠΎΠ³ΠΎΠΊΡ€Π°Ρ‚Π½ΠΎ ΡƒΡΠΈΠ»ΠΈΡ‚ΡŒ, Ссли ΠΎΠ±ΠΌΠΎΡ‚Π°Ρ‚ΡŒ ΠΏΡ€ΠΎΠ²ΠΎΠ΄ Π²ΠΎΠΊΡ€ΡƒΠ³ ΠΆΠ΅Π»Π΅Π·Π½ΠΎΠ³ΠΎ стСрТня β€” получится элСктромагнит. На этом ΠΏΡ€ΠΈΠ½Ρ†ΠΈΠΏΠ΅ основано дСйствиС Π°ΠΌΠΏΠ΅Ρ€ΠΌΠ΅Ρ‚Ρ€ΠΎΠ² магнитоэлСктричСской систСмы: элСктромагнит поворачиваСтся Π² ΠΏΠΎΠ»Π΅ постоянного ΠΌΠ°Π³Π½ΠΈΡ‚Π°, Π² Ρ€Π΅Π·ΡƒΠ»ΡŒΡ‚Π°Ρ‚Π΅ Ρ‡Π΅Π³ΠΎ стрСлка ΠΏΡ€ΠΈΠ±ΠΎΡ€Π° пСрСмСщаСтся ΠΏΠΎ шкалС.

3. Π₯имичСскоС дСйствиС Ρ‚ΠΎΠΊΠ°. ΠŸΡ€ΠΈ ΠΏΡ€ΠΎΡ…ΠΎΠΆΠ΄Π΅Π½ΠΈΠΈ Ρ‚ΠΎΠΊΠ° Ρ‡Π΅Ρ€Π΅Π· элСктролиты ΠΌΠΎΠΆΠ½ΠΎ Π½Π°Π±Π»ΡŽΠ΄Π°Ρ‚ΡŒ ΠΈΠ·ΠΌΠ΅Π½Π΅Π½ΠΈΠ΅ химичСского состава вСщСства. Π’Π°ΠΊ, Π² растворС ΠΏΠΎΠ»ΠΎΠΆΠΈΡ‚Π΅Π»ΡŒΠ½Ρ‹Π΅ ΠΈΠΎΠ½Ρ‹ Π΄Π²ΠΈΠ³Π°ΡŽΡ‚ΡΡ ΠΊ ΠΎΡ‚Ρ€ΠΈΡ†Π°Ρ‚Π΅Π»ΡŒΠ½ΠΎΠΌΡƒ элСктроду, ΠΈ этот элСктрод покрываСтся мСдью.

ЭлСктричСский Ρ‚ΠΎΠΊ называСтся постоянным, Ссли Π·Π° Ρ€Π°Π²Π½Ρ‹Π΅ ΠΏΡ€ΠΎΠΌΠ΅ΠΆΡƒΡ‚ΠΊΠΈ Π²Ρ€Π΅ΠΌΠ΅Π½ΠΈ Ρ‡Π΅Ρ€Π΅Π· ΠΏΠΎΠΏΠ΅Ρ€Π΅Ρ‡Π½ΠΎΠ΅ сСчСниС ΠΏΡ€ΠΎΠ²ΠΎΠ΄Π½ΠΈΠΊΠ° ΠΏΡ€ΠΎΡ…ΠΎΠ΄ΠΈΡ‚ ΠΎΠ΄ΠΈΠ½Π°ΠΊΠΎΠ²Ρ‹ΠΉ заряд.

ΠŸΠΎΡΡ‚ΠΎΡΠ½Π½Ρ‹ΠΉ Ρ‚ΠΎΠΊ Π½Π°ΠΈΠ±ΠΎΠ»Π΅Π΅ прост для изучСния. Π‘ Π½Π΅Π³ΠΎ ΠΌΡ‹ ΠΈ Π½Π°Ρ‡ΠΈΠ½Π°Π΅ΠΌ.

Β 

Π‘ΠΈΠ»Π° ΠΈ ΠΏΠ»ΠΎΡ‚Π½ΠΎΡΡ‚ΡŒ Ρ‚ΠΎΠΊΠ°

Β 

ΠšΠΎΠ»ΠΈΡ‡Π΅ΡΡ‚Π²Π΅Π½Π½ΠΎΠΉ характСристикой элСктричСского Ρ‚ΠΎΠΊΠ° являСтся сила Ρ‚ΠΎΠΊΠ°. Π’ случаС постоянного Ρ‚ΠΎΠΊΠ° Π°Π±ΡΠΎΠ»ΡŽΡ‚Π½Π°Ρ Π²Π΅Π»ΠΈΡ‡ΠΈΠ½Π° силы Ρ‚ΠΎΠΊΠ° Π΅ΡΡ‚ΡŒ ΠΎΡ‚Π½ΠΎΡˆΠ΅Π½ΠΈΠ΅ Π°Π±ΡΠΎΠ»ΡŽΡ‚Π½ΠΎΠΉ Π²Π΅Π»ΠΈΡ‡ΠΈΠ½Ρ‹ заряда , ΠΏΡ€ΠΎΡˆΠ΅Π΄ΡˆΠ΅Π³ΠΎ Ρ‡Π΅Ρ€Π΅Π· ΠΏΠΎΠΏΠ΅Ρ€Π΅Ρ‡Π½ΠΎΠ΅ сСчСниС ΠΏΡ€ΠΎΠ²ΠΎΠ΄Π½ΠΈΠΊΠ° Π·Π° врСмя , ΠΊ этому самому Π²Ρ€Π΅ΠΌΠ΅Π½ΠΈ:

(1)

Π˜Π·ΠΌΠ΅Ρ€ΡΠ΅Ρ‚ΡΡ сила Ρ‚ΠΎΠΊΠ° Π² Π°ΠΌΠΏΠ΅Ρ€Π°Ρ… (A). ΠŸΡ€ΠΈ силС Ρ‚ΠΎΠΊΠ° Π² А Ρ‡Π΅Ρ€Π΅Π· ΠΏΠΎΠΏΠ΅Ρ€Π΅Ρ‡Π½ΠΎΠ΅ сСчСниС ΠΏΡ€ΠΎΠ²ΠΎΠ΄Π½ΠΈΠΊΠ° Π·Π° с ΠΏΡ€ΠΎΡ…ΠΎΠ΄ΠΈΡ‚ заряд Π² Кл.

ΠŸΠΎΠ΄Ρ‡Π΅Ρ€ΠΊΠ½Ρ‘ΠΌ, Ρ‡Ρ‚ΠΎ Ρ„ΠΎΡ€ΠΌΡƒΠ»Π° (1) опрСдСляСт Π°Π±ΡΠΎΠ»ΡŽΡ‚Π½ΡƒΡŽ Π²Π΅Π»ΠΈΡ‡ΠΈΠ½Ρƒ, ΠΈΠ»ΠΈ ΠΌΠΎΠ΄ΡƒΠ»ΡŒ силы Ρ‚ΠΎΠΊΠ°.
Π‘ΠΈΠ»Π° Ρ‚ΠΎΠΊΠ° ΠΌΠΎΠΆΠ΅Ρ‚ ΠΈΠΌΠ΅Ρ‚ΡŒ Π΅Ρ‰Ρ‘ ΠΈ Π·Π½Π°ΠΊ! Π­Ρ‚ΠΎΡ‚ Π·Π½Π°ΠΊ Π½Π΅ связан со Π·Π½Π°ΠΊΠΎΠΌ зарядов, ΠΎΠ±Ρ€Π°Π·ΡƒΡŽΡ‰ΠΈΡ… Ρ‚ΠΎΠΊ, ΠΈ выбираСтся ΠΈΠ· ΠΈΠ½Ρ‹Ρ… сообраТСний. А ΠΈΠΌΠ΅Π½Π½ΠΎ, Π² рядС ситуаций (Π½Π°ΠΏΡ€ΠΈΠΌΠ΅Ρ€, Ссли Π·Π°Ρ€Π°Π½Π΅Π΅ Π½Π΅ ясно, ΠΊΡƒΠ΄Π° ΠΏΠΎΡ‚Π΅Ρ‡Ρ‘Ρ‚ Ρ‚ΠΎΠΊ) ΡƒΠ΄ΠΎΠ±Π½ΠΎ Π·Π°Ρ„ΠΈΠΊΡΠΈΡ€ΠΎΠ²Π°Ρ‚ΡŒ Π½Π΅ΠΊΠΎΡ‚ΠΎΡ€ΠΎΠ΅ Π½Π°ΠΏΡ€Π°Π²Π»Π΅Π½ΠΈΠ΅ ΠΎΠ±Ρ…ΠΎΠ΄Π° Ρ†Π΅ΠΏΠΈ (скаТСм, ΠΏΡ€ΠΎΡ‚ΠΈΠ² часовой стрСлки) ΠΈ ΡΡ‡ΠΈΡ‚Π°Ρ‚ΡŒ силу Ρ‚ΠΎΠΊΠ° ΠΏΠΎΠ»ΠΎΠΆΠΈΡ‚Π΅Π»ΡŒΠ½ΠΎΠΉ, Ссли Π½Π°ΠΏΡ€Π°Π²Π»Π΅Π½ΠΈΠ΅ Ρ‚ΠΎΠΊΠ° совпадаСт с Π½Π°ΠΏΡ€Π°Π²Π»Π΅Π½ΠΈΠ΅ΠΌ ΠΎΠ±Ρ…ΠΎΠ΄Π°, ΠΈ ΠΎΡ‚Ρ€ΠΈΡ†Π°Ρ‚Π΅Π»ΡŒΠ½ΠΎΠΉ, Ссли Ρ‚ΠΎΠΊ Ρ‚Π΅Ρ‡Ρ‘Ρ‚ ΠΏΡ€ΠΎΡ‚ΠΈΠ² направлСния ΠΎΠ±Ρ…ΠΎΠ΄Π° (сравнитС с тригономСтричСским ΠΊΡ€ΡƒΠ³ΠΎΠΌ: ΡƒΠ³Π»Ρ‹ ΡΡ‡ΠΈΡ‚Π°ΡŽΡ‚ΡΡ ΠΏΠΎΠ»ΠΎΠΆΠΈΡ‚Π΅Π»ΡŒΠ½Ρ‹ΠΌΠΈ, Ссли ΠΎΡ‚ΡΡ‡ΠΈΡ‚Ρ‹Π²Π°ΡŽΡ‚ΡΡ ΠΏΡ€ΠΎΡ‚ΠΈΠ² часовой стрСлки, ΠΈ ΠΎΡ‚Ρ€ΠΈΡ†Π°Ρ‚Π΅Π»ΡŒΠ½Ρ‹ΠΌΠΈ, Ссли ΠΏΠΎ часовой стрСлкС).

Π’ случаС постоянного Ρ‚ΠΎΠΊΠ° сила Ρ‚ΠΎΠΊΠ° Π΅ΡΡ‚ΡŒ Π²Π΅Π»ΠΈΡ‡ΠΈΠ½Π° постоянная. Она ΠΏΠΎΠΊΠ°Π·Ρ‹Π²Π°Π΅Ρ‚, ΠΊΠ°ΠΊΠΎΠΉ заряд ΠΏΡ€ΠΎΡ…ΠΎΠ΄ΠΈΡ‚ Ρ‡Π΅Ρ€Π΅Π· ΠΏΠΎΠΏΠ΅Ρ€Π΅Ρ‡Π½ΠΎΠ΅ сСчСниС ΠΏΡ€ΠΎΠ²ΠΎΠ΄Π½ΠΈΠΊΠ° Π·Π° с.

Часто Π±Ρ‹Π²Π°Π΅Ρ‚ ΡƒΠ΄ΠΎΠ±Π½ΠΎ Π½Π΅ ΡΠ²ΡΠ·Ρ‹Π²Π°Ρ‚ΡŒΡΡ с ΠΏΠ»ΠΎΡ‰Π°Π΄ΡŒΡŽ ΠΏΠΎΠΏΠ΅Ρ€Π΅Ρ‡Π½ΠΎΠ³ΠΎ сСчСния ΠΈ ввСсти Π²Π΅Π»ΠΈΡ‡ΠΈΠ½Ρƒ плотности Ρ‚ΠΎΠΊΠ°:

(2)

Π³Π΄Π΅ β€” сила Ρ‚ΠΎΠΊΠ°, β€” ΠΏΠ»ΠΎΡ‰Π°Π΄ΡŒ ΠΏΠΎΠΏΠ΅Ρ€Π΅Ρ‡Π½ΠΎΠ³ΠΎ сСчСния ΠΏΡ€ΠΎΠ²ΠΎΠ΄Π½ΠΈΠΊΠ° (разумССтся, это сСчСниС пСрпСндикулярно Π½Π°ΠΏΡ€Π°Π²Π»Π΅Π½ΠΈΡŽ Ρ‚ΠΎΠΊΠ°). Π‘ ΡƒΡ‡Ρ‘Ρ‚ΠΎΠΌ Ρ„ΠΎΡ€ΠΌΡƒΠ»Ρ‹ (1) ΠΈΠΌΠ΅Π΅ΠΌ Ρ‚Π°ΠΊΠΆΠ΅:

ΠŸΠ»ΠΎΡ‚Π½ΠΎΡΡ‚ΡŒ Ρ‚ΠΎΠΊΠ° ΠΏΠΎΠΊΠ°Π·Ρ‹Π²Π°Π΅Ρ‚, ΠΊΠ°ΠΊΠΎΠΉ заряд ΠΏΡ€ΠΎΡ…ΠΎΠ΄ΠΈΡ‚ Π·Π° Π΅Π΄ΠΈΠ½ΠΈΡ†Ρƒ Π²Ρ€Π΅ΠΌΠ΅Π½ΠΈ Ρ‡Π΅Ρ€Π΅Π· Π΅Π΄ΠΈΠ½ΠΈΡ†Ρƒ ΠΏΠ»ΠΎΡ‰Π°Π΄ΠΈ ΠΏΠΎΠΏΠ΅Ρ€Π΅Ρ‡Π½ΠΎΠ³ΠΎ сСчСния ΠΏΡ€ΠΎΠ²ΠΎΠ΄Π½ΠΈΠΊΠ°. Богласно Ρ„ΠΎΡ€ΠΌΡƒΠ»Π΅ (2), ΠΏΠ»ΠΎΡ‚Π½ΠΎΡΡ‚ΡŒ Ρ‚ΠΎΠΊΠ° измСряСтся Π² А/ΠΌ2.

Β 

Π‘ΠΊΠΎΡ€ΠΎΡΡ‚ΡŒ Π½Π°ΠΏΡ€Π°Π²Π»Π΅Π½Π½ΠΎΠ³ΠΎ двиТСния зарядов

Β 

Когда ΠΌΡ‹ Π²ΠΊΠ»ΡŽΡ‡Π°Π΅ΠΌ Π² ΠΊΠΎΠΌΠ½Π°Ρ‚Π΅ свСт, Π½Π°ΠΌ каТСтся, Ρ‡Ρ‚ΠΎ Π»Π°ΠΌΠΏΠΎΡ‡ΠΊΠ° загораСтся ΠΌΠ³Π½ΠΎΠ²Π΅Π½Π½ΠΎ. Π‘ΠΊΠΎΡ€ΠΎΡΡ‚ΡŒ распространСния Ρ‚ΠΎΠΊΠ° ΠΏΠΎ ΠΏΡ€ΠΎΠ²ΠΎΠ΄Π°ΠΌ ΠΎΡ‡Π΅Π½ΡŒ Π²Π΅Π»ΠΈΠΊΠ°: ΠΎΠ½Π° Π±Π»ΠΈΠ·ΠΊΠ° ΠΊ ΠΊΠΌ/с (скорости свСта Π² Π²Π°ΠΊΡƒΡƒΠΌΠ΅). Если Π±Ρ‹ Π»Π°ΠΌΠΏΠΎΡ‡ΠΊΠ° Π½Π°Ρ…ΠΎΠ΄ΠΈΠ»Π°ΡΡŒ Π½Π° Π›ΡƒΠ½Π΅, ΠΎΠ½Π° заТглась Π±Ρ‹ Ρ‡Π΅Ρ€Π΅Π· сСкунду с нСбольшим.

Однако Π½Π΅ слСдуСт Π΄ΡƒΠΌΠ°Ρ‚ΡŒ, Ρ‡Ρ‚ΠΎ с Ρ‚Π°ΠΊΠΎΠΉ Π³Ρ€Π°Π½Π΄ΠΈΠΎΠ·Π½ΠΎΠΉ ΡΠΊΠΎΡ€ΠΎΡΡ‚ΡŒΡŽ Π΄Π²ΠΈΠ³Π°ΡŽΡ‚ΡΡ свободныС заряды, ΠΎΠ±Ρ€Π°Π·ΡƒΡŽΡ‰ΠΈΠ΅ Ρ‚ΠΎΠΊ. ΠžΠΊΠ°Π·Ρ‹Π²Π°Π΅Ρ‚ΡΡ, ΠΈΡ… ΡΠΊΠΎΡ€ΠΎΡΡ‚ΡŒ составляСт всСго-навсСго Π΄ΠΎΠ»ΠΈ ΠΌΠΈΠ»Π»ΠΈΠΌΠ΅Ρ‚Ρ€Π° Π² сСкунду.

ΠŸΠΎΡ‡Π΅ΠΌΡƒ ΠΆΠ΅ Ρ‚ΠΎΠΊ распространяСтся ΠΏΠΎ ΠΏΡ€ΠΎΠ²ΠΎΠ΄Π°ΠΌ Ρ‚Π°ΠΊ быстро? Π”Π΅Π»ΠΎ Π² Ρ‚ΠΎΠΌ, Ρ‡Ρ‚ΠΎ свободныС заряды Π²Π·Π°ΠΈΠΌΠΎΠ΄Π΅ΠΉΡΡ‚Π²ΡƒΡŽΡ‚ Π΄Ρ€ΡƒΠ³ с Π΄Ρ€ΡƒΠ³ΠΎΠΌ ΠΈ, Π½Π°Ρ…ΠΎΠ΄ΡΡΡŒ ΠΏΠΎΠ΄ дСйствиСм элСктричСского поля источника Ρ‚ΠΎΠΊΠ°, ΠΏΡ€ΠΈ Π·Π°ΠΌΡ‹ΠΊΠ°Π½ΠΈΠΈ Ρ†Π΅ΠΏΠΈ приходят Π² Π΄Π²ΠΈΠΆΠ΅Π½ΠΈΠ΅ ΠΏΠΎΡ‡Ρ‚ΠΈ ΠΎΠ΄Π½ΠΎΠ²Ρ€Π΅ΠΌΠ΅Π½Π½ΠΎ вдоль всСго ΠΏΡ€ΠΎΠ²ΠΎΠ΄Π½ΠΈΠΊΠ°. Π‘ΠΊΠΎΡ€ΠΎΡΡ‚ΡŒ распространСния Ρ‚ΠΎΠΊΠ° Π΅ΡΡ‚ΡŒ ΡΠΊΠΎΡ€ΠΎΡΡ‚ΡŒ ΠΏΠ΅Ρ€Π΅Π΄Π°Ρ‡ΠΈ элСктричСского взаимодСйствия ΠΌΠ΅ΠΆΠ΄Ρƒ свободными зарядами, ΠΈ ΠΎΠ½Π° Π±Π»ΠΈΠ·ΠΊΠ° ΠΊ скорости свСта Π² Π²Π°ΠΊΡƒΡƒΠΌΠ΅. Π‘ΠΊΠΎΡ€ΠΎΡΡ‚ΡŒ ΠΆΠ΅, с ΠΊΠΎΡ‚ΠΎΡ€ΠΎΠΉ сами заряды ΠΏΠ΅Ρ€Π΅ΠΌΠ΅Ρ‰Π°ΡŽΡ‚ΡΡ Π²Π½ΡƒΡ‚Ρ€ΠΈ ΠΏΡ€ΠΎΠ²ΠΎΠ΄Π½ΠΈΠΊΠ°, ΠΌΠΎΠΆΠ΅Ρ‚ Π±Ρ‹Ρ‚ΡŒ Π½Π° ΠΌΠ½ΠΎΠ³ΠΎ порядков мСньшС.

Π˜Ρ‚Π°ΠΊ, ΠΏΠΎΠ΄Ρ‡Π΅Ρ€ΠΊΠ½Ρ‘ΠΌ Π΅Ρ‰Ρ‘ Ρ€Π°Π·, Ρ‡Ρ‚ΠΎ ΠΌΡ‹ Ρ€Π°Π·Π»ΠΈΡ‡Π°Π΅ΠΌ Π΄Π²Π΅ скорости.

1. Π‘ΠΊΠΎΡ€ΠΎΡΡ‚ΡŒ распространСния Ρ‚ΠΎΠΊΠ°. Π­Ρ‚ΠΎ β€” ΡΠΊΠΎΡ€ΠΎΡΡ‚ΡŒ ΠΏΠ΅Ρ€Π΅Π΄Π°Ρ‡ΠΈ элСктричСского сигнала ΠΏΠΎ Ρ†Π΅ΠΏΠΈ. Π‘Π»ΠΈΠ·ΠΊΠ° ΠΊ ΠΊΠΌ/с.

2. Π‘ΠΊΠΎΡ€ΠΎΡΡ‚ΡŒ Π½Π°ΠΏΡ€Π°Π²Π»Π΅Π½Π½ΠΎΠ³ΠΎ двиТСния свободных зарядов. Π­Ρ‚ΠΎ β€” срСдняя ΡΠΊΠΎΡ€ΠΎΡΡ‚ΡŒ пСрСмСщСния зарядов, ΠΎΠ±Ρ€Π°Π·ΡƒΡŽΡ‰ΠΈΡ… Ρ‚ΠΎΠΊ. НазываСтся Π΅Ρ‰Ρ‘ ΡΠΊΠΎΡ€ΠΎΡΡ‚ΡŒΡŽ Π΄Ρ€Π΅ΠΉΡ„Π°.

ΠœΡ‹ сСйчас Π²Ρ‹Π²Π΅Π΄Π΅ΠΌ Ρ„ΠΎΡ€ΠΌΡƒΠ»Ρƒ, Π²Ρ‹Ρ€Π°ΠΆΠ°ΡŽΡ‰ΡƒΡŽ силу Ρ‚ΠΎΠΊΠ° Ρ‡Π΅Ρ€Π΅Π· ΡΠΊΠΎΡ€ΠΎΡΡ‚ΡŒ Π½Π°ΠΏΡ€Π°Π²Π»Π΅Π½Π½ΠΎΠ³ΠΎ двиТСния зарядов ΠΏΡ€ΠΎΠ²ΠΎΠ΄Π½ΠΈΠΊΠ°.

ΠŸΡƒΡΡ‚ΡŒ ΠΏΡ€ΠΎΠ²ΠΎΠ΄Π½ΠΈΠΊ ΠΈΠΌΠ΅Π΅Ρ‚ ΠΏΠ»ΠΎΡ‰Π°Π΄ΡŒ ΠΏΠΎΠΏΠ΅Ρ€Π΅Ρ‡Π½ΠΎΠ³ΠΎ сСчСния (рис. 2). Π‘Π²ΠΎΠ±ΠΎΠ΄Π½Ρ‹Π΅ заряды ΠΏΡ€ΠΎΠ²ΠΎΠ΄Π½ΠΈΠΊΠ° Π±ΡƒΠ΄Π΅ΠΌ ΡΡ‡ΠΈΡ‚Π°Ρ‚ΡŒ ΠΏΠΎΠ»ΠΎΠΆΠΈΡ‚Π΅Π»ΡŒΠ½Ρ‹ΠΌΠΈ; Π²Π΅Π»ΠΈΡ‡ΠΈΠ½Ρƒ свободного заряда ΠΎΠ±ΠΎΠ·Π½Π°Ρ‡ΠΈΠΌ (Π² Π½Π°ΠΈΠ±ΠΎΠ»Π΅Π΅ Π²Π°ΠΆΠ½ΠΎΠΌ для ΠΏΡ€Π°ΠΊΡ‚ΠΈΠΊΠΈ случая мСталличСского ΠΏΡ€ΠΎΠ²ΠΎΠ΄Π½ΠΈΠΊΠ° это Π΅ΡΡ‚ΡŒ заряд элСктрона). ΠšΠΎΠ½Ρ†Π΅Π½Ρ‚Ρ€Π°Ρ†ΠΈΡ свободных зарядов (Ρ‚. Π΅. ΠΈΡ… число Π² Π΅Π΄ΠΈΠ½ΠΈΡ†Π΅ ΠΎΠ±ΡŠΡ‘ΠΌΠ°) Ρ€Π°Π²Π½Π° .

Рис. 2. К Π²Ρ‹Π²ΠΎΠ΄Ρƒ Ρ„ΠΎΡ€ΠΌΡƒΠ»Ρ‹

Какой заряд ΠΏΡ€ΠΎΠΉΠ΄Ρ‘Ρ‚ Ρ‡Π΅Ρ€Π΅Π· ΠΏΠΎΠΏΠ΅Ρ€Π΅Ρ‡Π½ΠΎΠ΅ сСчСниС нашСго ΠΏΡ€ΠΎΠ²ΠΎΠ΄Π½ΠΈΠΊΠ° Π·Π° врСмя ?

Π‘ ΠΎΠ΄Π½ΠΎΠΉ стороны, разумССтся,

(3)

Π‘ Π΄Ρ€ΡƒΠ³ΠΎΠΉ стороны, сСчСниС пСрСсСкут всС Ρ‚Π΅ свободныС заряды, ΠΊΠΎΡ‚ΠΎΡ€Ρ‹Π΅ спустя врСмя окаТутся Π²Π½ΡƒΡ‚Ρ€ΠΈ Ρ†ΠΈΠ»ΠΈΠ½Π΄Ρ€Π° с высотой . Π˜Ρ… число Ρ€Π°Π²Π½ΠΎ:

Π‘Π»Π΅Π΄ΠΎΠ²Π°Ρ‚Π΅Π»ΡŒΠ½ΠΎ, ΠΈΡ… ΠΎΠ±Ρ‰ΠΈΠΉ заряд Π±ΡƒΠ΄Π΅Ρ‚ Ρ€Π°Π²Π΅Π½:

(4)

ΠŸΡ€ΠΈΡ€Π°Π²Π½ΠΈΠ²Π°Ρ ΠΏΡ€Π°Π²Ρ‹Π΅ части Ρ„ΠΎΡ€ΠΌΡƒΠ» (3) ΠΈ (4) ΠΈ сокращая Π½Π° , ΠΏΠΎΠ»ΡƒΡ‡ΠΈΠΌ:

(5)

БоотвСтствСнно, ΠΏΠ»ΠΎΡ‚Π½ΠΎΡΡ‚ΡŒ Ρ‚ΠΎΠΊΠ° оказываСтся Ρ€Π°Π²Π½Π°:

Π”Π°Π²Π°ΠΉΡ‚Π΅ Π² качСствС ΠΏΡ€ΠΈΠΌΠ΅Ρ€Π° посчитаСм, ΠΊΠ°ΠΊΠΎΠ²Π° ΡΠΊΠΎΡ€ΠΎΡΡ‚ΡŒ двиТСния свободных элСктронов Π² ΠΌΠ΅Π΄Π½ΠΎΠΌ ΠΏΡ€ΠΎΠ²ΠΎΠ΄Π΅ ΠΏΡ€ΠΈ силС Ρ‚ΠΎΠΊΠ° A.

Заряд элСктрона извСстСн: Кл.

Π§Π΅ΠΌΡƒ Ρ€Π°Π²Π½Π° концСнтрация свободных элСктронов? Она совпадаСт с ΠΊΠΎΠ½Ρ†Π΅Π½Ρ‚Ρ€Π°Ρ†ΠΈΠ΅ΠΉ Π°Ρ‚ΠΎΠΌΠΎΠ² ΠΌΠ΅Π΄ΠΈ, ΠΏΠΎΡΠΊΠΎΠ»ΡŒΠΊΡƒ ΠΎΡ‚ ΠΊΠ°ΠΆΠ΄ΠΎΠ³ΠΎ Π°Ρ‚ΠΎΠΌΠ° отщСпляСтся ΠΏΠΎ ΠΎΠ΄Π½ΠΎΠΌΡƒ Π²Π°Π»Π΅Π½Ρ‚Π½ΠΎΠΌΡƒ элСктрону. Ну Π° ΠΊΠΎΠ½Ρ†Π΅Π½Ρ‚Ρ€Π°Ρ†ΠΈΡŽ Π°Ρ‚ΠΎΠΌΠΎΠ² ΠΌΡ‹ Π½Π°Ρ…ΠΎΠ΄ΠΈΡ‚ΡŒ ΡƒΠΌΠ΅Π΅ΠΌ:

ΠΌ

ПолоТим ΠΌΠΌ . Из Ρ„ΠΎΡ€ΠΌΡƒΠ»Ρ‹ (5) ΠΏΠΎΠ»ΡƒΡ‡ΠΈΠΌ:

м/с.

Π­Ρ‚ΠΎ порядка ΠΎΠ΄Π½ΠΎΠΉ дСсятой ΠΌΠΈΠ»Π»ΠΈΠΌΠ΅Ρ‚Ρ€Π° Π² сСкунду.

Β 

Π‘Ρ‚Π°Ρ†ΠΈΠΎΠ½Π°Ρ€Π½ΠΎΠ΅ элСктричСскоС ΠΏΠΎΠ»Π΅

Β 

ΠœΡ‹ всё врСмя Π³ΠΎΠ²ΠΎΡ€ΠΈΠΌ ΠΎ Π½Π°ΠΏΡ€Π°Π²Π»Π΅Π½Π½ΠΎΠΌ Π΄Π²ΠΈΠΆΠ΅Π½ΠΈΠΈ зарядов, Π½ΠΎ Π΅Ρ‰Ρ‘ Π½Π΅ касались вопроса ΠΎ Ρ‚ΠΎΠΌ, ΠΏΠΎΡ‡Π΅ΠΌΡƒ свободныС заряды ΡΠΎΠ²Π΅Ρ€ΡˆΠ°ΡŽΡ‚ Ρ‚Π°ΠΊΠΎΠ΅ Π΄Π²ΠΈΠΆΠ΅Π½ΠΈΠ΅. ΠŸΠΎΡ‡Π΅ΠΌΡƒ, собствСнно, Π²ΠΎΠ·Π½ΠΈΠΊΠ°Π΅Ρ‚ элСктричСский Ρ‚ΠΎΠΊ?

Для упорядочСнного пСрСмСщСния зарядов Π²Π½ΡƒΡ‚Ρ€ΠΈ ΠΏΡ€ΠΎΠ²ΠΎΠ΄Π½ΠΈΠΊΠ° Π½Π΅ΠΎΠ±Ρ…ΠΎΠ΄ΠΈΠΌΠ° сила, Π΄Π΅ΠΉΡΡ‚Π²ΡƒΡŽΡ‰Π°Ρ Π½Π° заряды Π² ΠΎΠΏΡ€Π΅Π΄Π΅Π»Ρ‘Π½Π½ΠΎΠΌ Π½Π°ΠΏΡ€Π°Π²Π»Π΅Π½ΠΈΠΈ. ΠžΡ‚ΠΊΡƒΠ΄Π° бСрётся эта сила? Π‘ΠΎ стороны элСктричСского поля!

Π§Ρ‚ΠΎΠ±Ρ‹ Π² ΠΏΡ€ΠΎΠ²ΠΎΠ΄Π½ΠΈΠΊΠ΅ ΠΏΡ€ΠΎΡ‚Π΅ΠΊΠ°Π» постоянный Ρ‚ΠΎΠΊ, Π²Π½ΡƒΡ‚Ρ€ΠΈ ΠΏΡ€ΠΎΠ²ΠΎΠ΄Π½ΠΈΠΊΠ° Π΄ΠΎΠ»ΠΆΠ½ΠΎ ΡΡƒΡ‰Π΅ΡΡ‚Π²ΠΎΠ²Π°Ρ‚ΡŒ стационарноС (Ρ‚ΠΎ Π΅ΡΡ‚ΡŒ β€” постоянноС, Π½Π΅ зависящСС ΠΎΡ‚ Π²Ρ€Π΅ΠΌΠ΅Π½ΠΈ) элСктричСскоС ΠΏΠΎΠ»Π΅. Π˜Π½Ρ‹ΠΌΠΈ словами, ΠΌΠ΅ΠΆΠ΄Ρƒ ΠΊΠΎΠ½Ρ†Π°ΠΌΠΈ ΠΏΡ€ΠΎΠ²ΠΎΠ΄Π½ΠΈΠΊΠ° Π½ΡƒΠΆΠ½ΠΎ ΠΏΠΎΠ΄Π΄Π΅Ρ€ΠΆΠΈΠ²Π°Ρ‚ΡŒ ΠΏΠΎΡΡ‚ΠΎΡΠ½Π½ΡƒΡŽ Ρ€Π°Π·Π½ΠΎΡΡ‚ΡŒ ΠΏΠΎΡ‚Π΅Π½Ρ†ΠΈΠ°Π»ΠΎΠ².

Π‘Ρ‚Π°Ρ†ΠΈΠΎΠ½Π°Ρ€Π½ΠΎΠ΅ элСктричСскоС ΠΏΠΎΠ»Π΅ Π΄ΠΎΠ»ΠΆΠ½ΠΎ ΡΠΎΠ·Π΄Π°Π²Π°Ρ‚ΡŒΡΡ зарядами ΠΏΡ€ΠΎΠ²ΠΎΠ΄Π½ΠΈΠΊΠΎΠ², входящих Π² ΡΠ»Π΅ΠΊΡ‚Ρ€ΠΈΡ‡Π΅ΡΠΊΡƒΡŽ Ρ†Π΅ΠΏΡŒ. Однако заряТСнныС ΠΏΡ€ΠΎΠ²ΠΎΠ΄Π½ΠΈΠΊΠΈ сами ΠΏΠΎ сСбС Π½Π΅ смогут ΠΎΠ±Π΅ΡΠΏΠ΅Ρ‡ΠΈΡ‚ΡŒ ΠΏΡ€ΠΎΡ‚Π΅ΠΊΠ°Π½ΠΈΠ΅ постоянного Ρ‚ΠΎΠΊΠ°.

Рассмотрим, ΠΊ ΠΏΡ€ΠΈΠΌΠ΅Ρ€Ρƒ, Π΄Π²Π° проводящих ΡˆΠ°Ρ€Π°, заряТСнных Ρ€Π°Π·Π½ΠΎΠΈΠΌΡ‘Π½Π½ΠΎ. Π‘ΠΎΠ΅Π΄ΠΈΠ½ΠΈΠΌ ΠΈΡ… ΠΏΡ€ΠΎΠ²ΠΎΠ΄ΠΎΠΌ. ΠœΠ΅ΠΆΠ΄Ρƒ ΠΊΠΎΠ½Ρ†Π°ΠΌΠΈ ΠΏΡ€ΠΎΠ²ΠΎΠ΄Π° Π²ΠΎΠ·Π½ΠΈΠΊΠ½Π΅Ρ‚ Ρ€Π°Π·Π½ΠΎΡΡ‚ΡŒ ΠΏΠΎΡ‚Π΅Π½Ρ†ΠΈΠ°Π»ΠΎΠ², Π° Π²Π½ΡƒΡ‚Ρ€ΠΈ ΠΏΡ€ΠΎΠ²ΠΎΠ΄Π° β€” элСктричСскоС ΠΏΠΎΠ»Π΅. По ΠΏΡ€ΠΎΠ²ΠΎΠ΄Ρƒ ΠΏΠΎΡ‚Π΅Ρ‡Ρ‘Ρ‚ Ρ‚ΠΎΠΊ. Но ΠΏΠΎ ΠΌΠ΅Ρ€Π΅ прохоТдСния Ρ‚ΠΎΠΊΠ° Ρ€Π°Π·Π½ΠΎΡΡ‚ΡŒ ΠΏΠΎΡ‚Π΅Π½Ρ†ΠΈΠ°Π»ΠΎΠ² ΠΌΠ΅ΠΆΠ΄Ρƒ ΡˆΠ°Ρ€Π°ΠΌΠΈ Π±ΡƒΠ΄Π΅Ρ‚ ΡƒΠΌΠ΅Π½ΡŒΡˆΠ°Ρ‚ΡŒΡΡ, вслСд Π·Π° Π½Π΅ΠΉ станСт ΡƒΠ±Ρ‹Π²Π°Ρ‚ΡŒ ΠΈ Π½Π°ΠΏΡ€ΡΠΆΡ‘Π½Π½ΠΎΡΡ‚ΡŒ поля Π² ΠΏΡ€ΠΎΠ²ΠΎΠ΄Π΅. Π’ ΠΊΠΎΠ½Ρ†Π΅ ΠΊΠΎΠ½Ρ†ΠΎΠ² ΠΏΠΎΡ‚Π΅Π½Ρ†ΠΈΠ°Π»Ρ‹ ΡˆΠ°Ρ€ΠΎΠ² станут Ρ€Π°Π²Π½Ρ‹ Π΄Ρ€ΡƒΠ³ Π΄Ρ€ΡƒΠ³Ρƒ, ΠΏΠΎΠ»Π΅ Π² ΠΏΡ€ΠΎΠ²ΠΎΠ΄Π΅ обратится Π² Π½ΡƒΠ»ΡŒ, ΠΈ Ρ‚ΠΎΠΊ исчСзнСт. ΠœΡ‹ оказались Π² элСктростатикС: ΡˆΠ°Ρ€Ρ‹ плюс ΠΏΡ€ΠΎΠ²ΠΎΠ΄ ΠΎΠ±Ρ€Π°Π·ΡƒΡŽΡ‚ Π΅Π΄ΠΈΠ½Ρ‹ΠΉ ΠΏΡ€ΠΎΠ²ΠΎΠ΄Π½ΠΈΠΊ, Π² ΠΊΠ°ΠΆΠ΄ΠΎΠΉ Ρ‚ΠΎΡ‡ΠΊΠ΅ ΠΊΠΎΡ‚ΠΎΡ€ΠΎΠ³ΠΎ ΠΏΠΎΡ‚Π΅Π½Ρ†ΠΈΠ°Π» ΠΏΡ€ΠΈΠ½ΠΈΠΌΠ°Π΅Ρ‚ ΠΎΠ΄Π½ΠΎ ΠΈ Ρ‚ΠΎ ΠΆΠ΅ Π·Π½Π°Ρ‡Π΅Π½ΠΈΠ΅; Π½Π°ΠΏΡ€ΡΠΆΡ‘Π½Π½ΠΎΡΡ‚ΡŒ
поля Π²Π½ΡƒΡ‚Ρ€ΠΈ ΠΏΡ€ΠΎΠ²ΠΎΠ΄Π½ΠΈΠΊΠ° Ρ€Π°Π²Π½Π° Π½ΡƒΠ»ΡŽ, Π½ΠΈΠΊΠ°ΠΊΠΎΠ³ΠΎ Ρ‚ΠΎΠΊΠ° Π½Π΅Ρ‚.

Π’ΠΎ, Ρ‡Ρ‚ΠΎ элСктростатичСскоС ΠΏΠΎΠ»Π΅ само ΠΏΠΎ сСбС Π½Π΅ годится Π½Π° Ρ€ΠΎΠ»ΡŒ стационарного поля, ΡΠΎΠ·Π΄Π°ΡŽΡ‰Π΅Π³ΠΎ Ρ‚ΠΎΠΊ, ясно ΠΈ ΠΈΠ· Π±ΠΎΠ»Π΅Π΅ ΠΎΠ±Ρ‰ΠΈΡ… сообраТСний. Π’Π΅Π΄ΡŒ элСктростатичСскоС ΠΏΠΎΠ»Π΅ ΠΏΠΎΡ‚Π΅Π½Ρ†ΠΈΠ°Π»ΡŒΠ½ΠΎ, Π΅Π³ΠΎ Ρ€Π°Π±ΠΎΡ‚Π° ΠΏΡ€ΠΈ ΠΏΠ΅Ρ€Π΅ΠΌΠ΅Ρ‰Π΅Π½ΠΈΠΈ заряда ΠΏΠΎ Π·Π°ΠΌΠΊΠ½ΡƒΡ‚ΠΎΠΌΡƒ ΠΏΡƒΡ‚ΠΈ Ρ€Π°Π²Π½Π° Π½ΡƒΠ»ΡŽ. Π‘Π»Π΅Π΄ΠΎΠ²Π°Ρ‚Π΅Π»ΡŒΠ½ΠΎ, ΠΎΠ½ΠΎ Π½Π΅ ΠΌΠΎΠΆΠ΅Ρ‚ Π²Ρ‹Π·Ρ‹Π²Π°Ρ‚ΡŒ Ρ†ΠΈΡ€ΠΊΡƒΠ»ΠΈΡ€ΠΎΠ²Π°Π½ΠΈΠ΅ зарядов ΠΏΠΎ Π·Π°ΠΌΠΊΠ½ΡƒΡ‚ΠΎΠΉ элСктричСской Ρ†Π΅ΠΏΠΈ β€” для этого трСбуСтся ΡΠΎΠ²Π΅Ρ€ΡˆΠ°Ρ‚ΡŒ Π½Π΅Π½ΡƒΠ»Π΅Π²ΡƒΡŽ Ρ€Π°Π±ΠΎΡ‚Ρƒ.

ΠšΡ‚ΠΎ ΠΆΠ΅ Π±ΡƒΠ΄Π΅Ρ‚ ΡΠΎΠ²Π΅Ρ€ΡˆΠ°Ρ‚ΡŒ эту Π½Π΅Π½ΡƒΠ»Π΅Π²ΡƒΡŽ Ρ€Π°Π±ΠΎΡ‚Ρƒ? ΠšΡ‚ΠΎ Π±ΡƒΠ΄Π΅Ρ‚ ΠΏΠΎΠ΄Π΄Π΅Ρ€ΠΆΠΈΠ²Π°Ρ‚ΡŒ Π² Ρ†Π΅ΠΏΠΈ Ρ€Π°Π·Π½ΠΎΡΡ‚ΡŒ ΠΏΠΎΡ‚Π΅Π½Ρ†ΠΈΠ°Π»ΠΎΠ² ΠΈ ΠΎΠ±Π΅ΡΠΏΠ΅Ρ‡ΠΈΠ²Π°Ρ‚ΡŒ стационарноС элСктричСскоС ΠΏΠΎΠ»Π΅, ΡΠΎΠ·Π΄Π°ΡŽΡ‰Π΅Π΅ Ρ‚ΠΎΠΊ Π² ΠΏΡ€ΠΎΠ²ΠΎΠ΄Π½ΠΈΠΊΠ°Ρ…?

ΠžΡ‚Π²Π΅Ρ‚ β€” источник Ρ‚ΠΎΠΊΠ°, ваТнСйший элСмСнт элСктричСской Ρ†Π΅ΠΏΠΈ.

Π§Ρ‚ΠΎΠ±Ρ‹ Π² ΠΏΡ€ΠΎΠ²ΠΎΠ΄Π½ΠΈΠΊΠ΅ ΠΏΡ€ΠΎΡ‚Π΅ΠΊΠ°Π» постоянный Ρ‚ΠΎΠΊ, ΠΊΠΎΠ½Ρ†Ρ‹ ΠΏΡ€ΠΎΠ²ΠΎΠ΄Π½ΠΈΠΊΠ° Π΄ΠΎΠ»ΠΆΠ½Ρ‹ Π±Ρ‹Ρ‚ΡŒ присоСдинСны ΠΊ ΠΊΠ»Π΅ΠΌΠΌΠ°ΠΌ источника Ρ‚ΠΎΠΊΠ° (Π±Π°Ρ‚Π°Ρ€Π΅ΠΉΠΊΠΈ, аккумулятора ΠΈ Ρ‚. Π΄.).

ΠšΠ»Π΅ΠΌΠΌΡ‹ источника β€” это заряТСнныС ΠΏΡ€ΠΎΠ²ΠΎΠ΄Π½ΠΈΠΊΠΈ. Если Ρ†Π΅ΠΏΡŒ Π·Π°ΠΌΠΊΠ½ΡƒΡ‚Π°, Ρ‚ΠΎ заряды с ΠΊΠ»Π΅ΠΌΠΌ ΠΏΠ΅Ρ€Π΅ΠΌΠ΅Ρ‰Π°ΡŽΡ‚ΡΡ ΠΏΠΎ Ρ†Π΅ΠΏΠΈ β€” ΠΊΠ°ΠΊ Π² рассмотрСнном Π²Ρ‹ΡˆΠ΅ ΠΏΡ€ΠΈΠΌΠ΅Ρ€Π΅ с ΡˆΠ°Ρ€Π°ΠΌΠΈ. Но Ρ‚Π΅ΠΏΠ΅Ρ€ΡŒ Ρ€Π°Π·Π½ΠΎΡΡ‚ΡŒ ΠΏΠΎΡ‚Π΅Π½Ρ†ΠΈΠ°Π»ΠΎΠ² ΠΌΠ΅ΠΆΠ΄Ρƒ ΠΊΠ»Π΅ΠΌΠΌΠ°ΠΌΠΈ Π½Π΅ ΡƒΠΌΠ΅Π½ΡŒΡˆΠ°Π΅Ρ‚ΡΡ: источник Ρ‚ΠΎΠΊΠ° Π½Π΅ΠΏΡ€Π΅Ρ€Ρ‹Π²Π½ΠΎ восполняСт заряды Π½Π° ΠΊΠ»Π΅ΠΌΠΌΠ°Ρ…, поддСрТивая Ρ€Π°Π·Π½ΠΎΡΡ‚ΡŒ ΠΏΠΎΡ‚Π΅Π½Ρ†ΠΈΠ°Π»ΠΎΠ² ΠΌΠ΅ΠΆΠ΄Ρƒ ΠΊΠΎΠ½Ρ†Π°ΠΌΠΈ Ρ†Π΅ΠΏΠΈ Π½Π° Π½Π΅ΠΈΠ·ΠΌΠ΅Π½Π½ΠΎΠΌ ΡƒΡ€ΠΎΠ²Π½Π΅.

Π’ этом ΠΈ состоит ΠΏΡ€Π΅Π΄Π½Π°Π·Π½Π°Ρ‡Π΅Π½ΠΈΠ΅ источника постоянного Ρ‚ΠΎΠΊΠ°. Π’Π½ΡƒΡ‚Ρ€ΠΈ Π½Π΅Π³ΠΎ ΠΏΡ€ΠΎΡ‚Π΅ΠΊΠ°ΡŽΡ‚ процСссы нСэлСктричСского (Ρ‡Π°Ρ‰Π΅ всСго β€” химичСского) происхоТдСния, ΠΊΠΎΡ‚ΠΎΡ€Ρ‹Π΅ ΠΎΠ±Π΅ΡΠΏΠ΅Ρ‡ΠΈΠ²Π°ΡŽΡ‚ Π½Π΅ΠΏΡ€Π΅Ρ€Ρ‹Π²Π½ΠΎΠ΅ Ρ€Π°Π·Π΄Π΅Π»Π΅Π½ΠΈΠ΅ зарядов. Π­Ρ‚ΠΈ заряды ΠΏΠΎΡΡ‚Π°Π²Π»ΡΡŽΡ‚ΡΡ Π½Π° ΠΊΠ»Π΅ΠΌΠΌΡ‹ источника Π² Π½Π΅ΠΎΠ±Ρ…ΠΎΠ΄ΠΈΠΌΠΎΠΌ количСствС.

ΠšΠΎΠ»ΠΈΡ‡Π΅ΡΡ‚Π²Π΅Π½Π½ΡƒΡŽ характСристику нСэлСктричСских процСссов раздСлСния зарядов Π²Π½ΡƒΡ‚Ρ€ΠΈ источника β€” Ρ‚Π°ΠΊ Π½Π°Π·Ρ‹Π²Π°Π΅ΠΌΡƒΡŽ Π­Π”Π‘ β€” ΠΌΡ‹ ΠΈΠ·ΡƒΡ‡ΠΈΠΌ ΠΏΠΎΠ·ΠΆΠ΅, Π² ΡΠΎΠΎΡ‚Π²Π΅Ρ‚ΡΡ‚Π²ΡƒΡŽΡ‰Π΅ΠΌ листкС.

А сСйчас вСрнёмся ΠΊ стационарному элСктричСскому полю. Каким ΠΆΠ΅ ΠΎΠ±Ρ€Π°Π·ΠΎΠΌ ΠΎΠ½ΠΎ Π²ΠΎΠ·Π½ΠΈΠΊΠ°Π΅Ρ‚ Π² ΠΏΡ€ΠΎΠ²ΠΎΠ΄Π½ΠΈΠΊΠ°Ρ… Ρ†Π΅ΠΏΠΈ ΠΏΡ€ΠΈ Π½Π°Π»ΠΈΡ‡ΠΈΠΈ источника Ρ‚ΠΎΠΊΠ°?

ЗаряТСнныС ΠΊΠ»Π΅ΠΌΠΌΡ‹ источника ΡΠΎΠ·Π΄Π°ΡŽΡ‚ Π½Π° ΠΊΠΎΠ½Ρ†Π°Ρ… ΠΏΡ€ΠΎΠ²ΠΎΠ΄Π½ΠΈΠΊΠ° элСктричСскоС ΠΏΠΎΠ»Π΅. Π‘Π²ΠΎΠ±ΠΎΠ΄Π½Ρ‹Π΅ заряды ΠΏΡ€ΠΎΠ²ΠΎΠ΄Π½ΠΈΠΊΠ°, находящиСся Π²Π±Π»ΠΈΠ·ΠΈ ΠΊΠ»Π΅ΠΌΠΌ, приходят Π² Π΄Π²ΠΈΠΆΠ΅Π½ΠΈΠ΅ ΠΈ Π΄Π΅ΠΉΡΡ‚Π²ΡƒΡŽΡ‚ своим элСктричСским ΠΏΠΎΠ»Π΅ΠΌ Π½Π° сосСдниС заряды. Π‘ΠΎ ΡΠΊΠΎΡ€ΠΎΡΡ‚ΡŒΡŽ, Π±Π»ΠΈΠ·ΠΊΠΎΠΉ ΠΊ скорости свСта, это взаимодСйствиС пСрСдаётся вдоль всСй Ρ†Π΅ΠΏΠΈ, ΠΈ Π² Ρ†Π΅ΠΏΠΈ устанавливаСтся постоянный элСктричСский Ρ‚ΠΎΠΊ. БтабилизируСтся ΠΈ элСктричСскоС ΠΏΠΎΠ»Π΅, создаваСмоС двиТущимися зарядами.

Π‘Ρ‚Π°Ρ†ΠΈΠΎΠ½Π°Ρ€Π½ΠΎΠ΅ элСктричСскоС ΠΏΠΎΠ»Π΅ β€” это ΠΏΠΎΠ»Π΅ свободных зарядов ΠΏΡ€ΠΎΠ²ΠΎΠ΄Π½ΠΈΠΊΠ°, ΡΠΎΠ²Π΅Ρ€ΡˆΠ°ΡŽΡ‰ΠΈΡ… Π½Π°ΠΏΡ€Π°Π²Π»Π΅Π½Π½ΠΎΠ΅ Π΄Π²ΠΈΠΆΠ΅Π½ΠΈΠ΅.

Π‘Ρ‚Π°Ρ†ΠΈΠΎΠ½Π°Ρ€Π½ΠΎΠ΅ элСктричСскоС ΠΏΠΎΠ»Π΅ Π½Π΅ мСняСтся со Π²Ρ€Π΅ΠΌΠ΅Π½Π΅ΠΌ ΠΏΠΎΡ‚ΠΎΠΌΡƒ, Ρ‡Ρ‚ΠΎ ΠΏΡ€ΠΈ постоянном Ρ‚ΠΎΠΊΠ΅ Π½Π΅ мСняСтся ΠΊΠ°Ρ€Ρ‚ΠΈΠ½Π° распрСдСлСния зарядов Π² ΠΏΡ€ΠΎΠ²ΠΎΠ΄Π½ΠΈΠΊΠ΅: Π½Π° мСсто заряда, ΠΏΠΎΠΊΠΈΠ½ΡƒΠ²ΡˆΠ΅Π³ΠΎ Π΄Π°Π½Π½Ρ‹ΠΉ участок ΠΏΡ€ΠΎΠ²ΠΎΠ΄Π½ΠΈΠΊΠ°, Π² ΡΠ»Π΅Π΄ΡƒΡŽΡ‰ΠΈΠΉ ΠΌΠΎΠΌΠ΅Π½Ρ‚ Π²Ρ€Π΅ΠΌΠ΅Π½ΠΈ поступаСт Ρ‚ΠΎΡ‡Π½ΠΎ Ρ‚Π°ΠΊΠΎΠΉ ΠΆΠ΅ заряд. По этой ΠΏΡ€ΠΈΡ‡ΠΈΠ½Π΅ стационарноС ΠΏΠΎΠ»Π΅ Π²ΠΎ ΠΌΠ½ΠΎΠ³ΠΎΠΌ (Π½ΠΎ Π½Π΅ Π²ΠΎ всём) Π°Π½Π°Π»ΠΎΠ³ΠΈΡ‡Π½ΠΎ полю элСктростатичСскому.

А ΠΈΠΌΠ΅Π½Π½ΠΎ, справСдливы ΡΠ»Π΅Π΄ΡƒΡŽΡ‰ΠΈΠ΅ Π΄Π²Π° утвСрТдСния, ΠΊΠΎΡ‚ΠΎΡ€Ρ‹Π΅ понадобятся Π½Π°ΠΌ Π² дальнСйшСм (ΠΈΡ… Π΄ΠΎΠΊΠ°Π·Π°Ρ‚Π΅Π»ΡŒΡΡ‚Π²ΠΎ даётся Π² вузовском курсС Ρ„ΠΈΠ·ΠΈΠΊΠΈ).

1. Как ΠΈ элСктростатичСскоС ΠΏΠΎΠ»Π΅, стационарноС элСктричСскоС ΠΏΠΎΠ»Π΅ ΠΏΠΎΡ‚Π΅Π½Ρ†ΠΈΠ°Π»ΡŒΠ½ΠΎ. Π­Ρ‚ΠΎ позволяСт Π³ΠΎΠ²ΠΎΡ€ΠΈΡ‚ΡŒ ΠΎ разности ΠΏΠΎΡ‚Π΅Π½Ρ†ΠΈΠ°Π»ΠΎΠ² (Ρ‚. Π΅. напряТСнии) Π½Π° любом участкС Ρ†Π΅ΠΏΠΈ (ΠΈΠΌΠ΅Π½Π½ΠΎ эту Ρ€Π°Π·Π½ΠΎΡΡ‚ΡŒ ΠΏΠΎΡ‚Π΅Π½Ρ†ΠΈΠ°Π»ΠΎΠ² ΠΌΡ‹ измСряСм Π²ΠΎΠ»ΡŒΡ‚ΠΌΠ΅Ρ‚Ρ€ΠΎΠΌ).
ΠŸΠΎΡ‚Π΅Π½Ρ†ΠΈΠ°Π»ΡŒΠ½ΠΎΡΡ‚ΡŒ, Π½Π°ΠΏΠΎΠΌΠ½ΠΈΠΌ, ΠΎΠ·Π½Π°Ρ‡Π°Π΅Ρ‚, Ρ‡Ρ‚ΠΎ Ρ€Π°Π±ΠΎΡ‚Π° стационарного поля ΠΏΠΎ ΠΏΠ΅Ρ€Π΅ΠΌΠ΅Ρ‰Π΅Π½ΠΈΡŽ заряда Π½Π΅ зависит ΠΎΡ‚ Ρ„ΠΎΡ€ΠΌΡ‹ Ρ‚Ρ€Π°Π΅ΠΊΡ‚ΠΎΡ€ΠΈΠΈ. ИмСнно поэтому ΠΏΡ€ΠΈ ΠΏΠ°Ρ€Π°Π»Π»Π΅Π»ΡŒΠ½ΠΎΠΌ соСдинСнии ΠΏΡ€ΠΎΠ²ΠΎΠ΄Π½ΠΈΠΊΠΎΠ² напряТСниС Π½Π° ΠΊΠ°ΠΆΠ΄ΠΎΠΌ ΠΈΠ· Π½ΠΈΡ… ΠΎΠ΄ΠΈΠ½Π°ΠΊΠΎΠ²ΠΎ: ΠΎΠ½ΠΎ Ρ€Π°Π²Π½ΠΎ разности ΠΏΠΎΡ‚Π΅Π½Ρ†ΠΈΠ°Π»ΠΎΠ² стационарного поля ΠΌΠ΅ΠΆΠ΄Ρƒ Ρ‚Π΅ΠΌΠΈ двумя Ρ‚ΠΎΡ‡ΠΊΠ°ΠΌΠΈ, ΠΊ ΠΊΠΎΡ‚ΠΎΡ€Ρ‹ΠΌ ΠΏΠΎΠ΄ΠΊΠ»ΡŽΡ‡Π΅Π½Ρ‹ ΠΏΡ€ΠΎΠ²ΠΎΠ΄Π½ΠΈΠΊΠΈ.
2. Π’ ΠΎΡ‚Π»ΠΈΡ‡ΠΈΠ΅ ΠΎΡ‚ элСктростатичСского поля, стационарноС ΠΏΠΎΠ»Π΅ двиТущихся зарядов ΠΏΡ€ΠΎΠ½ΠΈΠΊΠ°Π΅Ρ‚ Π²Π½ΡƒΡ‚Ρ€ΡŒ ΠΏΡ€ΠΎΠ²ΠΎΠ΄Π½ΠΈΠΊΠ° (Π΄Π΅Π»ΠΎ Π² Ρ‚ΠΎΠΌ, Ρ‡Ρ‚ΠΎ свободныС заряды, участвуя Π² Π½Π°ΠΏΡ€Π°Π²Π»Π΅Π½Π½ΠΎΠΌ Π΄Π²ΠΈΠΆΠ΅Π½ΠΈΠΈ, Π½Π΅ ΡƒΡΠΏΠ΅Π²Π°ΡŽΡ‚ Π΄ΠΎΠ»ΠΆΠ½Ρ‹ΠΌ ΠΎΠ±Ρ€Π°Π·ΠΎΠΌ ΠΏΠ΅Ρ€Π΅ΡΡ‚Ρ€Π°ΠΈΠ²Π°Ρ‚ΡŒΡΡ ΠΈ ΠΏΡ€ΠΈΠ½ΠΈΠΌΠ°Ρ‚ΡŒ «элСктростатичСскиС» ΠΊΠΎΠ½Ρ„ΠΈΠ³ΡƒΡ€Π°Ρ†ΠΈΠΈ).
Π›ΠΈΠ½ΠΈΠΈ напряТённости стационарного поля Π²Π½ΡƒΡ‚Ρ€ΠΈ ΠΏΡ€ΠΎΠ²ΠΎΠ΄Π½ΠΈΠΊΠ° ΠΏΠ°Ρ€Π°Π»Π»Π΅Π»ΡŒΠ½Ρ‹ Π΅Π³ΠΎ повСрхности, ΠΊΠ°ΠΊ Π±Ρ‹ Π½ΠΈ изгибался ΠΏΡ€ΠΎΠ²ΠΎΠ΄Π½ΠΈΠΊ. ΠŸΠΎΡΡ‚ΠΎΠΌΡƒ, ΠΊΠ°ΠΊ ΠΈ Π² ΠΎΠ΄Π½ΠΎΡ€ΠΎΠ΄Π½ΠΎΠΌ элСктростатичСском ΠΏΠΎΠ»Π΅, справСдлива Ρ„ΠΎΡ€ΠΌΡƒΠ»Π° , Π³Π΄Π΅ β€” напряТСниС Π½Π° ΠΊΠΎΠ½Ρ†Π°Ρ… ΠΏΡ€ΠΎΠ²ΠΎΠ΄Π½ΠΈΠΊΠ°, β€” Π½Π°ΠΏΡ€ΡΠΆΡ‘Π½Π½ΠΎΡΡ‚ΡŒ стационарного поля Π² ΠΏΡ€ΠΎΠ²ΠΎΠ΄Π½ΠΈΠΊΠ΅, β€” Π΄Π»ΠΈΠ½Π° ΠΏΡ€ΠΎΠ²ΠΎΠ΄Π½ΠΈΠΊΠ°.

НаправлСниС Ρ‚ΠΎΠΊΠ° Π² ΠΏΡ€ΠΎΠ²ΠΎΠ΄Π½ΠΈΠΊΠ΅, ΠΊΠ°ΠΊ, ΠΎΡ‚ΠΊΡƒΠ΄Π° ΠΈ ΠΊΡƒΠ΄Π° Ρ‚Π΅Ρ‡Π΅Ρ‚ элСктричСский Ρ‚ΠΎΠΊ Π² ΠΏΡ€ΠΎΠ²ΠΎΠ΄Π½ΠΈΠΊΠ°Ρ….

ЭлСктричСский Ρ‚ΠΎΠΊ прСдставляСт собой упорядочСнноС Π΄Π²ΠΈΠΆΠ΅Π½ΠΈΠ΅ заряТСнных частиц. Π’ Ρ‚Π²Π΅Ρ€Π΄Ρ‹Ρ… Ρ‚Π΅Π»Π°Ρ… это Π΄Π²ΠΈΠΆΠ΅Π½ΠΈΠ΅ элСктронов (ΠΎΡ‚Ρ€ΠΈΡ†Π°Ρ‚Π΅Π»ΡŒΠ½ΠΎ заряТСнных частиц) Π² ΠΆΠΈΠ΄ΠΊΠΈΡ… ΠΈ Π³Π°Π·ΠΎΠΎΠ±Ρ€Π°Π·Π½Ρ‹Ρ… Ρ‚Π΅Π»Π°Ρ… это Π΄Π²ΠΈΠΆΠ΅Π½ΠΈΠ΅ ΠΈΠΎΠ½ΠΎΠ² (ΠΏΠΎΠ»ΠΎΠΆΠΈΡ‚Π΅Π»ΡŒΠ½ΠΎ заряТСнных частиц). Π‘ΠΎΠ»Π΅Π΅ Ρ‚ΠΎΠ³ΠΎ Ρ‚ΠΎΠΊ Π±Ρ‹Π²Π°Π΅Ρ‚ постоянным ΠΈ ΠΏΠ΅Ρ€Π΅ΠΌΠ΅Π½Π½Ρ‹ΠΌ, ΠΈ Ρƒ Π½ΠΈΡ… совсСм Ρ€Π°Π·Π½ΠΎΠ΅ Π΄Π²ΠΈΠΆΠ΅Π½ΠΈΠ΅ элСктричСских зарядов. Π§Ρ‚ΠΎΠ±Ρ‹ Ρ…ΠΎΡ€ΠΎΡˆΠΎ ΠΏΠΎΠ½ΡΡ‚ΡŒ ΠΈ ΡƒΡΠ²ΠΎΠΈΡ‚ΡŒ Ρ‚Π΅ΠΌΡƒ Π΄Π²ΠΈΠΆΠ΅Π½ΠΈΠ΅ Ρ‚ΠΎΠΊΠ° Π² ΠΏΡ€ΠΎΠ²ΠΎΠ΄Π½ΠΈΠΊΠ°Ρ… ΠΏΠΎΠΆΠ°Π»ΡƒΠΉ сначала Π½ΡƒΠΆΠ½ΠΎ Π±ΠΎΠ»Π΅Π΅ ΠΏΠΎΠ΄Ρ€ΠΎΠ±Π½ΠΎ Ρ€Π°Π·ΠΎΠ±Ρ€Π°Ρ‚ΡŒΡΡ с основами элСктрофизики. ИмСнно с этого я ΠΈ Π½Π°Ρ‡Π½Ρƒ.

Π˜Ρ‚Π°ΠΊ, ΠΊΠ°ΠΊ Π²ΠΎΠΎΠ±Ρ‰Π΅ происходит Π΄Π²ΠΈΠΆΠ΅Π½ΠΈΠ΅ элСктричСского Ρ‚ΠΎΠΊΠ°? Π˜Π·Π²Π΅ΡΡ‚Π½ΠΎ, Ρ‡Ρ‚ΠΎ вСщСства состоят ΠΈΠ· Π°Ρ‚ΠΎΠΌΠΎΠ². Π­Ρ‚ΠΎ элСмСнтарныС частицы вСщСства. Π‘Ρ‚Ρ€ΠΎΠ΅Π½ΠΈΠ΅ Π°Ρ‚ΠΎΠΌΠ° Π½Π°ΠΏΠΎΠΌΠΈΠ½Π°Π΅Ρ‚ Π½Π°ΡˆΡƒ ΡΠΎΠ»Π½Π΅Ρ‡Π½ΡƒΡŽ систСму, Π³Π΄Π΅ Π² Ρ†Π΅Π½Ρ‚Ρ€Π΅ располоТСно ядро Π°Ρ‚ΠΎΠΌΠ°. Оно состоит ΠΈΠ· ΠΏΠ»ΠΎΡ‚Π½ΠΎ ΠΏΡ€ΠΈΠΆΠ°Ρ‚Ρ‹Ρ… Π΄Ρ€ΡƒΠ³ ΠΊ Π΄Ρ€ΡƒΠ³Ρƒ ΠΏΡ€ΠΎΡ‚ΠΎΠ½ΠΎΠ² (ΠΏΠΎΠ»ΠΎΠΆΠΈΡ‚Π΅Π»ΡŒΠ½Ρ‹Ρ… элСктричСских частиц) ΠΈ Π½Π΅ΠΉΡ‚Ρ€ΠΎΠ½ΠΎΠ² (элСктричСски Π½Π΅ΠΉΡ‚Ρ€Π°Π»ΡŒΠ½Ρ‹Ρ… частиц). Π’ΠΎΠΊΡ€ΡƒΠ³ этого ядра с ΠΎΠ³Ρ€ΠΎΠΌΠ½ΠΎΠΉ ΡΠΊΠΎΡ€ΠΎΡΡ‚ΡŒΡŽ ΠΏΠΎ своим ΠΎΡ€Π±ΠΈΡ‚Π°ΠΌ Π²Ρ€Π°Ρ‰Π°ΡŽΡ‚ΡΡ элСктроны (Π±ΠΎΠ»Π΅Π΅ ΠΌΠ΅Π»ΠΊΠΈΠ΅ частицы, ΠΈΠΌΠ΅ΡŽΡ‰ΠΈΠ΅ ΠΎΡ‚Ρ€ΠΈΡ†Π°Ρ‚Π΅Π»ΡŒΠ½Ρ‹ΠΉ заряд). Π£ Ρ€Π°Π·Π½Ρ‹Ρ… вСщСств количСство элСктронов ΠΈ ΠΎΡ€Π±ΠΈΡ‚, ΠΏΠΎ ΠΊΠΎΡ‚ΠΎΡ€Ρ‹ΠΌ ΠΎΠ½ΠΈ Π²Ρ€Π°Ρ‰Π°ΡŽΡ‚ΡΡ, ΠΌΠΎΠΆΠ΅Ρ‚ Π±Ρ‹Ρ‚ΡŒ Ρ€Π°Π·Π»ΠΈΡ‡Π½Ρ‹ΠΌ. Атомы Ρ‚Π²Π΅Ρ€Π΄Ρ‹Ρ… вСщСств ΠΈΠΌΠ΅ΡŽΡ‚ Ρ‚Π°ΠΊ Π½Π°Π·Ρ‹Π²Π°Π΅ΠΌΡƒΡŽ ΠΊΡ€ΠΈΡΡ‚Π°Π»Π»ΠΈΡ‡Π΅ΡΠΊΡƒΡŽ Ρ€Π΅ΡˆΠ΅Ρ‚ΠΊΡƒ. Π­Ρ‚ΠΎ структура вСщСства, ΠΏΠΎ ΠΊΠΎΡ‚ΠΎΡ€ΠΎΠΉ Π² ΠΎΠΏΡ€Π΅Π΄Π΅Π»Π΅Π½Π½ΠΎΠΉ порядкС Ρ€Π°ΡΠΏΠΎΠ»Π°Π³Π°ΡŽΡ‚ΡΡ Π°Ρ‚ΠΎΠΌΡ‹ ΠΎΡ‚Π½ΠΎΡΠΈΡ‚Π΅Π»ΡŒΠ½ΠΎ Π΄Ρ€ΡƒΠ³ Π΄Ρ€ΡƒΠ³Π°.

А Π³Π΄Π΅ ΠΆΠ΅ Ρ‚ΡƒΡ‚ ΠΌΠΎΠΆΠ΅Ρ‚ Π²ΠΎΠ·Π½ΠΈΠΊΠ½ΡƒΡ‚ΡŒ элСктричСский Ρ‚ΠΎΠΊ? ΠžΠΊΠ°Π·Ρ‹Π²Π°Π΅Ρ‚ΡΡ, Ρ‡Ρ‚ΠΎ Ρƒ Π½Π΅ΠΊΠΎΡ‚ΠΎΡ€Ρ‹Ρ… вСщСств (ΠΏΡ€ΠΎΠ²ΠΎΠ΄Π½ΠΈΠΊΠΎΠ² Ρ‚ΠΎΠΊΠ°) элСктроны, Ρ‡Ρ‚ΠΎ Π½Π°ΠΈΠ±ΠΎΠ»Π΅Π΅ ΡƒΠ΄Π°Π»Π΅Π½Ρ‹ ΠΎΡ‚ своСго ядра, ΠΌΠΎΠ³ΡƒΡ‚ ΠΎΡ‚Ρ€Ρ‹Π²Π°Ρ‚ΡŒΡΡ ΠΎΡ‚ Π°Ρ‚ΠΎΠΌΠ° ΠΈ ΠΏΠ΅Ρ€Π΅Ρ…ΠΎΠ΄ΠΈΡ‚ΡŒ Π½Π° сосСдний Π°Ρ‚ΠΎΠΌ. Π­Ρ‚ΠΎ Π΄Π²ΠΈΠΆΠ΅Π½ΠΈΠ΅ элСктронов называСтся свободным. ΠŸΡ€ΠΎΡΡ‚ΠΎ элСктроны ΠΏΠ΅Ρ€Π΅ΠΌΠ΅Ρ‰Π°ΡŽΡ‚ΡΡ Π²Π½ΡƒΡ‚Ρ€ΠΈ вСщСства ΠΎΡ‚ ΠΎΠ΄Π½ΠΎΠ³ΠΎ Π°Ρ‚ΠΎΠΌΠ° ΠΊ Π΄Ρ€ΡƒΠ³ΠΎΠΌΡƒ. Но Π²ΠΎΡ‚ Ссли ΠΊ этому вСщСству (элСктричСскому ΠΏΡ€ΠΎΠ²ΠΎΠ΄Π½ΠΈΠΊΡƒ) ΠΏΠΎΠ΄ΠΊΠ»ΡŽΡ‡ΠΈΡ‚ΡŒ внСшнСС элСктромагнитноС ΠΏΠΎΠ»Π΅, Ρ‚Π΅ΠΌ самым создав ΡΠ»Π΅ΠΊΡ‚Ρ€ΠΈΡ‡Π΅ΡΠΊΡƒΡŽ Ρ†Π΅ΠΏΡŒ, Ρ‚ΠΎ всС свободныС элСктроны Π½Π°Ρ‡Π½ΡƒΡ‚ Π΄Π²ΠΈΠ³Π°Ρ‚ΡŒΡΡ Π² ΠΎΠ΄Π½ΠΎΠΌ Π½Π°ΠΏΡ€Π°Π²Π»Π΅Π½ΠΈΠΈ. ИмСнно это ΠΈ Π΅ΡΡ‚ΡŒ Π΄Π²ΠΈΠΆΠ΅Π½ΠΈΠ΅ элСктричСского Ρ‚ΠΎΠΊΠ° Π²Π½ΡƒΡ‚Ρ€ΠΈ ΠΏΡ€ΠΎΠ²ΠΎΠ΄Π½ΠΈΠΊΠ°.

Π’Π΅ΠΏΠ΅Ρ€ΡŒ Π΄Π°Π²Π°ΠΉΡ‚Π΅ разбСрСмся с Ρ‚Π΅ΠΌ, Ρ‡Ρ‚ΠΎ собой прСдставляСт постоянный ΠΈ ΠΏΠ΅Ρ€Π΅ΠΌΠ΅Π½Π½Ρ‹ΠΉ Ρ‚ΠΎΠΊ. Π˜Ρ‚Π°ΠΊ, постоянный Ρ‚ΠΎΠΊ всСгда двиТСтся Ρ‚ΠΎΠ»ΡŒΠΊΠΎ Π² ΠΎΠ΄Π½ΠΎΠΌ Π½Π°ΠΏΡ€Π°Π²Π»Π΅Π½ΠΈΠΈ. Как Π³ΠΎΠ²ΠΎΡ€ΠΈΠ»ΠΎΡΡŒ Π² самом Π½Π°Ρ‡Π°Π»Π΅ β€” Π² Ρ‚Π²Π΅Ρ€Π΄Ρ‹Ρ… Ρ‚Π΅Π»Π°Ρ… двиТутся элСктроны, Π° Π² ΠΆΠΈΠ΄ΠΊΠΈΡ… ΠΈ Π³Π°Π·ΠΎΠΎΠ±Ρ€Π°Π·Π½Ρ‹Ρ… двиТутся ΠΈΠΎΠ½Ρ‹. Π­Π»Π΅ΠΊΡ‚Ρ€ΠΎΠ½Ρ‹, это ΠΎΡ‚Ρ€ΠΈΡ†Π°Ρ‚Π΅Π»ΡŒΠ½ΠΎΒ  заряТСнныС частицы. Π‘Π»Π΅Π΄ΠΎΠ²Π°Ρ‚Π΅Π»ΡŒΠ½ΠΎ, Π² Ρ‚Π²Π΅Ρ€Π΄Ρ‹Ρ… Ρ‚Π΅Π»Π°Ρ… элСктричСский Ρ‚ΠΎΠΊ Ρ‚Π΅Ρ‡Π΅Ρ‚ ΠΎΡ‚ минуса ΠΊ ΠΏΠ»ΡŽΡΡƒ источника питания (ΠΏΠ΅Ρ€Π΅ΠΌΠ΅Ρ‰Π°ΡŽΡ‚ΡΡ элСктроны ΠΏΠΎ элСктричСской Ρ†Π΅ΠΏΠΈ). Π’ Тидкостях ΠΈ Π³Π°Π·Π°Ρ… Ρ‚ΠΎΠΊ двиТСтся сразу Π² Π΄Π²ΡƒΡ… направлСниях, Π° Ρ‚ΠΎΡ‡Π½Π΅Π΅, ΠΎΠ΄Π½ΠΎΠ²Ρ€Π΅ΠΌΠ΅Π½Π½ΠΎ, элСктроны Ρ‚Π΅ΠΊΡƒΡ‚ ΠΊ ΠΏΠ»ΡŽΡΡƒ, Π° ΠΈΠΎΠ½Ρ‹ (ΠΎΡ‚Π΄Π΅Π»ΡŒΠ½Ρ‹Π΅ Π°Ρ‚ΠΎΠΌΡ‹, Ρ‡Ρ‚ΠΎ Π½Π΅ связаны ΠΌΠ΅ΠΆΠ΄Ρƒ собой кристалличСской Ρ€Π΅ΡˆΠ΅Ρ‚ΠΊΠΎΠΉ, ΠΎΠ½ΠΈ ΠΊΠ°ΠΆΠ΄Ρ‹ΠΉ сам ΠΏΠΎ сСбС) Ρ‚Π΅ΠΊΡƒΡ‚ ΠΊ минусу источника питания.

Π£Ρ‡Π΅Π½Ρ‹ΠΌΠΈ ΠΆΠ΅ Π±Ρ‹Π»ΠΎ принято ΠΎΡ„ΠΈΡ†ΠΈΠ°Π»ΡŒΠ½ΠΎ ΡΡ‡ΠΈΡ‚Π°Ρ‚ΡŒ, Ρ‡Ρ‚ΠΎ Π΄Π²ΠΈΠΆΠ΅Π½ΠΈΠ΅ происходит ΠΎΡ‚ плюса ΠΊ минусу (Π½Π°ΠΎΠ±ΠΎΡ€ΠΎΡ‚, Ρ‡Π΅ΠΌ это происходит Π² Π΄Π΅ΠΉΡΡ‚Π²ΠΈΡ‚Π΅Π»ΡŒΠ½ΠΎΡΡ‚ΠΈ). Π’Π°ΠΊ Ρ‡Ρ‚ΠΎ, с Π½Π°ΡƒΡ‡Π½ΠΎΠΉ Ρ‚ΠΎΡ‡ΠΊΠΈ зрСния ΠΏΡ€Π°Π²ΠΈΠ»ΡŒΠ½ΠΎ Π³ΠΎΠ²ΠΎΡ€ΠΈΡ‚ΡŒ, Ρ‡Ρ‚ΠΎ элСктричСский Ρ‚ΠΎΠΊ двиТСтся ΠΎΡ‚ плюса ΠΊ минусу, Π° с  Ρ€Π΅Π°Π»ΡŒΠ½ΠΎΠΉ Ρ‚ΠΎΡ‡ΠΊΠΈ зрСния (элСктрофизичСская ΠΏΡ€ΠΈΡ€ΠΎΠ΄Π°) ΠΏΡ€Π°Π²ΠΈΠ»ΡŒΠ½Π΅Π΅ ΠΏΠΎΠ»Π°Π³Π°Ρ‚ΡŒ, Ρ‡Ρ‚ΠΎ Ρ‚ΠΎΠΊ Ρ‚Π΅Ρ‡Π΅Ρ‚ ΠΎΡ‚ минуса ΠΊ ΠΏΠ»ΡŽΡΡƒ (Π² Ρ‚Π²Π΅Ρ€Π΄Ρ‹Ρ… Ρ‚Π΅Π»Π°Ρ…). НавСрноС это сдСлано для ΠΊΠ°ΠΊΠΎΠ³ΠΎ-Ρ‚ΠΎ удобства.

Π’Π΅ΠΏΠ΅Ρ€ΡŒ, Ρ‡Ρ‚ΠΎ касаСтся ΠΏΠ΅Ρ€Π΅ΠΌΠ΅Π½Π½ΠΎΠ³ΠΎ элСктричСского Ρ‚ΠΎΠΊΠ°. Π’ΡƒΡ‚ ΡƒΠΆΠ΅ Π½Π΅ΠΌΠ½ΠΎΠ³ΠΎ всС слоТнСС. Если Π² случаС постоянного Ρ‚ΠΎΠΊΠ° Π΄Π²ΠΈΠΆΠ΅Π½ΠΈΠ΅ заряТСнных частиц ΠΈΠΌΠ΅Π΅Ρ‚ Ρ‚ΠΎΠ»ΡŒΠΊΠΎ ΠΎΠ΄Π½ΠΎ Π½Π°ΠΏΡ€Π°Π²Π»Π΅Π½ΠΈΠ΅ (физичСски элСктроны со Π·Π½Π°ΠΊΠΎΠΌ минус Ρ‚Π΅ΠΊΡƒΡ‚ ΠΊ ΠΏΠ»ΡŽΡΡƒ), Ρ‚ΠΎ ΠΏΡ€ΠΈ ΠΏΠ΅Ρ€Π΅ΠΌΠ΅Π½Π½ΠΎΠΌ Ρ‚ΠΎΠΊΠ΅ Π½Π°ΠΏΡ€Π°Π²Π»Π΅Π½ΠΈΠ΅ двиТСния пСриодичСски мСняСтся Π½Π° ΠΏΡ€ΠΎΡ‚ΠΈΠ²ΠΎΠΏΠΎΠ»ΠΎΠΆΠ½ΠΎΠ΅. Π’Ρ‹ Π½Π°Π²Π΅Ρ€Π½ΠΎΠ΅ ΡΠ»Ρ‹ΡˆΠ°Π»ΠΈ, Ρ‡Ρ‚ΠΎ Π² ΠΎΠ±Ρ‹Ρ‡Π½ΠΎΠΉ городской элСктросСти ΠΏΠ΅Ρ€Π΅ΠΌΠ΅Π½Π½ΠΎΠ΅ напряТСниС Π²Π΅Π»ΠΈΡ‡ΠΈΠ½ΠΎΠΉ 220 Π²ΠΎΠ»ΡŒΡ‚ ΠΈ стандартной частотой 50 Π³Π΅Ρ€Ρ†. Π’Π°ΠΊ Π²ΠΎΡ‚ эти 50 Π³Π΅Ρ€Ρ† говорят ΠΎ Ρ‚ΠΎΠΌ, Ρ‡Ρ‚ΠΎ элСктричСский Ρ‚ΠΎΠΊ Π·Π° ΠΎΠ΄Π½Ρƒ сСкунду успСваСт 50 Ρ€Π°Π· ΠΏΡ€ΠΎΠΉΡ‚ΠΈ ΠΏΠΎΠ»Π½Ρ‹ΠΉ Ρ†ΠΈΠΊΠ», ΠΈΠΌΠ΅ΡŽΡ‰ΠΈΠΉ ΡΠΈΠ½ΡƒΡΠΎΠΈΠ΄Π°Π»ΡŒΠ½ΡƒΡŽ Ρ„ΠΎΡ€ΠΌΡƒ. ЀактичСски Π·Π° ΠΎΠ΄Π½Ρƒ сСкунду Π½Π°ΠΏΡ€Π°Π²Π»Π΅Π½ΠΈΠ΅ Ρ‚ΠΎΠΊΠ° мСняСтся Π°ΠΆ 100 Ρ€Π°Π· (Π·Π° ΠΎΠ΄ΠΈΠ½ Ρ†ΠΈΠΊΠ» мСняСтся Π΄Π²Π° Ρ€Π°Π·Π°).

P.S. НаправлСниС Ρ‚ΠΎΠΊΠ° Π² элСктричСских схСмах ΠΈΠΌΠ΅Π΅Ρ‚ Π²Π°ΠΆΠ½ΠΎΠ΅ Π·Π½Π°Ρ‡Π΅Π½ΠΈΠ΅. Π’ΠΎ ΠΌΠ½ΠΎΠ³ΠΈΡ… случаях Ссли схСма рассчитана Π½Π° ΠΎΠ΄Π½ΠΎ Π½Π°ΠΏΡ€Π°Π²Π»Π΅Π½ΠΈΠ΅ Ρ‚ΠΎΠΊΠ°, Π° Π²Ρ‹ случайно Π΅Π³ΠΎ помСняСтС Π½Π° ΠΏΡ€ΠΎΡ‚ΠΈΠ²ΠΎΠΏΠΎΠ»ΠΎΠΆΠ½Ρ‹ΠΉ ΠΈΠ»ΠΈ вмСсто постоянного Ρ‚ΠΎΠΊΠ° ΠΏΠΎΠ΄ΠΊΠ»ΡŽΡ‡ΠΈΡ‚Π΅ ΠΏΠ΅Ρ€Π΅ΠΌΠ΅Π½Π½Ρ‹ΠΉ, Ρ‚ΠΎ скорСС всСго устройство просто Π²Ρ‹ΠΉΠ΄Π΅Ρ‚ ΠΈΠ· строя. МногиС ΠΏΠΎΠ»ΡƒΠΏΡ€ΠΎΠ²ΠΎΠ΄Π½ΠΈΠΊΠΈ, Ρ‡Ρ‚ΠΎ Ρ€Π°Π±ΠΎΡ‚Π°ΡŽΡ‚ Π² схСмах, ΠΏΡ€ΠΈ ΠΎΠ±Ρ€Π°Ρ‚Π½ΠΎΠΌ Π½Π°ΠΏΡ€Π°Π²Π»Π΅Π½ΠΈΠΈ Ρ‚ΠΎΠΊΠ° ΠΌΠΎΠ³ΡƒΡ‚ ΠΏΡ€ΠΎΠ±ΠΈΠ²Π°Ρ‚ΡŒΡΡ ΠΈ ΡΠ³ΠΎΡ€Π°Ρ‚ΡŒ. Π’Π°ΠΊ Ρ‡Ρ‚ΠΎ ΠΏΡ€ΠΈ ΠΏΠΎΠ΄ΠΊΠ»ΡŽΡ‡Π΅Π½ΠΈΠΈ элСктричСского питания Π½Π°ΠΏΡ€Π°Π²Π»Π΅Π½ΠΈΠ΅ Ρ‚ΠΎΠΊΠ° Π΄ΠΎΠ»ΠΆΠ½ΠΎ Π±Ρ‹Ρ‚ΡŒ Π²Π°ΠΌΠΈ строго ΡΠΎΠ±Π»ΡŽΠ΄Π°Ρ‚ΡŒΡΡ.

ΠŸΠΎΠ½ΡΡ‚ΠΈΠ΅ элСктричСского Ρ‚ΠΎΠΊΠ° элСктричСский Ρ‚ΠΎΠΊ понятиС ΠΎΠΏΡ€Π΅Π΄Π΅Π»Π΅Π½ΠΈΠ΅

ЭлСктричСство стало постоянным ΠΈ Π½Π΅Π·Π°ΠΌΠ΅Π½ΠΈΠΌΡ‹ΠΌ спутником чСловСчСства. НСсмотря Π½Π° Ρ‚ΠΎ Ρ‡Ρ‚ΠΎ Π² ΠΎΠΊΡ€ΡƒΠΆΠ΅Π½ΠΈΠΈ любого обыватСля постоянно находятся Ρ€Π°Π·Π»ΠΈΡ‡Π½Ρ‹Π΅ устройства, само понятиС элСктричСского Ρ‚ΠΎΠΊΠ° остаСтся Ρ‡Π΅ΠΌ-Ρ‚ΠΎ Π·Π°Π³Π°Π΄ΠΎΡ‡Π½Ρ‹ΠΌ ΠΈ нСдоступным ΠΏΠΎ сути. И это Π½Π΅ΡƒΠ΄ΠΈΠ²ΠΈΡ‚Π΅Π»ΡŒΠ½ΠΎ. ЭлСктричСский Ρ‚ΠΎΠΊ Π½Π΅Π²ΠΈΠ΄ΠΈΠΌ, Π½Π΅ ΠΎΠ±Π»Π°Π΄Π°Π΅Ρ‚ Π·Π°ΠΏΠ°Ρ…ΠΎΠΌ, Π° всС ΠΏΠΎΠΏΡ‹Ρ‚ΠΊΠΈ ΠΏΠΎΡ‚Ρ€ΠΎΠ³Π°Ρ‚ΡŒ Π΅Π³ΠΎ Ρ€ΡƒΠΊΠ°ΠΌΠΈ, ΠΊΠ°ΠΊ ΠΏΡ€Π°Π²ΠΈΠ»ΠΎ, приводят ΠΊ нСприятным ощущСниям ΠΈ грозят ΡΠ΅Ρ€ΡŒΠ΅Π·Π½Ρ‹ΠΌΠΈ Ρ‚Ρ€Π°Π²ΠΌΠ°ΠΌΠΈ, Π° Π² ΠΎΡ‚Π΄Π΅Π»ΡŒΠ½Ρ‹Ρ… случаях Π΄Π°ΠΆΠ΅ нСсовмСстимыми с Тизнью.

Если Π·Π°Π΄Π°Ρ‚ΡŒ Π±ΠΎΠ»ΡŒΡˆΠΈΠ½ΡΡ‚Π²Ρƒ людСй вопрос: Β«Π’Π°ΠΊ Ρ‡Ρ‚ΠΎ ΠΆΠ΅ Ρ‚Π°ΠΊΠΎΠ΅ элСктричСство?Β» Π’ΠΎ Π² Π»ΡƒΡ‡ΡˆΠ΅ΠΌ случаС ΠΎΡ‚Π²Π΅Ρ‚ΠΎΠΌ Π±ΡƒΠ΄Π΅Ρ‚: «УпорядочСнноС Π΄Π²ΠΈΠΆΠ΅Π½ΠΈΠ΅ элСктронов». И Π² ΠΏΡ€ΠΈΠ½Ρ†ΠΈΠΏΠ΅ это ΠΏΡ€Π°Π²ΠΈΠ»ΡŒΠ½ΠΎ. Однако элСктричСский Ρ‚ΠΎΠΊ ΠΎΠ±Π»Π°Π΄Π°Π΅Ρ‚ рядом ΠΏΠ°Ρ€Π°ΠΌΠ΅Ρ‚Ρ€ΠΎΠ², ΠΊΠΎΡ‚ΠΎΡ€Ρ‹Π΅ ΠΈΠΌΠ΅ΡŽΡ‚ количСствСнныС ΠΈ качСствСнныС опрСдСлСния, с Ρ‡Π΅ΠΌ ΠΈ ΠΏΠΎΠΏΡ€ΠΎΠ±ΡƒΠ΅ΠΌ Ρ€Π°Π·ΠΎΠ±Ρ€Π°Ρ‚ΡŒΡΡ.

ΠžΠ±Ρ‰ΠΈΠ΅ понятия ΠΎΠ± элСктричСском Ρ‚ΠΎΠΊΠ΅

Π’Π°ΠΊ Ρ‡Ρ‚ΠΎ ΠΆΠ΅ прСдставляСт собой элСктричСский Ρ‚ΠΎΠΊ? Π‘ΠΎΠ²Π΅Ρ€ΡˆΠ΅Π½Π½ΠΎ Π²Π΅Ρ€Π½ΠΎ – это упорядочСнноС ΠΏΠ΅Ρ€Π΅ΠΌΠ΅Ρ‰Π΅Π½ΠΈΠ΅ (Π΄Π²ΠΈΠΆΠ΅Π½ΠΈΠ΅) заряТСнных частиц, Π² качСствС ΠΊΠΎΡ‚ΠΎΡ€Ρ‹Ρ… Π²Ρ‹ΡΡ‚ΡƒΠΏΠ°ΡŽΡ‚ элСктроны, Ρ‡Ρ‚ΠΎ Ρ…Π°Ρ€Π°ΠΊΡ‚Π΅Ρ€Π½ΠΎ для ΠΌΠ΅Ρ‚Π°Π»Π»ΠΎΠ², ΠΈΠ»ΠΈ ΠΈΠΎΠ½Ρ‹, Ρ‡Ρ‚ΠΎ свойствСнно для элСктролитов.

Рассмотрим ΡΡƒΡ‰Π½ΠΎΡΡ‚ΡŒ элСктричСского Ρ‚ΠΎΠΊΠ° Π½Π° ΠΏΡ€ΠΈΠΌΠ΅Ρ€Π΅ ΠΌΠ΅Ρ‚Π°Π»Π»ΠΎΠ². Π’ солСвых растворах (элСктролитах) ΠΌΠ΅Ρ…Π°Π½ΠΈΠ·ΠΌ образования элСктричСства Π±ΡƒΠ΄Π΅Ρ‚ Π°Π½Π°Π»ΠΎΠ³ΠΈΡ‡Π΅Π½ ΠΈ Π½Π΅Ρ‚ смысла ΠΎΠΏΠΈΡΡ‹Π²Π°Ρ‚ΡŒ Π΅Π³ΠΎ Π΄ΠΎΠΏΠΎΠ»Π½ΠΈΡ‚Π΅Π»ΡŒΠ½ΠΎ. ΠšΡ€ΠΈΡΡ‚Π°Π»Π»ΠΈΡ‡Π΅ΡΠΊΠ°Ρ Ρ€Π΅ΡˆΠ΅Ρ‚ΠΊΠ° ΠΌΠ΅Ρ‚Π°Π»Π»ΠΎΠ² состоит ΠΈΠ· Π°Ρ‚ΠΎΠΌΠΎΠ², Π²ΠΎΠΊΡ€ΡƒΠ³ ядСр ΠΊΠΎΡ‚ΠΎΡ€Ρ‹Ρ… Π²Ρ€Π°Ρ‰Π°ΡŽΡ‚ΡΡ элСктроны. Но ΠΎΠ½ΠΈ Π½Π΅ ΠΎΠ±Π»Π°Π΄Π°ΡŽΡ‚ ΡΡ‚Π΅ΠΏΠ΅Π½ΡŒΡŽ свободы. Для Ρ‚ΠΎΠ³ΠΎ Ρ‡Ρ‚ΠΎΠ±Ρ‹ элСктрон стал свободным носитСлСм заряда Π½Π΅ΠΎΠ±Ρ…ΠΎΠ΄ΠΈΠΌΡ‹ ΠΎΠΏΡ€Π΅Π΄Π΅Π»Π΅Π½Π½Ρ‹Π΅ процСссы. НапримСр, химичСская рСакция. Π’.ΠΊ. сами ΠΏΠΎ сСбС заряды Π½Π΅ ΠΌΠΎΠ³ΡƒΡ‚ ΠΏΠ΅Ρ€Π΅ΠΌΠ΅Ρ‰Π°Ρ‚ΡŒΡΡ, Π½Π΅ΠΎΠ±Ρ…ΠΎΠ΄ΠΈΠΌΡ‹ΠΌ условиСм для сущСствования Ρ‚ΠΎΠΊΠ° являСтся Π½Π°Π»ΠΈΡ‡ΠΈΠ΅ элСктричСского Ρ‚ΠΎΠΊΠ°, ΠΊΠΎΡ‚ΠΎΡ€ΠΎΠ΅ Π²ΠΎΠ·Π½ΠΈΠΊΠ°Π΅Ρ‚, ΠΊΠΎΠ³Π΄Π° ΠΊΠ°Ρ‚ΠΎΠ΄ ΠΈ Π°Π½ΠΎΠ΄ Π±ΡƒΠ΄Π΅Ρ‚ Π·Π°ΠΌΠΊΠ½ΡƒΡ‚ ΠΏΡ€ΠΎΠ²ΠΎΠ΄Π½ΠΈΠΊΠΎΠΌ.

ΠŸΠ΅Ρ€Π²Ρ‹ΠΉ являСтся ΠΎΡ‚Ρ€ΠΈΡ†Π°Ρ‚Π΅Π»ΡŒΠ½Ρ‹ΠΌ полюсом источника, Π° Π²Ρ‚ΠΎΡ€ΠΎΠΉ соотвСтствСнно ΠΏΠΎΠ»ΠΎΠΆΠΈΡ‚Π΅Π»ΡŒΠ½Ρ‹ΠΌ. Π’ ΠΈΡ‚ΠΎΠ³Π΅ образуСтся элСктричСская Ρ†Π΅ΠΏΡŒ, Π² ΠΊΠΎΡ‚ΠΎΡ€ΡƒΡŽ ΠΌΠΎΠ³ΡƒΡ‚ Π±Ρ‹Ρ‚ΡŒ Π²ΠΊΠ»ΡŽΡ‡Π΅Π½Ρ‹ Ρ€Π°Π·Π»ΠΈΡ‡Π½Ρ‹Π΅ элСмСнты Π²Π»ΠΈΡΡŽΡ‰ΠΈΠ΅ качСствСнныС характСристики элСктротока.

ΠšΠ°Ρ‡Π΅ΡΡ‚Π²Π΅Π½Π½Ρ‹Π΅ ΠΈ количСствСнныС характСристики элСктричСского Ρ‚ΠΎΠΊΠ°

ΠžΡΠ½ΠΎΠ²Π½Ρ‹ΠΌΠΈ понятиями, ΠΊΠΎΡ‚ΠΎΡ€Ρ‹Π΅ Ρ…Π°Ρ€Π°ΠΊΡ‚Π΅Ρ€ΠΈΠ·ΡƒΡŽΡ‚ элСктричСский Ρ‚ΠΎΠΊ ΡΠ²Π»ΡΡŽΡ‚ΡΡ Π΅Π³ΠΎ сила, напряТСниС ΠΈ сопротивлСниС. Π­Ρ‚ΠΎ Π½Π°ΠΈΠ±ΠΎΠ»Π΅Π΅ распространСнныС ΠΏΠ°Ρ€Π°ΠΌΠ΅Ρ‚Ρ€Ρ‹, ΠΎ ΠΊΠΎΡ‚ΠΎΡ€Ρ‹Ρ… ΠΌΠΎΠΆΠ½ΠΎ ΡƒΡΠ»Ρ‹ΡˆΠ°Ρ‚ΡŒ. Π˜ΠΌΠ΅Π΅Ρ‚ мСсто ΠΈ ряд Π΄Ρ€ΡƒΠ³ΠΈΡ… характСристик. НапримСр, ΠΌΠΎΡ‰Π½ΠΎΡΡ‚ΡŒ Ρ‚ΠΎΠΊΠ°, Π΅Π³ΠΎ ΠΏΠ»ΠΎΡ‚Π½ΠΎΡΡ‚ΡŒ ΠΈ Ρ‚.Π΄. Но ΠΎ Π½ΠΈΡ… ΠΏΠΎ своСй спСцификС, Π½Π΅ часто ΠΌΠΎΠΆΠ½ΠΎ ΡƒΡΠ»Ρ‹ΡˆΠ°Ρ‚ΡŒ Π² Π±Ρ‹Ρ‚Ρƒ.

Π§Ρ‚ΠΎ Ρ‚Π°ΠΊΠΎΠ΅ сила Ρ‚ΠΎΠΊΠ°? ВсС просто, это количСство носитСлСй заряда ΠΊΠΎΡ‚ΠΎΡ€ΠΎΠ΅ пСрСсСкло сСчСниС проводящСго элСмСнта Π·Π° Π΅Π΄ΠΈΠ½ΠΈΡ†Ρƒ Π²Ρ€Π΅ΠΌΠ΅Π½ΠΈ. Π‘ΠΈΠ»Π° Ρ‚ΠΎΠΊΠ° ΠΈΠΌΠ΅Π΅Ρ‚ ΠΎΠ±ΠΎΠ·Π½Π°Ρ‡Π΅Π½ΠΈΠ΅ с систСмы БИ – Β«IΒ», Π° Π΅Π΄ΠΈΠ½ΠΈΡ†Π΅ΠΉ измСрСния установлСн Π°ΠΌΠΏΠ΅Ρ€. Π€ΠΎΡ€ΠΌΡƒΠ»Π° для силы Ρ‚ΠΎΠΊΠ°, ΠΈΠ· опрСдСлСния, Π±ΡƒΠ΄Π΅Ρ‚ ΡΠ»Π΅Π΄ΡƒΡŽΡ‰Π΅ΠΉ:

I=βˆ†Q/βˆ†t, Π³Π΄Π΅ βˆ†Q – количСство заряда, Π° βˆ†t – ΠΏΡ€ΠΎΠΌΠ΅ΠΆΡƒΡ‚ΠΎΠΊ Π²Ρ€Π΅ΠΌΠ΅Π½ΠΈ Π·Π° ΠΊΠΎΡ‚ΠΎΡ€Ρ‹ΠΉ ΠΎΠ½ΠΎ пСрСсСкаСт сСчСниС ΠΏΡ€ΠΎΠ²ΠΎΠ΄Π½ΠΈΠΊΠ°.

Π‘Π»Π΅Π΄ΡƒΡŽΡ‰ΠΈΠΌ понятиСм, ΠΊΠΎΡ‚ΠΎΡ€ΠΎΠ΅ Ρ…Π°Ρ€Π°ΠΊΡ‚Π΅Ρ€ΠΈΠ·ΡƒΠ΅Ρ‚ элСктричСский Ρ‚ΠΎΠΊ являСтся напряТСниС. Оно ΠΏΠΎ своСй сути являСтся Ρ€Π°Π±ΠΎΡ‚ΠΎΠΉ элСктричСского поля ΠΏΠΎ ΠΏΠ΅Ρ€Π΅ΠΌΠ΅Ρ‰Π΅Π½ΠΈΡŽ заряда ΠΎΡ‚ ΠΎΠ΄Π½ΠΎΠ³ΠΎ элСктрода ΠΊ Π΄Ρ€ΡƒΠ³ΠΎΠΌΡƒ. ΠŸΠΎΡΠΊΠΎΠ»ΡŒΠΊΡƒ ΠΏΠΎΡ‚Π΅Π½Ρ†ΠΈΠ°Π»Ρ‹ ΠΊΠΎΠ½Ρ‚Π°ΠΊΡ‚ΠΎΠ² источника элСктричСского Ρ‚ΠΎΠΊΠ° Ρ€Π°Π·Π½Ρ‹Π΅, Ρ‚ΠΎ напряТСниС ΠΌΠΎΠΆΠ½ΠΎ ΡΡ‡ΠΈΡ‚Π°Ρ‚ΡŒ Ρ€Π°Π·Π½ΠΎΡΡ‚ΡŒΡŽ этих ΠΏΠΎΡ‚Π΅Π½Ρ†ΠΈΠ°Π»ΠΎΠ². Π’Π°ΠΊΠΈΠΌ ΠΎΠ±Ρ€Π°Π·ΠΎΠΌ, напряТСниС являСтся ΠΎΡ‚Π½ΠΎΡˆΠ΅Π½ΠΈΠ΅ΠΌ Ρ€Π°Π±ΠΎΡ‚Ρ‹ ΠΊ заряду. Π€ΠΎΡ€ΠΌΡƒΠ»Π° выглядит ΡΠ»Π΅Π΄ΡƒΡŽΡ‰ΠΈΠΌ ΠΎΠ±Ρ€Π°Π·ΠΎΠΌ: U=Axq. Π•Π΄ΠΈΠ½ΠΈΡ†Π΅ΠΉ измСрСния разности ΠΏΠΎΡ‚Π΅Π½Ρ†ΠΈΠ°Π»ΠΎΠ² Π² систСмС Π‘ΠΈ принят Π²ΠΎΠ»ΡŒΡ‚. Π‘ Ρ€Π°Π±ΠΎΡ‚ΠΎΠΉ нСпосрСдствСнно связано понятиС мощности элСктричСского Ρ‚ΠΎΠΊΠ°, которая являСтся ΠΏΡ€ΠΎΠΈΠ·Π²ΠΎΠ΄Π½ΠΎΠΉ ΠΎΡ‚ Π½Π΅Π΅. Π§Π΅Ρ€Π΅Π· напряТСниС ΠΈ силу, ΠΌΠΎΡ‰Π½ΠΎΡΡ‚ΡŒ (Π ) ΠΌΠΎΠΆΠ½ΠΎ Π²Ρ‹Ρ€Π°Π·ΠΈΡ‚ΡŒ ΡΠ»Π΅Π΄ΡƒΡŽΡ‰ΠΈΠΌ ΠΎΠ±Ρ€Π°Π·ΠΎΠΌ – Π =UxI.

Π’Π·Π°ΠΈΠΌΠΎΡΠ²ΡΠ·ΡŒ напряТСния ΠΈ силы Ρ‚ΠΎΠΊΠ° Π±Ρ‹Π»Π° ΠΎΠΏΡ€Π΅Π΄Π΅Π»Π΅Π½Π° ΠΎΠΏΡ‹Ρ‚Π½Ρ‹ΠΌ ΠΏΡƒΡ‚Π΅ΠΌ Π“. Омом. ΠŸΡ€Π°Π²Π΄Π° для этого ΠΏΡ€ΠΈΡˆΠ»ΠΎΡΡŒ ввСсти Ρ‚Π°ΠΊΠΎΠ΅ понятиС ΠΊΠ°ΠΊ сопротивлСниС ΠΏΡ€ΠΎΠ²ΠΎΠ΄Π½ΠΈΠΊΠ°. Π­Ρ‚Π° Π²Π΅Π»ΠΈΡ‡ΠΈΠ½Π° Π½Π΅ ΠΎΡ‚Π½ΠΎΡΠΈΡ‚ΡŒΡΡ ΠΊ количСствСнным характСристикам элСктричСского Ρ‚ΠΎΠΊΠ°, Π½ΠΎ позволяСт ΠΎΠΏΡ€Π΅Π΄Π΅Π»ΠΈΡ‚ΡŒ Π½Π΅ΠΎΠ±Ρ…ΠΎΠ΄ΠΈΠΌΡ‹Π΅ ΠΏΠ°Ρ€Π°ΠΌΠ΅Ρ‚Ρ€Ρ‹. По Π·Π°ΠΊΠΎΠ½Ρƒ Ома взаимосвязь силы Ρ‚ΠΎΠΊΠ°, напряТСния ΠΈ сопротивлСния Π²ΠΈΠ΄Π½Π° ΠΈΠ· выраТСния I=U/R.

ΠŸΠΎΠ½ΡΡ‚ΠΈΠ΅ постоянного ΠΈ ΠΏΠ΅Ρ€Π΅ΠΌΠ΅Π½Π½ΠΎΠ³ΠΎ Ρ‚ΠΎΠΊΠ°

ΠŸΠ΅Ρ€Π΅ΠΌΠ΅Ρ‰Π΅Π½ΠΈΠ΅ носитСлСй заряда, ΠΊΠΎΡ‚ΠΎΡ€ΠΎΠ΅ ΠΏΠΎ своСму Π½Π°ΠΏΡ€Π°Π²Π»Π΅Π½ΠΈΡŽ Π½Π΅ΠΈΠ·ΠΌΠ΅Π½Π½ΠΎ называСтся постоянным элСктричСским Ρ‚ΠΎΠΊΠΎΠΌ. НСобходимо ΠΏΠΎΡΡΠ½ΠΈΡ‚ΡŒ, Ρ‡Ρ‚ΠΎ ΠΏΡ€ΠΈ нСвозмоТности измСнСния своСго направлСния, Π΄Π°Π½Π½Ρ‹ΠΉ Π²ΠΈΠ΄ Ρ‚ΠΎΠΊΠ° ΠΌΠΎΠΆΠ΅Ρ‚ ΠΌΠ΅Π½ΡΡ‚ΡŒ свои количСствСнныС характСристики. НапримСр, Π² зависимости ΠΎΡ‚ элСктричСской схСмы ΠΏΠΎΠ΄ΠΊΠ»ΡŽΡ‡Π΅Π½ΠΈΡ ΠΏΡ€ΠΎΠ²ΠΎΠ΄Π½ΠΈΠΊΠΎΠ² (элСмСнтов) ΠΌΠΎΠΆΠ΅Ρ‚ измСнятся сила, напряТСниС Π½Π° участкС ΠΈ Ρ‚.Π΄. ΠŸΠΎΡΡ‚ΠΎΡΠ½Π½Ρ‹ΠΉ Ρ‚ΠΎΠΊ Ρ…Π°Ρ€Π°ΠΊΡ‚Π΅Ρ€Π΅Π½, Π½Π°ΠΏΡ€ΠΈΠΌΠ΅Ρ€, для Π³Π°Π»ΡŒΠ²Π°Π½ΠΈΡ‡Π΅ΡΠΊΠΈΡ… элСмСнтов.

А Π²ΠΎΡ‚ ΠΏΠ΅Ρ€Π΅ΠΌΠ΅Π½Π½Ρ‹ΠΉ Ρ‚ΠΎΠΊ, Π½Π° протяТСнии ΠΊΠΎΠ½ΠΊΡ€Π΅Ρ‚Π½ΠΎΠ³ΠΎ ΠΏΡ€ΠΎΠΌΠ΅ΠΆΡƒΡ‚ΠΊΠ° Π²Ρ€Π΅ΠΌΠ΅Π½ΠΈ, мСняСт своС Π½Π°ΠΏΡ€Π°Π²Π»Π΅Π½ΠΈΠ΅ Π² ΠΎΠΏΡ€Π΅Π΄Π΅Π»Π΅Π½Π½ΠΎΠΉ закономСрности. Π Π΅Π³ΡƒΠ»ΡΡ€Π½ΠΎΡΡ‚ΡŒ ΠΈΠ»ΠΈ ΠΏΠ΅Ρ€ΠΈΠΎΠ΄ΠΈΡ‡Π½ΠΎΡΡ‚ΡŒ измСнСния называСтся частотой ΠΈ измСряСтся Π² Π³Π΅Ρ€Ρ†Π°Ρ… (Π“Ρ†). Π Π°ΡΠΏΡ€ΠΎΡΡ‚Ρ€Π°Π½Π΅Π½Π½ΠΎΡΡ‚ΡŒ элСктричСского Ρ‚ΠΎΠΊΠ° ΠΏΠ΅Ρ€Π΅ΠΌΠ΅Π½Π½ΠΎΠ³ΠΎ Ρ‚ΠΈΠΏΠ° Π² ΠΏΠ΅Ρ€Π²ΡƒΡŽ ΠΎΡ‡Π΅Ρ€Π΅Π΄ΡŒ обусловлСна Π½ΠΈΠ·ΠΊΠΈΠΌΠΈ потСрями ΠΏΡ€ΠΈ ΠΏΠ΅Ρ€Π΅Π΄Π°Ρ‡Π΅ Π½Π° большиС расстояния.


ΠΊΠ°ΠΊΠΎΠ²Ρ‹, Π² ΠΏΡ€ΠΎΠ²ΠΎΠ΄Π½ΠΈΠΊΠ΅, Π² Ρ†Π΅ΠΏΠΈ

Π£ ΠΌΠ½ΠΎΠ³ΠΈΡ… Π·Π°ΠΏΠΎΠΌΠΈΠ½Π°Π½ΠΈΠ΅ основных понятий элСктродинамики Π²Ρ‹Π·Ρ‹Π²Π°Π΅Ρ‚ ΠΏΡ€ΠΎΠ±Π»Π΅ΠΌΡƒ. Для Π»ΡƒΡ‡ΡˆΠ΅Π³ΠΎ усвоСния ΠΌΠ°Ρ‚Π΅Ρ€ΠΈΠ°Π»Π° Π½Π΅ΠΎΠ±Ρ…ΠΎΠ΄ΠΈΠΌΠ° ассоциация, которая ΠΏΠΎΠΊΠ°ΠΆΠ΅Ρ‚ смысл Π³Π»Π°Π²Π½Ρ‹Ρ… ΠΎΠΏΡ€Π΅Π΄Π΅Π»Π΅Π½ΠΈΠΉ ΠΈ явлСний. Рассмотрим Π½Π° ΠΏΡ€ΠΎΡΡ‚Π΅ΠΉΡˆΠΈΡ… ΠΏΡ€ΠΈΠΌΠ΅Ρ€Π°Ρ… Ρ€Π°ΡΡΠΌΠ°Ρ‚Ρ€ΠΈΠ²Π°ΡŽΡ‚ΡΡ ваТнСйшиС понятия, связанныС с элСктричСством.

Π‘ΡƒΡ‚ΡŒ элСктричСского Ρ‚ΠΎΠΊΠ° ΠΈ поля

ЭлСктричСский Ρ‚ΠΎΠΊ – Π΄Π²ΠΈΠΆΠ΅Π½ΠΈΠ΅ заряТСнных частиц Π² ΠΎΠ΄Π½ΠΎΠΌ Π½Π°ΠΏΡ€Π°Π²Π»Π΅Π½ΠΈΠΈ. Он Π½Π°ΠΏΡ€Π°Π²Π»Π΅Π½ Ρ‚ΡƒΠ΄Π°, ΠΊΡƒΠ΄Π° двиТутся ΠΏΠΎΠ»ΠΎΠΆΠΈΡ‚Π΅Π»ΡŒΠ½Ρ‹Π΅ заряды. Π”Π°Π½Π½ΠΎΠ΅ явлСниС ΠΌΠΎΠΆΠ½ΠΎ ΡΡ€Π°Π²Π½ΠΈΡ‚ΡŒ с двиТСниями ΠΌΠΎΠ»Π΅ΠΊΡƒΠ» Π²ΠΎΠ΄Ρ‹ Π² Ρ‚Ρ€ΡƒΠ±Π΅. Π₯Π°ΠΎΡ‚ΠΈΡ‡Π½Ρ‹Π΅ колСбания Π½Π΅ ΡΠ²Π»ΡΡŽΡ‚ΡΡ Ρ‚Π΅Ρ‡Π΅Π½ΠΈΠ΅ΠΌ, Ρ‚Π°ΠΊ ΠΊΠ°ΠΊ ΠΈΡ… суммарная Π½Π°ΠΏΡ€Π°Π²Π»Π΅Π½Π½ΠΎΡΡ‚ΡŒ Ρ€Π°Π²Π½Π° Π½ΡƒΠ»ΡŽ.

Π§Ρ‚ΠΎΠ±Ρ‹ всС ΠΌΠΎΠ»Π΅ΠΊΡƒΠ»Ρ‹ Π½Π°Ρ‡Π°Π»ΠΈ ΠΏΠ΅Ρ€Π΅Π΄Π²ΠΈΠ³Π°Ρ‚ΡŒΡΡ Π² ΠΎΠ΄Π½ΠΎΠΌ Π½Π°ΠΏΡ€Π°Π²Π»Π΅Π½ΠΈΠΈ, Π½ΡƒΠΆΠ½ΠΎ ΠΏΡ€ΠΈΠ»ΠΎΠΆΠΈΡ‚ΡŒ силу. НапримСр, ΠΏΠΎΠ΄Π½ΡΡ‚ΡŒ ΠΎΠ΄ΠΈΠ½ ΠΊΠΎΠ½Π΅Ρ† Ρ‚Ρ€ΡƒΠ±Ρ‹, Ρ‚ΠΎΠ³Π΄Π° Π²ΠΎΠ΄Π° Π½Π°Ρ‡Π½Ρ‘Ρ‚ ΠΏΠ΅Ρ€Π΅Ρ‚Π΅ΠΊΠ°Ρ‚ΡŒ Π² ΠΏΡ€ΠΎΡ‚ΠΈΠ²ΠΎΠΏΠΎΠ»ΠΎΠΆΠ½ΠΎΠΌ Π½Π°ΠΏΡ€Π°Π²Π»Π΅Π½ΠΈΠΈ. А для появлСния Ρ‚ΠΎΠΊΠ° Π½ΡƒΠΆΠ½ΠΎ ΡΠΎΠ·Π΄Π°Ρ‚ΡŒ элСктричСскоС ΠΏΠΎΠ»Π΅. Π’Π°ΠΊ ΠΊΠ°ΠΊ частицы заряТСны (ΠΎΠ½ΠΈ ΠΏΠΎΠ»ΠΎΠΆΠΈΡ‚Π΅Π»ΡŒΠ½Ρ‹Π΅), Ρ‚ΠΎ Π² присутствии поля ΠΎΠ½ΠΈ Π±ΡƒΠ΄ΡƒΡ‚ ΠΏΠ΅Ρ€Π΅Π΄Π²ΠΈΠ³Π°Ρ‚ΡŒΡΡ Π² ΠΎΠ΄Π½ΠΎΠΌ Π½Π°ΠΏΡ€Π°Π²Π»Π΅Π½ΠΈΠΈ.

БущСствуСт Π΄Π²Π΅ разновидности Ρ‚ΠΎΠΊΠ°:

  • постоянный – Ρƒ ΠΊΠΎΡ‚ΠΎΡ€ΠΎΠ³ΠΎ Π½Π°ΠΏΡ€Π°Π²Π»Π΅Π½ΠΈΠ΅ двиТСния заряда Π½Π΅ мСняСтся со Π²Ρ€Π΅ΠΌΠ΅Π½Π΅ΠΌ;
  • ΠΏΠ΅Ρ€Π΅ΠΌΠ΅Π½Π½Ρ‹ΠΉ – Ρƒ ΠΊΠΎΡ‚ΠΎΡ€ΠΎΠ³ΠΎ Π½Π°ΠΏΡ€Π°Π²Π»Π΅Π½ΠΈΠ΅ двиТСния пСриодичСски мСняСтся.

Π”Π°Π½Π½ΠΎΠ΅ явлСниС сопровоТдаСтся Π½Π΅ΠΊΠΎΡ‚ΠΎΡ€Ρ‹ΠΌΠΈ процСссами:

  • ΠΌΠ°Π³Π½ΠΈΡ‚Π½Ρ‹ΠΌΠΈ, Ρ‡Ρ‚ΠΎ Ρ…Π°Ρ€Π°ΠΊΡ‚Π΅Ρ€Π½ΠΎ для всСх ΠΏΡ€ΠΎΠ²ΠΎΠ΄Π½ΠΈΠΊΠΎΠ²;
  • Ρ‚Π΅ΠΏΠ»ΠΎΠ²Ρ‹ΠΌΠΈ, Ρ‡Ρ‚ΠΎ Ρ‚Π°ΠΊΠΆΠ΅ Π΅ΡΡ‚ΡŒ Π²Π΅Π·Π΄Π΅, ΠΊΡ€ΠΎΠΌΠ΅ свСрхпроводников;
  • химичСскими, Ρ‡Ρ‚ΠΎ имССтся лишь Π² элСктролитах.

Условия сущСствования элСктричСского Ρ‚ΠΎΠΊΠ° Π² ΠΏΡ€ΠΎΠ²ΠΎΠ΄Π½ΠΈΠΊΠ΅:

  • присутствиС Π² ΠΏΡ€ΠΎΠ²ΠΎΠ΄Π½ΠΈΠΊΠ΅ свободных заряТСнных частиц;
  • появлСниС элСктричСского поля.

ΠžΡΠ½ΠΎΠ²Π½Ρ‹Π΅ понятия

ΠŸΡ€Π΅ΠΆΠ΄Π΅ Ρ‡Π΅ΠΌ Ρ€Π°Π·Π±ΠΈΡ€Π°Ρ‚ΡŒ, ΠΊΠ°ΠΊΠΎΠ²Ρ‹ условия сущСствования элСктричСского Ρ‚ΠΎΠΊΠ°, ΠΏΠΎΠ»Π΅Π·Π½ΠΎ ΠΏΠΎΠ·Π½Π°ΠΊΠΎΠΌΠΈΡ‚ΡŒΡΡ с основными понятиями, ΠΊΠΎΡ‚ΠΎΡ€Ρ‹Π΅ часто ΠΈΡΠΏΠΎΠ»ΡŒΠ·ΡƒΡŽΡ‚ΡΡ ΠΏΡ€ΠΈ описании процСссов.

Π‘ΠΈΠ»Π° Ρ‚ΠΎΠΊΠ°

Π’ Ρ„ΠΎΡ€ΠΌΡƒΠ»Π°Ρ… обозначаСтся ΠΊΠ°ΠΊ I (ΠΈ), Π΅Π΄ΠΈΠ½ΠΈΡ†Π΅ΠΉ измСрСния являСтся Π°ΠΌΠΏΠ΅Ρ€ (А). Π‘ΠΈΠ»ΠΎΠΉ Ρ‚ΠΎΠΊΠ° Π½Π°Π·Ρ‹Π²Π°ΡŽΡ‚ Ρ‚ΠΎ, ΠΊΠ°ΠΊΠΎΠΉ заряд ΠΏΡ€ΠΎΡ‚Π΅ΠΊΠ°Π΅Ρ‚ Ρ‡Π΅Ρ€Π΅Π· ΠΏΠΎΠΏΠ΅Ρ€Π΅Ρ‡Π½ΠΎΠ΅ сСчСниС ΠΏΡ€ΠΎΠ²ΠΎΠ΄Π½ΠΈΠΊΠ° Π·Π° Π΅Π΄ΠΈΠ½ΠΈΡ†Ρƒ Π²Ρ€Π΅ΠΌΠ΅Π½ΠΈ. АссоциациСй ΠΌΠΎΠΆΠ΅Ρ‚ ΡΡ‚Π°Ρ‚ΡŒ Ρ‚ΠΎ, сколько Π²ΠΎΠ΄Ρ‹ ΠΏΡ€ΠΎΡ‚Π΅ΠΊΠ°Π΅Ρ‚ Ρ‡Π΅Ρ€Π΅Π· сСчСниС Ρ‚Ρ€ΡƒΠ±Ρ‹ Π·Π° Π΅Π΄ΠΈΠ½ΠΈΡ†Ρƒ Π²Ρ€Π΅ΠΌΠ΅Π½ΠΈ.

НапряТСниС

Π’ Ρ„ΠΎΡ€ΠΌΡƒΠ»Π°Ρ… обознаСтся ΠΊΠ°ΠΊ U, Π΅Π΄ΠΈΠ½ΠΈΡ†Π΅ΠΉ измСрСния являСтся Π’ΠΎΠ»ΡŒΡ‚ (Π’). НапряТСниС – это Ρ€Π°Π±ΠΎΡ‚Π°, которая ΡΠΎΠ²Π΅Ρ€ΡˆΠ°Π΅Ρ‚ΡΡ для пСрСмСщСния ΠΏΠΎ элСктричСской Ρ†Π΅ΠΏΠΈ ΠΎΠ΄Π½ΠΎΠΉ ΠΏΠΎΠ»ΠΎΠΆΠΈΡ‚Π΅Π»ΡŒΠ½ΠΎ заряТСнной частицы. Π’ сравнСнии с Π²ΠΎΠ΄ΠΎΠΉ ΠΌΠΎΠΆΠ½ΠΎ привСсти Ρ‚Π°ΠΊΡƒΡŽ Π°ΡΡΠΎΡ†ΠΈΠ°Ρ†ΠΈΡŽ, Ρ‡Ρ‚ΠΎ это ΠΎΠ±ΡŠΡ‘ΠΌ Ρ€Π°Π±ΠΎΡ‚Ρ‹, ΠΊΠΎΡ‚ΠΎΡ€Ρ‹ΠΉ Π½ΡƒΠΆΠ΅Π½ для пСрСмСщСния Π»ΠΈΡ‚Ρ€Π° Тидкости ΠΈΠ· ΠΎΠ΄Π½ΠΎΠ³ΠΎ ΠΊΠΎΠ½Ρ†Π° Ρ‚Ρ€ΡƒΠ±Ρ‹ Π² Π΄Ρ€ΡƒΠ³ΠΎΠΉ.

Π‘ΠΎΠΏΡ€ΠΎΡ‚ΠΈΠ²Π»Π΅Π½ΠΈΠ΅

Π‘ΠΎΠΏΡ€ΠΎΡ‚ΠΈΠ²Π»Π΅Π½ΠΈΠ΅ Π² Ρ„ΠΎΡ€ΠΌΡƒΠ»Π°Ρ… обозначаСтся ΠΊΠ°ΠΊ R, Π΅Π΄ΠΈΠ½ΠΈΡ†Π΅ΠΉ измСрСния слуТит Ом (Ом). ЀизичСский смысл явлСния Π·Π°ΠΊΠ»ΡŽΡ‡Π°Π΅Ρ‚ΡΡ Π² сопротивлСнии, ΠΊΠΎΡ‚ΠΎΡ€ΠΎΠ΅ ΠΎΠΊΠ°Π·Ρ‹Π²Π°Π΅Ρ‚ ΠΏΡ€ΠΎΠ²ΠΎΠ΄Π½ΠΈΠΊ ΠΏΡ€ΠΈ Π΄Π²ΠΈΠΆΠ΅Π½ΠΈΠΈ частиц. Π’ Π²Π°ΠΊΡƒΡƒΠΌΠ΅ заряды ΠΏΠ΅Ρ€Π΅ΠΌΠ΅Ρ‰Π°ΡŽΡ‚ΡΡ Π³ΠΎΡ€Π°Π·Π΄ΠΎ быстрСС, Ρ‚Π°ΠΊ ΠΊΠ°ΠΊ ΠΈΠΌ Π½ΠΈΡ‡Ρ‚ΠΎ Π½Π΅ ΠΌΠ΅ΡˆΠ°Π΅Ρ‚, Π½ΠΎ Π² вСщСствС Π½Π° Π½ΠΈΡ… лоТится Π΄ΠΎΠΏΠΎΠ»Π½ΠΈΡ‚Π΅Π»ΡŒΠ½Π°Ρ Π½Π°Π³Ρ€ΡƒΠ·ΠΊΠ° ΠΈΠ·-Π·Π° кристалличСской Ρ€Π΅ΡˆΡ‘Ρ‚ΠΊΠΈ.

УдСльноС сопротивлСниС для ΠΊΠ°ΠΆΠ΄ΠΎΠ³ΠΎ ΠΌΠ°Ρ‚Π΅Ρ€ΠΈΠ°Π»Π° своё, это статичныС Ρ†ΠΈΡ„Ρ€Ρ‹, ΠΊΠΎΡ‚ΠΎΡ€Ρ‹Π΅ ΠΌΠΎΠΆΠ½ΠΎ ΠΏΠΎΡΠΌΠΎΡ‚Ρ€Π΅Ρ‚ΡŒ Π² справочникС. Π˜Π½Ρ‚Π΅Ρ€Π΅ΡΠ½ΠΎ, Ρ‡Ρ‚ΠΎ этот ΠΏΠ°Ρ€Π°ΠΌΠ΅Ρ‚Ρ€ сильно зависит ΠΎΡ‚ Ρ‚Π΅ΠΌΠΏΠ΅Ρ€Π°Ρ‚ΡƒΡ€Ρ‹, Ρ‡Ρ‚ΠΎ ΠΎΠ±ΡŠΡΡΠ½ΡΠ΅Ρ‚ΡΡ Π΄Π²ΠΈΠΆΠ΅Π½ΠΈΠ΅ΠΌ элСмСнтов кристалличСской Ρ€Π΅ΡˆΡ‘Ρ‚ΠΊΠΈ, Π° Ρ‚Π°ΠΊΠΆΠ΅ высвобоТдСниСм частиц ΠΏΡ€ΠΈ ΠΏΠΎΠ²Ρ‹ΡˆΠ΅Π½ΠΈΠΈ Ρ‚Π΅ΠΌΠΏΠ΅Ρ€Π°Ρ‚ΡƒΡ€Ρ‹.

Π§Ρ‚ΠΎΠ±Ρ‹ Π±Ρ‹Π»ΠΎ понятнСй, Π½ΡƒΠΆΠ½ΠΎ Π²ΡΠΏΠΎΠΌΠ½ΠΈΡ‚ΡŒ Π΄Π²ΠΈΠΆΠ΅Π½ΠΈΠ΅ Π²ΠΎΠ΄Ρ‹ ΠΏΠΎ Ρ‚Ρ€ΡƒΠ±Π΅. Π’Π΅Π΄ΡŒ ΠΆΠΈΠ΄ΠΊΠΎΡΡ‚ΡŒ ΠΏΡ€ΠΈ Ρ‚Π΅Ρ‡Π΅Π½ΠΈΠΈ Ρ‚ΠΎΠΆΠ΅ испытываСт сопротивлСниС, ΠΊΠΎΡ‚ΠΎΡ€ΠΎΠ΅ выраТаСтся Π² силС трСния. Но для Π±ΠΎΠ»Π΅Π΅ схоТСй ΠΊΠ°Ρ€Ρ‚ΠΈΠ½Ρ‹ Π½ΡƒΠΆΠ½ΠΎ ΠΏΡ€Π΅Π΄ΡΡ‚Π°Π²ΠΈΡ‚ΡŒ, Ρ‡Ρ‚ΠΎ вся Ρ‚Ρ€ΡƒΠ±Π° уставлСна Ρ€Π΅ΡˆΡ‘Ρ‚ΠΊΠ°ΠΌΠΈ, ΠΊΠΎΡ‚ΠΎΡ€Ρ‹Π΅ Ρ‚ΠΎΠΆΠ΅ ΠΏΡ€ΠΈΡ‚ΠΎΡ€ΠΌΠ°ΠΆΠΈΠ²Π°ΡŽΡ‚ Π΄Π²ΠΈΠΆΠ΅Π½ΠΈΠ΅ Π²ΠΎΠ΄Ρ‹.

ЭлСктричСскоС ΠΏΠΎΠ»Π΅

Π’Π°ΠΆΠ½Ρ‹ΠΌ условиСм сущСствования элСктричСского Ρ‚ΠΎΠΊΠ° являСтся ΠΎΠ΄Π½ΠΎΠΈΠΌΡ‘Π½Π½ΠΎΠ΅ ΠΏΠΎΠ»Π΅. Оно трСбуСтся для Ρ‚ΠΎΠ³ΠΎ, Ρ‡Ρ‚ΠΎΠ±Ρ‹ всС заряТСнныС частицы Π½Π°Ρ‡Π°Π»ΠΈ Π΄Π²ΠΈΠ³Π°Ρ‚ΡŒΡΡ Π² ΠΎΠ΄Π½ΠΎΠΌ Π½Π°ΠΏΡ€Π°Π²Π»Π΅Π½ΠΈΠΈ. Π€ΠΎΡ€ΠΌΡƒΠ»Π° этого явлСния такая: Π½Π° заряд q Π² созданном ΠΏΠΎΠ»Π΅ с напряТСниСм E дСйствуСт сила F = q*E. ИмСнно ΠΎΠ½Π° ΠΈ Π²Ρ‹Π½ΡƒΠΆΠ΄Π°Π΅Ρ‚ заряТСнныС частицы ΠΏΠ΅Ρ€Π΅ΠΌΠ΅Ρ‰Π°Ρ‚ΡŒΡΡ Π² ΠΎΠΏΡ€Π΅Π΄Π΅Π»Ρ‘Π½Π½ΠΎΠΌ Π½Π°ΠΏΡ€Π°Π²Π»Π΅Π½ΠΈΠΈ.

ΠŸΡ€ΠΈ отсутствии поля Π² ΠΏΡ€ΠΎΠ²ΠΎΠ΄Π½ΠΈΠΊΠ΅ Π½Π΅Ρ‚ разности ΠΏΠΎΡ‚Π΅Π½Ρ†ΠΈΠ°Π»ΠΎΠ² (Π² Π»ΡŽΠ±Ρ‹Ρ… Π΅Π³ΠΎ Π΄Π²ΡƒΡ… Ρ‚ΠΎΡ‡ΠΊΠ°Ρ…). ΠŸΠΎΡΡ‚ΠΎΠΌΡƒ Π΄ΠΎΠΊΠ°Π·Π°Ρ‚Π΅Π»ΡŒΡΡ‚Π²ΠΎΠΌ наличия Π² ΠΏΡ€ΠΎΠ²ΠΎΠ΄Π½ΠΈΠΊΠ΅ поля ΠΌΠΎΠΆΠ΅Ρ‚ ΠΏΠΎΡΠ»ΡƒΠΆΠΈΡ‚ΡŒ нСравСнство ΠΏΠΎΡ‚Π΅Π½Ρ†ΠΈΠ°Π»ΠΎΠ². ДвиТущиСся ΠΆΠ΅ заряды стрСмятся эту Ρ€Π°Π·Π½ΠΎΡΡ‚ΡŒ Π½Π΅ΠΉΡ‚Ρ€Π°Π»ΠΈΠ·ΠΎΠ²Π°Ρ‚ΡŒ. ИмСнно ΠΈΠ·-Π·Π° этого ΠΏΠΎΠ»Π΅ Π½Π΅ способно Π²Π΅Ρ‡Π½ΠΎ ΠΏΠΎΠ΄Π΄Π΅Ρ€ΠΆΠΈΠ²Π°Ρ‚ΡŒ Ρ‚ΠΎΠΊ. Π’Π°ΠΊ ΠΊΠ°ΠΊ с Ρ‚Π΅Ρ‡Π΅Π½ΠΈΠ΅ΠΌ Π²Ρ€Π΅ΠΌΠ΅Π½ΠΈ заряТСнныС частицы ΡƒΠ±Π΅Ρ€ΡƒΡ‚ нСравСнство ΠΏΠΎΡ‚Π΅Π½Ρ†ΠΈΠ°Π»ΠΎΠ², Ρ‡Ρ‚ΠΎ ΠΏΡ€ΠΈΠ²Π΅Π΄Ρ‘Ρ‚ ΠΊ ΠΈΡΡ‡Π΅Π·Π½ΠΎΠ²Π΅Π½ΠΈΡŽ элСктричСского поля.

Π§Ρ‚ΠΎΠ±Ρ‹ Π΄Π²ΠΈΠΆΠ΅Π½ΠΈΠ΅ зарядов Π±Ρ‹Π»ΠΎ Π² Π·Π°ΠΌΠΊΠ½ΡƒΡ‚ΠΎΠΉ Ρ†Π΅ΠΏΠΈ постоянно, Π½ΡƒΠΆΠ΅Π½ внСшний источник энСргии нСэлСктричСской ΠΏΡ€ΠΈΡ€ΠΎΠ΄Ρ‹, ΠΊΠΎΡ‚ΠΎΡ€Ρ‹ΠΉ Π±ΡƒΠ΄Π΅Ρ‚ ΡΠΎΠ·Π΄Π°Π²Π°Ρ‚ΡŒ Ρ€Π°Π·Π½ΠΎΡΡ‚ΡŒ ΠΏΠΎΡ‚Π΅Π½Ρ†ΠΈΠ°Π»ΠΎΠ² Π² систСмС. Π’Π°ΠΊΠΎΠΉ ΠΏΡ€ΠΈΠ±ΠΎΡ€ имСнуСтся источником Ρ‚ΠΎΠΊΠ°, ΠΎΠ±ΡΠ·Π°Ρ‚Π΅Π»ΡŒΠ½Ρ‹ΠΉ конструктивный элСмСнт Π² Π½Ρ‘ΠΌ – устройство, ΠΏΡ€Π΅ΠΎΠ±Ρ€Π°Π·ΡƒΡŽΡ‰Π΅Π΅ Ρ…ΠΈΠΌΠΈΡ‡Π΅ΡΠΊΡƒΡŽ, ΠΌΠ΅Ρ…Π°Π½ΠΈΡ‡Π΅ΡΠΊΡƒΡŽ, Ρ‚Π΅ΠΏΠ»ΠΎΠ²ΡƒΡŽ ΠΈΠ»ΠΈ ΡΠ΄Π΅Ρ€Π½ΡƒΡŽ ΡΠ½Π΅Ρ€Π³ΠΈΡŽ Π² ΡΠ»Π΅ΠΊΡ‚Ρ€ΠΈΡ‡Π΅ΡΠΊΡƒΡŽ.

Π‘Π²ΠΎΠ±ΠΎΠ΄Π½Ρ‹Π΅ заряды

Π”Ρ€ΡƒΠ³ΠΈΠΌ ваТнСйшим условиСм сущСствования элСктричСского Ρ‚ΠΎΠΊΠ° Π² ΠΏΡ€ΠΎΠ²ΠΎΠ΄Π½ΠΈΠΊΠ΅ являСтся Π½Π°Π»ΠΈΡ‡ΠΈΠ΅ заряТСнных частиц. БущСствуСт всСго 2 разновидности: элСктрон ΠΈ ΠΏΡ€ΠΎΡ‚ΠΎΠ½. Π­Π»Π΅ΠΊΡ‚Ρ€ΠΎΠ½ – ΠΎΡ‚Ρ€ΠΈΡ†Π°Ρ‚Π΅Π»ΡŒΠ½Ρ‹ΠΉ заряд, ΠΊΠΎΡ‚ΠΎΡ€Ρ‹ΠΉ Π² Π°Ρ‚ΠΎΠΌΠ°Ρ… вращаСтся ΠΏΠΎ слоТной ΠΎΡ€Π±ΠΈΡ‚Π΅ Π²ΠΎΠΊΡ€ΡƒΠ³ ядра. ΠŸΡ€ΠΎΡ‚ΠΎΠ½ – ΠΏΠΎΠ»ΠΎΠΆΠΈΡ‚Π΅Π»ΡŒΠ½ΠΎ заряТСнная частица, которая Π³ΠΎΡ€Π°Π·Π΄ΠΎ тяТСлСС ΠΈ ΠΊΡ€ΡƒΠΏΠ½Π΅Π΅ элСктрона, ΠΎΠ½Π° вмСстС с Π½Π΅ΠΉΡ‚Ρ€ΠΎΠ½Π°ΠΌΠΈ (нСзаряТСнными) создаёт ядро Π°Ρ‚ΠΎΠΌΠ°.

ΠžΡ‚ΡΡŽΠ΄Π° Π²ΠΈΠ΄Π½ΠΎ, Ρ‡Ρ‚ΠΎ большая Ρ‡Π°ΡΡ‚ΡŒ ΠΏΡ€ΠΎΡ‚ΠΎΠ½ΠΎΠ² ΠΈ элСктронов Π² вСщСствС Π½Π΅ свободна, Π° соСдинСна Π² ΠΌΠΎΠ»Π΅ΠΊΡƒΠ»Π°Ρ…. Однако элСктричСский Ρ‚ΠΎΠΊ ΠΌΠΎΠ³ΡƒΡ‚ ΡΠΎΠ·Π΄Π°Π²Π°Ρ‚ΡŒ лишь свободныС частицы, Ρ‚Π°ΠΊ ΠΊΠ°ΠΊ связанныС заряды Π½Π΅ΠΉΡ‚Ρ€Π°Π»ΠΈΠ·ΡƒΡŽΡ‚ Π΄Ρ€ΡƒΠ³ Π΄Ρ€ΡƒΠ³Π°. Π’Π΅ΠΌ Π½Π΅ ΠΌΠ΅Π½Π΅Π΅, Π²ΠΎ ΠΌΠ½ΠΎΠ³ΠΈΡ… соСдинСниях ΠΈΠΌΠ΅ΡŽΡ‚ΡΡ свободныС частицы, ΠΊ Ρ‚Π°ΠΊΠΈΠΌ вСщСствам относится Π²ΠΎΠ΄Π°, ΠΌΠ½ΠΎΠ³ΠΈΠ΅ ΠΌΠ΅Ρ‚Π°Π»Π»Ρ‹.

Π˜Π½Ρ‚Π΅Ρ€Π΅ΡΠ½ΠΎ, Ρ‡Ρ‚ΠΎ количСство свободных зарядов ΠΌΠΎΠΆΠ½ΠΎ ΡƒΠ²Π΅Π»ΠΈΡ‡ΠΈΡ‚ΡŒ, Ссли ΠΏΠ΅Ρ€Π΅Π΄Π°Ρ‚ΡŒ ΡΠ½Π΅Ρ€Π³ΠΈΡŽ Π°Ρ‚ΠΎΠΌΡƒ. Π’ΠΎΠ³Π΄Π° элСктрон слСтит с ΠΎΡ€Π±ΠΈΡ‚Ρ‹ ΠΈ ΠΎΡΠ²ΠΎΠ±ΠΎΠ΄ΠΈΡ‚ΡŒΡΡ. Π­Π½Π΅Ρ€Π³ΠΈΡŽ ΠΌΠΎΠΆΠ½ΠΎ ΠΏΠ΅Ρ€Π΅Π΄Π°Ρ‚ΡŒ с ΠΏΠΎΠΌΠΎΡ‰ΡŒΡŽ нагрСвания. Π˜Π·Π²Π΅ΡΡ‚Π½ΠΎ, Ρ‡Ρ‚ΠΎ Ρƒ ΠΏΠΎΠ»ΡƒΠΏΡ€ΠΎΠ²ΠΎΠ΄Π½ΠΈΠΊΠΎΠ² увСличиваСтся количСство свободных заряТСнных частиц ΠΏΡ€ΠΈ ΠΏΠΎΠ²Ρ‹ΡˆΠ΅Π½ΠΈΠΈ Ρ‚Π΅ΠΌΠΏΠ΅Ρ€Π°Ρ‚ΡƒΡ€Ρ‹.

Π‘ΠΊΠΎΡ€ΠΎΡΡ‚ΡŒ Ρ‚ΠΎΠΊΠ°

ΠŸΡ€ΠΈ создании условий сущСствования элСктричСского Ρ‚ΠΎΠΊΠ° Π² Ρ†Π΅ΠΏΠΈ, ΠΎΠ½ Π²ΠΎΠ·Π½ΠΈΠΊΠ°Π΅Ρ‚ Π²ΠΎ всём ΠΏΡ€ΠΎΠ²ΠΎΠ΄Π½ΠΈΠΊΠ΅ ΠΌΠ³Π½ΠΎΠ²Π΅Π½Π½ΠΎ, Ρ‚ΠΎ Π΅ΡΡ‚ΡŒ со ΡΠΊΠΎΡ€ΠΎΡΡ‚ΡŒΡŽ свСта, нСсмотря Π½Π° Ρ‚ΠΎ, Ρ‡Ρ‚ΠΎ ΠΏΡ€ΠΎΡ‚ΠΎΠ½Ρ‹ двиТутся Π³ΠΎΡ€Π°Π·Π΄ΠΎ ΠΌΠ΅Π΄Π»Π΅Π½Π½Π΅Π΅. Π­Ρ‚ΠΎ связано с Ρ‚Π΅ΠΌ, Ρ‡Ρ‚ΠΎ ΠΏΡ€ΠΎΡ‚ΠΎΠ½Ρ‹, ΠΊΠ°ΠΊ Π±ΡƒΠ΄Ρ‚ΠΎ Ρ‚ΠΎΠ»ΠΊΠ°ΡŽΡ‚ Π΄Ρ€ΡƒΠ³ Π΄Ρ€ΡƒΠ³Π°, поэтому, ΠΊΠΎΠ³Π΄Π° ΠΏΡ€ΠΎΡ‚ΠΎΠ½ Π² ΠΎΠ΄Π½ΠΎΠΌ ΠΊΠΎΠ½Ρ†Π΅ ΠΏΡ€ΠΎΠ²ΠΎΠ΄Π½ΠΈΠΊΠ° Π½Π°Ρ‡ΠΈΠ½Π°Π΅Ρ‚ своё Π΄Π²ΠΈΠΆΠ΅Π½ΠΈΠ΅, Ρ‚ΠΎ ΠΏΠΎΡ‡Ρ‚ΠΈ сразу Π½Π°Ρ‡ΠΈΠ½Π°ΡŽΡ‚ Π΄Π²ΠΈΠ³Π°Ρ‚ΡŒΡΡ частицы Π² Π΄Ρ€ΡƒΠ³ΠΎΠΌ ΠΊΠΎΠ½Ρ†Π΅ Ρ†Π΅ΠΏΠΈ.

ΠŸΡ€ΠΈΠ²Π΅Π΄Ρ‘ΠΌ ΠΏΡ€ΠΈΠΌΠ΅Ρ€ с Π²ΠΎΠ΄ΠΎΠΉ. Если Π² Ρ‚Ρ€ΡƒΠ±Π΅ ΡƒΠΆΠ΅ Π΅ΡΡ‚ΡŒ ΠΆΠΈΠ΄ΠΊΠΎΡΡ‚ΡŒ, Ρ‚ΠΎ, закачивая Π² ΠΎΠ΄ΠΈΠ½ Π΅Ρ‘ ΠΊΠΎΠ½Π΅Ρ† Π΅Ρ‰Ρ‘ ΠΎΠ΄ΠΈΠ½ Π»ΠΈΡ‚Ρ€, ΠΌΠΎΠΆΠ½ΠΎ ΠΎΠ±Π½Π°Ρ€ΡƒΠΆΠΈΡ‚ΡŒ, Ρ‡Ρ‚ΠΎ ΠΈΠ· Π΄Ρ€ΡƒΠ³ΠΎΠ³ΠΎ ΠΊΠΎΠ½Ρ†Π° сразу ΠΆΠ΅ полилась Π²ΠΎΠ΄Π°. НСсмотря Π½Π° Ρ‚ΠΎ, Ρ‡Ρ‚ΠΎ залитая Π²ΠΎΠ΄Π° Π½Π΅ ΠΏΡ€ΠΎΠ΄Π²ΠΈΠ½ΡƒΠ»Π°ΡΡŒ ΠΏΠΎ Ρ‚Ρ€ΡƒΠ±Π΅. ΠŸΡ€ΠΎΡΡ‚ΠΎ Π½ΠΎΠ²Ρ‹ΠΉ Π»ΠΈΡ‚Ρ€ ΠΏΡ€ΠΈΠ²ΠΎΠ΄ΠΈΡ‚ Π² Π΄Π²ΠΈΠΆΠ΅Π½ΠΈΠ΅ сразу всю ΠΆΠΈΠ΄ΠΊΠΎΡΡ‚ΡŒ, которая находится Π² ёмкости.

Π§ΠΈΡ‚Π°ΠΉΡ‚Π΅ Ρ‚Π°ΠΊΠΆΠ΅:

ΠžΠ±Ρ‰ΠΈΠ΅ свСдСния ΠΎΠ± элСктричСствС — Π£Π·Π½Π°ΠΉΡ‚Π΅ ΠΎΠ± элСктричСствС, Ρ‚ΠΎΠΊΠ΅, напряТСнии ΠΈ сопротивлСнии


Π”ΠΎΠΌ > ΠŸΠΎΠ΄Π΄Π΅Ρ€ΠΆΠΊΠ°> ЭлСктроэнСргСтика
ЭлСктроэнСргСтика

Π§Ρ‚ΠΎ Ρ‚Π°ΠΊΠΎΠ΅ элСктричСство?

Π›ΡŽΠ±Π°Ρ Ρ‚Π΅Ρ…Π½ΠΈΠΊΠ°, ΠΊΠΎΡ‚ΠΎΡ€ΡƒΡŽ ΠΌΡ‹ ΠΈΡΠΏΠΎΠ»ΡŒΠ·ΡƒΠ΅ΠΌ Π² нашСй повсСднСвной ΠΆΠΈΠ·Π½ΠΈ, Π½Π°ΠΏΡ€ΠΈΠΌΠ΅Ρ€, бытовая Ρ‚Π΅Ρ…Π½ΠΈΠΊΠ°, ΠΎΡ€Π³Ρ‚Π΅Ρ…Π½ΠΈΠΊΠ° ΠΈ ΠΏΡ€ΠΎΠΌΡ‹ΡˆΠ»Π΅Π½Π½ΠΎΠ΅ ΠΎΠ±ΠΎΡ€ΡƒΠ΄ΠΎΠ²Π°Π½ΠΈΠ΅, ΠΏΠΎΡ‡Ρ‚ΠΈ всС это Ρ‚Ρ€Π΅Π±ΡƒΠ΅Ρ‚ элСктричСства.Π‘Π»Π΅Π΄ΠΎΠ²Π°Ρ‚Π΅Π»ΡŒΠ½ΠΎ, ΠΌΡ‹ Π΄ΠΎΠ»ΠΆΠ½Ρ‹ ΠΏΠΎΠ½ΠΈΠΌΠ°Ρ‚ΡŒ элСктричСство.

ΠŸΠ΅Ρ€Π²Ρ‹ΠΉ вопрос, ΠΊΠΎΡ‚ΠΎΡ€Ρ‹ΠΉ ΠΌΡ‹ ΡƒΠ·Π½Π°Π΅ΠΌ ΠΎΡ‚Π²Π΅Ρ‚ « Π³Π΄Π΅ элСктричСство Ρ€ΠΎΠ΄ΠΎΠΌ ΠΈΠ·? «

ВсС Π΄Π΅Π»Π° состоят ΠΈΠ· Π°Ρ‚ΠΎΠΌΡ‹. Π—Π°Ρ‚Π΅ΠΌ Π·Π°Π΄Π°ΠΉΡ‚Π΅ ΡΠ»Π΅Π΄ΡƒΡŽΡ‰ΠΈΠΉ вопрос: Β« Π§Ρ‚ΠΎ Ρ‚Π°ΠΊΠΎΠ΅ Π°Ρ‚ΠΎΠΌΡ‹? Β».
Атомы — это самая малСнькая Ρ‡Π°ΡΡ‚ΡŒ элСмСнта.Они состоят ядра ΠΈ элСктронов, элСктроны ΠΎΠΊΡ€ΡƒΠΆΠ°ΡŽΡ‚ ядро. Π­Π»Π΅ΠΌΠ΅Π½Ρ‚Ρ‹ ΠΈΠ΄Π΅Π½Ρ‚ΠΈΡ„ΠΈΡ†ΠΈΡ€ΡƒΡŽΡ‚ΡΡ ΠΏΠΎ количСству элСктронов Π½Π° ΠΎΡ€Π±ΠΈΡ‚Π΅ Π²ΠΎΠΊΡ€ΡƒΠ³ ядра Π°Ρ‚ΠΎΠΌΠΎΠ² ΠΈ числом ΠΏΡ€ΠΎΡ‚ΠΎΠ½ΠΎΠ² Π² ядрС.


Π―Π΄Ρ€ΠΎ состоит ΠΈΠ· ΠΏΡ€ΠΎΡ‚ΠΎΠ½ΠΎΠ² ΠΈ Π½Π΅ΠΉΡ‚Ρ€ΠΎΠ½ΠΎΠ², Π° количСство ΠΏΡ€ΠΎΡ‚ΠΎΠ½Ρ‹ ΠΈ Π½Π΅ΠΉΡ‚Ρ€ΠΎΠ½Ρ‹ ΡƒΡ€Π°Π²Π½ΠΎΠ²Π΅ΡˆΠ΅Π½Ρ‹. Π£ Π½Π΅ΠΉΡ‚Ρ€ΠΎΠ½ΠΎΠ² Π½Π΅Ρ‚ элСктричСского заряда, ΠΏΡ€ΠΎΡ‚ΠΎΠ½Ρ‹ ΠΈΠΌΠ΅ΡŽΡ‚ ΠΏΠΎΠ»ΠΎΠΆΠΈΡ‚Π΅Π»ΡŒΠ½Ρ‹ΠΉ заряд (+), Π° элСктроны — ΠΎΡ‚Ρ€ΠΈΡ†Π°Ρ‚Π΅Π»ΡŒΠ½Ρ‹ΠΉ заряды (-).ΠŸΠΎΠ»ΠΎΠΆΠΈΡ‚Π΅Π»ΡŒΠ½Ρ‹ΠΉ заряд ΠΏΡ€ΠΎΡ‚ΠΎΠ½Π° Ρ€Π°Π²Π΅Π½ ΠΎΡ‚Ρ€ΠΈΡ†Π°Ρ‚Π΅Π»ΡŒΠ½ΠΎΠΌΡƒ заряду элСктрона.

Π­Π»Π΅ΠΊΡ‚Ρ€ΠΎΠ½Ρ‹ связаны ΠΏΠΎ своСй ΠΎΡ€Π±ΠΈΡ‚Π΅ Π·Π° счСт притяТСния ΠΏΡ€ΠΎΡ‚ΠΎΠ½ΠΎΠ², Π½ΠΎ элСктроны Π²ΠΎ внСшнСй Π·ΠΎΠ½Π΅ ΠΌΠΎΠ³ΡƒΡ‚ ΠΏΠΎΠΊΠΈΠ½ΡƒΡ‚ΡŒ свою ΠΎΡ€Π±ΠΈΡ‚Ρƒ Π·Π° счСт Π½Π΅ΠΊΠΎΡ‚ΠΎΡ€Ρ‹Π΅ внСшниС силы. Π˜Ρ… Π½Π°Π·Ρ‹Π²Π°ΡŽΡ‚ свободными элСктронами, ΠΊΠΎΡ‚ΠΎΡ€Ρ‹Π΅ ΠΏΠ΅Ρ€Π΅ΠΌΠ΅Ρ‰Π°ΡŽΡ‚ΡΡ ΠΎΡ‚ ΠΎΠ΄Π½ΠΎΠ³ΠΎ Π°Ρ‚ΠΎΠΌΠ° ΠΊ Π΄Ρ€ΡƒΠ³ΠΎΠΌΡƒ, ΠΎΠ±Ρ€Π°Π·ΡƒΡŽΡ‚ΡΡ ΠΏΠΎΡ‚ΠΎΠΊΠΈ элСктронов. Π­Ρ‚ΠΎ основа элСктричСства. ΠœΠ°Ρ‚Π΅Ρ€ΠΈΠ°Π»Ρ‹, ΠΏΠΎΠ·Π²ΠΎΠ»ΡΡŽΡ‰ΠΈΠ΅ свободно ΠΏΠ΅Ρ€Π΅ΠΌΠ΅Ρ‰Π°ΡŽΡ‰ΠΈΠ΅ΡΡ элСктроны Π½Π°Π·Ρ‹Π²Π°ΡŽΡ‚ΡΡ ΠΏΡ€ΠΎΠ²ΠΎΠ΄Π½ΠΈΠΊΠ°ΠΌΠΈ Π° ΠΌΠ°Ρ‚Π΅Ρ€ΠΈΠ°Π»Ρ‹, ΠΊΠΎΡ‚ΠΎΡ€Ρ‹Π΅ ΠΏΠΎΠ·Π²ΠΎΠ»ΡΡŽΡ‚ ΠΏΠ΅Ρ€Π΅ΠΌΠ΅Ρ‰Π°Ρ‚ΡŒΡΡ Π½Π΅Π±ΠΎΠ»ΡŒΡˆΠΎΠΌΡƒ количСству свободных элСктронов, Π½Π°Π·Ρ‹Π²Π°ΡŽΡ‚ΡΡ изоляторы .

ВсС вСщСства состоят ΠΈΠ· Π°Ρ‚ΠΎΠΌΠΎΠ², ΠΈΠΌΠ΅ΡŽΡ‰ΠΈΡ… элСктричСскиС заряды. Π‘Π»Π΅Π΄ΠΎΠ²Π°Ρ‚Π΅Π»ΡŒΠ½ΠΎ, Ρƒ Π½ΠΈΡ… Π΅ΡΡ‚ΡŒ элСктричСскиС заряды. Π§Ρ‚ΠΎ касаСтся сбалансированного количСство ΠΏΡ€ΠΎΡ‚ΠΎΠ½ΠΎΠ² ΠΈ элСктронов, сила ΠΏΠΎΠ»ΠΎΠΆΠΈΡ‚Π΅Π»ΡŒΠ½ΠΎΠ³ΠΎ заряда ΠΈ сила ΠΎΡ‚Ρ€ΠΈΡ†Π°Ρ‚Π΅Π»ΡŒΠ½ΠΎΠ³ΠΎ заряда ΡƒΡ€Π°Π²Π½ΠΎΠ²Π΅ΡˆΠ΅Π½Π°. Π­Ρ‚ΠΎ называСтся Π½Π΅ΠΉΡ‚Ρ€Π°Π»ΡŒΠ½Ρ‹ΠΌ состояниСм. Π°Ρ‚ΠΎΠΌΠ°. (Число ΠΏΡ€ΠΎΡ‚ΠΎΠ½ΠΎΠ² ΠΈ элСктронов остаСтся Ρ€Π°Π²Π½Ρ‹ΠΌ.)

Β« БтатичСскоС элСктричСство Β» прСдставляСт собой ΡΠΈΡ‚ΡƒΠ°Ρ†ΠΈΡŽ, ΠΊΠΎΠ³Π΄Π° всС Π²Π΅Ρ‰ΠΈ состоят ΠΈΠ· элСктричСских обвинСния.НапримСр, Ρ‚Ρ€Π΅Π½ΠΈΠ΅ ΠΌΠ°Ρ‚Π΅Ρ€ΠΈΠ°Π»Π° ΠΎ Π΄Ρ€ΡƒΠ³ΠΎΠΉ ΠΌΠΎΠΆΠ΅Ρ‚ Π²Ρ‹Π·Π²Π°Ρ‚ΡŒ статичСскоС элСктричСство. Π‘Π²ΠΎΠ±ΠΎΠ΄Π½Ρ‹Π΅ элСктроны ΠΎΠ΄Π½ΠΎΠ³ΠΎ ΠΌΠ°Ρ‚Π΅Ρ€ΠΈΠ°Π»Π° Π΄Π²ΠΈΠ³Π°Ρ‚ΡŒΡΡ с силой, ΠΏΠΎΠΊΠ° ΠΎΠ½ΠΈ Π½Π΅ освободятся ΠΎΡ‚ своих ΠΎΡ€Π±ΠΈΡ‚ Π²ΠΎΠΊΡ€ΡƒΠ³ ядра ΠΈ ΠΏΠ΅Ρ€Π΅ΠΉΡ‚ΠΈ ΠΊ Π΄Ρ€ΡƒΠ³ΠΎΠΌΡƒ. Π­Π»Π΅ΠΊΡ‚Ρ€ΠΎΠ½Ρ‹ ΠΎΠ΄Π½ΠΎΠ³ΠΎ ΠΌΠ°Ρ‚Π΅Ρ€ΠΈΠ°Π»Π° ΡƒΠΌΠ΅Π½ΡŒΡˆΠ°ΡŽΡ‚ΡΡ, ΠΎΠ½ прСдставляСт собой ΠΏΠΎΠ»ΠΎΠΆΠΈΡ‚Π΅Π»ΡŒΠ½Ρ‹ΠΉ заряд. Π’ Ρ‚ΠΎ ΠΆΠ΅ врСмя элСктроны Π΄Ρ€ΡƒΠ³ΠΎΠ³ΠΎ ΡƒΠ²Π΅Π»ΠΈΡ‡ΠΈΠ²Π°ΡŽΡ‚ΡΡ, ΠΎΠ½ ΠΈΠΌΠ΅Π΅Ρ‚ ΠΎΡ‚Ρ€ΠΈΡ†Π°Ρ‚Π΅Π»ΡŒΠ½Ρ‹Π΅ заряды.

Π’ ΠΎΠ±Ρ‰Π΅ΠΌ заряд производство ΠΌΠ°Ρ‚Π΅Ρ€ΠΈΠΈ ΠΎΠ·Π½Π°Ρ‡Π°Π΅Ρ‚, Ρ‡Ρ‚ΠΎ матСрия ΠΈΠΌΠ΅Π΅Ρ‚ элСктричСскиС заряды.Он ΠΈΠΌΠ΅Π΅Ρ‚ ΠΏΠΎΠ»ΠΎΠΆΠΈΡ‚Π΅Π»ΡŒΠ½Ρ‹ΠΉ ΠΈ ΠΎΡ‚Ρ€ΠΈΡ†Π°Ρ‚Π΅Π»ΡŒΠ½Ρ‹ΠΉ заряды, Ρ‡Ρ‚ΠΎ выраТаСтся Π² ΠΊΡƒΠ»ΠΎΠ½.


Π’ΠΎΠΊ, НапряТСниС ΠΈ сопротивлСниС


Π§Ρ‚ΠΎ сСйчас?

ЭлСктричСскоС явлСниС Π²Ρ‹Π·Π²Π°Π½ΠΎ ΠΏΠΎΡ‚ΠΎΠΊΠΎΠΌ свободныС элСктроны ΠΎΡ‚ ΠΎΠ΄Π½ΠΎΠ³ΠΎ Π°Ρ‚ΠΎΠΌΠ° ΠΊ Π΄Ρ€ΡƒΠ³ΠΎΠΌΡƒ.Π₯арактСристики Ρ‚Π΅ΠΊΡƒΡ‰Π΅Π΅ элСктричСство ΠΏΡ€ΠΎΡ‚ΠΈΠ²ΠΎΠΏΠΎΠ»ΠΎΠΆΠ½Ρ‹ Ρ‚Π΅ΠΌ статичСского элСктричСства.

ΠŸΡ€ΠΎΠ²ΠΎΠ΄Π° состоят ΠΈΠ· ΠΏΡ€ΠΎΠ²ΠΎΠ΄Π½ΠΈΠΊΠΎΠ², Ρ‚Π°ΠΊΠΈΡ… ΠΊΠ°ΠΊ мСдь. ΠΈΠ»ΠΈ алюминий. Атомы ΠΌΠ΅Ρ‚Π°Π»Π»Π° состоят ΠΈΠ· свободных элСктронов, ΠΊΠΎΡ‚ΠΎΡ€Ρ‹Π΅ свободно ΠΏΠ΅Ρ€Π΅Ρ…ΠΎΠ΄ΠΈΡ‚ΡŒ ΠΎΡ‚ ΠΎΠ΄Π½ΠΎΠ³ΠΎ Π°Ρ‚ΠΎΠΌΠ° ΠΊ Π΄Ρ€ΡƒΠ³ΠΎΠΌΡƒ. Если Π΄ΠΎΠ±Π°Π²Π»Π΅Π½ элСктрон Π² ΠΏΡ€ΠΎΠ²ΠΎΠ΄Π΅ свободный элСктрон притягиваСтся ΠΊ ΠΏΡ€ΠΎΡ‚ΠΎΠ½Ρƒ, Ρ‡Ρ‚ΠΎΠ±Ρ‹ ΠΎΡΡ‚Π°Π²Π°Ρ‚ΡŒΡΡ Π½Π΅ΠΉΡ‚Ρ€Π°Π»ΡŒΠ½Ρ‹ΠΌ. ВытСснСниС элСктронов с ΠΈΡ… ΠΎΡ€Π±ΠΈΡ‚ ΠΌΠΎΠΆΠ΅Ρ‚ Π²Ρ‹Π·Π²Π°Ρ‚ΡŒ нСдостаток элСктронов.Π­Π»Π΅ΠΊΡ‚Ρ€ΠΎΠ½Ρ‹, ΠΊΠΎΡ‚ΠΎΡ€Ρ‹Π΅ Π½Π΅ΠΏΡ€Π΅Ρ€Ρ‹Π²Π½ΠΎ двиТутся ΠΏΠΎ ΠΏΡ€ΠΎΠ²ΠΎΠ»ΠΎΠΊΠ΅, Π½Π°Π·Ρ‹Π²Π°ΡŽΡ‚ΡΡ Electric. Π’Π΅ΠΊΡƒΡ‰ΠΈΠΉ .



Для ΠΎΠ΄Π½ΠΎΠΆΠΈΠ»ΡŒΠ½Ρ‹Ρ… ΠΏΡ€ΠΎΠ²ΠΎΠ΄ΠΎΠ²
элСктричСский Ρ‚ΠΎΠΊ относится ΠΊ Π½Π°ΠΏΡ€Π°Π²Π»Π΅Π½Π½Ρ‹ΠΌ ΠΎΡ‚Ρ€ΠΈΡ†Π°Ρ‚Π΅Π»ΡŒΠ½ΠΎ-ΠΏΠΎΠ»ΠΎΠΆΠΈΡ‚Π΅Π»ΡŒΠ½Ρ‹Π΅ элСктроны ΠΎΡ‚ ΠΎΠ΄Π½ΠΎΠ³ΠΎ Π°Ρ‚ΠΎΠΌΠ° ΠΊ Π΄Ρ€ΡƒΠ³ΠΎΠΌΡƒ. Π–ΠΈΠ΄ΠΊΠΎΡΡ‚ΡŒ ΠΏΡ€ΠΎΠ²ΠΎΠ΄Π½ΠΈΠΊΠΈ ΠΈ Π³Π°Π·ΠΎΠΏΡ€ΠΎΠ²ΠΎΠ΄Ρ‹, элСктричСский Ρ‚ΠΎΠΊ относится ΠΊ элСктронам Π° ΠΏΡ€ΠΎΡ‚ΠΎΠ½Ρ‹ Ρ‚Π΅ΠΊΡƒΡ‚ Π² ΠΎΠ±Ρ€Π°Ρ‚Π½ΠΎΠΌ Π½Π°ΠΏΡ€Π°Π²Π»Π΅Π½ΠΈΠΈ.

Π’ΠΎΠΊ — это ΠΏΠΎΡ‚ΠΎΠΊ элСктронов, Π½ΠΎ Ρ‚ΠΎΠΊ ΠΈ элСктроны Ρ‚Π΅ΠΊΡƒΡ‚ Π² ΠΏΡ€ΠΎΡ‚ΠΈΠ²ΠΎΠΏΠΎΠ»ΠΎΠΆΠ½ΠΎΠ΅ Π½Π°ΠΏΡ€Π°Π²Π»Π΅Π½ΠΈΠ΅. Π’ΠΎΠΊ Ρ‚Π΅Ρ‡Π΅Ρ‚ ΠΎΡ‚ ΠΏΠΎΠ»ΠΎΠΆΠΈΡ‚Π΅Π»ΡŒΠ½ΠΎΠ³ΠΎ ΠΊ ΠΎΡ‚Ρ€ΠΈΡ†Π°Ρ‚Π΅Π»ΡŒΠ½ΠΎΠΌΡƒ ΠΈ элСктроны ΠΏΠ΅Ρ€Π΅Ρ‚Π΅ΠΊΠ°ΡŽΡ‚ с ΠΎΡ‚Ρ€ΠΈΡ†Π°Ρ‚Π΅Π»ΡŒΠ½ΠΎΠ³ΠΎ Π½Π° ΠΏΠΎΠ»ΠΎΠΆΠΈΡ‚Π΅Π»ΡŒΠ½Ρ‹ΠΉ.


Π’ΠΎΠΊ опрСдСляСтся количСством элСктронов, проходящих Ρ‡Π΅Ρ€Π΅Π· сСчСниС ΠΏΡ€ΠΎΠ²ΠΎΠ΄Π½ΠΈΠΊΠ° Π·Π° ΠΎΠ΄Π½Ρƒ сСкунду. Π’ΠΎΠΊ измСряСтся Π² Π°ΠΌΠΏΠ΅Ρ€ , Ρ‡Ρ‚ΠΎ сокращСнно Β« Π°ΠΌΠΏΠ΅Ρ€ Β».ΠžΠ±ΠΎΠ·Π½Π°Ρ‡Π΅Π½ΠΈΠ΅ усилитСля — это Π±ΡƒΠΊΠ²Π° Β« A Β».

А Ρ‚ΠΎΠΊ Π² ΠΎΠ΄ΠΈΠ½ Π°ΠΌΠΏΠ΅Ρ€ ΠΎΠ·Π½Π°Ρ‡Π°Π΅Ρ‚ Ρ‡Ρ‚ΠΎ Ρ‚ΠΎΠΊ ΠΏΡ€ΠΎΡ…ΠΎΠ΄ΠΈΡ‚ Ρ‡Π΅Ρ€Π΅Π· ΠΏΠΎΠΏΠ΅Ρ€Π΅Ρ‡Π½ΠΎΠ΅ сСчСниС Π΄Π²ΡƒΡ… ΠΏΡ€ΠΎΠ²ΠΎΠ΄Π½ΠΈΠΊΠΎΠ², ΠΊΠΎΡ‚ΠΎΡ€Ρ‹Π΅ располоТСны ΠΏΠ°Ρ€Π°Π»Π»Π΅Π»ΡŒΠ½ΠΎ Π½Π° расстоянии 1 ΠΌ Π΄Ρ€ΡƒΠ³ ΠΎΡ‚ Π΄Ρ€ΡƒΠ³Π° с 2×10 -7 ΠΡŒΡŽΡ‚ΠΎΠ½ сила Π½Π° ΠΌΠ΅Ρ‚Ρ€ Π²ΠΎΠ·Π½ΠΈΠΊΠ°Π΅Ρ‚ Π² ΠΊΠ°ΠΆΠ΄ΠΎΠΌ ΠΏΡ€ΠΎΠ²ΠΎΠ΄Π½ΠΈΠΊΠ΅. Π­Ρ‚ΠΎ Ρ‚Π°ΠΊΠΆΠ΅ ΠΌΠΎΠΆΠ΅Ρ‚ ΠΎΠ·Π½Π°Ρ‡Π°Ρ‚ΡŒ сборы ΠΎΠ΄Π½ΠΎΠ³ΠΎ ΠΊΡƒΠ»ΠΎΠ½Π° (ΠΈΠ»ΠΈ 6,24Ρ…10 18 элСктронов), проходящСго Ρ‡Π΅Ρ€Π΅Π· ΠΏΠΎΠΏΠ΅Ρ€Π΅Ρ‡Π½ΠΎΠ΅ сСчСниС ΠΏΡ€ΠΎΠ²ΠΎΠ΄Π½ΠΈΠΊΠ° Π·Π° ΠΎΠ΄Π½Ρƒ сСкунду.


Π§Ρ‚ΠΎ Ρ‚Π°ΠΊΠΎΠ΅ напряТСниС?

ЭлСктричСский Ρ‚ΠΎΠΊ — это ΠΏΠΎΡ‚ΠΎΠΊ элСктронов Π² ΠΏΡ€ΠΎΠ²ΠΎΠ΄Π½ΠΈΠΊΠ΅. Π‘ΠΈΠ»Π° Π½Π΅ΠΎΠ±Ρ…ΠΎΠ΄ΠΈΠΌ для протСкания Ρ‚ΠΎΠΊΠ° Ρ‡Π΅Ρ€Π΅Π· ΠΏΡ€ΠΎΠ²ΠΎΠ΄Π½ΠΈΠΊ, называСтся напряТСниС ΠΈ ΠΏΠΎΡ‚Π΅Π½Ρ†ΠΈΠ°Π» Π΄Ρ€ΡƒΠ³ΠΎΠΉ срок напряТСния. НапримСр, Ρƒ ΠΏΠ΅Ρ€Π²ΠΎΠ³ΠΎ элСмСнта большС ΠΏΠΎΠ»ΠΎΠΆΠΈΡ‚Π΅Π»ΡŒΠ½Ρ‹Π΅ заряды, поэтому ΠΎΠ½ ΠΈΠΌΠ΅Π΅Ρ‚ Π±ΠΎΠ»Π΅Π΅ высокий ΠΏΠΎΡ‚Π΅Π½Ρ†ΠΈΠ°Π». Π‘ Π΄Ρ€ΡƒΠ³ΠΎΠΉ стороны, Π²Ρ‚ΠΎΡ€ΠΎΠΉ элСмСнт ΠΈΠΌΠ΅Π΅Ρ‚ Π±ΠΎΠ»Π΅Π΅ ΠΎΡ‚Ρ€ΠΈΡ†Π°Ρ‚Π΅Π»ΡŒΠ½Ρ‹Π΅ заряды, поэтому ΠΎΠ½ ΠΈΠΌΠ΅Π΅Ρ‚ Π±ΠΎΠ»Π΅Π΅ Π½ΠΈΠ·ΠΊΠΈΠΉ ΠΏΠΎΡ‚Π΅Π½Ρ†ΠΈΠ°Π».Π Π°Π·Π½ΠΈΡ†Π° ΠΌΠ΅ΠΆΠ΄Ρƒ двумя Ρ‚ΠΎΡ‡ΠΊΠ°ΠΌΠΈ называСтся Ρ€Π°Π·Π½ΠΎΡΡ‚ΡŒ ΠΏΠΎΡ‚Π΅Π½Ρ†ΠΈΠ°Π»ΠΎΠ² .

ЭлСктродвиТущая сила ΠΎΠ·Π½Π°Ρ‡Π°Π΅Ρ‚ силу, которая заставляСт Ρ‚ΠΎΠΊ Π½Π΅ΠΏΡ€Π΅Ρ€Ρ‹Π²Π½ΠΎ Ρ‚Π΅Ρ‡ΡŒ Ρ‡Π΅Ρ€Π΅Π· Π΄ΠΈΡ€ΠΈΠΆΠ΅Ρ€. Π­Ρ‚Π° сила ΠΌΠΎΠΆΠ΅Ρ‚ ΡΠΎΠ·Π΄Π°Π²Π°Ρ‚ΡŒΡΡ Π³Π΅Π½Π΅Ρ€Π°Ρ‚ΠΎΡ€ΠΎΠΌ энСргии, аккумулятор, аккумулятор Ρ„ΠΎΠ½Π°Ρ€ΠΈΠΊΠ° ΠΈ Ρ‚ΠΎΠΏΠ»ΠΈΠ²Π½Ρ‹ΠΉ элСмСнт ΠΈ Ρ‚. Π΄.

Π’ΠΎΠ»ΡŒΡ‚, сокращСнно Β« Π’ Β», это Π΅Π΄ΠΈΠ½ΠΈΡ†Π° измСрСния измСрСния, ΠΈΡΠΏΠΎΠ»ΡŒΠ·ΡƒΠ΅ΠΌΡ‹Π΅ взаимозамСняСмо для напряТСния, ΠΏΠΎΡ‚Π΅Π½Ρ†ΠΈΠ°Π»Π°, ΠΈ элСктродвиТущая сила.Один Π²ΠΎΠ»ΡŒΡ‚ ΠΎΠ·Π½Π°Ρ‡Π°Π΅Ρ‚ силу, которая заставляСт Ρ‚ΠΎΠΊ Π² ΠΎΠ΄ΠΈΠ½ Π°ΠΌΠΏΠ΅Ρ€ ΠΏΡ€ΠΎΡ…ΠΎΠ΄ΠΈΡ‚ Ρ‡Π΅Ρ€Π΅Π· сопротивлСниС Π² ΠΎΠ΄ΠΈΠ½ Ом.

Π§Ρ‚ΠΎ Ρ‚Π°ΠΊΠΎΠ΅ сопротивлСниС?
Π­Π»Π΅ΠΊΡ‚Ρ€ΠΎΠ½Ρ‹ двиТутся Ρ‡Π΅Ρ€Π΅Π· ΠΏΡ€ΠΎΠ²ΠΎΠ΄Π½ΠΈΠΊ ΠΏΡ€ΠΈ ΠΏΡ€ΠΎΡ‚Π΅ΠΊΠ°Π½ΠΈΠΈ элСктричСского Ρ‚ΠΎΠΊΠ°. ВсС ΠΌΠ°Ρ‚Π΅Ρ€ΠΈΠ°Π»Ρ‹ ΠΌΠ΅ΡˆΠ°ΡŽΡ‚ ΠΏΡ€ΠΎΡ‚Π΅ΠΊΠ°Π½ΠΈΠ΅ элСктричСского Ρ‚ΠΎΠΊΠ° Π΄ΠΎ Π½Π΅ΠΊΠΎΡ‚ΠΎΡ€ΠΎΠΉ стСпСни. Π­Ρ‚Π° характСристика называСтся сопротивлСниСм .Π‘ΠΎΠΏΡ€ΠΎΡ‚ΠΈΠ²Π»Π΅Π½ΠΈΠ΅ увСличиваСтся с ΡƒΠ²Π΅Π»ΠΈΡ‡Π΅Π½ΠΈΠ΅ΠΌ Π΄Π»ΠΈΠ½Ρ‹ ΠΈΠ»ΠΈ ΡƒΠΌΠ΅Π½ΡŒΡˆΠ΅Π½ΠΈΠ΅ΠΌ ΠΏΠΎΠΏΠ΅Ρ€Π΅Ρ‡Π½ΠΎΠ³ΠΎ сСчСния ΠΌΠ°Ρ‚Π΅Ρ€ΠΈΠ°Π».

Π•Π΄ΠΈΠ½ΠΈΡ†Π° измСрСния сопротивлСния Ом ΠΈ Π΅Π³ΠΎ символ — грСчСская Π±ΡƒΠΊΠ²Π° ΠΎΠΌΠ΅Π³Π° ( Ξ© ). Π‘ΠΎΠΏΡ€ΠΎΡ‚ΠΈΠ²Π»Π΅Π½ΠΈΠ΅ Π² ΠΎΠ΄ΠΈΠ½ Ом ΠΎΠ·Π½Π°Ρ‡Π°Π΅Ρ‚, Ρ‡Ρ‚ΠΎ ΠΏΡ€ΠΎΠ²ΠΎΠ΄Π½ΠΈΠΊ пропускаСт Ρ‚ΠΎΠΊ. ΠΎΠ΄Π½ΠΎΠ³ΠΎ Π°ΠΌΠΏΠ΅Ρ€Π° Π½Π° ΠΏΠΎΡ‚ΠΎΠΊ с напряТСниСм ΠΎΠ΄ΠΈΠ½ Π²ΠΎΠ»ΡŒΡ‚.

ВсС ΠΌΠ°Ρ‚Π΅Ρ€ΠΈΠ°Π»Ρ‹ Ρ€Π°Π·Π»ΠΈΡ‡Π°ΡŽΡ‚ΡΡ ΠΏΠΎ ΠΏΡ€ΠΎΠΏΡƒΡΠΊΠ°Π½ΠΈΡŽ элСктронов.ΠœΠ°Ρ‚Π΅Ρ€ΠΈΠ°Π»Ρ‹ ΠΊΠΎΡ‚ΠΎΡ€Ρ‹Π΅ ΠΏΠΎΠ·Π²ΠΎΠ»ΡΡŽΡ‚ свободно ΠΏΠ΅Ρ€Π΅ΠΌΠ΅Ρ‰Π°Ρ‚ΡŒΡΡ Π±ΠΎΠ»ΡŒΡˆΠΎΠΌΡƒ количСству элСктронов, Π½Π°Π·Ρ‹Π²Π°ΡŽΡ‚ΡΡ ΠΏΡ€ΠΎΠ²ΠΎΠ΄Π½ΠΈΠΊΠ°ΠΌΠΈ Ρ‚Π°ΠΊΠΈΠ΅ ΠΊΠ°ΠΊ мСдь, сСрСбро, алюминий, раствор хлористоводородной, сСрной кислота ΠΈ солСная Π²ΠΎΠ΄Π°. Напротив, ΠΌΠ°Ρ‚Π΅Ρ€ΠΈΠ°Π»Ρ‹, ΠΏΡ€ΠΎΠΏΡƒΡΠΊΠ°ΡŽΡ‰ΠΈΠ΅ ΠΌΠ°Π»ΠΎ элСктронов для протСкания Π½Π°Π·Ρ‹Π²Π°ΡŽΡ‚ΡΡ изоляторы , Ρ‚Π°ΠΊΠΈΠ΅ ΠΊΠ°ΠΊ пластмассовыС, Ρ€Π΅Π·ΠΈΠ½Π°, стСкло ΠΈ сухая Π±ΡƒΠΌΠ°Π³Π°. Π”Ρ€ΡƒΠ³ΠΎΠΉ Π²ΠΈΠ΄ ΠΌΠ°Ρ‚Π΅Ρ€ΠΈΠ°Π»ΠΎΠ², ΠΏΠΎΠ»ΡƒΠΏΡ€ΠΎΠ²ΠΎΠ΄Π½ΠΈΠΊΠΈ ΠΈΠΌΠ΅ΡŽΡ‚ характСристики ΠΊΠ°ΠΊ ΠΏΡ€ΠΎΠ²ΠΎΠ΄Π½ΠΈΠΊΠΎΠ², Ρ‚Π°ΠΊ ΠΈ изоляторов.Они ΠΏΠΎΠ·Π²ΠΎΠ»ΡΡŽΡ‚ элСктронам Π΄Π²ΠΈΠ³Π°Ρ‚ΡŒΡΡ, имСя Π²ΠΎΠ·ΠΌΠΎΠΆΠ½ΠΎΡΡ‚ΡŒ ΠΊΠΎΠ½Ρ‚Ρ€ΠΎΠ»ΠΈΡ€ΠΎΠ²Π°Ρ‚ΡŒ ΠΏΠΎΡ‚ΠΎΠΊ элСктронами ΠΈ ΠΏΡ€ΠΈΠΌΠ΅Ρ€Π°ΠΌΠΈ ΡΠ²Π»ΡΡŽΡ‚ΡΡ ΡƒΠ³Π»Π΅Ρ€ΠΎΠ΄, ΠΊΡ€Π΅ΠΌΠ½ΠΈΠΉ, Π³Π΅Ρ€ΠΌΠ°Π½ΠΈΠΉ ΠΈ Ρ‚. Π΄.

Π‘ΠΎΠΏΡ€ΠΎΡ‚ΠΈΠ²Π»Π΅Π½ΠΈΠ΅ ΠΏΡ€ΠΎΠ²ΠΎΠ΄Π½ΠΈΠΊΠ° зависит ΠΎΡ‚ ΡΠ»Π΅Π΄ΡƒΡŽΡ‰ΠΈΡ… Π΄Π²ΡƒΡ… основных Ρ„Π°ΠΊΡ‚ΠΎΡ€ΠΎΠ²:

1. Π’ΠΈΠ΄Ρ‹ ΠΌΠ°Ρ‚Π΅Ρ€ΠΈΠ°Π»Π°
2. Π’Π΅ΠΌΠΏΠ΅Ρ€Π°Ρ‚ΡƒΡ€Π° ΠΌΠ°Ρ‚Π΅Ρ€ΠΈΠ°Π»Π°

Как ΠΈΠ·ΠΌΠ΅Ρ€ΠΈΡ‚ΡŒ Ρ‚ΠΎΠΊ

ΠŸΡ€ΠΈΠ±ΠΎΡ€ для измСрСния силы Ρ‚ΠΎΠΊΠ° называСтся АмпСрмСтр ΠΈΠ»ΠΈ АмпСрмСтр .
Π¨Π°Π³ΠΈ для измСрСния Ρ‚ΠΎΠΊΠ° ΠŸΠΎΠ΄ΠΊΠ»ΡŽΡ‡ΠΈΡ‚Π΅ Π½Π΅Π±ΠΎΠ»ΡŒΡˆΡƒΡŽ Π»Π°ΠΌΠΏΠΎΡ‡ΠΊΡƒ ΠΊ сухой Π±Π°Ρ‚Π°Ρ€Π΅Π΅.Π˜Π·ΠΌΠ΅Ρ€ΡŒΡ‚Π΅ Ρ‚ΠΎΠΊ ΠΊΠΎΡ‚ΠΎΡ€Ρ‹ΠΉ ΠΏΡ€ΠΎΡ…ΠΎΠ΄ΠΈΡ‚ Ρ‡Π΅Ρ€Π΅Π· Π»Π°ΠΌΠΏΠΎΡ‡ΠΊΡƒ ΠΏΡ€ΠΈ ΠΏΠΎΠ΄ΠΊΠ»ΡŽΡ‡Π΅Π½ΠΈΠΈ ΠΏΠΎΠ»ΠΎΠΆΠΈΡ‚Π΅Π»ΡŒΠ½ΠΎΠΉ ΠΊΠ»Π΅ΠΌΠΌΡ‹ (+) Π°ΠΌΠΏΠ΅Ρ€ΠΌΠ΅Ρ‚Ρ€Π° ΠΊ ΠΎΡ‚Ρ€ΠΈΡ†Π°Ρ‚Π΅Π»ΡŒΠ½ΠΎΠΉ ΠΊΠ»Π΅ΠΌΠΌΠ΅ (-) сухого элСмСнта (см. рисунок)
Указания ΠΏΠΎ Ρ‚Π΅Ρ…Π½ΠΈΠΊΠ΅ бСзопасности ΠΏΡ€ΠΈ ΠΈΠ·ΠΌΠ΅Ρ€Π΅Π½ΠΈΠΈ силы Ρ‚ΠΎΠΊΠ°;
1. ΠžΡ†Π΅Π½ΠΈΡ‚ΡŒ Ρ‚ΠΎΠΊ, Ρ‚Ρ€Π΅Π±ΡƒΡŽΡ‰ΠΈΠΉ измСрСния Π·Π°Ρ‚Π΅ΠΌ Π²Ρ‹Π±Π΅Ρ€ΠΈΡ‚Π΅ подходящий Π°ΠΌΠΏΠ΅Ρ€ΠΌΠ΅Ρ‚Ρ€, Ρ‚Π°ΠΊ ΠΊΠ°ΠΊ ΠΊΠ°ΠΆΠ΄Ρ‹ΠΉ Π°ΠΌΠΏΠ΅Ρ€ΠΌΠ΅Ρ‚Ρ€ ΠΈΠΌΠ΅Π΅Ρ‚ Ρ€Π°Π·Π½Ρ‹Π΅ ΠΏΡ€Π΅Π΄Π΅Π» измСрСния Ρ‚ΠΎΠΊΠ°.
2. Π£Π±Π΅Π΄ΠΈΡ‚Π΅ΡΡŒ, Ρ‡Ρ‚ΠΎ соСдинСниС с плюсовой ΠΊΠ»Π΅ΠΌΠΌΠΎΠΉ (+) ΠΈ ΠΎΡ‚Ρ€ΠΈΡ†Π°Ρ‚Π΅Π»ΡŒΠ½Π°Ρ ΠΊΠ»Π΅ΠΌΠΌΠ° (-) Π°ΠΌΠΏΠ΅Ρ€ΠΌΠ΅Ρ‚Ρ€Π° ΠΏΡ€Π°Π²ΠΈΠ»ΡŒΠ½Ρ‹Π΅.
3. НС ΠΏΠΎΠ΄ΠΊΠ»ΡŽΡ‡Π°ΠΉΡ‚Π΅ ΠΊΠ»Π΅ΠΌΠΌΡ‹ Π°ΠΌΠΏΠ΅Ρ€ΠΌΠ΅Ρ‚Ρ€Π° Π½Π°ΠΏΡ€ΡΠΌΡƒΡŽ ΡΡƒΡˆΠΈΡ‚ΡŒ ΠΊΠ»Π΅ΠΌΠΌΡ‹ ячССк. Π’Π°ΠΊ ΠΊΠ°ΠΊ это ΠΌΠΎΠΆΠ΅Ρ‚ ΠΏΠΎΠ²Ρ€Π΅Π΄ΠΈΡ‚ΡŒ счСтчик.

Как ΠΈΠ·ΠΌΠ΅Ρ€ΠΈΡ‚ΡŒ напряТСниС
ΠŸΡ€ΠΈΠ±ΠΎΡ€ для измСрСния напряТСния, Ρ€Π°Π·Π½ΠΈΡ†Ρ‹ ΠŸΠΎΡ‚Π΅Π½Ρ†ΠΈΠ°Π»ΡŒΠ½Π°Ρ ΠΈΠ»ΠΈ элСктродвиТущая сила называСтся Π²ΠΎΠ»ΡŒΡ‚ΠΌΠ΅Ρ‚Ρ€ΠΎΠΌ .

Π¨Π°Π³ΠΈ для измСрСния напряТСния
ΠŸΠΎΠ΄ΠΊΠ»ΡŽΡ‡ΠΈΡ‚Π΅ Π½Π΅Π±ΠΎΠ»ΡŒΡˆΡƒΡŽ Π»Π°ΠΌΠΏΠΎΡ‡ΠΊΡƒ ΠΊ сухому элСмСнту. Π’ΠΎΠ»ΡŒΡ‚ΠΌΠ΅Ρ‚Ρ€ Π΅ΡΡ‚ΡŒ ΠΏΠΎΠ΄ΠΊΠ»ΡŽΡ‡Π΅Π½ ΠΏΠ°Ρ€Π°Π»Π»Π΅Π»ΡŒΠ½ΠΎ Π»Π°ΠΌΠΏΠΎΡ‡ΠΊΠ΅ для измСрСния напряТСния ΠΏΠΎΠΏΠ΅Ρ€Π΅ΠΊ Π»Π°ΠΌΠΏΠΎΡ‡ΠΊΠΈ. ΠŸΠΎΠ΄ΠΊΠ»ΡŽΡ‡ΠΈΡ‚Π΅ ΠΏΠΎΠ»ΠΎΠΆΠΈΡ‚Π΅Π»ΡŒΠ½ΡƒΡŽ ΠΊΠ»Π΅ΠΌΠΌΡƒ (+) Π²ΠΎΠ»ΡŒΡ‚ΠΌΠ΅Ρ‚Ρ€ ΠΊ плюсовой ΠΊΠ»Π΅ΠΌΠΌΠ΅ (+) сухого элСмСнта ΠΈ ΠΏΠΎΠ΄ΠΊΠ»ΡŽΡ‡ΠΈΡ‚Π΅ ΠΎΡ‚Ρ€ΠΈΡ†Π°Ρ‚Π΅Π»ΡŒΠ½Π°Ρ ΠΊΠ»Π΅ΠΌΠΌΠ° (-) Π²ΠΎΠ»ΡŒΡ‚ΠΌΠ΅Ρ‚Ρ€Π° ΠΊ ΠΎΡ‚Ρ€ΠΈΡ†Π°Ρ‚Π΅Π»ΡŒΠ½ΠΎΠΉ ΠΊΠ»Π΅ΠΌΠΌΠ΅ (-) сухой ячСйки (см. рисунок).
Указания ΠΏΠΎ Ρ‚Π΅Ρ…Π½ΠΈΠΊΠ΅ бСзопасности ΠΏΡ€ΠΈ ΠΈΠ·ΠΌΠ΅Ρ€Π΅Π½ΠΈΠΈ НапряТСниС;
1. ΠžΡ†Π΅Π½ΠΈΡ‚ΡŒ напряТСниС, Ρ‚Ρ€Π΅Π±ΡƒΡŽΡ‰Π΅Π΅ измСрСния Π·Π°Ρ‚Π΅ΠΌ Π²Ρ‹Π±Π΅Ρ€ΠΈΡ‚Π΅ подходящий Π²ΠΎΠ»ΡŒΡ‚ΠΌΠ΅Ρ‚Ρ€
, ΠΏΠΎΡΠΊΠΎΠ»ΡŒΠΊΡƒ ΠΊΠ°ΠΆΠ΄Ρ‹ΠΉ Π²ΠΎΠ»ΡŒΡ‚ΠΌΠ΅Ρ‚Ρ€ рассчитан Π½Π°
ΠΏΡ€Π΅Π΄Π΅Π» измСрСния напряТСния.
2. Π£Π±Π΅Π΄ΠΈΡ‚Π΅ΡΡŒ, Ρ‡Ρ‚ΠΎ ΠΏΠΎΠ΄ΠΊΠ»ΡŽΡ‡Π΅Π½ΠΈΠ΅ ΠΏΠΎΠ»ΠΎΠΆΠΈΡ‚Π΅Π»ΡŒΠ½ΠΎΠΉ ΠΊΠ»Π΅ΠΌΠΌΡ‹ (+) ΠΈ ΠΎΡ‚Ρ€ΠΈΡ†Π°Ρ‚Π΅Π»ΡŒΠ½Π°Ρ ΠΊΠ»Π΅ΠΌΠΌΠ° (-) Π²ΠΎΠ»ΡŒΡ‚ΠΌΠ΅Ρ‚Ρ€Π° ΠΏΡ€Π°Π²ΠΈΠ»ΡŒΠ½Ρ‹Π΅.

Как ΠΈΠ·ΠΌΠ΅Ρ€ΠΈΡ‚ΡŒ сопротивлСниС
Π˜Π½ΡΡ‚Ρ€ΡƒΠΌΠ΅Π½Ρ‚, ΠΈΡΠΏΠΎΠ»ΡŒΠ·ΡƒΠ΅ΠΌΡ‹ΠΉ для измСрСния Π‘ΠΎΠΏΡ€ΠΎΡ‚ΠΈΠ²Π»Π΅Π½ΠΈΠ΅ называСтся тСстСром ΠΈΠ»ΠΈ ΠΌΡƒΠ»ΡŒΡ‚ΠΈΠΌΠ΅Ρ‚Ρ€ΠΎΠΌ .ΠœΡƒΠ»ΡŒΡ‚ΠΈΠΌΠ΅Ρ‚Ρ€ ΠΈΠ»ΠΈ тСстовый ΠΌΠ΅Ρ‚Ρ€ ΠΈΡΠΏΠΎΠ»ΡŒΠ·ΡƒΠ΅Ρ‚ΡΡ для изготовлСния Ρ€Π°Π·Π»ΠΈΡ‡Π½Ρ‹Ρ… элСктричСских измСрСния, Ρ‚Π°ΠΊΠΈΠ΅ ΠΊΠ°ΠΊ Ρ‚ΠΎΠΊ, напряТСниС ΠΈ сопротивлСниС. Он сочСтаСт Π² сСбС Ρ„ΡƒΠ½ΠΊΡ†ΠΈΠΈ Π°ΠΌΠΏΠ΅Ρ€ΠΌΠ΅Ρ‚Ρ€Π°, Π²ΠΎΠ»ΡŒΡ‚ΠΌΠ΅Ρ‚Ρ€Π° ΠΈ ΠΎΠΌΠΌΠ΅Ρ‚Ρ€Π°.

Π¨Π°Π³ΠΈ для измСрСния сопротивлСния
ΠŸΠΎΠ²Π΅Ρ€Π½ΠΈΡ‚Π΅ Π»ΠΈΡ†Π΅Π²ΡƒΡŽ ΡˆΠΊΠ°Π»Ρƒ Π² ΠΏΠΎΠ»ΠΎΠΆΠ΅Π½ΠΈΠ΅ для Ρ‚Ρ€Π΅Π±ΡƒΠ΅ΠΌΠΎΠ³ΠΎ измСрСния, сопротивлСния, Π·Π°Ρ‚Π΅ΠΌ ΠΊΠΎΡΠ½ΠΈΡ‚Π΅ΡΡŒ ΠΎΠ±ΠΎΠΈΡ… Π²Ρ‹Π²ΠΎΠ΄ΠΎΠ² ΠΌΡƒΠ»ΡŒΡ‚ΠΈΠΌΠ΅Ρ‚Ρ€Π° (см. рисунок 1) ΠΈ ΠΎΡ‚Ρ€Π΅Π³ΡƒΠ»ΠΈΡ€ΡƒΠΉΡ‚Π΅ Π΄ΠΈΠ°ΠΏΠ°Π·ΠΎΠ½ измСритСля Π½Π° 0 Ом.Π’Ρ€ΠΎΠ³Π°Ρ‚ΡŒ ΠΎΠ±Π° Π²Ρ‹Π²ΠΎΠ΄Π° измСритСля ΠΊ ΡΠΎΠΏΡ€ΠΎΡ‚ΠΈΠ²Π»Π΅Π½ΠΈΡŽ ΠΈ Π²ΠΎΠ·ΡŒΠΌΠΈΡ‚Π΅ Ρ‡Ρ‚Π΅Π½ΠΈΠ΅ (см. рисунок 2).


Как Ρ€Π°Π±ΠΎΡ‚Π°Π΅Ρ‚ элСктричСство?

ЭлСктричСский Ρ‚ΠΎΠΊ — это ΡΠΏΠΎΡΠΎΠ±Π½ΠΎΡΡ‚ΡŒ Π΄Π΅Π»Π°Ρ‚ΡŒ Ρ€Π°Π±ΠΎΡ‚Ρƒ.ЭлСктричСский Ρ‚ΠΎΠΊ ΠΌΠΎΠΆΠ½ΠΎ ΠΏΡ€Π΅ΠΎΠ±Ρ€Π°Π·ΠΎΠ²Π°Ρ‚ΡŒ Π² Ρ‚Π΅ΠΏΠ»ΠΎ, ΠΌΠΎΡ‰Π½ΠΎΡΡ‚ΡŒ ΠΈ ΠΌΠ°Π³Π½Π΅Ρ‚ΠΈΠ·ΠΌ, Ρ‡Ρ‚ΠΎΠ±Ρ‹ Π½Π°Π·Π²Π°Ρ‚ΡŒ нСсколько.

ЭлСктричСский Ρ‚ΠΎΠΊ классифицирован ΠΏΠΎ своим функциям ΠΈ Ρ‚Ρ€Π΅ΠΌ основным Ρ‚ΠΈΠΏΠ°ΠΌ:

1.

ВСплоэнСргСтика

2.

ЭлСктрохимия

3.

ΠœΠ°Π³Π½Π΅Ρ‚ΠΈΠ·ΠΌ


1. Π’Π΅ΠΏΠ»ΠΎ ΠΈ энСргия ΠΈΡΠΏΠΎΠ»ΡŒΠ·ΡƒΠ΅Ρ‚ΡΡ для производства Ρ‚Π΅ΠΏΠ»Π° ΠΈ элСктроэнСргии.
НапримСр, нихромовая токовСдущая ΠΏΡ€ΠΎΠ²ΠΎΠ»ΠΎΠΊΠ°. ΠΏΡ€ΠΎΠ²ΠΎΠ»ΠΎΠΊΠ° ΠΈΠΌΠ΅Π΅Ρ‚ высокоС сопротивлСниС ΠΈ выдСляСт Ρ‚Π΅ΠΏΠ»ΠΎ.Π­Ρ‚ΠΎ примСняСтся Π±Ρ‹Ρ‚ΡŒ составной Ρ‡Π°ΡΡ‚ΡŒΡŽ элСктричСских Π΄ΡƒΡ…ΠΎΠ²ΠΎΠΊ, тостСров, элСктричСских ΡƒΡ‚ΡŽΠ³ΠΎΠ² ΠΈ Π»Π°ΠΌΠΏΠΎΡ‡ΠΊΠΈ ΠΈ Π΄Ρ€.

ЭкспСримСнт проводится ΠΏΡƒΡ‚Π΅ΠΌ измСрСния Π½Π°Π³Ρ€Π΅Ρ‚ΡŒ количСство Π²ΠΎΠ΄Ρ‹ ΠΊΠ°Π»ΠΎΡ€ΠΈΠΌΠ΅Ρ‚Ρ€ΠΎΠΌ. Π£Π²Π΅Π»ΠΈΡ‡ΡŒΡ‚Π΅ напряТСниС Π½Π° ΠΏΡ€ΠΎΠ²ΠΎΠ΄ Π²Π°Ρ€ΠΈΠ°ΠΊΠΎΠΌ ΠΈ ΠΏΠΎΠ΄ΠΊΠ»ΡŽΡ‡ΠΈΡ‚Π΅ Π°ΠΌΠΏΠ΅Ρ€ΠΌΠ΅Ρ‚Ρ€ ΠΈ Π²ΠΎΠ»ΡŒΡ‚ΠΌΠ΅Ρ‚Ρ€ для измСрСния Ρ‚ΠΎΠΊ ΠΈ напряТСниС.
УстановитС ΡˆΠΊΠ°Π»Ρƒ ΠΏΠ΅Ρ€Π΅ΠΌΠ΅Π½Π½ΠΎΠ³ΠΎ Ρ‚ΠΎΠΊΠ°, Ρ‡Ρ‚ΠΎΠ±Ρ‹ ΠΎΡ‚Ρ€Π΅Π³ΡƒΠ»ΠΈΡ€ΠΎΠ²Π°Ρ‚ΡŒ напряТСниС ΠΈ Ρ‚Π΅ΠΊΡƒΡ‰Π΅Π΅ Π·Π½Π°Ρ‡Π΅Π½ΠΈΠ΅ нихромовая ΠΏΡ€ΠΎΠ²ΠΎΠ»ΠΎΠΊΠ° ΠΈ Ρ‚ΠΎΠΊ пСриодичСски пропускаСтся ΠΈ ΠΈΠ·ΠΌΠ΅Ρ€ΠΈΡ‚ΡŒ количСство Ρ‚Π΅ΠΏΠ»Π° ΠΎΡ‚ Π½ΠΈΡ…Ρ€ΠΎΠΌΠΎΠ²ΠΎΠΉ ΠΏΡ€ΠΎΠ²ΠΎΠ»ΠΎΠΊΠΈ.Π•ΡΡ‚ΡŒ ΠΊΠ°ΠΊΠΈΠ΅-Ρ‚ΠΎ указания напряТСния ΠΈ Ρ‚ΠΎΠΊΠ°. Если напряТСниС, Ρ‚ΠΎΠΊ ΠΈ врСмя ΡƒΠ²Π΅Π»ΠΈΡ‡ΠΈΠ²Π°ΡŽΡ‚ΡΡ, количСство Ρ‚Π΅ΠΏΠ»Π° Ρ‚Π°ΠΊΠΆΠ΅ увСличится. Они Π²Ρ‹Ρ€Π°ΠΆΠ°ΡŽΡ‚ΡΡ ΠΎΡ‚Π½ΠΎΡˆΠ΅Π½ΠΈΠ΅, ΠΊΠ°ΠΊ ΠΏΠΎΠΊΠ°Π·Π°Π½ΠΎ Π½ΠΈΠΆΠ΅.

Π­Ρ‚ΠΎ называСтся ДТоуля. Π—Π°ΠΊΠΎΠ½ . ΠšΠΎΠ»ΠΈΡ‡Π΅ΡΡ‚Π²ΠΎ Ρ‚Π΅ΠΏΠ»Π° зависит ΠΎΡ‚ напряТСния врСмя Ρ‚ΠΎΠΊΠ° ΠΈ ΠΈΠ½Ρ‚Π΅Ρ€Π²Π°Π» Π²Ρ€Π΅ΠΌΠ΅Π½ΠΈ.По Π·Π°ΠΊΠΎΠ½Ρƒ Ома V (напряТСниС) = I (Ρ‚ΠΎΠΊ) x R (Π‘ΠΎΠΏΡ€ΠΎΡ‚ΠΈΠ²Π»Π΅Π½ΠΈΠ΅), ΡΠ»Π΅Π΄ΠΎΠ²Π°Ρ‚Π΅Π»ΡŒΠ½ΠΎ,

ΠšΠΎΠ»ΠΈΡ‡Π΅ΡΡ‚Π²ΠΎ Ρ‚Π΅ΠΏΠ»Π° зависит ΠΎΡ‚ Ρ‚Π΅ΠΊΡƒΡ‰ΠΈΠΉ ΠΊΠ²Π°Π΄Ρ€Π°Ρ‚, ΡƒΠΌΠ½ΠΎΠΆΠ΅Π½Π½Ρ‹ΠΉ Π½Π° сопротивлСниС ΠΈ ΠΈΠ½Ρ‚Π΅Ρ€Π²Π°Π» Π²Ρ€Π΅ΠΌΠ΅Π½ΠΈ.

ΠŸΡ€ΠΈ пропускании Ρ‚ΠΎΠΊΠ° Ρ‡Π΅Ρ€Π΅Π· Π½ΠΈΡ…Ρ€ΠΎΠΌΠΎΠ²ΡƒΡŽ ΠΏΡ€ΠΎΠ²ΠΎΠ»ΠΎΠΊΡƒ Π² Π²ΠΎΠ΄Π΅ Ρ‚ΠΎΠΊ прСвращаСтся Π² Ρ‚Π΅ΠΏΠ»ΠΎ, ΠΈ Ρ‚Π΅ΠΌΠΏΠ΅Ρ€Π°Ρ‚ΡƒΡ€Π° ΠΏΠΎΠ²Ρ‹ΡˆΠ°Π΅Ρ‚ΡΡ. Π Π°Π±ΠΎΡ‚Ρƒ выполняСт Ρ‚Π΅ΠΏΠ»ΠΎ, выдСляСмоС Π² элСктричСской Ρ†Π΅ΠΏΠΈ, которая называСтся Electric ΠΌΠΎΡ‰Π½ΠΎΡΡ‚ΡŒ .

Π˜Π·ΠΌΠ΅Ρ€ΡΠ΅Ρ‚ΡΡ элСктричСская ΠΌΠΎΡ‰Π½ΠΎΡΡ‚ΡŒ Π² Π²Π°Ρ‚Ρ‚-часах (Π’Ρ‚Ρ‡), Π° количСство Ρ‚Π΅ΠΏΠ»Π° измСряСтся Π² калориях. (Cal).

Π Π°Π±ΠΎΡ‚Π° выполняСтся Π·Π° счСт выдСляСмого Ρ‚Π΅ΠΏΠ»Π° Π² элСктричСской Ρ†Π΅ΠΏΠΈ написано ΠΌΠΎΡ‰Π½ΠΎΡΡ‚ΡŒ, Ρ‡Ρ‚ΠΎ ΠΎΠ·Π½Π°Ρ‡Π°Π΅Ρ‚ Ρ‡Ρ‚ΠΎ номинальная Ρ€Π°Π±ΠΎΡ‚Π° выполняСтся Π² Ρ†Π΅ΠΏΠΈ, ΠΊΠΎΠ³Π΄Π° Ρ‚ΠΎΠΊ 1 А с ΠŸΡ€ΠΈΠΌΠ΅Π½ΡΠ΅Ρ‚ΡΡ 1 Π’ΠΎΠ»ΡŒΡ‚, Π° Π΅Π³ΠΎ Π΅Π΄ΠΈΠ½ΠΈΡ†Π° измСрСния — Π²Π°Ρ‚Ρ‚.

2. ЭлСктрохимия

НапримСр, ΠΊΠΎΠ³Π΄Π° Ρ‚ΠΎΠΊ ΠΏΡ€ΠΎΡ…ΠΎΠ΄ΠΈΡ‚ Ρ‡Π΅Ρ€Π΅Π· Ρ…Π»ΠΎΡ€ΠΈΠ΄ натрия (NaCl), химичСская рСакция, называСмая элСктролизом. ΠΈΠΌΠ΅Π΅Ρ‚ мСсто. ΠŸΡ€ΠΈΠΌΠ΅Π½ΡΠ΅Ρ‚ΡΡ для производства элСктролиза, цинкования. ΠΈ аккумулятор ΠΈ Ρ‚. Π΄.


ЭкспСримСнт проводится ΠΏΡƒΡ‚Π΅ΠΌ пропитывания Π΄Π²ΡƒΡ… ΠΏΠ»Π°Ρ‚ΠΈΠ½ΠΎΠ²Ρ‹Ρ… (Pt) пластин. Π² расплавС соли. ΠŸΠΎΠ΄ΠΊΠ»ΡŽΡ‡ΠΈΡ‚Π΅ Π±Π°Ρ‚Π°Ρ€Π΅ΠΈ ΠΊ Π΄Π²ΡƒΠΌ ΠΏΠ»Π°Ρ‚ΠΈΠ½ΠΎΠ²Ρ‹ΠΌ пластинам, Ρ‚ΠΎΠΊ ΠΏΡ€ΠΎΡ…ΠΎΠ΄ΠΈΡ‚ Ρ‡Π΅Ρ€Π΅Π· расплав соли ΠΈ ΠΏΡ€ΠΎΠΈΠ·Π²ΠΎΠ΄ΠΈΡ‚ Ρ…Π»ΠΎΡ€ ΠΏΡƒΠ·Ρ‹Ρ€ΠΈ Π²ΠΎΠΊΡ€ΡƒΠ³ ΠΏΠΎΠ»ΠΎΠΆΠΈΡ‚Π΅Π»ΡŒΠ½ΠΎΠΉ пластины (+) ΠΈ ΠΏΡƒΠ·Ρ‹Ρ€ΡŒΠΊΠΈ Π²ΠΎΠ΄ΠΎΡ€ΠΎΠ΄Π° Π²ΠΎΠΊΡ€ΡƒΠ³ ΠΎΡ‚Ρ€ΠΈΡ†Π°Ρ‚Π΅Π»ΡŒΠ½ΠΎΠΉ пластины (-), ΠΏΠΎΡΠΊΠΎΠ»ΡŒΠΊΡƒ Ρ…Π»ΠΎΡ€ΠΈΠ΄ натрия составляСт натрия (Na) ΠΈ Ρ…Π»ΠΎΡ€ΠΈΠ΄Π° (Cl).Когда Ρ…Π»ΠΎΡ€ΠΈΠ΄ натрия Ρ‚Π°Π΅Ρ‚ Π² Π²ΠΎΠ΄Π΅, элСмСнты Ρ€Π°Π·Π΄Π΅Π»ΡΡŽΡ‚ΡΡ. Натрий ΠΈΠΌΠ΅Π΅Ρ‚ ΠΏΠΎΠ»ΠΎΠΆΠΈΡ‚Π΅Π»ΡŒΠ½Ρ‹Π΅ заряды (+), Π° Ρƒ Ρ…Π»ΠΎΡ€Π° ΠΎΡ‚Ρ€ΠΈΡ†Π°Ρ‚Π΅Π»ΡŒΠ½Ρ‹Π΅ заряды (-) ΠΈ эти заряды Π½Π°Π·Ρ‹Π²Π°ΡŽΡ‚ΡΡ ΠΈΠΎΠ½Π°ΠΌΠΈ . Расплав соли ΠΈΠΌΠ΅Π΅Ρ‚ ΠΎΠ±Π° ΠΏΠΎΠ»ΠΎΠΆΠΈΡ‚Π΅Π»ΡŒΠ½Ρ‹Ρ… заряда, Π½Π°Π·Ρ‹Π²Π°Π΅ΠΌΡ‹Π΅ Π°Π½ΠΎΠ΄Π°ΠΌΠΈ ΠΈ , Π° ΠΎΡ‚Ρ€ΠΈΡ†Π°Ρ‚Π΅Π»ΡŒΠ½Ρ‹Π΅ заряды Π½Π°Π·Ρ‹Π²Π°ΡŽΡ‚ΡΡ ΠΊΠ°Ρ‚ΠΎΠ΄Π°ΠΌΠΈ . БостояниС Ρ€Π°Π·Π΄Π΅Π»Π΅Π½Π½Ρ‹Ρ… элСмСнтов называСтся ионизация .Если соль растапливаСтся Π²ΠΎΠ΄ΠΎΠΉ, Π² растворС ΠΈΠΌΠ΅ΡŽΡ‚ΡΡ ΠΈΠΎΠ½Ρ‹, называСтся раствор элСктролита . И Ссли Ρ‚Π΅ΠΊΡƒΡ‰ΠΈΠΉ ΠΏΡ€ΠΎΡ…ΠΎΠ΄ΠΈΡ‚ Ρ‡Π΅Ρ€Π΅Π· раствор элСктролита, химичСская рСакция происходит элСктролиз.

3. ΠœΠ°Π³Π½Π΅Ρ‚ΠΈΠ·ΠΌ

ΠŸΡ€ΠΈΠΌΠ΅Ρ€ Π΄Π°Π½Π½ΠΎΠΉ элСктромонтаТной Ρ€Π°Π±ΠΎΡ‚Ρ‹ — Ρ‚ΠΎΠΊΠΎΠ²Π΅Π΄ΡƒΡ‰ΠΈΠΉ ΠΏΡ€ΠΎΠ²ΠΎΠ»ΠΎΠΊΠ°, Π²ΠΎΠ·Π½ΠΈΠΊΠ°ΡŽΡ‚ ΠΌΠ°Π³Π½ΠΈΡ‚Π½Ρ‹Π΅ Π»ΠΈΠ½ΠΈΠΈ ΠΏΠΎΡ‚ΠΎΠΊΠ°.Π­Ρ‚ΠΎ примСняСтся для производства элСктродвигатСли, элСктричСскиС трансформаторы ΠΈ ΠΌΠ°Π³Π½ΠΈΡ‚ΠΎΡ„ΠΎΠ½Ρ‹, ΠΏΡ€.

ПониманиС смысла ΠΌΠ°Π³Π½Π΅Ρ‚ΠΈΠ·ΠΌΠ°:
Π§Ρ‚ΠΎ Ρ‚Π°ΠΊΠΎΠ΅ ΠΌΠ°Π³Π½Π΅Ρ‚ΠΈΠ·ΠΌ?

Боставная Ρ„ΠΎΡ€ΠΌΡƒΠ»Π° ΠΌΠ°Π³Π½ΠΈΡ‚Π°: Fe 3 O 4 . ВсС ΠΌΠ°Π³Π½ΠΈΡ‚Ρ‹ ΠΎΠ±Π»Π°Π΄Π°ΡŽΡ‚ двумя характСристиками. Π’ΠΎ-ΠΏΠ΅Ρ€Π²Ρ‹Ρ…, ΠΎΠ½ΠΈ ΠΏΡ€ΠΈΠ²Π»Π΅ΠΊΠ°ΡŽΡ‚ ΠΈ Π΄Π΅Ρ€ΠΆΠΈ ΠΆΠ΅Π»Π΅Π·ΠΎ.Π’Ρ‚ΠΎΡ€ΠΈΡ‡Π½Ρ‹ΠΉ, Ссли свободно Π΄Π²ΠΈΠ³Π°Ρ‚ΡŒΡΡ, ΠΊΠ°ΠΊ компас ΠΈΠ³Π»Π°, ΠΎΠ½ΠΈ Π·Π°ΠΉΠΌΡƒΡ‚ ΠΏΠΎΠ»ΠΎΠΆΠ΅Π½ΠΈΠ΅ сСвСр-юг. Π›ΡŽΠ±Ρ‹Π΅ ΠΌΠ°Ρ‚Π΅Ρ€ΠΈΠ°Π»Ρ‹ Π˜ΠΌΠ΅ΡŽΡ‚ Ρ‚Π°ΠΊΠΈΠ΅ характСристики, ΠΎΠ½ΠΈ Π½Π°Π·Ρ‹Π²Π°ΡŽΡ‚ΡΡ ΠΌΠ°Π³Π½ΠΈΡ‚ .

Π₯арактСристики ΠΌΠ°Π³Π½ΠΈΡ‚Π°
ΠšΠ°ΠΆΠ΄Ρ‹ΠΉ ΠΌΠ°Π³Π½ΠΈΡ‚ ΠΈΠΌΠ΅Π΅Ρ‚ Π΄Π²Π° полюса, ΠΎΠ΄ΠΈΠ½ сСвСрный полюс ΠΈ ΠΎΠ΄ΠΈΠ½ ΡŽΠΆΠ½Ρ‹ΠΉ полюс.
ΠŸΡ€ΠΎΡ‚ΠΈΠ²ΠΎΠΏΠΎΠ»ΠΎΠΆΠ½Ρ‹Π΅ полюса ΠΏΡ€ΠΈΡ‚ΡΠ³ΠΈΠ²Π°ΡŽΡ‚ΡΡ Π΄Ρ€ΡƒΠ³ ΠΊ Π΄Ρ€ΡƒΠ³Ρƒ, Π² Ρ‚ΠΎ врСмя ΠΊΠ°ΠΊ полюса ΠΎΡ‚Ρ‚Π°Π»ΠΊΠΈΠ²Π°ΡŽΡ‚ Π΄Ρ€ΡƒΠ³ Π΄Ρ€ΡƒΠ³Π°.

ЭлСктричСство ΠΈ ΠΌΠ°Π³Π½ΠΈΡ‚Π½ΠΎΠ΅ ΠΏΠΎΠ»Π΅

Когда магнитная стрСлка находится рядом с элСктричСским ΠΏΡ€ΠΎΠ²ΠΎΠ΄ΠΎΠΌ, ΠΊΠΎΡ‚ΠΎΡ€Ρ‹ΠΉ Ρ‚ΠΎΠΊ пропускаСтся, магнитная стрСлка Π²ΠΊΠ»ΡŽΡ‡Π°Π΅Ρ‚ Π½Π°ΠΏΡ€Π°Π²Π»Π΅Π½ΠΈΠ΅ Ρ‚ΠΎΠΊΠ° (см. рисунок 1 ΠΈ 2).Π‘Π»Π΅Π΄ΠΎΠ²Π°Ρ‚Π΅Π»ΡŒΠ½ΠΎ, элСктричСский Ρ‚ΠΎΠΊ Ρ‚Π°ΠΊΠΆΠ΅ создаСт связанный ΠΌΠ°Π³Π½ΠΈΡ‚Π½Ρ‹ΠΉ силу ΠΈΠ»ΠΈ говорят, Ρ‡Ρ‚ΠΎ элСктричСство способно ΠΏΡ€ΠΎΠΈΠ·Π²ΠΎΠ΄ΠΈΡ‚ΡŒ ΠΌΠ°Π³Π½ΠΈΡ‚Π½ΠΎΠ΅ ΠΏΠΎΠ»Π΅.

Когда магнитная ΠΈΠ³Π»Π° ΠΏΠΎΠΌΠ΅Ρ‰Π΅Π½Π° Π² ΠΊΠ°Ρ‚ΡƒΡˆΠΊΡƒ с ΠΏΡ€ΠΎΠ²ΠΎΠ»ΠΎΠΊΠΎΠΉ с ΠΎΠ΄Π½ΠΎΠΉ ΠΏΠ΅Ρ‚Π»Π΅ΠΉ (см. рисунок), ΠΈ Ρ‚ΠΎΠΊ ΠΏΡ€ΠΎΡ…ΠΎΠ΄ΠΈΡ‚ Ρ‡Π΅Ρ€Π΅Π· ΠΊΠ°Ρ‚ΡƒΡˆΠΊΡƒ с ΠΏΡ€ΠΎΠ²ΠΎΠ»ΠΎΠΊΠΎΠΉ, ΠΌΠ°Π³Π½ΠΈΡ‚Π½Ρ‹ΠΉ ΠΈΠ³Π»Π° поворачиваСтся Π² Π½Π°ΠΏΡ€Π°Π²Π»Π΅Π½ΠΈΠΈ, ΠΏΠΎΠΊΠ°Π·Π°Π½Π½ΠΎΠΌ Π½Π° рисункС Π²Ρ‹ΡˆΠ΅.А направлСния ΠΌΠ°Π³Π½ΠΈΡ‚Π½Ρ‹Ρ… Π»ΠΈΠ½ΠΈΠΉ ΠΏΠΎΡ‚ΠΎΠΊΠ° ΠΏΠΎΠΊΠ°Π·Π°Π½Ρ‹ стрСлки.

Когда магнитная ΠΈΠ³Π»Π° ΠΏΠΎΠΌΠ΅Ρ‰Π΅Π½Π° Π² ΠΏΡ€ΠΎΠ²ΠΎΠ»ΠΎΡ‡Π½ΡƒΡŽ ΠΊΠ°Ρ‚ΡƒΡˆΠΊΡƒ с мноТСством ΠΏΠ΅Ρ‚Π΅Π»ΡŒ ΠΊΠ°ΠΊ ΠΏΠΎΠΊΠ°Π·Π°Π½ΠΎ Π½Π° ΠΏΡ€Π°Π²ΠΎΠΌ рисункС, Ρ‚ΠΎΠΊ ΠΏΡ€ΠΎΡ…ΠΎΠ΄ΠΈΡ‚ Ρ‡Π΅Ρ€Π΅Π· ΠΊΠ°Ρ‚ΡƒΡˆΠΊΠ°. НаправлСниС ΠΌΠ°Π³Π½ΠΈΡ‚Π½Ρ‹Ρ… Π»ΠΈΠ½ΠΈΠΉ ΠΌΠ°Π³Π½ΠΈΡ‚Π½Ρ‹Ρ… ΠΏΠ°Ρ€Π°Π»Π»Π΅Π»Π΅ΠΉ ΠΊΠ°Ρ‚ΡƒΡˆΠΊΠ° ΠΏΡ€ΠΎΠ²ΠΎΠ»ΠΎΠΊΠΈ. Π₯арактСристики ΠΌΠ°Π³Π½ΠΈΡ‚Π½Ρ‹Ρ… Π»ΠΈΠ½ΠΈΠΉ ΠΏΠΎΡ‚ΠΎΠΊΠ° ΠΊΠ°ΠΊ характСристики ΠΌΠ°Π³Π½ΠΈΡ‚Π°, Π½ΠΎ Π±Π΅Π· ΠΌΠ°Π³Π½ΠΈΡ‚Π½ΠΎΠ³ΠΎ полюса.

Когда ΠΊΠ°Ρ‚ΡƒΡˆΠΊΠ° с Ρ‚ΠΎΠΊΠΎΠ²Π΅Π΄ΡƒΡ‰ΠΈΠΌ ΠΏΡ€ΠΎΠ²ΠΎΠ΄ΠΎΠΌ находится рядом с ΠΆΠ΅Π»Π΅Π·Π½Ρ‹ΠΌ стСрТнСм, ΠΆΠ΅Π»Π΅Π·Π½Ρ‹ΠΉ ΡΡ‚Π΅Ρ€ΠΆΠ΅Π½ΡŒ Π½Π΅ΠΌΠ½ΠΎΠ³ΠΎ сдвинСтся (см. рисунок 1). Если сСрдСчник Ρ€Π°Π·ΠΌΠ΅Ρ‰Π΅Π½ Π² ΠΊΠ°Ρ‚ΡƒΡˆΠΊΠ΅ ΠΈΠ· ΠΏΡ€ΠΎΠ²ΠΎΠ»ΠΎΠΊΠΈ ΠΆΠ΅Π»Π΅Π·Π½Ρ‹ΠΉ ΡΡ‚Π΅Ρ€ΠΆΠ΅Π½ΡŒ сильно притягиваСтся (см. Ρ„ΠΈΠ³ΡƒΡ€Π° 2). ΠŸΠΎΡΠΊΠΎΠ»ΡŒΠΊΡƒ сСрдСчник — это мягкоС ΠΆΠ΅Π»Π΅Π·ΠΎ, ΠΊΠΎΡ‚ΠΎΡ€ΠΎΠ΅ ΠΏΡ€ΠΎΠ²ΠΎΠ΄ΠΈΡ‚ ΠΌΠ°Π³Π½ΠΈΡ‚Π½Ρ‹Π΅ силовыС Π»ΠΈΠ½ΠΈΠΈ, ΠΊΠΎΠ³Π΄Π° Ρ‚ΠΎΠΊ ΠΏΡ€ΠΎΡ…ΠΎΠ΄ΠΈΡ‚ Ρ‡Π΅Ρ€Π΅Π· ΠΏΡ€ΠΎΠ²ΠΎΠ»ΠΎΡ‡Π½ΡƒΡŽ ΠΊΠ°Ρ‚ΡƒΡˆΠΊΡƒ Π²ΠΎΠΊΡ€ΡƒΠ³ сСрдСчника сСрдСчник намагничиваСтся с высокой ΠΌΠΎΡ‰Π½ΠΎΡΡ‚ΡŒΡŽ Ρ‡Ρ‚ΠΎ называСтся элСктромагнитов .Π­Ρ‚Π° функция ΡˆΠΈΡ€ΠΎΠΊΠΎ примСняСтся Π² ΠΏΡ€ΠΎΠΌΡ‹ΡˆΠ»Π΅Π½Π½ΠΎΡΡ‚ΠΈ.

Π£Ρ‡Π΅Π±Π½ΠΈΠΊ ΠΏΠΎ Ρ„ΠΈΠ·ΠΈΠΊΠ΅: элСктричСский Ρ‚ΠΎΠΊ

Если Π΄Π²Π° трСбования элСктричСской Ρ†Π΅ΠΏΠΈ ΡΠΎΠ±Π»ΡŽΠ΄Π΅Π½Ρ‹, заряд Π±ΡƒΠ΄Π΅Ρ‚ ΠΏΡ€ΠΎΡ…ΠΎΠ΄ΠΈΡ‚ΡŒ Ρ‡Π΅Ρ€Π΅Π· внСшнюю Ρ†Π΅ΠΏΡŒ.Говорят, Ρ‡Ρ‚ΠΎ Π΅ΡΡ‚ΡŒ Ρ‚ΠΎΠΊ — ΠΏΠΎΡ‚ΠΎΠΊ заряда. ИспользованиС слова Ρ‚ΠΎΠΊ Π² этом контСкстС ΠΎΠ·Π½Π°Ρ‡Π°Π΅Ρ‚ просто ΠΈΡΠΏΠΎΠ»ΡŒΠ·ΠΎΠ²Π°Ρ‚ΡŒ Π΅Π³ΠΎ, Ρ‡Ρ‚ΠΎΠ±Ρ‹ ΡΠΊΠ°Π·Π°Ρ‚ΡŒ, Ρ‡Ρ‚ΠΎ Ρ‡Ρ‚ΠΎ-Ρ‚ΠΎ происходит Π² ΠΏΡ€ΠΎΠ²ΠΎΠ΄Π°Ρ… — заряд двиТСтся. Однако Ρ‚ΠΎΠΊ — это физичСская Π²Π΅Π»ΠΈΡ‡ΠΈΠ½Π°, ΠΊΠΎΡ‚ΠΎΡ€ΡƒΡŽ ΠΌΠΎΠΆΠ½ΠΎ ΠΈΠ·ΠΌΠ΅Ρ€ΠΈΡ‚ΡŒ ΠΈ Π²Ρ‹Ρ€Π°Π·ΠΈΡ‚ΡŒ числСнно. Как физичСская Π²Π΅Π»ΠΈΡ‡ΠΈΠ½Π°, , Ρ‚ΠΎΠΊ, — это ΡΠΊΠΎΡ€ΠΎΡΡ‚ΡŒ, с ΠΊΠΎΡ‚ΠΎΡ€ΠΎΠΉ заряд ΠΏΡ€ΠΎΡ…ΠΎΠ΄ΠΈΡ‚ Ρ‡Π΅Ρ€Π΅Π· Ρ‚ΠΎΡ‡ΠΊΡƒ Π² Ρ†Π΅ΠΏΠΈ. Как ΠΏΠΎΠΊΠ°Π·Π°Π½ΠΎ Π½Π° Π΄ΠΈΠ°Π³Ρ€Π°ΠΌΠΌΠ΅ Π½ΠΈΠΆΠ΅, Ρ‚ΠΎΠΊ Π² Ρ†Π΅ΠΏΠΈ ΠΌΠΎΠΆΠ½ΠΎ ΠΎΠΏΡ€Π΅Π΄Π΅Π»ΠΈΡ‚ΡŒ, Ссли ΠΌΠΎΠΆΠ½ΠΎ ΠΈΠ·ΠΌΠ΅Ρ€ΠΈΡ‚ΡŒ количСство заряда Q , проходящСго Ρ‡Π΅Ρ€Π΅Π· ΠΏΠΎΠΏΠ΅Ρ€Π΅Ρ‡Π½ΠΎΠ΅ сСчСниС ΠΏΡ€ΠΎΠ²ΠΎΠ΄Π° Π·Π° врСмя t .Π’ΠΎΠΊ — это просто ΡΠΎΠΎΡ‚Π½ΠΎΡˆΠ΅Π½ΠΈΠ΅ количСства заряда ΠΈ Π²Ρ€Π΅ΠΌΠ΅Π½ΠΈ.

Current — это Π²Π΅Π»ΠΈΡ‡ΠΈΠ½Π° ставки. Π’ Ρ„ΠΈΠ·ΠΈΠΊΠ΅ Π΅ΡΡ‚ΡŒ нСсколько скоростных Π²Π΅Π»ΠΈΡ‡ΠΈΠ½. НапримСр, ΡΠΊΠΎΡ€ΠΎΡΡ‚ΡŒ — это Π²Π΅Π»ΠΈΡ‡ΠΈΠ½Π° скорости — ΡΠΊΠΎΡ€ΠΎΡΡ‚ΡŒ, с ΠΊΠΎΡ‚ΠΎΡ€ΠΎΠΉ ΠΎΠ±ΡŠΠ΅ΠΊΡ‚ мСняСт своС ΠΏΠΎΠ»ΠΎΠΆΠ΅Π½ΠΈΠ΅. ΠœΠ°Ρ‚Π΅ΠΌΠ°Ρ‚ΠΈΡ‡Π΅ΡΠΊΠΈ ΡΠΊΠΎΡ€ΠΎΡΡ‚ΡŒ — это ΠΎΡ‚Π½ΠΎΡˆΠ΅Π½ΠΈΠ΅ измСнСния полоТСния ΠΊ Π²Ρ€Π΅ΠΌΠ΅Π½ΠΈ. УскорСниС — это Π²Π΅Π»ΠΈΡ‡ΠΈΠ½Π° скорости — ΡΠΊΠΎΡ€ΠΎΡΡ‚ΡŒ, с ΠΊΠΎΡ‚ΠΎΡ€ΠΎΠΉ ΠΎΠ±ΡŠΠ΅ΠΊΡ‚ мСняСт свою ΡΠΊΠΎΡ€ΠΎΡΡ‚ΡŒ. ΠœΠ°Ρ‚Π΅ΠΌΠ°Ρ‚ΠΈΡ‡Π΅ΡΠΊΠΈ ускорСниС — это ΠΎΡ‚Π½ΠΎΡˆΠ΅Π½ΠΈΠ΅ измСнСния скорости ΠΊ Π²Ρ€Π΅ΠΌΠ΅Π½ΠΈ. А ΠΌΠΎΡ‰Π½ΠΎΡΡ‚ΡŒ — это Π²Π΅Π»ΠΈΡ‡ΠΈΠ½Π° скорости — ΡΠΊΠΎΡ€ΠΎΡΡ‚ΡŒ, с ΠΊΠΎΡ‚ΠΎΡ€ΠΎΠΉ Ρ€Π°Π±ΠΎΡ‚Π° выполняСтся Π½Π° ΠΎΠ±ΡŠΠ΅ΠΊΡ‚Π΅.ΠœΠ°Ρ‚Π΅ΠΌΠ°Ρ‚ΠΈΡ‡Π΅ΡΠΊΠΈ ΠΌΠΎΡ‰Π½ΠΎΡΡ‚ΡŒ — это ΠΎΡ‚Π½ΠΎΡˆΠ΅Π½ΠΈΠ΅ Ρ€Π°Π±ΠΎΡ‚Ρ‹ ΠΊ Π²Ρ€Π΅ΠΌΠ΅Π½ΠΈ. Π’ ΠΊΠ°ΠΆΠ΄ΠΎΠΌ случаС Π²Π΅Π»ΠΈΡ‡ΠΈΠ½Ρ‹ скорости матСматичСскоС ΡƒΡ€Π°Π²Π½Π΅Π½ΠΈΠ΅ Π²ΠΊΠ»ΡŽΡ‡Π°Π΅Ρ‚ Π½Π΅ΠΊΠΎΡ‚ΠΎΡ€ΡƒΡŽ Π²Π΅Π»ΠΈΡ‡ΠΈΠ½Ρƒ Π²ΠΎ Π²Ρ€Π΅ΠΌΠ΅Π½ΠΈ. Π’Π°ΠΊΠΈΠΌ ΠΎΠ±Ρ€Π°Π·ΠΎΠΌ, Ρ‚ΠΎΠΊ ΠΊΠ°ΠΊ Π²Π΅Π»ΠΈΡ‡ΠΈΠ½Π° скорости Π±ΡƒΠ΄Π΅Ρ‚ матСматичСски Π²Ρ‹Ρ€Π°ΠΆΠ΅Π½ ΠΊΠ°ΠΊ

ΠžΠ±Ρ€Π°Ρ‚ΠΈΡ‚Π΅ Π²Π½ΠΈΠΌΠ°Π½ΠΈΠ΅, Ρ‡Ρ‚ΠΎ Π² ΠΏΡ€ΠΈΠ²Π΅Π΄Π΅Π½Π½ΠΎΠΌ Π²Ρ‹ΡˆΠ΅ ΡƒΡ€Π°Π²Π½Π΅Π½ΠΈΠΈ ΠΈΡΠΏΠΎΠ»ΡŒΠ·ΡƒΠ΅Ρ‚ΡΡ символ I для обозначСния Π²Π΅Π»ΠΈΡ‡ΠΈΠ½Ρ‹ Ρ‚ΠΎΠΊΠ°.

Как ΠΎΠ±Ρ‹Ρ‡Π½ΠΎ, ΠΊΠΎΠ³Π΄Π° количСство вводится Π² ЀизичСском классС, Ρ‚Π°ΠΊΠΆΠ΅ вводится стандартная мСтричСская Π΅Π΄ΠΈΠ½ΠΈΡ†Π°, ΠΈΡΠΏΠΎΠ»ΡŒΠ·ΡƒΠ΅ΠΌΠ°Ρ для выраТСния этой Π²Π΅Π»ΠΈΡ‡ΠΈΠ½Ρ‹.Бтандартная мСтричСская Π΅Π΄ΠΈΠ½ΠΈΡ†Π° измСрСния Ρ‚ΠΎΠΊΠ° — Π°ΠΌΠΏΠ΅Ρ€ . АмпСр часто сокращаСтся Π΄ΠΎ А ΠΈ обозначаСтся символом A . Π’ΠΎΠΊ Π² 1 Π°ΠΌΠΏΠ΅Ρ€ ΠΎΠ·Π½Π°Ρ‡Π°Π΅Ρ‚, Ρ‡Ρ‚ΠΎ 1 ΠΊΡƒΠ»ΠΎΠ½ заряда ΠΏΡ€ΠΎΡ…ΠΎΠ΄ΠΈΡ‚ Ρ‡Π΅Ρ€Π΅Π· ΠΏΠΎΠΏΠ΅Ρ€Π΅Ρ‡Π½ΠΎΠ΅ сСчСниС ΠΏΡ€ΠΎΠ²ΠΎΠ΄Π° ΠΊΠ°ΠΆΠ΄ΡƒΡŽ 1 сСкунду.

1 Π°ΠΌΠΏΠ΅Ρ€ = 1 ΠΊΡƒΠ»ΠΎΠ½ / 1 сСкунда

Π§Ρ‚ΠΎΠ±Ρ‹ ΠΏΡ€ΠΎΠ²Π΅Ρ€ΠΈΡ‚ΡŒ своС ΠΏΠΎΠ½ΠΈΠΌΠ°Π½ΠΈΠ΅, ΠΎΠΏΡ€Π΅Π΄Π΅Π»ΠΈΡ‚Π΅ Ρ‚ΠΎΠΊ для ΡΠ»Π΅Π΄ΡƒΡŽΡ‰ΠΈΡ… Π΄Π²ΡƒΡ… ситуаций. ΠžΠ±Ρ€Π°Ρ‚ΠΈΡ‚Π΅ Π²Π½ΠΈΠΌΠ°Π½ΠΈΠ΅, Ρ‡Ρ‚ΠΎ Π² ΠΊΠ°ΠΆΠ΄ΠΎΠΉ ситуации даСтся нСкоторая посторонняя информация.НаТмитС ΠΊΠ½ΠΎΠΏΠΊΡƒ ΠŸΡ€ΠΎΠ²Π΅Ρ€ΠΈΡ‚ΡŒ ΠΎΡ‚Π²Π΅Ρ‚ , Ρ‡Ρ‚ΠΎΠ±Ρ‹ ΡƒΠ±Π΅Π΄ΠΈΡ‚ΡŒΡΡ, Ρ‡Ρ‚ΠΎ Π²Ρ‹ ΠΏΡ€Π°Π²Ρ‹.

ΠŸΡ€ΠΎΠ²ΠΎΠ΄ ΠΈΠ·ΠΎΠ»ΠΈΡ€ΡƒΡŽΡ‚ ΠΏΠΎΠΏΠ΅Ρ€Π΅Ρ‡Π½Ρ‹ΠΌ сСчСниСм 2 ΠΌΠΌ ΠΈ ΠΎΠΏΡ€Π΅Π΄Π΅Π»ΡΡŽΡ‚, Ρ‡Ρ‚ΠΎ заряд 20 C ΠΏΡ€ΠΎΠΉΠ΄Π΅Ρ‚ Ρ‡Π΅Ρ€Π΅Π· Π½Π΅Π³ΠΎ Π·Π° 40 с.

Π‘Π΅Ρ‡Π΅Π½ΠΈΠ΅ ΠΏΡ€ΠΎΠ²ΠΎΠ΄Π° Π΄Π»ΠΈΠ½ΠΎΠΉ 1 ΠΌΠΌ изолируСтся, ΠΈ опрСдСляСтся, Ρ‡Ρ‚ΠΎ заряд 2 Кл ΠΏΡ€ΠΎΡ…ΠΎΠ΄ΠΈΡ‚ Ρ‡Π΅Ρ€Π΅Π· Π½Π΅Π³ΠΎ Π·Π° 0,5 с.

I = _____ АмпСр

I = _____ АмпСр

ΠžΠ±Ρ‹Ρ‡Π½ΠΎΠ΅ Π½Π°ΠΏΡ€Π°Π²Π»Π΅Π½ΠΈΠ΅ Ρ‚ΠΎΠΊΠ°

Частицы, ΠΊΠΎΡ‚ΠΎΡ€Ρ‹Π΅ пСрСносят заряд ΠΏΠΎ ΠΏΡ€ΠΎΠ²ΠΎΠ΄Π°ΠΌ Π² Ρ†Π΅ΠΏΠΈ, ΡΠ²Π»ΡΡŽΡ‚ΡΡ ΠΏΠΎΠ΄Π²ΠΈΠΆΠ½Ρ‹ΠΌΠΈ элСктронами.НаправлСниС элСктричСского поля Π² Ρ†Π΅ΠΏΠΈ ΠΏΠΎ ΠΎΠΏΡ€Π΅Π΄Π΅Π»Π΅Π½ΠΈΡŽ являСтся Π½Π°ΠΏΡ€Π°Π²Π»Π΅Π½ΠΈΠ΅ΠΌ, Π² ΠΊΠΎΡ‚ΠΎΡ€ΠΎΠΌ ΠΏΡ€ΠΎΡ‚Π°Π»ΠΊΠΈΠ²Π°ΡŽΡ‚ΡΡ ΠΏΠΎΠ»ΠΎΠΆΠΈΡ‚Π΅Π»ΡŒΠ½Ρ‹Π΅ ΠΈΡΠΏΡ‹Ρ‚Π°Ρ‚Π΅Π»ΡŒΠ½Ρ‹Π΅ заряды. Π’Π°ΠΊΠΈΠΌ ΠΎΠ±Ρ€Π°Π·ΠΎΠΌ, эти ΠΎΡ‚Ρ€ΠΈΡ†Π°Ρ‚Π΅Π»ΡŒΠ½ΠΎ заряТСнныС элСктроны двиТутся Π² Π½Π°ΠΏΡ€Π°Π²Π»Π΅Π½ΠΈΠΈ, ΠΏΡ€ΠΎΡ‚ΠΈΠ²ΠΎΠΏΠΎΠ»ΠΎΠΆΠ½ΠΎΠΌ элСктричСскому полю. Но Π² Ρ‚ΠΎ врСмя ΠΊΠ°ΠΊ элСктроны ΡΠ²Π»ΡΡŽΡ‚ΡΡ носитСлями заряда Π² мСталличСских ΠΏΡ€ΠΎΠ²ΠΎΠ΄Π°Ρ…, носитСлями заряда Π² Π΄Ρ€ΡƒΠ³ΠΈΡ… цСпях ΠΌΠΎΠ³ΡƒΡ‚ Π±Ρ‹Ρ‚ΡŒ ΠΏΠΎΠ»ΠΎΠΆΠΈΡ‚Π΅Π»ΡŒΠ½Ρ‹Π΅ заряды, ΠΎΡ‚Ρ€ΠΈΡ†Π°Ρ‚Π΅Π»ΡŒΠ½Ρ‹Π΅ заряды ΠΈΠ»ΠΈ ΠΈ Ρ‚ΠΎ, ΠΈ Π΄Ρ€ΡƒΠ³ΠΎΠ΅. ЀактичСски, носитСли заряда Π² ΠΏΠΎΠ»ΡƒΠΏΡ€ΠΎΠ²ΠΎΠ΄Π½ΠΈΠΊΠ°Ρ…, ΡƒΠ»ΠΈΡ‡Π½Ρ‹Ρ… фонарях ΠΈ Π»ΡŽΠΌΠΈΠ½Π΅ΡΡ†Π΅Π½Ρ‚Π½Ρ‹Ρ… Π»Π°ΠΌΠΏΠ°Ρ… ΠΎΠ΄Π½ΠΎΠ²Ρ€Π΅ΠΌΠ΅Π½Π½ΠΎ ΡΠ²Π»ΡΡŽΡ‚ΡΡ ΠΊΠ°ΠΊ ΠΏΠΎΠ»ΠΎΠΆΠΈΡ‚Π΅Π»ΡŒΠ½Ρ‹ΠΌΠΈ, Ρ‚Π°ΠΊ ΠΈ ΠΎΡ‚Ρ€ΠΈΡ†Π°Ρ‚Π΅Π»ΡŒΠ½Ρ‹ΠΌΠΈ зарядами, двиТущимися Π² ΠΏΡ€ΠΎΡ‚ΠΈΠ²ΠΎΠΏΠΎΠ»ΠΎΠΆΠ½Ρ‹Ρ… направлСниях.

Π‘Π΅Π½ Π€Ρ€Π°Π½ΠΊΠ»ΠΈΠ½, ΠΏΡ€ΠΎΠ²ΠΎΠ΄ΠΈΠ²ΡˆΠΈΠΉ ΠΎΠ±ΡˆΠΈΡ€Π½Ρ‹Π΅ Π½Π°ΡƒΡ‡Π½Ρ‹Π΅ исслСдования статичСского ΠΈ Ρ‚Π΅ΠΊΡƒΡ‰Π΅Π³ΠΎ элСктричСства, рассматривал ΠΏΠΎΠ»ΠΎΠΆΠΈΡ‚Π΅Π»ΡŒΠ½Ρ‹Π΅ заряды ΠΊΠ°ΠΊ носитСли заряда. Π’Π°ΠΊΠΈΠΌ ΠΎΠ±Ρ€Π°Π·ΠΎΠΌ, Ρ€Π°Π½Π½Π΅Π΅ соглашСниС ΠΎ Π½Π°ΠΏΡ€Π°Π²Π»Π΅Π½ΠΈΠΈ элСктричСского Ρ‚ΠΎΠΊΠ° Π±Ρ‹Π»ΠΎ установлСно Π² Ρ‚ΠΎΠΌ Π½Π°ΠΏΡ€Π°Π²Π»Π΅Π½ΠΈΠΈ, Π² ΠΊΠΎΡ‚ΠΎΡ€ΠΎΠΌ Π±ΡƒΠ΄ΡƒΡ‚ Π΄Π²ΠΈΠ³Π°Ρ‚ΡŒΡΡ ΠΏΠΎΠ»ΠΎΠΆΠΈΡ‚Π΅Π»ΡŒΠ½Ρ‹Π΅ заряды. ΠšΠΎΠ½Π²Π΅Π½Ρ†ΠΈΡ ΠΏΡ€ΠΈΠΆΠΈΠ»Π°ΡΡŒ ΠΈ ΠΈΡΠΏΠΎΠ»ΡŒΠ·ΡƒΠ΅Ρ‚ΡΡ Π΄ΠΎ сих ΠΏΠΎΡ€. НаправлСниС элСктричСского Ρ‚ΠΎΠΊΠ° условно являСтся Π½Π°ΠΏΡ€Π°Π²Π»Π΅Π½ΠΈΠ΅ΠΌ, Π² ΠΊΠΎΡ‚ΠΎΡ€ΠΎΠΌ Π΄ΠΎΠ»ΠΆΠ΅Π½ Π΄Π²ΠΈΠ³Π°Ρ‚ΡŒΡΡ ΠΏΠΎΠ»ΠΎΠΆΠΈΡ‚Π΅Π»ΡŒΠ½Ρ‹ΠΉ заряд. Π’Π°ΠΊΠΈΠΌ ΠΎΠ±Ρ€Π°Π·ΠΎΠΌ, Ρ‚ΠΎΠΊ Π²ΠΎ внСшнСй Ρ†Π΅ΠΏΠΈ Π½Π°ΠΏΡ€Π°Π²Π»Π΅Π½ ΠΎΡ‚ ΠΏΠΎΠ»ΠΎΠΆΠΈΡ‚Π΅Π»ΡŒΠ½ΠΎΠΉ ΠΊΠ»Π΅ΠΌΠΌΡ‹ ΠΊ ΠΎΡ‚Ρ€ΠΈΡ†Π°Ρ‚Π΅Π»ΡŒΠ½ΠΎΠΉ ΠΊΠ»Π΅ΠΌΠΌΠ΅ Π±Π°Ρ‚Π°Ρ€Π΅ΠΈ.Π­Π»Π΅ΠΊΡ‚Ρ€ΠΎΠ½Ρ‹ Π΄Π΅ΠΉΡΡ‚Π²ΠΈΡ‚Π΅Π»ΡŒΠ½ΠΎ Π±ΡƒΠ΄ΡƒΡ‚ Π΄Π²ΠΈΠ³Π°Ρ‚ΡŒΡΡ ΠΏΠΎ ΠΏΡ€ΠΎΠ²ΠΎΠ΄Π°ΠΌ Π² ΠΏΡ€ΠΎΡ‚ΠΈΠ²ΠΎΠΏΠΎΠ»ΠΎΠΆΠ½ΠΎΠΌ Π½Π°ΠΏΡ€Π°Π²Π»Π΅Π½ΠΈΠΈ. Зная, Ρ‡Ρ‚ΠΎ настоящими носитСлями заряда Π² ΠΏΡ€ΠΎΠ²ΠΎΠ΄Π°Ρ… ΡΠ²Π»ΡΡŽΡ‚ΡΡ ΠΎΡ‚Ρ€ΠΈΡ†Π°Ρ‚Π΅Π»ΡŒΠ½ΠΎ заряТСнныС элСктроны, это соглашСниС ΠΌΠΎΠΆΠ΅Ρ‚ ΠΏΠΎΠΊΠ°Π·Π°Ρ‚ΡŒΡΡ Π½Π΅ΠΌΠ½ΠΎΠ³ΠΎ странным ΠΈ ΡƒΡΡ‚Π°Ρ€Π΅Π²ΡˆΠΈΠΌ. Π’Π΅ΠΌ Π½Π΅ ΠΌΠ΅Π½Π΅Π΅, это соглашСниС, ΠΊΠΎΡ‚ΠΎΡ€ΠΎΠ΅ ΠΈΡΠΏΠΎΠ»ΡŒΠ·ΡƒΠ΅Ρ‚ΡΡ Π²ΠΎ всСм ΠΌΠΈΡ€Π΅, ΠΈ ΠΊ ΠΊΠΎΡ‚ΠΎΡ€ΠΎΠΌΡƒ студСнт-Ρ„ΠΈΠ·ΠΈΠΊ ΠΌΠΎΠΆΠ΅Ρ‚ Π»Π΅Π³ΠΊΠΎ ΠΏΡ€ΠΈΠ²Ρ‹ΠΊΠ½ΡƒΡ‚ΡŒ.

Π—Π°Π²ΠΈΡΠΈΠΌΠΎΡΡ‚ΡŒ Ρ‚ΠΎΠΊΠ° ΠΎΡ‚ скорости Π΄Ρ€Π΅ΠΉΡ„Π°

Π’ΠΎΠΊ связан с количСством ΠΊΡƒΠ»ΠΎΠ½ΠΎΠ² заряда, ΠΊΠΎΡ‚ΠΎΡ€Ρ‹Π΅ проходят Ρ‚ΠΎΡ‡ΠΊΡƒ Π² Ρ†Π΅ΠΏΠΈ Π·Π° Π΅Π΄ΠΈΠ½ΠΈΡ†Ρƒ Π²Ρ€Π΅ΠΌΠ΅Π½ΠΈ.Из-Π·Π° своСго опрСдСлСния Π΅Π³ΠΎ часто ΠΏΡƒΡ‚Π°ΡŽΡ‚ со ΡΠΊΠΎΡ€ΠΎΡΡ‚ΡŒΡŽ Π΄Ρ€Π΅ΠΉΡ„Π° количСства. Π‘ΠΊΠΎΡ€ΠΎΡΡ‚ΡŒ Π΄Ρ€Π΅ΠΉΡ„Π° ΠΎΠ·Π½Π°Ρ‡Π°Π΅Ρ‚ срСднСС расстояниС, ΠΏΡ€ΠΎΠΉΠ΄Π΅Π½Π½ΠΎΠ΅ носитСлСм заряда Π·Π° Π΅Π΄ΠΈΠ½ΠΈΡ†Ρƒ Π²Ρ€Π΅ΠΌΠ΅Π½ΠΈ. Как ΠΈ ΡΠΊΠΎΡ€ΠΎΡΡ‚ΡŒ любого ΠΎΠ±ΡŠΠ΅ΠΊΡ‚Π°, ΡΠΊΠΎΡ€ΠΎΡΡ‚ΡŒ Π΄Ρ€Π΅ΠΉΡ„Π° элСктрона, двиТущСгося ΠΏΠΎ ΠΏΡ€ΠΎΠ²ΠΎΠ΄Ρƒ, — это ΠΎΡ‚Π½ΠΎΡˆΠ΅Π½ΠΈΠ΅ расстояния ΠΊΠΎ Π²Ρ€Π΅ΠΌΠ΅Π½ΠΈ. ΠŸΡƒΡ‚ΡŒ Ρ‚ΠΈΠΏΠΈΡ‡Π½ΠΎΠ³ΠΎ элСктрона Ρ‡Π΅Ρ€Π΅Π· ΠΏΡ€ΠΎΠ²ΠΎΠ»ΠΎΠΊΡƒ ΠΌΠΎΠΆΠ½ΠΎ ΠΎΠΏΠΈΡΠ°Ρ‚ΡŒ ΠΊΠ°ΠΊ довольно хаотичСский Π·ΠΈΠ³Π·Π°Π³ΠΎΠΎΠ±Ρ€Π°Π·Π½Ρ‹ΠΉ ΠΏΡƒΡ‚ΡŒ, Ρ…Π°Ρ€Π°ΠΊΡ‚Π΅Ρ€ΠΈΠ·ΡƒΡŽΡ‰ΠΈΠΉΡΡ столкновСниями с Π½Π΅ΠΏΠΎΠ΄Π²ΠΈΠΆΠ½Ρ‹ΠΌΠΈ Π°Ρ‚ΠΎΠΌΠ°ΠΌΠΈ. КаТдоС столкновСниС ΠΏΡ€ΠΈΠ²ΠΎΠ΄ΠΈΡ‚ ΠΊ измСнСнию направлСния элСктрона.Однако ΠΈΠ·-Π·Π° столкновСний с Π°Ρ‚ΠΎΠΌΠ°ΠΌΠΈ Π² Ρ‚Π²Π΅Ρ€Π΄ΠΎΠΉ сСти мСталличСского ΠΏΡ€ΠΎΠ²ΠΎΠ΄Π½ΠΈΠΊΠ° Π½Π° ΠΊΠ°ΠΆΠ΄Ρ‹Π΅ Ρ‚Ρ€ΠΈ шага Π²ΠΏΠ΅Ρ€Π΅Π΄ приходится Π΄Π²Π° шага Π½Π°Π·Π°Π΄. Π‘ элСктричСским ΠΏΠΎΡ‚Π΅Π½Ρ†ΠΈΠ°Π»ΠΎΠΌ, установлСнным Π½Π° Π΄Π²ΡƒΡ… ΠΊΠΎΠ½Ρ†Π°Ρ… Ρ†Π΅ΠΏΠΈ, элСктрон ΠΏΡ€ΠΎΠ΄ΠΎΠ»ΠΆΠ°Π΅Ρ‚ Π΄Π²ΠΈΠΆΠ΅Π½ΠΈΠ΅ Π²ΠΏΠ΅Ρ€Π΅Π΄ Π½Π° . ΠŸΡ€ΠΎΠ³Ρ€Π΅ΡΡ всСгда ΠΈΠ΄Π΅Ρ‚ ΠΊ ΠΏΠΎΠ»ΠΎΠΆΠΈΡ‚Π΅Π»ΡŒΠ½ΠΎΠΌΡƒ ΠΏΠΎΠ»ΡŽΡΡƒ. Однако ΠΎΠ±Ρ‰ΠΈΠΉ эффСкт бСсчислСнных столкновСний ΠΈ высоких скоростСй ΠΌΠ΅ΠΆΠ΄Ρƒ столкновСниями состоит Π² Ρ‚ΠΎΠΌ, Ρ‡Ρ‚ΠΎ общая ΡΠΊΠΎΡ€ΠΎΡΡ‚ΡŒ Π΄Ρ€Π΅ΠΉΡ„Π° элСктрона Π² Ρ†Π΅ΠΏΠΈ Π½Π΅Π½ΠΎΡ€ΠΌΠ°Π»ΡŒΠ½ΠΎ ΠΌΠ°Π»Π°. Випичная ΡΠΊΠΎΡ€ΠΎΡΡ‚ΡŒ Π΄Ρ€Π΅ΠΉΡ„Π° ΠΌΠΎΠΆΠ΅Ρ‚ ΡΠΎΡΡ‚Π°Π²Π»ΡΡ‚ΡŒ 1 ΠΌΠ΅Ρ‚Ρ€ Π² час.Π­Ρ‚ΠΎ ΠΌΠ΅Π΄Π»Π΅Π½Π½ΠΎ!

Π’ΠΎΠ³Π΄Π° ΠΌΠΎΠΆΠ½ΠΎ ΡΠΏΡ€ΠΎΡΠΈΡ‚ΡŒ: ΠΊΠ°ΠΊ ΠΌΠΎΠΆΠ΅Ρ‚ Π±Ρ‹Ρ‚ΡŒ Ρ‚ΠΎΠΊ порядка 1 ΠΈΠ»ΠΈ 2 Π°ΠΌΠΏΠ΅Ρ€ Π² Ρ†Π΅ΠΏΠΈ, Ссли ΡΠΊΠΎΡ€ΠΎΡΡ‚ΡŒ Π΄Ρ€Π΅ΠΉΡ„Π° составляСт всСго ΠΎΠΊΠΎΠ»ΠΎ 1 ΠΌΠ΅Ρ‚Ρ€Π° Π² час? ΠžΡ‚Π²Π΅Ρ‚ Ρ‚Π°ΠΊΠΎΠ²: сущСствуСт мноТСство носитСлСй заряда, двиТущихся ΠΎΠ΄Π½ΠΎΠ²Ρ€Π΅ΠΌΠ΅Π½Π½ΠΎ ΠΏΠΎ всСй Π΄Π»ΠΈΠ½Π΅ Ρ†Π΅ΠΏΠΈ. Π’ΠΎΠΊ — это ΡΠΊΠΎΡ€ΠΎΡΡ‚ΡŒ, с ΠΊΠΎΡ‚ΠΎΡ€ΠΎΠΉ заряд пСрСсСкаСт Ρ‚ΠΎΡ‡ΠΊΡƒ Π² Ρ†Π΅ΠΏΠΈ. Π‘ΠΈΠ»ΡŒΠ½Ρ‹ΠΉ Ρ‚ΠΎΠΊ являСтся Ρ€Π΅Π·ΡƒΠ»ΡŒΡ‚Π°Ρ‚ΠΎΠΌ Π½Π΅ΡΠΊΠΎΠ»ΡŒΠΊΠΈΡ… ΠΊΡƒΠ»ΠΎΠ½ΠΎΠ² заряда, ΠΏΠ΅Ρ€Π΅ΡΠ΅ΠΊΠ°ΡŽΡ‰ΠΈΡ… ΠΏΠΎΠΏΠ΅Ρ€Π΅Ρ‡Π½ΠΎΠ΅ сСчСниС ΠΏΡ€ΠΎΠ²ΠΎΠ΄Π° Π² Ρ†Π΅ΠΏΠΈ. Если носитСли заряда ΠΏΠ»ΠΎΡ‚Π½ΠΎ ΡƒΠΏΠ°ΠΊΠΎΠ²Π°Π½Ρ‹ Π² ΠΏΡ€ΠΎΠ²ΠΎΠ΄, Ρ‚ΠΎΠ³Π΄Π° Π½Π΅ ΠΎΠ±ΡΠ·Π°Ρ‚Π΅Π»ΡŒΠ½ΠΎ Π΄ΠΎΠ»ΠΆΠ½Π° Π±Ρ‹Ρ‚ΡŒ высокая ΡΠΊΠΎΡ€ΠΎΡΡ‚ΡŒ, Ρ‡Ρ‚ΠΎΠ±Ρ‹ ΠΈΠΌΠ΅Ρ‚ΡŒ большой Ρ‚ΠΎΠΊ.Π’ΠΎ Π΅ΡΡ‚ΡŒ носитСлям заряда Π½Π΅ Π½ΡƒΠΆΠ½ΠΎ ΠΏΡ€Π΅ΠΎΠ΄ΠΎΠ»Π΅Π²Π°Ρ‚ΡŒ большоС расстояниС Π·Π° сСкунду, ΠΈΡ… просто Π΄ΠΎΠ»ΠΆΠ½ΠΎ Π±Ρ‹Ρ‚ΡŒ ΠΌΠ½ΠΎΠ³ΠΎ, проходящих Ρ‡Π΅Ρ€Π΅Π· ΠΏΠΎΠΏΠ΅Ρ€Π΅Ρ‡Π½ΠΎΠ΅ сСчСниС. Π’ΠΎΠΊ Π½Π΅ ΠΈΠΌΠ΅Π΅Ρ‚ ΠΎΡ‚Π½ΠΎΡˆΠ΅Π½ΠΈΡ ΠΊ Ρ‚ΠΎΠΌΡƒ, насколько Π΄Π°Π»Π΅ΠΊΠΎ заряды ΠΏΠ΅Ρ€Π΅ΠΌΠ΅Ρ‰Π°ΡŽΡ‚ΡΡ Π·Π° сСкунду, Π° скорСС ΠΊ Ρ‚ΠΎΠΌΡƒ, сколько зарядов ΠΏΡ€ΠΎΡ…ΠΎΠ΄ΠΈΡ‚ Ρ‡Π΅Ρ€Π΅Π· ΠΏΠΎΠΏΠ΅Ρ€Π΅Ρ‡Π½ΠΎΠ΅ сСчСниС ΠΏΡ€ΠΎΠ²ΠΎΠ΄Π° Π² Ρ†Π΅ΠΏΠΈ.

Π§Ρ‚ΠΎΠ±Ρ‹ ΠΏΡ€ΠΎΠΈΠ»Π»ΡŽΡΡ‚Ρ€ΠΈΡ€ΠΎΠ²Π°Ρ‚ΡŒ, насколько ΠΏΠ»ΠΎΡ‚Π½ΠΎ ΡƒΠΏΠ°ΠΊΠΎΠ²Π°Π½Ρ‹ носитСли заряда, ΠΌΡ‹ рассмотрим Ρ‚ΠΈΠΏΠΈΡ‡Π½Ρ‹ΠΉ ΠΏΡ€ΠΎΠ²ΠΎΠ΄, ΠΈΡΠΏΠΎΠ»ΡŒΠ·ΡƒΠ΅ΠΌΡ‹ΠΉ Π² цСпях домашнСго освСщСния — ΠΌΠ΅Π΄Π½Ρ‹ΠΉ ΠΏΡ€ΠΎΠ²ΠΎΠ΄ 14-Π³ΠΎ ΠΊΠ°Π»ΠΈΠ±Ρ€Π°. Π’ срСзС этого ΠΏΡ€ΠΎΠ²ΠΎΠ΄Π° Π΄Π»ΠΈΠ½ΠΎΠΉ 0,01 см (ΠΎΡ‡Π΅Π½ΡŒ Ρ‚ΠΎΠ½ΠΊΠΎΠΌ) ΠΈΡ… Π±ΡƒΠ΄Π΅Ρ‚ Ρ†Π΅Π»Ρ‹Ρ… 3.51 x 10 20 Π°Ρ‚ΠΎΠΌΠΎΠ² ΠΌΠ΅Π΄ΠΈ. ΠšΠ°ΠΆΠ΄Ρ‹ΠΉ Π°Ρ‚ΠΎΠΌ ΠΌΠ΅Π΄ΠΈ ΠΈΠΌΠ΅Π΅Ρ‚ 29 элСктронов; маловСроятно, Ρ‡Ρ‚ΠΎ Π΄Π°ΠΆΠ΅ 11 Π²Π°Π»Π΅Π½Ρ‚Π½Ρ‹Ρ… элСктронов ΠΎΠ΄Π½ΠΎΠ²Ρ€Π΅ΠΌΠ΅Π½Π½ΠΎ Π±ΡƒΠ΄ΡƒΡ‚ Π΄Π²ΠΈΠ³Π°Ρ‚ΡŒΡΡ ΠΊΠ°ΠΊ носитСли заряда. Если ΠΌΡ‹ ΠΏΡ€Π΅Π΄ΠΏΠΎΠ»ΠΎΠΆΠΈΠΌ, Ρ‡Ρ‚ΠΎ ΠΊΠ°ΠΆΠ΄Ρ‹ΠΉ Π°Ρ‚ΠΎΠΌ ΠΌΠ΅Π΄ΠΈ вносит Ρ‚ΠΎΠ»ΡŒΠΊΠΎ ΠΎΠ΄ΠΈΠ½ элСктрон, Ρ‚ΠΎ Π½Π° Ρ‚ΠΎΠ½ΠΊΠΎΠΌ 0,01-сантимСтровом ΠΏΡ€ΠΎΠ²ΠΎΠ΄Π΅ Π±ΡƒΠ΄Π΅Ρ‚ Ρ†Π΅Π»Ρ‹Ρ… 56 ΠΊΡƒΠ»ΠΎΠ½ΠΎΠ² заряда. ΠŸΡ€ΠΈ Ρ‚Π°ΠΊΠΎΠΌ большом количСствС ΠΏΠΎΠ΄Π²ΠΈΠΆΠ½ΠΎΠ³ΠΎ заряда Π² Ρ‚Π°ΠΊΠΎΠΌ малСньком пространствС малая ΡΠΊΠΎΡ€ΠΎΡΡ‚ΡŒ Π΄Ρ€Π΅ΠΉΡ„Π° ΠΌΠΎΠΆΠ΅Ρ‚ привСсти ΠΊ ΠΎΡ‡Π΅Π½ΡŒ Π±ΠΎΠ»ΡŒΡˆΠΎΠΌΡƒ Ρ‚ΠΎΠΊΡƒ.

Π§Ρ‚ΠΎΠ±Ρ‹ Π΅Ρ‰Π΅ большС ΠΏΡ€ΠΎΠΈΠ»Π»ΡŽΡΡ‚Ρ€ΠΈΡ€ΠΎΠ²Π°Ρ‚ΡŒ это Ρ€Π°Π·Π»ΠΈΡ‡ΠΈΠ΅ ΠΌΠ΅ΠΆΠ΄Ρƒ ΡΠΊΠΎΡ€ΠΎΡΡ‚ΡŒΡŽ заноса ΠΈ Ρ‚Π΅Ρ‡Π΅Π½ΠΈΠ΅ΠΌ, рассмотрим аналогию с Π³ΠΎΠ½ΠΊΠ°ΠΌΠΈ.ΠŸΡ€Π΅Π΄ΠΏΠΎΠ»ΠΎΠΆΠΈΠΌ, Ρ‡Ρ‚ΠΎ Π±Ρ‹Π»Π° ΠΎΡ‡Π΅Π½ΡŒ большая Π³ΠΎΠ½ΠΊΠ° Ρ‡Π΅Ρ€Π΅ΠΏΠ°Ρ… с ΠΌΠΈΠ»Π»ΠΈΠΎΠ½Π°ΠΌΠΈ ΠΈ ΠΌΠΈΠ»Π»ΠΈΠΎΠ½Π°ΠΌΠΈ Ρ‡Π΅Ρ€Π΅ΠΏΠ°Ρ… Π½Π° ΠΎΡ‡Π΅Π½ΡŒ ΡˆΠΈΡ€ΠΎΠΊΠΎΠΉ Π³ΠΎΠ½ΠΎΡ‡Π½ΠΎΠΉ трассС. Π§Π΅Ρ€Π΅ΠΏΠ°Ρ…ΠΈ Π½Π΅ ΠΎΡ‡Π΅Π½ΡŒ быстро Π΄Π²ΠΈΠ³Π°ΡŽΡ‚ΡΡ — Ρƒ Π½ΠΈΡ… ΠΎΡ‡Π΅Π½ΡŒ низкая ΡΠΊΠΎΡ€ΠΎΡΡ‚ΡŒ Π΄Ρ€Π΅ΠΉΡ„ . ΠŸΡ€Π΅Π΄ΠΏΠΎΠ»ΠΎΠΆΠΈΠΌ, Ρ‡Ρ‚ΠΎ Π³ΠΎΠ½ΠΊΠ° Π±Ρ‹Π»Π° довольно ΠΊΠΎΡ€ΠΎΡ‚ΠΊΠΎΠΉ — скаТСм, Π΄Π»ΠΈΠ½ΠΎΠΉ 1 ΠΌΠ΅Ρ‚Ρ€ — ΠΈ Ρ‡Ρ‚ΠΎ Π·Π½Π°Ρ‡ΠΈΡ‚Π΅Π»ΡŒΠ½Ρ‹ΠΉ ΠΏΡ€ΠΎΡ†Π΅Π½Ρ‚ Ρ‡Π΅Ρ€Π΅ΠΏΠ°Ρ… достиг Ρ„ΠΈΠ½ΠΈΡˆΠ½ΠΎΠΉ Ρ‡Π΅Ρ€Ρ‚Ρ‹ Π² ΠΎΠ΄Π½ΠΎ ΠΈ Ρ‚ΠΎ ΠΆΠ΅ врСмя — Ρ‡Π΅Ρ€Π΅Π· 30 ΠΌΠΈΠ½ΡƒΡ‚ послС Π½Π°Ρ‡Π°Π»Π° Π³ΠΎΠ½ΠΊΠΈ. Π’ Ρ‚Π°ΠΊΠΎΠΌ случаС Ρ‚Π΅Ρ‡Π΅Π½ΠΈΠ΅ Π±ΡƒΠ΄Π΅Ρ‚ ΠΎΡ‡Π΅Π½ΡŒ большим — ΠΌΠΈΠ»Π»ΠΈΠΎΠ½Ρ‹ Ρ‡Π΅Ρ€Π΅ΠΏΠ°Ρ… ΠΏΠ΅Ρ€Π΅ΡΠ΅ΠΊΠ°ΡŽΡ‚ Ρ‚ΠΎΡ‡ΠΊΡƒ Π·Π° ΠΊΠΎΡ€ΠΎΡ‚ΠΊΠΈΠΉ ΠΏΡ€ΠΎΠΌΠ΅ΠΆΡƒΡ‚ΠΎΠΊ Π²Ρ€Π΅ΠΌΠ΅Π½ΠΈ.Π’ этой Π°Π½Π°Π»ΠΎΠ³ΠΈΠΈ ΡΠΊΠΎΡ€ΠΎΡΡ‚ΡŒ связана с Ρ‚Π΅ΠΌ, насколько Π΄Π°Π»Π΅ΠΊΠΎ Ρ‡Π΅Ρ€Π΅ΠΏΠ°Ρ…ΠΈ ΠΏΠ΅Ρ€Π΅ΠΌΠ΅Ρ‰Π°ΡŽΡ‚ΡΡ Π·Π° ΠΎΠΏΡ€Π΅Π΄Π΅Π»Π΅Π½Π½Ρ‹ΠΉ ΠΏΡ€ΠΎΠΌΠ΅ΠΆΡƒΡ‚ΠΎΠΊ Π²Ρ€Π΅ΠΌΠ΅Π½ΠΈ; Π° Ρ‚ΠΎΠΊ зависит ΠΎΡ‚ Ρ‚ΠΎΠ³ΠΎ, сколько Ρ‡Π΅Ρ€Π΅ΠΏΠ°Ρ… пСрСсСкли Ρ„ΠΈΠ½ΠΈΡˆΠ½ΡƒΡŽ Ρ‡Π΅Ρ€Ρ‚Ρƒ Π·Π° ΠΎΠΏΡ€Π΅Π΄Π΅Π»Π΅Π½Π½Ρ‹ΠΉ ΠΏΡ€ΠΎΠΌΠ΅ΠΆΡƒΡ‚ΠΎΠΊ Π²Ρ€Π΅ΠΌΠ΅Π½ΠΈ.

ΠŸΡ€ΠΈΡ€ΠΎΠ΄Π° ΠΏΠΎΡ‚ΠΎΠΊΠ° заряда

Как Ρ‚ΠΎΠ»ΡŒΠΊΠΎ Π±Ρ‹Π»ΠΎ установлСно, Ρ‡Ρ‚ΠΎ срСдняя ΡΠΊΠΎΡ€ΠΎΡΡ‚ΡŒ Π΄Ρ€Π΅ΠΉΡ„Π° элСктрона ΠΎΡ‡Π΅Π½ΡŒ ΠΈ ΠΎΡ‡Π΅Π½ΡŒ ΠΌΠ°Π»Π°, вскорС Π²ΠΎΠ·Π½ΠΈΠΊΠ°Π΅Ρ‚ вопрос: ΠΏΠΎΡ‡Π΅ΠΌΡƒ свСт Π² ΠΊΠΎΠΌΠ½Π°Ρ‚Π΅ ΠΈΠ»ΠΈ Π² Ρ„ΠΎΠ½Π°Ρ€ΠΈΠΊΠ΅ загораСтся сразу послС Π²ΠΊΠ»ΡŽΡ‡Π΅Π½ΠΈΡ ΠΏΠ΅Ρ€Π΅ΠΊΠ»ΡŽΡ‡Π°Ρ‚Π΅Π»Ρ? Π Π°Π·Π²Π΅ Π½Π΅ Π±ΡƒΠ΄Π΅Ρ‚ Π·Π°ΠΌΠ΅Ρ‚Π½ΠΎΠΉ Π·Π°Π΄Π΅Ρ€ΠΆΠΊΠΈ ΠΏΠΎ Π²Ρ€Π΅ΠΌΠ΅Π½ΠΈ ΠΏΠ΅Ρ€Π΅Π΄ Ρ‚Π΅ΠΌ, ΠΊΠ°ΠΊ Π½ΠΎΡΠΈΡ‚Π΅Π»ΡŒ заряда ΠΏΠ΅Ρ€Π΅ΠΉΠ΄Π΅Ρ‚ ΠΎΡ‚ ΠΏΠ΅Ρ€Π΅ΠΊΠ»ΡŽΡ‡Π°Ρ‚Π΅Π»Ρ ΠΊ Π½ΠΈΡ‚ΠΈ Π½Π°ΠΊΠ°Π»Π° Π»Π°ΠΌΠΏΠΎΡ‡ΠΊΠΈ? ΠžΡ‚Π²Π΅Ρ‚ — Π½Π΅Ρ‚! ΠΈ объяснСниС Ρ‚ΠΎΠ³ΠΎ, ΠΏΠΎΡ‡Π΅ΠΌΡƒ раскрываСт Π·Π½Π°Ρ‡ΠΈΡ‚Π΅Π»ΡŒΠ½ΡƒΡŽ ΠΈΠ½Ρ„ΠΎΡ€ΠΌΠ°Ρ†ΠΈΡŽ ΠΎ ΠΏΡ€ΠΈΡ€ΠΎΠ΄Π΅ ΠΏΠΎΡ‚ΠΎΠΊΠ° заряда Π² Ρ†Π΅ΠΏΠΈ.

Как ΡƒΠΏΠΎΠΌΠΈΠ½Π°Π»ΠΎΡΡŒ Π²Ρ‹ΡˆΠ΅, носитСлями заряда Π² ΠΏΡ€ΠΎΠ²ΠΎΠ΄Π°Ρ… элСктричСских Ρ†Π΅ΠΏΠ΅ΠΉ ΡΠ²Π»ΡΡŽΡ‚ΡΡ элСктроны. Π­Ρ‚ΠΈ элСктроны просто ΠΏΠΎΡΡ‚Π°Π²Π»ΡΡŽΡ‚ΡΡ Π°Ρ‚ΠΎΠΌΠ°ΠΌΠΈ ΠΌΠ΅Π΄ΠΈ (ΠΈΠ»ΠΈ любого Π΄Ρ€ΡƒΠ³ΠΎΠ³ΠΎ ΠΌΠ°Ρ‚Π΅Ρ€ΠΈΠ°Π»Π°, ΠΈΠ· ΠΊΠΎΡ‚ΠΎΡ€ΠΎΠ³ΠΎ сдСлана ΠΏΡ€ΠΎΠ²ΠΎΠ»ΠΎΠΊΠ°) Π²Π½ΡƒΡ‚Ρ€ΠΈ мСталличСской ΠΏΡ€ΠΎΠ²ΠΎΠ»ΠΎΠΊΠΈ. Как Ρ‚ΠΎΠ»ΡŒΠΊΠΎ ΠΏΠ΅Ρ€Π΅ΠΊΠ»ΡŽΡ‡Π°Ρ‚Π΅Π»ΡŒ поворачиваСтся Π² ΠΏΠΎΠ»ΠΎΠΆΠ΅Π½ΠΈΠ΅ Π½Π° , Ρ†Π΅ΠΏΡŒ замыкаСтся, ΠΈ Π½Π° Π΄Π²ΡƒΡ… ΠΊΠΎΠ½Ρ†Π°Ρ… внСшнСй Ρ†Π΅ΠΏΠΈ устанавливаСтся Ρ€Π°Π·Π½ΠΎΡΡ‚ΡŒ элСктричСских ΠΏΠΎΡ‚Π΅Π½Ρ†ΠΈΠ°Π»ΠΎΠ². Π‘ΠΈΠ³Π½Π°Π» элСктричСского поля распространяСтся ΠΏΠΎΡ‡Ρ‚ΠΈ со ΡΠΊΠΎΡ€ΠΎΡΡ‚ΡŒΡŽ свСта ΠΊΠΎ всСм ΠΌΠΎΠ±ΠΈΠ»ΡŒΠ½Ρ‹ΠΌ элСктронам Π² Ρ†Π΅ΠΏΠΈ, приказывая ΠΈΠΌ Π½Π°Ρ‡Π°Ρ‚ΡŒ ΠΌΠ°Ρ€Ρˆ ΠΈ ΠΌΠ°Ρ€Ρˆ .По ΠΏΠΎΠ»ΡƒΡ‡Π΅Π½ΠΈΠΈ сигнала элСктроны Π½Π°Ρ‡ΠΈΠ½Π°ΡŽΡ‚ Π΄Π²ΠΈΠ³Π°Ρ‚ΡŒΡΡ ΠΏΠΎ Π·ΠΈΠ³Π·Π°Π³ΠΎΠΎΠ±Ρ€Π°Π·Π½ΠΎΠΉ Ρ‚Ρ€Π°Π΅ΠΊΡ‚ΠΎΡ€ΠΈΠΈ Π² ΠΎΠ±Ρ‹Ρ‡Π½ΠΎΠΌ Π½Π°ΠΏΡ€Π°Π²Π»Π΅Π½ΠΈΠΈ. Π’Π°ΠΊΠΈΠΌ ΠΎΠ±Ρ€Π°Π·ΠΎΠΌ, Ρ‰Π΅Π»Ρ‡ΠΎΠΊ ΠΏΠ΅Ρ€Π΅ΠΊΠ»ΡŽΡ‡Π°Ρ‚Π΅Π»Ρ Π²Ρ‹Π·Ρ‹Π²Π°Π΅Ρ‚ Π½Π΅ΠΌΠ΅Π΄Π»Π΅Π½Π½ΡƒΡŽ Ρ€Π΅Π°ΠΊΡ†ΠΈΡŽ Π²ΠΎ всСх частях схСмы, заставляя носитСли заряда ΠΏΠΎΠ²ΡΡŽΠ΄Ρƒ Π΄Π²ΠΈΠ³Π°Ρ‚ΡŒΡΡ Π² ΠΎΠ΄Π½ΠΎΠΌ ΠΈ Ρ‚ΠΎΠΌ ΠΆΠ΅ Π½Π°ΠΏΡ€Π°Π²Π»Π΅Π½ΠΈΠΈ. Π’ Ρ‚ΠΎ врСмя ΠΊΠ°ΠΊ фактичСскоС Π΄Π²ΠΈΠΆΠ΅Π½ΠΈΠ΅ носитСлСй заряда происходит с ΠΌΠ΅Π΄Π»Π΅Π½Π½ΠΎΠΉ ΡΠΊΠΎΡ€ΠΎΡΡ‚ΡŒΡŽ, сигнал, ΠΊΠΎΡ‚ΠΎΡ€Ρ‹ΠΉ ΠΈΠ½Ρ„ΠΎΡ€ΠΌΠΈΡ€ΡƒΠ΅Ρ‚ ΠΎ Π½Π°Ρ‡Π°Π»Π΅ двиТСния, двиТСтся со ΡΠΊΠΎΡ€ΠΎΡΡ‚ΡŒΡŽ, ΡΠΎΡΡ‚Π°Π²Π»ΡΡŽΡ‰Π΅ΠΉ Ρ‡Π°ΡΡ‚ΡŒ скорости свСта.

Π­Π»Π΅ΠΊΡ‚Ρ€ΠΎΠ½Ρ‹, ΠΊΠΎΡ‚ΠΎΡ€Ρ‹Π΅ Π·Π°ΠΆΠΈΠ³Π°ΡŽΡ‚ Π»Π°ΠΌΠΏΠΎΡ‡ΠΊΡƒ Π² Ρ„ΠΎΠ½Π°Ρ€ΠΈΠΊΠ΅, Π½Π΅ Π΄ΠΎΠ»ΠΆΠ½Ρ‹ сначала ΠΏΡ€ΠΎΠΉΡ‚ΠΈ ΠΎΡ‚ ΠΏΠ΅Ρ€Π΅ΠΊΠ»ΡŽΡ‡Π°Ρ‚Π΅Π»Ρ Ρ‡Π΅Ρ€Π΅Π· 10 см ΠΏΡ€ΠΎΠ²ΠΎΠ΄Π° ΠΊ Π½ΠΈΡ‚ΠΈ Π½Π°ΠΊΠ°Π»Π°.Π‘ΠΊΠΎΡ€Π΅Π΅ элСктроны, ΠΊΠΎΡ‚ΠΎΡ€Ρ‹Π΅ Π·Π°ΠΆΠΈΠ³Π°ΡŽΡ‚ Π»Π°ΠΌΠΏΠΎΡ‡ΠΊΡƒ сразу послС Ρ‚ΠΎΠ³ΠΎ, ΠΊΠ°ΠΊ ΠΏΠ΅Ρ€Π΅ΠΊΠ»ΡŽΡ‡Π°Ρ‚Π΅Π»ΡŒ ΠΏΠΎΠ²Π΅Ρ€Π½ΡƒΡ‚ Π½Π° Π½Π° , ΡΠ²Π»ΡΡŽΡ‚ΡΡ элСктронами, ΠΊΠΎΡ‚ΠΎΡ€Ρ‹Π΅ ΠΏΡ€ΠΈΡΡƒΡ‚ΡΡ‚Π²ΡƒΡŽΡ‚ Π² самой Π½ΠΈΡ‚ΠΈ. Когда ΠΏΠ΅Ρ€Π΅ΠΊΠ»ΡŽΡ‡Π°Ρ‚Π΅Π»ΡŒ ΠΏΠΎΠ²Π΅Ρ€Π½ΡƒΡ‚, всС ΠΏΠΎΠ΄Π²ΠΈΠΆΠ½Ρ‹Π΅ элСктроны ΠΏΠΎΠ²ΡΡŽΠ΄Ρƒ Π½Π°Ρ‡ΠΈΠ½Π°ΡŽΡ‚ Π΄Π²ΠΈΠΆΠ΅Π½ΠΈΠ΅; ΠΈ ΠΈΠΌΠ΅Π½Π½ΠΎ ΠΏΠΎΠ΄Π²ΠΈΠΆΠ½Ρ‹Π΅ элСктроны, ΠΏΡ€ΠΈΡΡƒΡ‚ΡΡ‚Π²ΡƒΡŽΡ‰ΠΈΠ΅ Π² Π½ΠΈΡ‚ΠΈ Π½Π°ΠΊΠ°Π»Π°, нСпосрСдствСнно отвСтствСнны Π·Π° Π·Π°ΠΆΠΈΠ³Π°Π½ΠΈΠ΅ Π΅Π΅ ΠΊΠΎΠ»Π±Ρ‹. Когда эти элСктроны ΠΏΠΎΠΊΠΈΠ΄Π°ΡŽΡ‚ Π½ΠΈΡ‚ΡŒ Π½Π°ΠΊΠ°Π»Π°, Π² Π½Π΅Π΅ входят Π½ΠΎΠ²Ρ‹Π΅ элСктроны, ΠΊΠΎΡ‚ΠΎΡ€Ρ‹Π΅ отвСтствСнны Π·Π° Π·Π°ΠΆΠΈΠ³Π°Π½ΠΈΠ΅ Π»Π°ΠΌΠΏΡ‹. Π­Π»Π΅ΠΊΡ‚Ρ€ΠΎΠ½Ρ‹ двиТутся вмСстС, ΠΊΠ°ΠΊ Π²ΠΎΠ΄Π° Π² Ρ‚Ρ€ΡƒΠ±Π°Ρ… Π΄ΠΎΠΌΠ°.Когда ΠΊΡ€Π°Π½ поворачиваСтся с Π½Π° , Π²ΠΎΠ΄Π° Π² ΠΊΡ€Π°Π½Π΅ Π²Ρ‹Ρ…ΠΎΠ΄ΠΈΡ‚ ΠΈΠ· ΠΊΡ€Π°Π½Π°. НС Π½ΡƒΠΆΠ½ΠΎ Π΄ΠΎΠ»Π³ΠΎ ΠΆΠ΄Π°Ρ‚ΡŒ, ΠΏΠΎΠΊΠ° Π²ΠΎΠ΄Π° ΠΈΠ· Ρ‚ΠΎΡ‡ΠΊΠΈ Π²Ρ…ΠΎΠ΄Π° Π² ваш Π΄ΠΎΠΌ ΠΏΡ€ΠΎΠΉΠ΄Π΅Ρ‚ ΠΏΠΎ Ρ‚Ρ€ΡƒΠ±Π°ΠΌ ΠΊ ΠΊΡ€Π°Π½Ρƒ. Π’Ρ€ΡƒΠ±Ρ‹ ΡƒΠΆΠ΅ Π·Π°ΠΏΠΎΠ»Π½Π΅Π½Ρ‹ Π²ΠΎΠ΄ΠΎΠΉ, ΠΈ Π²ΠΎΠ΄Π° Π²ΠΎ всСм Π²ΠΎΠ΄Π½ΠΎΠΌ ΠΊΠΎΠ½Ρ‚ΡƒΡ€Π΅ ΠΎΠ΄Π½ΠΎΠ²Ρ€Π΅ΠΌΠ΅Π½Π½ΠΎ приводится Π² Π΄Π²ΠΈΠΆΠ΅Π½ΠΈΠ΅.

РазвиваСмая здСсь ΠΊΠ°Ρ€Ρ‚ΠΈΠ½Π° ΠΏΠΎΡ‚ΠΎΠΊΠ° заряда прСдставляСт собой ΠΊΠ°Ρ€Ρ‚ΠΈΠ½Ρƒ, Π² ΠΊΠΎΡ‚ΠΎΡ€ΠΎΠΉ носитСли заряда ΠΏΠΎΠ΄ΠΎΠ±Π½Ρ‹ солдатам, ΠΈΠ΄ΡƒΡ‰ΠΈΠΌ вмСстС, ΠΏΠΎΠ²ΡΡŽΠ΄Ρƒ с ΠΎΠ΄ΠΈΠ½Π°ΠΊΠΎΠ²ΠΎΠΉ ΡΠΊΠΎΡ€ΠΎΡΡ‚ΡŒΡŽ.Π˜Ρ… Π΄Π²ΠΈΠΆΠ΅Π½ΠΈΠ΅ начинаСтся Π½Π΅ΠΌΠ΅Π΄Π»Π΅Π½Π½ΠΎ Π² ΠΎΡ‚Π²Π΅Ρ‚ Π½Π° установлСниС элСктричСского ΠΏΠΎΡ‚Π΅Π½Ρ†ΠΈΠ°Π»Π° Π½Π° Π΄Π²ΡƒΡ… ΠΊΠΎΠ½Ρ†Π°Ρ… Ρ†Π΅ΠΏΠΈ. Π’ элСктричСской Ρ†Π΅ΠΏΠΈ Π½Π΅Ρ‚ мСста, Π³Π΄Π΅ носитСли заряда Ρ€Π°ΡΡ…ΠΎΠ΄ΡƒΡŽΡ‚ΡΡ ΠΈΠ»ΠΈ Ρ€Π°ΡΡ…ΠΎΠ΄ΡƒΡŽΡ‚ΡΡ. Π₯отя энСргия, ΠΊΠΎΡ‚ΠΎΡ€ΠΎΠΉ ΠΎΠ±Π»Π°Π΄Π°Π΅Ρ‚ заряд, ΠΌΠΎΠΆΠ΅Ρ‚ Π±Ρ‹Ρ‚ΡŒ израсходована (ΠΈΠ»ΠΈ Π»ΡƒΡ‡ΡˆΠ΅ ΡΠΊΠ°Π·Π°Ρ‚ΡŒ, Ρ‡Ρ‚ΠΎ элСктричСская энСргия прСобразуСтся Π² Π΄Ρ€ΡƒΠ³ΠΈΠ΅ Ρ„ΠΎΡ€ΠΌΡ‹ энСргии), сами носитСли заряда Π½Π΅ Ρ€Π°ΡΠΏΠ°Π΄Π°ΡŽΡ‚ΡΡ, Π½Π΅ ΠΈΡΡ‡Π΅Π·Π°ΡŽΡ‚ ΠΈΠ»ΠΈ ΠΈΠ½Ρ‹ΠΌ ΠΎΠ±Ρ€Π°Π·ΠΎΠΌ Π½Π΅ ΡƒΠ΄Π°Π»ΡΡŽΡ‚ΡΡ ΠΈΠ· схСма. И Π½Π΅Ρ‚ мСста Π² Ρ†Π΅ΠΏΠΈ, Π³Π΄Π΅ Π±Ρ‹ носитСли заряда Π½Π°Ρ‡Π°Π»ΠΈ ΡΠΊΠ°ΠΏΠ»ΠΈΠ²Π°Ρ‚ΡŒΡΡ ΠΈΠ»ΠΈ Π½Π°ΠΊΠ°ΠΏΠ»ΠΈΠ²Π°Ρ‚ΡŒΡΡ.Π‘ΠΊΠΎΡ€ΠΎΡΡ‚ΡŒ, с ΠΊΠΎΡ‚ΠΎΡ€ΠΎΠΉ заряд Π²Ρ…ΠΎΠ΄ΠΈΡ‚ Π²ΠΎ внСшнюю Ρ†Π΅ΠΏΡŒ Π½Π° ΠΎΠ΄Π½ΠΎΠΌ ΠΊΠΎΠ½Ρ†Π΅, такая ΠΆΠ΅, ΠΊΠ°ΠΊ ΡΠΊΠΎΡ€ΠΎΡΡ‚ΡŒ, с ΠΊΠΎΡ‚ΠΎΡ€ΠΎΠΉ заряд Π²Ρ‹Ρ…ΠΎΠ΄ΠΈΡ‚ ΠΈΠ· внСшнСй Ρ†Π΅ΠΏΠΈ Π½Π° Π΄Ρ€ΡƒΠ³ΠΎΠΌ ΠΊΠΎΠ½Ρ†Π΅. Π’ΠΎΠΊ — ΡΠΊΠΎΡ€ΠΎΡΡ‚ΡŒ ΠΏΠΎΡ‚ΠΎΠΊΠ° заряда — Π²Π΅Π·Π΄Π΅ ΠΎΠ΄ΠΈΠ½Π°ΠΊΠΎΠ²Π°. ΠŸΠΎΡ‚ΠΎΠΊ заряда ΠΏΠΎΠ΄ΠΎΠ±Π΅Π½ двиТСнию солдат, ΠΈΠ΄ΡƒΡ‰ΠΈΡ… вмСстС, ΠΏΠΎΠ²ΡΡŽΠ΄Ρƒ с ΠΎΠ΄ΠΈΠ½Π°ΠΊΠΎΠ²ΠΎΠΉ ΡΠΊΠΎΡ€ΠΎΡΡ‚ΡŒΡŽ.

ΠŸΡ€ΠΎΠ²Π΅Ρ€ΡŒΡ‚Π΅ своС ΠΏΠΎΠ½ΠΈΠΌΠ°Π½ΠΈΠ΅

1.Говорят, Ρ‡Ρ‚ΠΎ Ρ‚ΠΎΠΊ сущСствуСт всякий Ρ€Π°Π·, ΠΊΠΎΠ³Π΄Π° _____.

Π°. ΠΏΡ€ΠΎΠ²ΠΎΠ΄ заряТСн

Π³. батарСя присутствуСт

Π³. элСктричСскиС заряды нСсбалансированныС

Π³. элСктричСскиС заряды двиТутся ΠΏΠΎ ΠΏΠ΅Ρ‚Π»Π΅

2. Π£ Ρ‚ΠΎΠΊΠ° Π΅ΡΡ‚ΡŒ Π½Π°ΠΏΡ€Π°Π²Π»Π΅Π½ΠΈΠ΅. По соглашСнию Ρ‚ΠΎΠΊ ΠΈΠ΄Π΅Ρ‚ Π² Π½Π°ΠΏΡ€Π°Π²Π»Π΅Π½ΠΈΠΈ ___.

Π°. + заряды ΠΏΠ΅Ρ€Π΅ΠΌΠ΅Ρ‰Π°ΡŽΡ‚ΡΡ

Π³.- элСктроны двиТутся

Π³. + Π΄Π²ΠΈΠΆΠ΅Π½ΠΈΠ΅ элСктронов

3. Π‘ΠΊΠΎΡ€ΠΎΡΡ‚ΡŒ Π΄Ρ€Π΅ΠΉΡ„Π° ΠΏΠΎΠ΄Π²ΠΈΠΆΠ½Ρ‹Ρ… носитСлСй заряда Π² элСктричСских цСпях ____.

Π°. ΠΎΡ‡Π΅Π½ΡŒ быстро; мСньшС, Π½ΠΎ ΠΎΡ‡Π΅Π½ΡŒ Π±Π»ΠΈΠ·ΠΊΠΎ ΠΊ скорости свСта

Π³. быстрый; быстрСС, Ρ‡Π΅ΠΌ самая быстрая машина, Π½ΠΎ Π΄Π°Π»Π΅ΠΊΠΎ Π½Π΅ ΡΠΊΠΎΡ€ΠΎΡΡ‚ΡŒ свСта

Π³. ΠΌΠ΅Π΄Π»Π΅Π½Π½Ρ‹ΠΉ; ΠΌΠ΅Π΄Π»Π΅Π½Π½Π΅Π΅ Майкла ДТСксона ΠΏΡ€ΠΎΠ±Π΅Π³Π°Π΅Ρ‚ 220-ΠΌΠ΅Ρ‚Ρ€ΠΎΠ²ΡƒΡŽ

Π³.ΠΎΡ‡Π΅Π½ΡŒ ΠΌΠ΅Π΄Π»Π΅Π½Π½ΠΎ; ΠΌΠ΅Π΄Π»Π΅Π½Π½Π΅Π΅ ΡƒΠ»ΠΈΡ‚ΠΊΠΈ

4. Если Π±Ρ‹ ΡΠ»Π΅ΠΊΡ‚Ρ€ΠΈΡ‡Π΅ΡΠΊΡƒΡŽ Ρ†Π΅ΠΏΡŒ ΠΌΠΎΠΆΠ½ΠΎ Π±Ρ‹Π»ΠΎ ΡΡ€Π°Π²Π½ΠΈΡ‚ΡŒ с водяной Ρ†Π΅ΠΏΡŒΡŽ Π² Π°ΠΊΠ²Π°ΠΏΠ°Ρ€ΠΊΠ΅, Ρ‚ΠΎ Ρ‚ΠΎΠΊ Π±Ρ‹Π» Π±Ρ‹ Π°Π½Π°Π»ΠΎΠ³ΠΈΡ‡Π΅Π½ ____.

Π’Ρ‹Π±ΠΎΡ€:

A. Π΄Π°Π²Π»Π΅Π½ΠΈΠ΅ Π²ΠΎΠ΄Ρ‹

Π‘. Π³Π°Π»Π»ΠΎΠ½ΠΎΠ² Π²ΠΎΠ΄Ρ‹, ΡΡ‚Π΅ΠΊΠ°ΡŽΡ‰Π΅ΠΉ ΠΏΠΎ Π³ΠΎΡ€ΠΊΠ΅ Π² ΠΌΠΈΠ½ΡƒΡ‚Ρƒ

Π‘.Π²ΠΎΠ΄Π°

D. ниТняя Ρ‡Π°ΡΡ‚ΡŒ ΠΏΠΎΠ»Π·ΡƒΠ½Π°

E. водяной насос

F. Π²Π΅Ρ€Ρ… Π³ΠΎΡ€ΠΊΠΈ

5. На схСмС справа ΠΈΠ·ΠΎΠ±Ρ€Π°ΠΆΠ΅Π½ токопроводящий ΠΏΡ€ΠΎΠ²ΠΎΠ΄. Π”Π²Π΅ ΠΏΠ»ΠΎΡ‰Π°Π΄ΠΈ ΠΏΠΎΠΏΠ΅Ρ€Π΅Ρ‡Π½ΠΎΠ³ΠΎ сСчСния располоТСны Π½Π° расстоянии 50 см Π΄Ρ€ΡƒΠ³ ΠΎΡ‚ Π΄Ρ€ΡƒΠ³Π°. ΠšΠ°ΠΆΠ΄Ρ‹Π΅ 2,0 сСкунды Ρ‡Π΅Ρ€Π΅Π· ΠΊΠ°ΠΆΠ΄ΡƒΡŽ ΠΈΠ· этих областСй ΠΏΡ€ΠΎΡ…ΠΎΠ΄ΠΈΡ‚ заряд 10 Β° C.Π‘ΠΈΠ»Π° Ρ‚ΠΎΠΊΠ° Π² этом ΠΏΡ€ΠΎΠ²ΠΎΠ΄Π΅ ____ А.

Π°. 0,10

Π³. 0,25

Π³. 0,50

Π³. 1.0

e. 5,0

Ρ„. 20

Π³. 10

Ρ‡.40

я. Π½ΠΈ ΠΎΠ΄ΠΈΠ½ ΠΈΠ· этих

6. Π˜ΡΠΏΠΎΠ»ΡŒΠ·ΡƒΠΉΡ‚Π΅ Π΄ΠΈΠ°Π³Ρ€Π°ΠΌΠΌΡƒ справа, Ρ‡Ρ‚ΠΎΠ±Ρ‹ Π·Π°ΠΏΠΎΠ»Π½ΠΈΡ‚ΡŒ ΡΠ»Π΅Π΄ΡƒΡŽΡ‰ΠΈΠ΅ утвСрТдСния:

Π°. Π’ΠΎΠΊ Π² ΠΎΠ΄ΠΈΠ½ Π°ΠΌΠΏΠ΅Ρ€ — это ΠΏΠΎΡ‚ΠΎΠΊ заряда со ΡΠΊΠΎΡ€ΠΎΡΡ‚ΡŒΡŽ _______ ΠΊΡƒΠ»ΠΎΠ½ΠΎΠ² Π² сСкунду.

Π³. Когда заряд 8 Кл ΠΏΡ€ΠΎΡ…ΠΎΠ΄ΠΈΡ‚ Ρ‡Π΅Ρ€Π΅Π· Π»ΡŽΠ±ΡƒΡŽ Ρ‚ΠΎΡ‡ΠΊΡƒ Ρ†Π΅ΠΏΠΈ Π·Π° 2 сСкунды, Ρ‚ΠΎΠΊ составляСт ________ А.

Π³. Если Ρ‡Π΅Ρ€Π΅Π· Ρ‚ΠΎΡ‡ΠΊΡƒ A (Π΄ΠΈΠ°Π³Ρ€Π°ΠΌΠΌΠ° справа) Π·Π° 10 сСкунд расход заряда составляСт 5 Β° C, Ρ‚ΠΎ Ρ‚ΠΎΠΊ Ρ€Π°Π²Π΅Π½ _________ A.

Π³. Если Ρ‚ΠΎΠΊ Π² Ρ‚ΠΎΡ‡ΠΊΠ΅ D Ρ€Π°Π²Π΅Π½ 2,0 А, Ρ‚ΠΎ _______ C заряда ΠΏΡ€ΠΎΡ…ΠΎΠ΄ΠΈΡ‚ Ρ‡Π΅Ρ€Π΅Π· Ρ‚ΠΎΡ‡ΠΊΡƒ D Π·Π° 10 сСкунд.

e. Если 12 Β° C заряда ΠΏΡ€ΠΎΠΉΠ΄Π΅Ρ‚ ΠΌΠΈΠΌΠΎ Ρ‚ΠΎΡ‡ΠΊΠΈ A Π·Π° 3 сСкунды, Ρ‚ΠΎ 8 C заряда ΠΏΡ€ΠΎΠΉΠ΄ΡƒΡ‚ ΠΌΠΈΠΌΠΎ Ρ‚ΠΎΡ‡ΠΊΠΈ E Π·Π° ________ сСкунд.

Ρ„. Π’Π΅Ρ€Π½ΠΎ Π»ΠΈ:

Π’ΠΎΠΊ Π² Ρ‚ΠΎΡ‡ΠΊΠ΅ E Π·Π½Π°Ρ‡ΠΈΡ‚Π΅Π»ΡŒΠ½ΠΎ мСньшС Ρ‚ΠΎΠΊΠ° Π² Ρ‚ΠΎΡ‡ΠΊΠ΅ A, ΠΏΠΎΡΠΊΠΎΠ»ΡŒΠΊΡƒ Π² Π»Π°ΠΌΠΏΠΎΡ‡ΠΊΠ°Ρ… расходуСтся заряд.

ЭлСктричСский Ρ‚ΠΎΠΊ — Как гСнСрируСтся элСктричСский Ρ‚ΠΎΠΊ | ΠžΠΏΡ€Π΅Π΄Π΅Π»Π΅Π½ΠΈΠ΅

Как ΠΏΡ€Π°Π²ΠΈΠ»ΠΎ, Π’Π΅ΠΊΡƒΡ‰ΠΈΠΉ ΠΎΠ·Π½Π°Ρ‡Π°Π΅Ρ‚ ΠΏΠΎΡ‚ΠΎΠΊ Ρ‡Π΅Π³ΠΎ-Ρ‚ΠΎ ΠΈΠ· ΠΎΠ΄Π½ΠΎΠ³ΠΎ мСста Π² Π΄Ρ€ΡƒΠ³ΠΎΠ΅. Для НапримСр, Π²ΠΎΠ΄Π° ΠΏΠ°Π΄Π°Π΅Ρ‚ с Ρ…ΠΎΠ»ΠΌΠ°, рСчная Π²ΠΎΠ΄Π° двиТСтся с ΠΈΠ· ΠΎΠ΄Π½ΠΎΠ³ΠΎ мСста Π² Π΄Ρ€ΡƒΠ³ΠΎΠ΅, ΠΈ окСанская Π²ΠΎΠ΄Π° двиТСтся ΠΈΠ· ΠΎΠ΄Π½ΠΎΠ³ΠΎ мСсто Π² Π΄Ρ€ΡƒΠ³ΠΎΠ΅ мСсто извСстны ΠΊΠ°ΠΊ Π²ΠΎΠ΄Π½Ρ‹Π΅ ΠΏΠΎΡ‚ΠΎΠΊΠΈ.Π’ Ρ€Π΅ΠΊΠ΅ ΠΈ ΠΎΠΊΠ΅Π°Π½, ΠΌΠΎΠ»Π΅ΠΊΡƒΠ»Ρ‹ Π²ΠΎΠ΄Ρ‹ двиТутся ΠΈΠ· ΠΎΠ΄Π½ΠΎΠ³ΠΎ мСста Π² Π΄Ρ€ΡƒΠ³ΠΎΠ΅. Π΄Ρ€ΡƒΠ³ΠΎΠ΅ мСсто Π±ΡƒΠ΄Π΅Ρ‚ ΠΏΡ€ΠΎΠ²ΠΎΠ΄ΠΈΡ‚ΡŒ Ρ‚ΠΎΠΊ.

Π’ Π° ΠΏΠΎΠ΄ΠΎΠ±Π½Ρ‹ΠΌ ΠΎΠ±Ρ€Π°Π·ΠΎΠΌ пСрСносчики элСктричСского заряда двиТутся ΠΈΠ· ΠΎΠ΄Π½ΠΎΠ³ΠΎ ΡƒΠΊΠ°Π·Ρ‹Π²Π°Π΅Ρ‚ Π½Π° Π΄Ρ€ΡƒΠ³ΡƒΡŽ Ρ‚ΠΎΡ‡ΠΊΡƒ Π² ΠΏΡ€ΠΎΠ²ΠΎΠ΄Π½ΠΈΠΊΠ΅ ΠΈΠ»ΠΈ ΠΏΠΎΠ»ΡƒΠΏΡ€ΠΎΠ²ΠΎΠ΄Π½ΠΈΠΊΠ΅. ΠΏΡ€ΠΎΠ²ΠΎΠ΄ΠΈΡ‚ элСктричСский Ρ‚ΠΎΠΊ.

ЭлСктричСский Π’Π΅ΠΊΡƒΡ‰ΠΈΠΉ ΠΎΠΏΡ€Π΅Π΄Π΅Π»Π΅Π½ΠΈΠ΅

Π³. ΠΏΠΎΡ‚ΠΎΠΊ носитСлСй элСктричСского заряда Π² ΠΏΡ€ΠΎΠ²ΠΎΠ΄Π½ΠΈΠΊΠ΅ ΠΈΠ»ΠΈ ΠΏΠΎΠ»ΡƒΠΏΡ€ΠΎΠ²ΠΎΠ΄Π½ΠΈΠΊΠ΅ называСтся элСктричСский Ρ‚ΠΎΠΊ.

Π’ ΠΏΡ€ΠΎΠ²ΠΎΠ΄Π½ΠΈΠΊΠΈ ΠΈΠ»ΠΈ ΠΏΠΎΠ»ΡƒΠΏΡ€ΠΎΠ²ΠΎΠ΄Π½ΠΈΠΊΠΎΠ², элСктричСский Ρ‚ΠΎΠΊ проводится ΠΊΡ€ΠΎΡˆΠ΅Ρ‡Π½Ρ‹ΠΌΠΈ частицы. Π­Ρ‚ΠΈ ΠΊΡ€ΠΎΡˆΠ΅Ρ‡Π½Ρ‹Π΅ частицы извСстны ΠΊΠ°ΠΊ элСктричСский заряд. ΠΏΠ΅Ρ€Π΅Π²ΠΎΠ·Ρ‡ΠΈΠΊΠΈ.

Π³. НоситСлями элСктричСского заряда ΠΌΠΎΠ³ΡƒΡ‚ Π±Ρ‹Ρ‚ΡŒ элСктроны, Π΄Ρ‹Ρ€ΠΊΠΈ, ΠΏΡ€ΠΎΡ‚ΠΎΠ½Ρ‹, ΠΈΠΎΠ½Ρ‹ ΠΈ Ρ‚. Π΄. Однако элСктричСский Ρ‚ΠΎΠΊ часто проводятся элСктронами ΠΈ Π΄Ρ‹Ρ€ΠΊΠ°ΠΌΠΈ.

Π’ ΠΏΡ€ΠΎΠ²ΠΎΠ΄Π½ΠΈΠΊΠΈ, отвСрстия Π½Π΅Π·Π½Π°Ρ‡ΠΈΡ‚Π΅Π»ΡŒΠ½Ρ‹.Π’Π°ΠΊ элСктроны проводят элСктричСский Ρ‚ΠΎΠΊ. Π’ ΠΏΠΎΠ»ΡƒΠΏΡ€ΠΎΠ²ΠΎΠ΄Π½ΠΈΠΊΠ°Ρ… ΠΏΡ€ΠΈΡΡƒΡ‚ΡΡ‚Π²ΡƒΡŽΡ‚ ΠΊΠ°ΠΊ элСктроны, Ρ‚Π°ΠΊ ΠΈ Π΄Ρ‹Ρ€ΠΊΠΈ. Π’Π°ΠΊ ΠΈ элСктроны, ΠΈ Π΄Ρ‹Ρ€ΠΊΠΈ проводят элСктричСский Ρ‚ΠΎΠΊ.

ЭлСктричСский Ρ‚ΠΎΠΊ — ваТная Π²Π΅Π»ΠΈΡ‡ΠΈΠ½Π° Π² элСктронных схСмах. Когда напряТСниС наносится ΠΏΠΎΠΏΠ΅Ρ€Π΅ΠΊ ΠΏΡ€ΠΎΠ²ΠΎΠ΄Π½ΠΈΠΊΠ° ΠΈΠ»ΠΈ ΠΏΠΎΠ»ΡƒΠΏΡ€ΠΎΠ²ΠΎΠ΄Π½ΠΈΠΊΠ°, элСктричСский Ρ‚ΠΎΠΊ Π½Π°Ρ‡ΠΈΠ½Π°Π΅Ρ‚ Ρ‚Π΅Ρ‡ΡŒ.ЭлСктричСский Ρ‚ΠΎΠΊ часто Π±Ρ‹Π²Π°Π΅Ρ‚ для простоты называСтся Β«Ρ‚Π΅ΠΊΡƒΡ‰ΠΈΠΉΒ».

ЭлСктричСский Π’Π΅ΠΊΡƒΡ‰ΠΈΠΉ символ

ЭлСктричСский Ρ‚ΠΎΠΊ прСдставлСн символом Ιͺ. Π’ символ Ιͺ Π±Ρ‹Π» использовался французским Ρ„ΠΈΠ·ΠΈΠΊΠΎΠΌ АндрС-ΠœΠ°Ρ€ΠΈ АмпСр. Π’ Π΅Π³ΠΎ ΠΈΠΌΠ΅Π½Π΅ΠΌ Π½Π°Π·Π²Π°Π½Π° Π΅Π΄ΠΈΠ½ΠΈΡ†Π° элСктричСского Ρ‚ΠΎΠΊΠ° (Π°ΠΌΠΏΠ΅Ρ€).

Π§Ρ‚ΠΎ Ρ‚Π°ΠΊΠΎΠ΅ элСктричСский заряд?

ЭлСктричСский заряд — это Ρ„ΡƒΠ½Π΄Π°ΠΌΠ΅Π½Ρ‚Π°Π»ΡŒΠ½ΠΎΠ΅ свойство Ρ‚Π°ΠΊΠΈΡ… частиц, ΠΊΠ°ΠΊ элСктроны ΠΈ ΠΏΡ€ΠΎΡ‚ΠΎΠ½Ρ‹. ЭлСктричСский заряд Π½Π΅ ΠΌΠΎΠΆΠ΅Ρ‚ Π±Ρ‹Ρ‚ΡŒ создан Π½ΠΈ ΡƒΠ½ΠΈΡ‡Ρ‚ΠΎΠΆΠ΅Π½. Π­Ρ‚ΠΎ ΠΎΠ·Π½Π°Ρ‡Π°Π΅Ρ‚, Ρ‡Ρ‚ΠΎ Ссли Π΅ΡΡ‚ΡŒ элСктрон ΠΈΠ»ΠΈ ΠΏΡ€ΠΎΡ‚ΠΎΠ½ Ρ‚ΠΎΠ³Π΄Π° Π΅ΡΡ‚ΡŒ заряд.

Π­Π»Π΅ΠΊΡ‚Ρ€ΠΎΠ½ΠΎΠ² ΠΈΠΌΠ΅ΡŽΡ‚ ΠΎΡ‚Ρ€ΠΈΡ†Π°Ρ‚Π΅Π»ΡŒΠ½Ρ‹ΠΉ заряд, Π° ΠΏΡ€ΠΎΡ‚ΠΎΠ½Ρ‹ — ΠΏΠΎΠ»ΠΎΠΆΠΈΡ‚Π΅Π»ΡŒΠ½Ρ‹ΠΉ.ΠŸΡ€ΠΎΡ‚ΠΎΠ½Ρ‹ Π½Π°ΠΌΠ½ΠΎΠ³ΠΎ тяТСлСС элСктронов. Однако ΠΎΠ±Π²ΠΈΠ½Π΅Π½ΠΈΠ΅ ΠΏΡ€ΠΎΡ‚ΠΎΠ½Π° Ρ€Π°Π²Π΅Π½ заряду элСктрона.

ср Π·Π½Π°ΠΉΡ‚Π΅, Ρ‡Ρ‚ΠΎ Ссли Π΄Π²Π° ΠΏΡ€ΠΎΡ‚ΠΈΠ²ΠΎΠΏΠΎΠ»ΠΎΠΆΠ½Ρ‹Ρ… заряда ΠΏΠΎΠΌΠ΅Ρ‰Π΅Π½Ρ‹ рядом с ΠΊΠ°ΠΆΠ΄Ρ‹ΠΌ Π΄Ρ€ΡƒΠ³ΠΈΠ΅ ΠΎΠ½ΠΈ ΠΏΡ€ΠΈΠ²Π»Π΅ΠΊΠ°ΡŽΡ‚ΡΡ. Π‘ Π΄Ρ€ΡƒΠ³ΠΎΠΉ стороны, Ссли Π΄Π²Π° ΠΎΠ΄ΠΈΠ½Π°ΠΊΠΎΠ²Ρ‹Ρ… ΠΈΠ»ΠΈ ΠΊΠ°ΠΊ заряды ΠΏΠΎΠΌΠ΅Ρ‰Π°ΡŽΡ‚ΡΡ Π±Π»ΠΈΠ·ΠΊΠΎ Π΄Ρ€ΡƒΠ³ ΠΊ Π΄Ρ€ΡƒΠ³Ρƒ, ΠΎΠ½ΠΈ ΠΎΡ‚Ρ‚Π°Π»ΠΊΠΈΠ²Π°ΡŽΡ‚ΡΡ.

Когда ΠΏΡ€ΠΎΡ‚ΠΎΠ½ помСщаСтся Π±Π»ΠΈΠΆΠ΅ ΠΊ элСктрону, ΠΎΠ½ΠΈ ΠΏΡ€ΠΈΡ‚ΡΠ³ΠΈΠ²Π°ΡŽΡ‚ΡΡ.Π‘ Π΄Ρ€ΡƒΠ³ΠΎΠΉ стороны, ΠΊΠΎΠ³Π΄Π° Π΄Π²Π° ΠΏΡ€ΠΎΡ‚ΠΎΠ½Π° ΠΈΠ»ΠΈ Π΄Π²Π° элСктрона Ρ€Π°Π·ΠΌΠ΅Ρ‰Π΅Π½Π½Ρ‹Π΅ Π±Π»ΠΈΠ·ΠΊΠΎ Π΄Ρ€ΡƒΠ³ ΠΊ Π΄Ρ€ΡƒΠ³Ρƒ, ΠΎΠ½ΠΈ ΠΎΡ‚Ρ‚Π°Π»ΠΊΠΈΠ²Π°ΡŽΡ‚ΡΡ.

ЭлСктричСский Π·Π°Ρ€ΡΠΆΠ°Ρ‚ΡŒ измСряСтся Π² ΠΊΡƒΠ»ΠΎΠ½Π°Ρ… (C). Один ΠΊΡƒΠ»ΠΎΠ½ — это количСство заряд пСрСносится Ρ‚ΠΎΠΊΠΎΠΌ Π² 1 Π°ΠΌΠΏΠ΅Ρ€ Π·Π° 1 сСкунду. Для НапримСр, Ссли 4 ΠΊΡƒΠ»ΠΎΠ½ΠΎΠ² (Кл) заряда проходят Π·Π° 2 сСкунды, Ρ‚ΠΎΠΊ = 4 Γ· 2 = 2 Π°ΠΌΠΏΠ΅Ρ€Π° (А).

Как элСктричСский Ρ‚ΠΎΠΊ гСнСрируСтся?

Атомы ΡΠ²Π»ΡΡŽΡ‚ΡΡ основными ΡΡ‚Ρ€ΠΎΠΈΡ‚Π΅Π»ΡŒΠ½Ρ‹ΠΌΠΈ Π±Π»ΠΎΠΊΠ°ΠΌΠΈ ΠΌΠ°Ρ‚Π΅Ρ€ΠΈΠΈ. ΠšΠ°ΠΆΠ΄Ρ‹ΠΉ ΠΎΠ±ΡŠΠ΅ΠΊΡ‚ Π² ВсСлСнная состоит ΠΈΠ· Π°Ρ‚ΠΎΠΌΠΎΠ². Атомы ΠΊΡ€ΠΎΡˆΠ΅Ρ‡Π½Ρ‹Π΅ частицы. Π˜Ρ… Ρ€Π°Π·ΠΌΠ΅Ρ€ ΡƒΠΊΠ°Π·Π°Π½ Π² Π½Π°Π½ΠΎΠΌΠ΅Ρ‚Ρ€Π°Ρ….

ΠšΠ°ΠΆΠ΄Ρ‹ΠΉ Π°Ρ‚ΠΎΠΌ состоит ΠΈΠ· субатомных частицы, Ρ‚Π°ΠΊΠΈΠ΅ ΠΊΠ°ΠΊ элСктроны, ΠΏΡ€ΠΎΡ‚ΠΎΠ½Ρ‹ ΠΈ Π½Π΅ΠΉΡ‚Ρ€ΠΎΠ½Ρ‹. Π­Ρ‚ΠΈ субатомныС частицы мСньшС Π°Ρ‚ΠΎΠΌΠ°.

Π­Π»Π΅ΠΊΡ‚Ρ€ΠΎΠ½Ρ‹ ΠΎΡ‚Ρ€ΠΈΡ†Π°Ρ‚Π΅Π»ΡŒΠ½ΠΎ заряТСны частицы, ΠΏΡ€ΠΎΡ‚ΠΎΠ½Ρ‹ — ΠΏΠΎΠ»ΠΎΠΆΠΈΡ‚Π΅Π»ΡŒΠ½ΠΎ заряТСнныС частицы, ΠΈ Π½Π΅ΠΉΡ‚Ρ€ΠΎΠ½Ρ‹ — Π½Π΅ΠΉΡ‚Ρ€Π°Π»ΡŒΠ½Ρ‹Π΅ частицы (Π±Π΅Π· заряда).

ΠŸΡ€ΠΎΡ‚ΠΎΠ½Ρ‹ ΠΈ Π½Π΅ΠΉΡ‚Ρ€ΠΎΠ½Ρ‹ Π½Π°ΠΌΠ½ΠΎΠ³ΠΎ тяТСлСС Ρ‡Π΅ΠΌ элСктроны. Π’Π°ΠΊΠΈΠΌ ΠΎΠ±Ρ€Π°Π·ΠΎΠΌ, ΠΏΡ€ΠΎΡ‚ΠΎΠ½Ρ‹ ΠΈ Π½Π΅ΠΉΡ‚Ρ€ΠΎΠ½Ρ‹ всСгда находятся Π² Ρ†Π΅Π½Ρ‚Ρ€ Π°Ρ‚ΠΎΠΌΠ°. Π’ ΡΠΈΠ»ΡŒΠ½Ρ‹ΠΉ ядСрная сила ΠΌΠ΅ΠΆΠ΄Ρƒ ΠΏΡ€ΠΎΡ‚ΠΎΠ½Π°ΠΌΠΈ ΠΈ Π½Π΅ΠΉΡ‚Ρ€ΠΎΠ½Π°ΠΌΠΈ заставляСт ΠΈΡ… всСгда Π΄Π΅Ρ€ΠΆΠΈΡ‚Π΅ΡΡŒ вмСстС.

ΠŸΡ€ΠΎΡ‚ΠΎΠ½Ρ‹ ΠΈΠΌΠ΅ΡŽΡ‚ ΠΏΠΎΠ»ΠΎΠΆΠΈΡ‚Π΅Π»ΡŒΠ½Ρ‹ΠΉ заряд ΠΈ Π½Π΅ΠΉΡ‚Ρ€ΠΎΠ½Ρ‹ Π½Π΅ ΠΈΠΌΠ΅ΡŽΡ‚ заряда. Π˜Ρ‚Π°ΠΊ, ΠΎΠ±Ρ‰ΠΈΠΉ заряд ядра ΠΏΠΎΠ»ΠΎΠΆΠΈΡ‚Π΅Π»ΡŒΠ½Ρ‹ΠΉ.

Π­Π»Π΅ΠΊΡ‚Ρ€ΠΎΠ½Ρ‹ всСгда Π²Ρ€Π°Ρ‰Π°ΡŽΡ‚ΡΡ Π²ΠΎΠΊΡ€ΡƒΠ³ ядро ΠΈΠ·-Π·Π° элСктростатичСской силы притяТСния ΠΌΠ΅ΠΆΠ΄Ρƒ Π½ΠΈΠΌΠΈ.

Π­Π»Π΅ΠΊΡ‚Ρ€ΠΎΠ½Ρ‹ Π²Ρ€Π°Ρ‰Π°ΡŽΡ‚ΡΡ Π²ΠΎΠΊΡ€ΡƒΠ³ ядро Π½Π° Ρ€Π°Π·Π½Ρ‹Ρ… ΠΎΡ€Π±ΠΈΡ‚Π°Ρ…. КаТдая ΠΎΡ€Π±ΠΈΡ‚Π° ΠΈΠΌΠ΅Π΅Ρ‚ ΡƒΡ€ΠΎΠ²Π΅Π½ΡŒ энСргии связанныС с Π½ΠΈΠΌ.

Π­Π»Π΅ΠΊΡ‚Ρ€ΠΎΠ½Ρ‹, Π²Ρ€Π°Ρ‰Π°ΡŽΡ‰ΠΈΠ΅ΡΡ ΠΏΡ€ΠΈ Π·Π°ΠΊΡ€Ρ‹Ρ‚ΠΈΠΈ расстояниС ΠΎΡ‚ ядра ΠΈΠΌΠ΅ΡŽΡ‚ ΠΎΡ‡Π΅Π½ΡŒ Π½ΠΈΠ·ΠΊΡƒΡŽ ΡΠ½Π΅Ρ€Π³ΠΈΡŽ. Π‘ Π΄Ρ€ΡƒΠ³ΠΎΠΉ стороны, элСктроны Π²Ρ€Π°Ρ‰Π°ΡŽΡ‚ΡΡ Π½Π° большСм расстоянии ΠΎΡ‚ ядра ΠΎΠ±Π»Π°Π΄Π°ΡŽΡ‚ ΠΎΡ‡Π΅Π½ΡŒ высокой энСргиСй.

Π­Π»Π΅ΠΊΡ‚Ρ€ΠΎΠ½Ρ‹ Π½Π° внСшнСй ΠΎΡ€Π±ΠΈΡ‚Π΅ Π°Ρ‚ΠΎΠΌ Π½Π°Π·Ρ‹Π²Π°ΡŽΡ‚ Π²Π°Π»Π΅Π½Ρ‚Π½Ρ‹ΠΌ элСктроны. Π­Ρ‚ΠΈ элСктроны ΠΎΡ‡Π΅Π½ΡŒ слабо ΠΏΡ€ΠΈΠΊΡ€Π΅ΠΏΠ»Π΅Π½Ρ‹ ΠΊ Ρ€ΠΎΠ΄ΠΈΡ‚Π΅Π»ΡŒΡΠΊΠΈΠΉ Π°Ρ‚ΠΎΠΌ.Π˜Ρ‚Π°ΠΊ, ΠΏΡ€ΠΈΠ»ΠΎΠΆΠΈΠ² нСбольшоС количСство энСргии достаточно, Ρ‡Ρ‚ΠΎΠ±Ρ‹ ΠΎΡΠ²ΠΎΠ±ΠΎΠ΄ΠΈΡ‚ΡŒ ΠΈΡ… ΠΎΡ‚ Ρ€ΠΎΠ΄ΠΈΡ‚Π΅Π»ΡŒΡΠΊΠΎΠ³ΠΎ Π°Ρ‚ΠΎΠΌΠ°.

Когда нСбольшоС количСство энСргии Π² Ρ„ΠΎΡ€ΠΌΠ° Ρ‚Π΅ΠΏΠ»Π°, свСта ΠΈΠ»ΠΈ элСктричСства ΠΏΠΎΠ»Π΅ пСрСдаСтся Π²Π°Π»Π΅Π½Ρ‚Π½Ρ‹ΠΌ элСктронам, ΠΎΠ½ΠΈ ΠΏΠΎΠ»ΡƒΡ‡Π°ΡŽΡ‚ достаточной энСргии, Π° Π·Π°Ρ‚Π΅ΠΌ отдСляСтся ΠΎΡ‚ Ρ€ΠΎΠ΄ΠΈΡ‚Π΅Π»ΡŒΡΠΊΠΎΠ³ΠΎ Π°Ρ‚ΠΎΠΌΠ°.

Π­Π»Π΅ΠΊΡ‚Ρ€ΠΎΠ½Ρ‹, ΠΎΡ‚Π΄Π΅Π»Π΅Π½Π½Ρ‹Π΅ ΠΎΡ‚ Ρ€ΠΎΠ΄ΠΈΡ‚Π΅Π»ΡŒΡΠΊΠΈΠΉ Π°Ρ‚ΠΎΠΌ извСстСн ΠΊΠ°ΠΊ свободный элСктроны.Π­Ρ‚ΠΈ элСктроны свободно ΠΏΠ΅Ρ€Π΅ΠΌΠ΅Ρ‰Π°ΡŽΡ‚ΡΡ ΠΈΠ· ΠΎΠ΄Π½ΠΎΠ³ΠΎ мСста Π² Π΄Ρ€ΡƒΠ³ΠΎΠ΅. Π΄Ρ€ΡƒΠ³ΠΎΠ΅ мСсто.

ΠœΡ‹ Π·Π½Π°Π΅ΠΌ, Ρ‡Ρ‚ΠΎ элСктроны ΠΈΠΌΠ΅ΡŽΡ‚ ΠΎΡ‚Ρ€ΠΈΡ†Π°Ρ‚Π΅Π»ΡŒΠ½ΠΎΠ΅ Π·Π°Ρ€ΡΠΆΠ°Ρ‚ΡŒ. Π’Π°ΠΊΠΈΠΌ ΠΎΠ±Ρ€Π°Π·ΠΎΠΌ, свободныС элСктроны нСсут ΠΎΡ‚Ρ€ΠΈΡ†Π°Ρ‚Π΅Π»ΡŒΠ½Ρ‹ΠΉ заряд ΠΎΡ‚ ΠΎΠ΄Π½ΠΎΠ³ΠΎ мСсто Π² Π΄Ρ€ΡƒΠ³ΠΎΠ΅ мСсто.

ΠœΡ‹ Π·Π½Π°Π΅ΠΌ, Ρ‡Ρ‚ΠΎ элСктричСский Ρ‚ΠΎΠΊ ΠΎΠ·Π½Π°Ρ‡Π°Π΅Ρ‚ ΠΏΠΎΡ‚ΠΎΠΊ заряда. Π˜Ρ‚Π°ΠΊ, элСктроны свободно ΠΏΠ΅Ρ€Π΅ΠΌΠ΅Ρ‰Π°ΡŽΡ‚ΡΡ ΠΈΠ· ΠΎΠ΄Π½ΠΎΠ³ΠΎ мСста Π² Π΄Ρ€ΡƒΠ³ΠΎΠ΅ мСсто Π±ΡƒΠ΄Π΅Ρ‚ ΠΏΡ€ΠΎΠ²ΠΎΠ΄ΠΈΡ‚ΡŒ элСктричСский Ρ‚ΠΎΠΊ.

Π’ ΠΏΠΎΠ»ΡƒΠΏΡ€ΠΎΠ²ΠΎΠ΄Π½ΠΈΠΊΠ°Ρ… ΠΎΠ±Π° свободных элСктрона ΠΈ Π΄Ρ‹Ρ€ΠΊΠΈ ΠΏΡ€ΠΈΡΡƒΡ‚ΡΡ‚Π²ΡƒΡŽΡ‚. Π‘Π²ΠΎΠ±ΠΎΠ΄Π½Ρ‹Π΅ элСктроны ΠΎΡ‚Ρ€ΠΈΡ†Π°Ρ‚Π΅Π»ΡŒΠ½ΠΎ заряТСнныС частицы. Π’Π°ΠΊΠΈΠΌ ΠΎΠ±Ρ€Π°Π·ΠΎΠΌ, ΠΎΠ½ΠΈ нСсут ΠΎΡ‚Ρ€ΠΈΡ†Π°Ρ‚Π΅Π»ΡŒΠ½Ρ‹ΠΉ заряд (элСктричСский Π’Π΅ΠΊΡƒΡ‰ΠΈΠΉ). Π”Ρ‹Ρ€Ρ‹ — это ΠΏΠΎΠ»ΠΎΠΆΠΈΡ‚Π΅Π»ΡŒΠ½ΠΎ заряТСнныС частицы. ΠŸΠΎΡΡ‚ΠΎΠΌΡƒ ΠΎΠ½ΠΈ нСсут ΠΏΠΎΠ»ΠΎΠΆΠΈΡ‚Π΅Π»ΡŒΠ½Ρ‹ΠΉ заряд (элСктричСский Ρ‚ΠΎΠΊ).

Π’Π°ΠΊΠΈΠΌ ΠΎΠ±Ρ€Π°Π·ΠΎΠΌ, ΠΈ свободныС элСктроны, ΠΈ Π΄Ρ‹Ρ€ΠΊΠΈ ΠΏΡ€ΠΎΠ²ΠΎΠ΄ΠΈΡ‚ΡŒ элСктричСский Ρ‚ΠΎΠΊ Π² ΠΏΠΎΠ»ΡƒΠΏΡ€ΠΎΠ²ΠΎΠ΄Π½ΠΈΠΊΠ°Ρ….

Π’ ΠΏΡ€ΠΎΠ²ΠΎΠ΄Π½ΠΈΠΊΠ°Ρ… отвСрстия Π½Π΅Π·Π½Π°Ρ‡ΠΈΡ‚Π΅Π»ΡŒΠ½Ρ‹. Π’Π°ΠΊ свободныС элСктроны проводят элСктричСский Ρ‚ΠΎΠΊ.

ΠŸΡ€ΠΎΡ‚ΠΎΠ½Ρ‹ Ρ‚Π°ΠΊΠΆΠ΅ ΠΎΠ±Π»Π°Π΄Π°ΡŽΡ‚ ΡΠΏΠΎΡΠΎΠ±Π½ΠΎΡΡ‚ΡŒΡŽ ΠΏΡ€ΠΎΠ²ΠΎΠ΄ΠΈΡ‚ΡŒ элСктричСский Ρ‚ΠΎΠΊ. Однако ΠΏΡ€ΠΎΡ‚ΠΎΠ½Ρ‹ Π½Π΅ ΠΌΠΎΠ³ΡƒΡ‚ свободно ΠΏΠ΅Ρ€Π΅ΠΌΠ΅Ρ‰Π°Ρ‚ΡŒΡΡ ΠΈΠ· ΠΎΠ΄Π½ΠΎΠ³ΠΎ мСста Π² Π΄Ρ€ΡƒΠ³ΠΎΠ΅, ΠΊΠ°ΠΊ элСктроны. Они всСгда удСрТиваСтся Π² фиксированном ΠΏΠΎΠ»ΠΎΠΆΠ΅Π½ΠΈΠΈ. Π˜Ρ‚Π°ΠΊ, ΠΏΡ€ΠΎΡ‚ΠΎΠ½Ρ‹ Π½Π΅ проводят элСктричСский Ρ‚ΠΎΠΊ.

SI Π΅Π΄ΠΈΠ½ΠΈΡ†Π° элСктричСского Ρ‚ΠΎΠΊΠ°

Π•Π΄ΠΈΠ½ΠΈΡ†Π° измСрСния элСктричСского Ρ‚ΠΎΠΊΠ° Π² систСмС БИ — Π°ΠΌΠΏΠ΅Ρ€, Π½Π°Π·Π²Π°Π½Π½Ρ‹ΠΉ Π² Ρ‡Π΅ΡΡ‚ΡŒ французского Ρ„ΠΈΠ·ΠΈΠΊΠ° АндрС-ΠœΠ°Ρ€ΠΈ АмпСр. ЭлСктричСский Ρ‚ΠΎΠΊ, ΠΏΡ€ΠΎΡ‚Π΅ΠΊΠ°ΡŽΡ‰ΠΈΠΉ Π² ΠΏΡ€ΠΎΠ²ΠΎΠ΄Π½ΠΈΠΊΠ΅ ΠΈΠ»ΠΈ ΠΏΠΎΠ»ΡƒΠΏΡ€ΠΎΠ²ΠΎΠ΄Π½ΠΈΠΊ измСряСтся Π² Π°ΠΌΠΏΠ΅Ρ€Π°Ρ…. АмпСр Ρ‚ΠΎΠΆΠ΅ ΠΈΠ½ΠΎΠ³Π΄Π° Π½Π°Π·Ρ‹Π²Π°ΡŽΡ‚ΡΡ усилитСлями ΠΈΠ»ΠΈ А.

Π’ΠΎΠΊ, ΠΏΡ€ΠΎΡ‚Π΅ΠΊΠ°ΡŽΡ‰ΠΈΠΉ Ρ‡Π΅Ρ€Π΅Π· элСктронный ΠΊΠΎΠΌΠΏΠΎΠ½Π΅Π½Ρ‚ (Π½Π°ΠΏΡ€ΠΈΠΌΠ΅Ρ€, Π΄ΠΈΠΎΠ΄) Π² Ρ†Π΅ΠΏΠΈ измСряСтся с ΠΏΠΎΠΌΠΎΡ‰ΡŒΡŽ устройства, Π½Π°Π·Ρ‹Π²Π°Π΅ΠΌΠΎΠ³ΠΎ Π°ΠΌΠΏΠ΅Ρ€ΠΌΠ΅Ρ‚Ρ€ΠΎΠΌ.

Π’Π΅ΠΊΡƒΡ‰ΠΈΠΉ Π½Π°ΠΏΡ€Π°Π²Π»Π΅Π½ΠΈΠ΅

Когда напряТСниС подаСтся Π½Π° ΠΏΡ€ΠΎΠ²ΠΎΠ΄Π½ΠΈΠΊ ΠΈΠ»ΠΈ ΠΏΠΎΠ»ΡƒΠΏΡ€ΠΎΠ²ΠΎΠ΄Π½ΠΈΠΊ, Π½Π°Ρ‡ΠΈΠ½Π°Π΅Ρ‚ Ρ‚Π΅Ρ‡ΡŒ элСктричСский Ρ‚ΠΎΠΊ.

Π’ ΠΏΡ€ΠΎΠ²ΠΎΠ΄Π½ΠΈΠΊΠ°Ρ…, ΠΏΠΎΠ»ΠΎΠΆΠΈΡ‚Π΅Π»ΡŒΠ½ΠΎ заряТСнных ΠΏΡ€ΠΎΡ‚ΠΎΠ½Ρ‹ ΡƒΠ΄Π΅Ρ€ΠΆΠΈΠ²Π°ΡŽΡ‚ΡΡ Π² фиксированном ΠΏΠΎΠ»ΠΎΠΆΠ΅Π½ΠΈΠΈ, Π° ΠΎΡ‚Ρ€ΠΈΡ†Π°Ρ‚Π΅Π»ΡŒΠ½ΠΎ заряТСнныС элСктроны ΠΏΠ΅Ρ€Π΅ΠΌΠ΅Ρ‰Π°ΡŽΡ‚ΡΡ ΠΈΠ· ΠΎΠ΄Π½ΠΎΠ³ΠΎ мСста Π² Π΄Ρ€ΡƒΠ³ΠΎΠ΅ Π·Π° счСт нСсущий заряд. Π’Π°ΠΊΠΈΠΌ ΠΎΠ±Ρ€Π°Π·ΠΎΠΌ, элСктроны проводят элСктричСский Ρ‚ΠΎΠΊ. Π² ΠΏΡ€ΠΎΠ²ΠΎΠ΄Π½ΠΈΠΊΠ°Ρ….

Π’ ΠΏΠΎΠ»ΡƒΠΏΡ€ΠΎΠ²ΠΎΠ΄Π½ΠΈΠΊΠ°Ρ… ΠΎΠ±Π° свободных элСктрона Π° Π΄Ρ‹Ρ€Ρ‹ пСрСносят заряд ΠΈΠ· ΠΎΠ΄Π½ΠΎΠ³ΠΎ мСста Π² Π΄Ρ€ΡƒΠ³ΠΎΠ΅. Π’Π°ΠΊΠΈΠΌ ΠΎΠ±Ρ€Π°Π·ΠΎΠΌ, элСктроны ΠΈ Π΄Ρ‹Ρ€ΠΊΠΈ проводят элСктричСский Ρ‚ΠΎΠΊ Π² ΠΏΠΎΠ»ΡƒΠΏΡ€ΠΎΠ²ΠΎΠ΄Π½ΠΈΠΊΠΈ.

ΠŸΡ€ΠΈ ΠΏΠΎΠ΄Π°Ρ‡Π΅ напряТСния элСктроны (ΠΎΡ‚Ρ€ΠΈΡ†Π°Ρ‚Π΅Π»ΡŒΠ½Ρ‹Π΅ заряды) ΠΏΠ΅Ρ€Π΅ΠΌΠ΅Ρ‰Π°ΡŽΡ‚ΡΡ ΠΎΡ‚ ΠΎΡ‚Ρ€ΠΈΡ†Π°Ρ‚Π΅Π»ΡŒΠ½ΠΎΠ³ΠΎ ΠΊΠΎΠ½Ρ†Π° Π±Π°Ρ‚Π°Ρ€Π΅ΠΈ ΠΊ ΠΏΠΎΠ»ΠΎΠΆΠΈΡ‚Π΅Π»ΡŒΠ½Ρ‹ΠΉ ΠΊΠΎΠ½Π΅Ρ† Π±Π°Ρ‚Π°Ρ€Π΅ΠΈ. Π˜Ρ‚Π°ΠΊ, элСктроны (ΠΎΡ‚Ρ€ΠΈΡ†Π°Ρ‚Π΅Π»ΡŒΠ½Ρ‹Π΅ зарядов) Π½Π°ΠΏΡ€Π°Π²Π»Π΅Π½ΠΈΠ΅ Ρ‚ΠΎΠΊΠ° ΠΎΡ‚ ΠΎΡ‚Ρ€ΠΈΡ†Π°Ρ‚Π΅Π»ΡŒΠ½ΠΎΠ³ΠΎ ΠΊ ΠΏΠΎΠ»ΠΎΠΆΠΈΡ‚Π΅Π»ΡŒΠ½ΠΎΠΌΡƒ.

Π‘ Π΄Ρ€ΡƒΠ³ΠΎΠΉ стороны, отвСрстия (ΠΏΠΎΠ»ΠΎΠΆΠΈΡ‚Π΅Π»ΡŒΠ½Ρ‹ΠΉ заряды) ΠΏΠ΅Ρ€Π΅ΠΌΠ΅Ρ‰Π°ΡŽΡ‚ΡΡ ΠΎΡ‚ ΠΏΠΎΠ»ΠΎΠΆΠΈΡ‚Π΅Π»ΡŒΠ½ΠΎΠ³ΠΎ ΠΊΠΎΠ½Ρ†Π° Π±Π°Ρ‚Π°Ρ€Π΅ΠΈ ΠΊ ΠΎΡ‚Ρ€ΠΈΡ†Π°Ρ‚Π΅Π»ΡŒΠ½ΠΎΠΌΡƒ ΠΊΠΎΠ½Π΅Ρ† Π±Π°Ρ‚Π°Ρ€Π΅ΠΈ. Π’Π°ΠΊ Π΄Ρ‹Ρ€Ρ‹ (ΠΏΠΎΠ»ΠΎΠΆΠΈΡ‚Π΅Π»ΡŒΠ½Ρ‹Π΅ заряды) Ρ‚ΠΎΠΊ Π½Π°ΠΏΡ€Π°Π²Π»Π΅Π½ΠΈΠ΅ ΠΎΡ‚ ΠΏΠΎΠ»ΠΎΠΆΠΈΡ‚Π΅Π»ΡŒΠ½ΠΎΠ³ΠΎ ΠΊ ΠΎΡ‚Ρ€ΠΈΡ†Π°Ρ‚Π΅Π»ΡŒΠ½ΠΎΠΌΡƒ.

ΠžΠ±Ρ‹Ρ‡Π½Ρ‹ΠΉ Ρ‚Π΅ΠΊΡƒΡ‰Π΅Π΅ Π½Π°ΠΏΡ€Π°Π²Π»Π΅Π½ΠΈΠ΅ ΠΎΡ‚ ΠΏΠΎΠ»ΠΎΠΆΠΈΡ‚Π΅Π»ΡŒΠ½ΠΎΠ³ΠΎ ΠΊ ΠΎΡ‚Ρ€ΠΈΡ†Π°Ρ‚Π΅Π»ΡŒΠ½ΠΎΠΌΡƒ (Ρ‚ΠΎ ΠΆΠ΅, Ρ‡Ρ‚ΠΎ ΠΈ Ρ‚Π΅ΠΊΡƒΡ‰Π΅Π΅ Π½Π°ΠΏΡ€Π°Π²Π»Π΅Π½ΠΈΠ΅ ΠΏΠΎΠ»ΠΎΠΆΠΈΡ‚Π΅Π»ΡŒΠ½Ρ‹Ρ… зарядов).

Заряд ΠΏΠΎΠ»ΠΎΠΆΠΈΡ‚Π΅Π»ΡŒΠ½ΠΎ заряТСнного частица (Π΄Ρ‹Ρ€ΠΊΠ°) Ρ€Π°Π²Π½Π° заряду ΠΎΡ‚Ρ€ΠΈΡ†Π°Ρ‚Π΅Π»ΡŒΠ½ΠΎ заряТСнной частица (свободный элСктрон), Π½ΠΎ ΠΏΡ€ΠΎΡ‚ΠΈΠ²ΠΎΠΏΠΎΠ»ΠΎΠΆΠ½ΠΎΠΉ полярности.

ΠŸΠΎΡ‚ΠΎΠΊ ΠΎΡ‚Ρ€ΠΈΡ†Π°Ρ‚Π΅Π»ΡŒΠ½Ρ‹Ρ… зарядов Π² Ρ†Π΅ΠΏΠΈ Π±ΡƒΠ΄Π΅Ρ‚ ΠΏΡ€ΠΎΠΈΠ·Π²ΠΎΠ΄ΠΈΡ‚ΡŒ Ρ‚ΠΎΠΊ Ρ‚Π°ΠΊΠΎΠΉ ΠΆΠ΅, ΠΊΠ°ΠΊ ΠΏΠΎΡ‚ΠΎΠΊ ΠΏΠΎΠ»ΠΎΠΆΠΈΡ‚Π΅Π»ΡŒΠ½Ρ‹Ρ… зарядов ΠΏΡ€ΠΎΠΈΠ·Π²ΠΎΠ΄ΠΈΡ‚ΡŒ. Π’Π°ΠΊ Ρ‡Ρ‚ΠΎ Π½Π΅ ΠΈΠΌΠ΅Π΅Ρ‚ значСния, Ρ‚Π΅Ρ‡Π΅Ρ‚ Π»ΠΈ Ρ‚ΠΎΠΊ ΠΎΡ‚ ΠΏΠΎΠ»ΠΎΠΆΠΈΡ‚Π΅Π»ΡŒΠ½ΠΎΠ³ΠΎ ΠΊ ΠΎΡ‚Ρ€ΠΈΡ†Π°Ρ‚Π΅Π»ΡŒΠ½ΠΎΠΌΡƒ ΠΈΠ»ΠΈ ΠΎΡ‚Ρ€ΠΈΡ†Π°Ρ‚Π΅Π»ΡŒΠ½ΠΎΠ³ΠΎ ΠΊ ΠΏΠΎΠ»ΠΎΠΆΠΈΡ‚Π΅Π»ΡŒΠ½ΠΎΠΌΡƒ, Π³Π΅Π½Π΅Ρ€ΠΈΡ€ΡƒΠ΅ΠΌΡ‹ΠΉ Ρ‚ΠΎΠΊ Π±ΡƒΠ΄Π΅Ρ‚ Ρ‚Π°ΠΊΠΈΠΌ ΠΆΠ΅.

ЭлСктричСский Ρ‚ΠΎΠΊ | ЭнциклопСдия.com

Под элСктричСским Ρ‚ΠΎΠΊΠΎΠΌ ΠΎΠ±Ρ‹Ρ‡Π½ΠΎ ΠΏΠΎΠ½ΠΈΠΌΠ°ΡŽΡ‚ ΠΏΠΎΡ‚ΠΎΠΊ элСктронов. Когда Π΄Π²Π° ΠΊΠΎΠ½Ρ†Π° Π±Π°Ρ‚Π°Ρ€Π΅ΠΈ ΡΠΎΠ΅Π΄ΠΈΠ½ΡΡŽΡ‚ΡΡ Π΄Ρ€ΡƒΠ³ с Π΄Ρ€ΡƒΠ³ΠΎΠΌ с ΠΏΠΎΠΌΠΎΡ‰ΡŒΡŽ мСталличСского ΠΏΡ€ΠΎΠ²ΠΎΠ΄Π°, элСктроны выходят ΠΈΠ· ΠΎΠ΄Π½ΠΎΠ³ΠΎ ΠΊΠΎΠ½Ρ†Π° (элСктрода ΠΈΠ»ΠΈ полюса) Π±Π°Ρ‚Π°Ρ€Π΅ΠΈ Ρ‡Π΅Ρ€Π΅Π· ΠΏΡ€ΠΎΠ²ΠΎΠ΄ ΠΊ ΠΏΡ€ΠΎΡ‚ΠΈΠ²ΠΎΠΏΠΎΠ»ΠΎΠΆΠ½ΠΎΠΌΡƒ ΠΊΠΎΠ½Ρ†Ρƒ Π±Π°Ρ‚Π°Ρ€Π΅ΠΈ.

ЭлСктричСский Ρ‚ΠΎΠΊ Ρ‚Π°ΠΊΠΆΠ΅ ΠΌΠΎΠΆΠ½ΠΎ Ρ€Π°ΡΡΠΌΠ°Ρ‚Ρ€ΠΈΠ²Π°Ρ‚ΡŒ ΠΊΠ°ΠΊ ΠΏΠΎΡ‚ΠΎΠΊ ΠΏΠΎΠ»ΠΎΠΆΠΈΡ‚Π΅Π»ΡŒΠ½Ρ‹Ρ… Β«Π΄Ρ‹Ρ€ΠΎΠΊΒ». Β«Π”Ρ‹Ρ€Π°Β» Π² этом смыслС — это ΠΎΠ±Π»Π°ΡΡ‚ΡŒ пространства, Π³Π΄Π΅ ΠΎΠ±Ρ‹Ρ‡Π½ΠΎ ΠΌΠΎΠΆΠ½ΠΎ Π½Π°ΠΉΡ‚ΠΈ элСктрон, Π½ΠΎ Π½Π΅ сущСствуСт. ΠžΡ‚ΡΡƒΡ‚ΡΡ‚Π²ΠΈΠ΅ ΠΎΡ‚Ρ€ΠΈΡ†Π°Ρ‚Π΅Π»ΡŒΠ½ΠΎΠ³ΠΎ заряда элСктрона ΠΌΠΎΠΆΠ½ΠΎ Ρ€Π°ΡΡΠΌΠ°Ρ‚Ρ€ΠΈΠ²Π°Ρ‚ΡŒ ΠΊΠ°ΠΊ созданиС ΠΏΠΎΠ»ΠΎΠΆΠΈΡ‚Π΅Π»ΡŒΠ½ΠΎ заряТСнной Π΄Ρ‹Ρ€Ρ‹.

Π’ Π½Π΅ΠΊΠΎΡ‚ΠΎΡ€Ρ‹Ρ… случаях элСктричСский Ρ‚ΠΎΠΊ ΠΌΠΎΠΆΠ΅Ρ‚ Ρ‚Π°ΠΊΠΆΠ΅ ΡΠΎΡΡ‚ΠΎΡΡ‚ΡŒ ΠΈΠ· ΠΏΠΎΡ‚ΠΎΠΊΠ° ΠΏΠΎΠ»ΠΎΠΆΠΈΡ‚Π΅Π»ΡŒΠ½ΠΎ заряТСнных частиц, извСстных ΠΊΠ°ΠΊ ΠΊΠ°Ρ‚ΠΈΠΎΠ½Ρ‹. ΠšΠ°Ρ‚ΠΈΠΎΠ½ — это просто Π°Ρ‚ΠΎΠΌ ΠΈΠ»ΠΈ Π³Ρ€ΡƒΠΏΠΏΠ° Π°Ρ‚ΠΎΠΌΠΎΠ², нСсущих ΠΏΠΎΠ»ΠΎΠΆΠΈΡ‚Π΅Π»ΡŒΠ½Ρ‹ΠΉ заряд.

Π˜Π·ΠΌΠ΅Ρ€Π΅Π½ΠΈΠ΅ Ρ‚ΠΎΠΊΠ°

АмпСр (Π°ΠΌΠΏΠ΅Ρ€) ΠΈΡΠΏΠΎΠ»ΡŒΠ·ΡƒΠ΅Ρ‚ΡΡ для измСрСния Π²Π΅Π»ΠΈΡ‡ΠΈΠ½Ρ‹ ΠΏΡ€ΠΎΡ‚Π΅ΠΊΠ°ΡŽΡ‰Π΅Π³ΠΎ Ρ‚ΠΎΠΊΠ°. ΠžΡ‚Π΄Π΅Π»Π΅Π½ΠΈΠ΅ Π±Ρ‹Π»ΠΎ Π½Π°Π·Π²Π°Π½ΠΎ Π² Ρ‡Π΅ΡΡ‚ΡŒ французского ΠΌΠ°Ρ‚Π΅ΠΌΠ°Ρ‚ΠΈΠΊΠ° ΠΈ Ρ„ΠΈΠ·ΠΈΠΊΠ° АндрС ΠœΠ°Ρ€ΠΈ АмпСра (1775–1836), основавшСго соврСмСнныС исслСдования элСктричСских Ρ‚ΠΎΠΊΠΎΠ². АмпСр опрСдСляСтся ΠΊΠ°ΠΊ количСство элСктронов, ΠΊΠΎΡ‚ΠΎΡ€Ρ‹Π΅ проходят Ρ‡Π΅Ρ€Π΅Π· Π»ΡŽΠ±ΡƒΡŽ Π·Π°Π΄Π°Π½Π½ΡƒΡŽ Ρ‚ΠΎΡ‡ΠΊΡƒ Π·Π° ΠΎΠΏΡ€Π΅Π΄Π΅Π»Π΅Π½Π½ΡƒΡŽ Π΅Π΄ΠΈΠ½ΠΈΡ†Ρƒ Π²Ρ€Π΅ΠΌΠ΅Π½ΠΈ.ΠŸΠΎΡΠΊΠΎΠ»ΡŒΠΊΡƒ элСктричСский заряд измСряСтся Π² ΠΊΡƒΠ»ΠΎΠ½Π°Ρ…, Ρ‚ΠΎΡ‡Π½ΠΎΠ΅ ΠΎΠΏΡ€Π΅Π΄Π΅Π»Π΅Π½ΠΈΠ΅ Π°ΠΌΠΏΠ΅Ρ€Π° — это количСство ΠΊΡƒΠ»ΠΎΠ½ΠΎΠ², ΠΊΠΎΡ‚ΠΎΡ€Ρ‹Π΅ проходят Ρ‡Π΅Ρ€Π΅Π· Π·Π°Π΄Π°Π½Π½ΡƒΡŽ Ρ‚ΠΎΡ‡ΠΊΡƒ ΠΊΠ°ΠΆΠ΄ΡƒΡŽ сСкунду.

Π₯арактСристики элСктричСского Ρ‚ΠΎΠΊΠ°

Π Π°Π·Π½ΠΎΡΡ‚ΡŒ ΠΏΠΎΡ‚Π΅Π½Ρ†ΠΈΠ°Π»ΠΎΠ². Для протСкания элСктричСского Ρ‚ΠΎΠΊΠ° Π½Π΅ΠΎΠ±Ρ…ΠΎΠ΄ΠΈΠΌΠΎ Π²Ρ‹ΠΏΠΎΠ»Π½Π΅Π½ΠΈΠ΅ ряда условий. Π’ΠΎ-ΠΏΠ΅Ρ€Π²Ρ‹Ρ…, ΠΌΠ΅ΠΆΠ΄Ρƒ двумя Ρ‚ΠΎΡ‡ΠΊΠ°ΠΌΠΈ Π΄ΠΎΠ»ΠΆΠ½Π° ΡΡƒΡ‰Π΅ΡΡ‚Π²ΠΎΠ²Π°Ρ‚ΡŒ Ρ€Π°Π·Π½ΠΎΡΡ‚ΡŒ ΠΏΠΎΡ‚Π΅Π½Ρ†ΠΈΠ°Π»ΠΎΠ². Π’Π΅Ρ€ΠΌΠΈΠ½ Ρ€Π°Π·Π½ΠΎΡΡ‚ΡŒ ΠΏΠΎΡ‚Π΅Π½Ρ†ΠΈΠ°Π»ΠΎΠ² (ΠΈΠ»ΠΈ напряТСниС) ΠΎΠ·Π½Π°Ρ‡Π°Π΅Ρ‚, Ρ‡Ρ‚ΠΎ сила, создаваСмая Π³Ρ€ΡƒΠΏΠΏΠΎΠΉ элСктронов Π² ΠΎΠ΄Π½ΠΎΠΌ мСстС, большС, Ρ‡Π΅ΠΌ сила элСктронов Π² Π΄Ρ€ΡƒΠ³ΠΎΠΌ мСстС.Π‘ΠΎΠ»ΡŒΡˆΠ°Ρ сила ΠΎΡ‚Ρ‚Π°Π»ΠΊΠΈΠ²Π°Π΅Ρ‚ элСктроны ΠΎΡ‚ ΠΏΠ΅Ρ€Π²ΠΎΠ³ΠΎ мСста ΠΊΠΎ Π²Ρ‚ΠΎΡ€ΠΎΠΌΡƒ.

ΠŸΠΎΡ‚Π΅Π½Ρ†ΠΈΠ°Π»ΡŒΠ½Ρ‹Π΅ различия ΠΎΠ±Ρ‹Ρ‡Π½ΠΎ Π½Π΅ Π²ΡΡ‚Ρ€Π΅Ρ‡Π°ΡŽΡ‚ΡΡ Π² ΠΏΡ€ΠΈΡ€ΠΎΠ΄Π΅. Π’ Π±ΠΎΠ»ΡŒΡˆΠΈΠ½ΡΡ‚Π²Π΅ случаСв распрСдСлСниС элСктронов Π² ΠΎΠΊΡ€ΡƒΠΆΠ°ΡŽΡ‰Π΅ΠΌ нас ΠΌΠΈΡ€Π΅ довольно Ρ€Π°Π²Π½ΠΎΠΌΠ΅Ρ€Π½ΠΎΠ΅. Однако ΡƒΡ‡Π΅Π½Ρ‹Π΅ ΠΈΠ·ΠΎΠ±Ρ€Π΅Π»ΠΈ ΠΎΠΏΡ€Π΅Π΄Π΅Π»Π΅Π½Π½Ρ‹Π΅ Π²ΠΈΠ΄Ρ‹ устройств, Π² ΠΊΠΎΡ‚ΠΎΡ€Ρ‹Ρ… элСктроны ΠΌΠΎΠ³ΡƒΡ‚ Π½Π°ΠΊΠ°ΠΏΠ»ΠΈΠ²Π°Ρ‚ΡŒΡΡ, создавая Ρ€Π°Π·Π½ΠΎΡΡ‚ΡŒ ΠΏΠΎΡ‚Π΅Π½Ρ†ΠΈΠ°Π»ΠΎΠ². НапримСр, батарСя — это Π½Π΅ Ρ‡Ρ‚ΠΎ ΠΈΠ½ΠΎΠ΅, ΠΊΠ°ΠΊ устройство для производства Π±ΠΎΠ»ΡŒΡˆΠΈΡ… масс элСктронов Π½Π° ΠΎΠ΄Π½ΠΎΠΌ элСктродС (Ρ‚ΠΎΡ‡ΠΊΠ°, ΠΈΠ· ΠΊΠΎΡ‚ΠΎΡ€ΠΎΠΉ отправляСтся ΠΈΠ»ΠΈ принимаСтся элСктричСский Ρ‚ΠΎΠΊ) ΠΈ нСдостатка элСктронов Π½Π° Π΄Ρ€ΡƒΠ³ΠΎΠΌ элСктродС.Π­Ρ‚Π° Ρ€Π°Π·Π½ΠΈΡ†Π° ΠΎΠ±ΡŠΡΡΠ½ΡΠ΅Ρ‚ ΡΠΏΠΎΡΠΎΠ±Π½ΠΎΡΡ‚ΡŒ Π±Π°Ρ‚Π°Ρ€Π΅ΠΈ Π³Π΅Π½Π΅Ρ€ΠΈΡ€ΠΎΠ²Π°Ρ‚ΡŒ Ρ€Π°Π·Π½ΠΎΡΡ‚ΡŒ ΠΏΠΎΡ‚Π΅Π½Ρ†ΠΈΠ°Π»ΠΎΠ² ΠΈΠ»ΠΈ напряТСниС.

ЭлСктричСскоС сопротивлСниС. Π’Ρ‚ΠΎΡ€ΠΎΠ΅ условиС, Π½Π΅ΠΎΠ±Ρ…ΠΎΠ΄ΠΈΠΌΠΎΠ΅ для протСкания Ρ‚ΠΎΠΊΠ°, — это ΠΏΡƒΡ‚ΡŒ, ΠΏΠΎ ΠΊΠΎΡ‚ΠΎΡ€ΠΎΠΌΡƒ ΠΌΠΎΠ³ΡƒΡ‚ ΠΏΠ΅Ρ€Π΅ΠΌΠ΅Ρ‰Π°Ρ‚ΡŒΡΡ элСктроны. НСкоторыС ΠΌΠ°Ρ‚Π΅Ρ€ΠΈΠ°Π»Ρ‹ ΠΌΠΎΠ³ΡƒΡ‚ ΠΎΠ±Π΅ΡΠΏΠ΅Ρ‡ΠΈΡ‚ΡŒ Ρ‚Π°ΠΊΠΎΠΉ ΠΏΡƒΡ‚ΡŒ, Π° Π΄Ρ€ΡƒΠ³ΠΈΠ΅ — Π½Π΅Ρ‚. ΠœΠ°Ρ‚Π΅Ρ€ΠΈΠ°Π»Ρ‹, ΠΊΠΎΡ‚ΠΎΡ€Ρ‹Π΅ ΠΏΡ€ΠΎΠΏΡƒΡΠΊΠ°ΡŽΡ‚ элСктричСский Ρ‚ΠΎΠΊ, Π½Π°Π·Ρ‹Π²Π°ΡŽΡ‚ΡΡ ΠΏΡ€ΠΎΠ²ΠΎΠ΄Π½ΠΈΠΊΠ°ΠΌΠΈ. Π’Π΅, ΠΊΠΎΡ‚ΠΎΡ€Ρ‹Π΅ Π±Π»ΠΎΠΊΠΈΡ€ΡƒΡŽΡ‚ ΠΏΡ€ΠΎΡ…ΠΎΠΆΠ΄Π΅Π½ΠΈΠ΅ элСктричСского Ρ‚ΠΎΠΊΠ°, Π½Π°Π·Ρ‹Π²Π°ΡŽΡ‚ΡΡ Π½Π΅ΠΏΡ€ΠΎΠ²ΠΎΠ΄Π½ΠΈΠΊΠ°ΠΌΠΈ ΠΈΠ»ΠΈ изоляторами. ΠœΠ΅Ρ‚Π°Π»Π»ΠΈΡ‡Π΅ΡΠΊΠΈΠΉ ΠΏΡ€ΠΎΠ²ΠΎΠ΄, ΡΠΎΠ΅Π΄ΠΈΠ½ΡΡŽΡ‰ΠΈΠΉ Π΄Π²Π° полюса Π±Π°Ρ‚Π°Ρ€Π΅ΠΈ Π² ΠΏΡ€ΠΈΠ²Π΅Π΄Π΅Π½Π½ΠΎΠΌ Ρ€Π°Π½Π΅Π΅ ΠΏΡ€ΠΈΠΌΠ΅Ρ€Π΅, обСспСчиваСт ΠΏΡƒΡ‚ΡŒ для двиТСния элСктронов ΠΎΡ‚ ΠΎΠ΄Π½ΠΎΠ³ΠΎ полюса Π±Π°Ρ‚Π°Ρ€Π΅ΠΈ ΠΊ Π΄Ρ€ΡƒΠ³ΠΎΠΌΡƒ.

Π­Π»Π΅ΠΊΡ‚Ρ€ΠΎΠΏΡ€ΠΎΠ²ΠΎΠ΄Π½ΠΎΡΡ‚ΡŒ ΠΌΠ°Ρ‚Π΅Ρ€ΠΈΠ°Π»ΠΎΠ² — это Π²Π½ΡƒΡ‚Ρ€Π΅Π½Π½Π΅Π΅ (ΠΈΠ»ΠΈ СстСствСнноС) свойство, основанноС Π½Π° ΠΈΡ… сопротивлСнии двиТСнию элСктронов. Π­Π»Π΅ΠΊΡ‚Ρ€ΠΎΠ½Ρ‹ Π² Π½Π΅ΠΊΠΎΡ‚ΠΎΡ€Ρ‹Ρ… ΠΌΠ°Ρ‚Π΅Ρ€ΠΈΠ°Π»Π°Ρ… связаны химичСскими связями ΠΈ Π½Π΅ ΠΌΠΎΠ³ΡƒΡ‚ ΠΏΡ€ΠΎΠ²ΠΎΠ΄ΠΈΡ‚ΡŒ элСктричСский Ρ‚ΠΎΠΊ. Π’ Π΄Ρ€ΡƒΠ³ΠΈΡ… ΠΌΠ°Ρ‚Π΅Ρ€ΠΈΠ°Π»Π°Ρ… большоС количСство элСктронов свободно пСрСмСщаСтся, ΠΈ ΠΎΠ½ΠΈ Π»Π΅Π³ΠΊΠΎ ΠΏΠ΅Ρ€Π΅Π΄Π°ΡŽΡ‚ ΠΏΠΎΡ‚ΠΎΠΊ элСктронов.

ЭлСктричСскоС сопротивлСниС (ΠΈΠ»ΠΈ ΡƒΠ΄Π΅Π»ΡŒΠ½ΠΎΠ΅ сопротивлСниС) измСряСтся Π² Π΅Π΄ΠΈΠ½ΠΈΡ†Π°Ρ…, извСстных ΠΊΠ°ΠΊ ΠΎΠΌ (Ом). Устройство Π±Ρ‹Π»ΠΎ Π½Π°Π·Π²Π°Π½ΠΎ Π² Ρ‡Π΅ΡΡ‚ΡŒ Π½Π΅ΠΌΠ΅Ρ†ΠΊΠΎΠ³ΠΎ Ρ„ΠΈΠ·ΠΈΠΊΠ° Π“Π΅ΠΎΡ€Π³Π° Π‘ΠΈΠΌΠΎΠ½Π° Ома (1789–1854), ΠΏΠ΅Ρ€Π²ΠΎΠ³ΠΎ Ρ‡Π΅Π»ΠΎΠ²Π΅ΠΊΠ°, Π²Ρ‹Ρ€Π°Π·ΠΈΠ²ΡˆΠ΅Π³ΠΎ Π·Π°ΠΊΠΎΠ½Ρ‹ элСктропроводности.ΠŸΡ€ΠΎΡ‚ΠΈΠ²ΠΎΠΏΠΎΠ»ΠΎΠΆΠ½ΠΎΡΡ‚ΡŒΡŽ ΡΠΎΠΏΡ€ΠΎΡ‚ΠΈΠ²Π»Π΅Π½ΠΈΡŽ являСтся ΠΏΡ€ΠΎΠ²ΠΎΠ΄ΠΈΠΌΠΎΡΡ‚ΡŒ, свойство, ΠΊΠΎΡ‚ΠΎΡ€ΠΎΠ΅ измСряСтся Π΅Π΄ΠΈΠ½ΠΈΡ†Π΅ΠΉ, Π½Π°Π·Ρ‹Π²Π°Π΅ΠΌΠΎΠΉ mho (ΠΎΠΌ, записанный Π½Π°ΠΎΠ±ΠΎΡ€ΠΎΡ‚).

Π‘ΠΎΠΏΡ€ΠΎΡ‚ΠΈΠ²Π»Π΅Π½ΠΈΠ΅ куска ΠΏΡ€ΠΎΠ²ΠΎΠ΄Π°, ΠΈΡΠΏΠΎΠ»ΡŒΠ·ΡƒΠ΅ΠΌΠΎΠ³ΠΎ Π² элСктричСской Ρ†Π΅ΠΏΠΈ, зависит ΠΎΡ‚ Ρ‚Ρ€Π΅Ρ… Ρ„Π°ΠΊΡ‚ΠΎΡ€ΠΎΠ²: Π΄Π»ΠΈΠ½Ρ‹ ΠΏΡ€ΠΎΠ²ΠΎΠ΄Π°, Π΅Π³ΠΎ ΠΏΠ»ΠΎΡ‰Π°Π΄ΠΈ ΠΏΠΎΠΏΠ΅Ρ€Π΅Ρ‡Π½ΠΎΠ³ΠΎ сСчСния ΠΈ ΡƒΠ΄Π΅Π»ΡŒΠ½ΠΎΠ³ΠΎ сопротивлСния ΠΌΠ°Ρ‚Π΅Ρ€ΠΈΠ°Π»Π°, ΠΈΠ· ΠΊΠΎΡ‚ΠΎΡ€ΠΎΠ³ΠΎ ΠΎΠ½ сдСлан. Π§Ρ‚ΠΎΠ±Ρ‹ ΠΏΠΎΠ½ΡΡ‚ΡŒ влияниС элСктричСского сопротивлСния, ΠΏΡ€Π΅Π΄ΡΡ‚Π°Π²ΡŒΡ‚Π΅ сСбС Π²ΠΎΠ΄Ρƒ, Ρ‚Π΅ΠΊΡƒΡ‰ΡƒΡŽ ΠΏΠΎ ΡˆΠ»Π°Π½Π³Ρƒ.

ΠšΠΎΠ»ΠΈΡ‡Π΅ΡΡ‚Π²ΠΎ Π²ΠΎΠ΄Ρ‹, ΠΏΡ€ΠΎΡ‚Π΅ΠΊΠ°ΡŽΡ‰Π΅ΠΉ ΠΏΠΎ ΡˆΠ»Π°Π½Π³Ρƒ, Π°Π½Π°Π»ΠΎΠ³ΠΈΡ‡Π½ΠΎ Ρ‚ΠΎΠΊΡƒ Π² ΠΏΡ€ΠΎΠ²ΠΎΠ΄Π΅.Подобно Ρ‚ΠΎΠΌΡƒ, ΠΊΠ°ΠΊ Ρ‡Π΅Ρ€Π΅Π· толстый ΠΏΠΎΠΆΠ°Ρ€Π½Ρ‹ΠΉ шланг ΠΌΠΎΠΆΠ΅Ρ‚ ΠΏΡ€ΠΎΠΉΡ‚ΠΈ большС Π²ΠΎΠ΄Ρ‹, Ρ‡Π΅ΠΌ Ρ‡Π΅Ρ€Π΅Π· Ρ‚ΠΎΠ½ΠΊΠΈΠΉ садовый шланг, толстая мСталличСская ΠΏΡ€ΠΎΠ²ΠΎΠ»ΠΎΠΊΠ° ΠΌΠΎΠΆΠ΅Ρ‚ ΠΏΡ€ΠΎΠΏΡƒΡΠΊΠ°Ρ‚ΡŒ большС Ρ‚ΠΎΠΊΠ°, Ρ‡Π΅ΠΌ тонкая мСталличСская ΠΏΡ€ΠΎΠ²ΠΎΠ»ΠΎΠΊΠ°. Π£ ΠΏΡ€ΠΎΠ²ΠΎΠ΄Π° Ρ‡Π΅ΠΌ большС ΠΏΠ»ΠΎΡ‰Π°Π΄ΡŒ ΠΏΠΎΠΏΠ΅Ρ€Π΅Ρ‡Π½ΠΎΠ³ΠΎ сСчСния, Ρ‚Π΅ΠΌ мСньшС Π΅Π³ΠΎ сопротивлСниС; Ρ‡Π΅ΠΌ мСньшС ΠΏΠ»ΠΎΡ‰Π°Π΄ΡŒ ΠΏΠΎΠΏΠ΅Ρ€Π΅Ρ‡Π½ΠΎΠ³ΠΎ сСчСния, Ρ‚Π΅ΠΌ большС Π΅Π³ΠΎ сопротивлСниС.

АналогичноС сравнСниС ΠΌΠΎΠΆΠ½ΠΎ провСсти ΠΈ ΠΏΠΎ Π΄Π»ΠΈΠ½Π΅. Π’ΠΎΠ΄Ρƒ слоТнСС Ρ‚Π΅Ρ‡ΡŒ ΠΏΠΎ Π΄Π»ΠΈΠ½Π½ΠΎΠΌΡƒ ΡˆΠ»Π°Π½Π³Ρƒ просто ΠΏΠΎΡ‚ΠΎΠΌΡƒ, Ρ‡Ρ‚ΠΎ ΠΎΠ½Π° Π΄ΠΎΠ»ΠΆΠ½Π° Ρ‚Π΅Ρ‡ΡŒ дальшС. Π’ΠΎΡ‡Π½ΠΎ Ρ‚Π°ΠΊ ΠΆΠ΅ Ρ‚ΠΎΠΊΡƒ Ρ‚Ρ€ΡƒΠ΄Π½Π΅Π΅ ΠΏΡ€ΠΎΠΉΡ‚ΠΈ ΠΏΠΎ Π΄Π»ΠΈΠ½Π½ΠΎΠΌΡƒ ΠΏΡ€ΠΎΠ²ΠΎΠ΄Ρƒ, Ρ‡Π΅ΠΌ ΠΏΠΎ ΠΊΠΎΡ€ΠΎΡ‚ΠΊΠΎΠΌΡƒ.

УдСльноС сопротивлСниС — это свойство ΠΌΠ°Ρ‚Π΅Ρ€ΠΈΠ°Π»Π°, ΠΈΠ· ΠΊΠΎΡ‚ΠΎΡ€ΠΎΠ³ΠΎ ΠΈΠ·Π³ΠΎΡ‚ΠΎΠ²Π»Π΅Π½ сам ΠΏΡ€ΠΎΠ²ΠΎΠ΄, ΠΊΠΎΡ‚ΠΎΡ€ΠΎΠ΅ различаСтся ΠΎΡ‚ ΠΌΠ°Ρ‚Π΅Ρ€ΠΈΠ°Π»Π° ΠΊ ΠΌΠ°Ρ‚Π΅Ρ€ΠΈΠ°Π»Ρƒ. ΠŸΡ€Π΅Π΄ΡΡ‚Π°Π²ΡŒΡ‚Π΅, Ρ‡Ρ‚ΠΎ Π²Ρ‹ наполняСтС ΠΏΠΎΠΆΠ°Ρ€Π½Ρ‹ΠΉ шланг ΠΏΠ°Ρ‚ΠΎΠΊΠΎΠΉ, Π° Π½Π΅ Π²ΠΎΠ΄ΠΎΠΉ. МСласса Π±ΡƒΠ΄Π΅Ρ‚ Ρ‚Π΅Ρ‡ΡŒ ΠΌΠ΅Π΄Π»Π΅Π½Π½Π΅Π΅ просто ΠΈΠ·-Π·Π° Π΅Π΅ вязкости (липкости ΠΈΠ»ΠΈ сопротивлСния Ρ‚Π΅Ρ‡Π΅Π½ΠΈΡŽ). Π’ΠΎΡ‡Π½ΠΎ Ρ‚Π°ΠΊ ΠΆΠ΅ элСктричСский Ρ‚ΠΎΠΊ ΠΏΡ€ΠΎΡ…ΠΎΠ΄ΠΈΡ‚ Ρ‡Π΅Ρ€Π΅Π· Π½Π΅ΠΊΠΎΡ‚ΠΎΡ€Ρ‹Π΅ ΠΌΠ΅Ρ‚Π°Π»Π»Ρ‹ (Π½Π°ΠΏΡ€ΠΈΠΌΠ΅Ρ€, свинСц) с большим Ρ‚Ρ€ΡƒΠ΄ΠΎΠΌ, Ρ‡Π΅ΠΌ Ρ‡Π΅Ρ€Π΅Π· Π΄Ρ€ΡƒΠ³ΠΈΠ΅ ΠΌΠ΅Ρ‚Π°Π»Π»Ρ‹ (Π½Π°ΠΏΡ€ΠΈΠΌΠ΅Ρ€, сСрСбро).

ЭлСктричСскиС Ρ†Π΅ΠΏΠΈ

Π’ Π±ΠΎΠ»ΡŒΡˆΠΈΠ½ΡΡ‚Π²Π΅ случаСв ΠΏΡƒΡ‚ΡŒ, ΠΏΠΎ ΠΊΠΎΡ‚ΠΎΡ€ΠΎΠΌΡƒ ΠΏΡ€ΠΎΡ…ΠΎΠ΄ΠΈΡ‚ элСктричСский Ρ‚ΠΎΠΊ, извСстСн ΠΊΠ°ΠΊ элСктричСская Ρ†Π΅ΠΏΡŒ.Как ΠΌΠΈΠ½ΠΈΠΌΡƒΠΌ, схСма состоит ΠΈΠ· (1) источника элСктронов (Π½Π°ΠΏΡ€ΠΈΠΌΠ΅Ρ€, Π±Π°Ρ‚Π°Ρ€Π΅ΠΈ), ΠΊΠΎΡ‚ΠΎΡ€Ρ‹ΠΉ Π±ΡƒΠ΄Π΅Ρ‚ ΠΎΠ±Π΅ΡΠΏΠ΅Ρ‡ΠΈΠ²Π°Ρ‚ΡŒ Ρ€Π°Π·Π½ΠΎΡΡ‚ΡŒ ΠΏΠΎΡ‚Π΅Π½Ρ†ΠΈΠ°Π»ΠΎΠ², ΠΈ (2) ΠΏΡƒΡ‚ΠΈ, ΠΏΠΎ ΠΊΠΎΡ‚ΠΎΡ€ΠΎΠΌΡƒ ΠΌΠΎΠ³ΡƒΡ‚ ΠΏΠ΅Ρ€Π΅ΠΌΠ΅Ρ‰Π°Ρ‚ΡŒΡΡ элСктроны (Π½Π°ΠΏΡ€ΠΈΠΌΠ΅Ρ€, мСталличСской ΠΏΡ€ΠΎΠ²ΠΎΠ»ΠΎΠΊΠΈ). ВспомнитС, Ρ‡Ρ‚ΠΎ Ρ€Π°Π·Π½ΠΎΡΡ‚ΡŒ ΠΏΠΎΡ‚Π΅Π½Ρ†ΠΈΠ°Π»ΠΎΠ² (ΠΈΠ»ΠΈ напряТСниС) ΠΎΠ·Π½Π°Ρ‡Π°Π΅Ρ‚ Π±ΠΎΠ»ΡŒΡˆΡƒΡŽ силу элСктронов Π² ΠΎΠ΄Π½ΠΎΠΌ мСстС, Ρ‡Π΅ΠΌ Π² Π΄Ρ€ΡƒΠ³ΠΎΠΌ; эта большая сила Ρ‚ΠΎΠ»ΠΊΠ°Π΅Ρ‚ элСктроны ΠΊ мСсту с мСньшСй силой.

Для любого практичСского (ΠΈΠ»ΠΈ ΠΏΠΎΠ»Π΅Π·Π½ΠΎΠ³ΠΎ) примСнСния Ρ‚ΠΎΠΊ Ρ‚Π°ΠΊΠΆΠ΅ Ρ‚Ρ€Π΅Π±ΡƒΠ΅Ρ‚ (3) ΠΏΡ€ΠΈΠ±ΠΎΡ€Π°, Ρ€Π°Π±ΠΎΡ‚Π° ΠΊΠΎΡ‚ΠΎΡ€ΠΎΠ³ΠΎ зависит ΠΎΡ‚ протСкания элСктричСского Ρ‚ΠΎΠΊΠ°.К Ρ‚Π°ΠΊΠΈΠΌ ΠΏΡ€ΠΈΠ±ΠΎΡ€Π°ΠΌ относятся элСктричСскиС часы, тостСры, Ρ€Π°Π΄ΠΈΠΎ, Ρ‚Π΅Π»Π΅Π²ΠΈΠ·ΠΎΡ€Ρ‹ ΠΈ Ρ€Π°Π·Π»ΠΈΡ‡Π½Ρ‹Π΅ Ρ‚ΠΈΠΏΡ‹ элСктродвигатСлСй. Π’ΠΎ ΠΌΠ½ΠΎΠ³ΠΈΡ… случаях элСктричСскиС Ρ†Π΅ΠΏΠΈ Ρ‚Π°ΠΊΠΆΠ΅ содСрТат (4) ΠΊΠ°ΠΊΠΎΠΉ-Ρ‚ΠΎ ΠΈΠ·ΠΌΠ΅Ρ€ΠΈΡ‚Π΅Π»ΡŒ, ΠΊΠΎΡ‚ΠΎΡ€Ρ‹ΠΉ ΠΏΠΎΠΊΠ°Π·Ρ‹Π²Π°Π΅Ρ‚ Π²Π΅Π»ΠΈΡ‡ΠΈΠ½Ρƒ элСктричСского Ρ‚ΠΎΠΊΠ° ΠΈΠ»ΠΈ Ρ€Π°Π·Π½ΠΎΡΡ‚ΡŒ ΠΏΠΎΡ‚Π΅Π½Ρ†ΠΈΠ°Π»ΠΎΠ² Π² Ρ†Π΅ΠΏΠΈ. НаконСц, схСма, вСроятно, Π±ΡƒΠ΄Π΅Ρ‚ Π²ΠΊΠ»ΡŽΡ‡Π°Ρ‚ΡŒ (5) Ρ€Π°Π·Π»ΠΈΡ‡Π½Ρ‹Π΅ устройства для управлСния ΠΏΠΎΡ‚ΠΎΠΊΠΎΠΌ элСктричСского Ρ‚ΠΎΠΊΠ°, Ρ‚Π°ΠΊΠΈΠ΅ ΠΊΠ°ΠΊ выпрямитСли, трансформаторы, кондСнсаторы ΠΈ автоматичСскиС Π²Ρ‹ΠΊΠ»ΡŽΡ‡Π°Ρ‚Π΅Π»ΠΈ.

ΠŸΡ€ΠΈΠ±ΠΎΡ€Ρ‹ ΠΌΠΎΠΆΠ½ΠΎ Π²ΠΊΠ»ΡŽΡ‡Π°Ρ‚ΡŒ Π² ΡΠ»Π΅ΠΊΡ‚Ρ€ΠΈΡ‡Π΅ΡΠΊΡƒΡŽ Ρ†Π΅ΠΏΡŒ ΠΎΠ΄Π½ΠΈΠΌ ΠΈΠ· Π΄Π²ΡƒΡ… способов.Π’ ΠΏΠΎΡΠ»Π΅Π΄ΠΎΠ²Π°Ρ‚Π΅Π»ΡŒΠ½ΠΎΠΉ Ρ†Π΅ΠΏΠΈ Ρ‚ΠΎΠΊ Ρ‚Π΅Ρ‡Π΅Ρ‚ Ρ‡Π΅Ρ€Π΅Π· ΠΏΡ€ΠΈΠ±ΠΎΡ€Ρ‹ ΠΎΠ΄ΠΈΠ½ Π·Π° Π΄Ρ€ΡƒΠ³ΠΈΠΌ. Π’ ΠΏΠ°Ρ€Π°Π»Π»Π΅Π»ΡŒΠ½ΠΎΠΉ Ρ†Π΅ΠΏΠΈ входящий Ρ‚ΠΎΠΊ раздСляСтся ΠΈ пСрСдаСтся Ρ‡Π΅Ρ€Π΅Π· ΠΊΠ°ΠΆΠ΄ΡƒΡŽ ΠΎΡ‚Π΄Π΅Π»ΡŒΠ½ΡƒΡŽ Ρ†Π΅ΠΏΡŒ нСзависимо.

Π’Π°ΠΆΠ½Ρ‹ΠΌ прСимущСством ΠΏΠ°Ρ€Π°Π»Π»Π΅Π»ΡŒΠ½Ρ‹Ρ… Ρ†Π΅ΠΏΠ΅ΠΉ являСтся ΠΈΡ… ΡƒΡΡ‚ΠΎΠΉΡ‡ΠΈΠ²ΠΎΡΡ‚ΡŒ ΠΊ поврСТдСниям. ΠŸΡ€Π΅Π΄ΠΏΠΎΠ»ΠΎΠΆΠΈΠΌ, Ρ‡Ρ‚ΠΎ ΠΊΠ°ΠΊΠΎΠΉ-Π»ΠΈΠ±ΠΎ ΠΈΠ· ΠΏΡ€ΠΈΠ±ΠΎΡ€ΠΎΠ² Π² ΠΏΠΎΡΠ»Π΅Π΄ΠΎΠ²Π°Ρ‚Π΅Π»ΡŒΠ½ΠΎΠΉ Ρ†Π΅ΠΏΠΈ ΠΏΠΎΠ²Ρ€Π΅ΠΆΠ΄Π΅Π½, ΠΈ Ρ‚ΠΎΠΊ Π½Π΅ ΠΌΠΎΠΆΠ΅Ρ‚ Ρ‚Π΅Ρ‡ΡŒ Ρ‡Π΅Ρ€Π΅Π· Π½Π΅Π³ΠΎ. Π­Ρ‚ΠΎΡ‚ ΠΏΡ€ΠΎΠ±ΠΎΠΉ ΠΏΡ€Π΅Π΄ΠΎΡ‚Π²Ρ€Π°Ρ‰Π°Π΅Ρ‚ ΠΏΡ€ΠΎΡ‚Π΅ΠΊΠ°Π½ΠΈΠ΅ Ρ‚ΠΎΠΊΠ° Π² любом ΠΈΠ· ΠΏΡ€ΠΈΠ±ΠΎΡ€ΠΎΠ². ΠŸΡ€ΠΈ ΠΏΠ°Ρ€Π°Π»Π»Π΅Π»ΡŒΠ½ΠΎΠΉ схСмС Ρ‚Π°ΠΊΠΎΠΉ ΠΏΡ€ΠΎΠ±Π»Π΅ΠΌΡ‹ Π½Π΅ Π²ΠΎΠ·Π½ΠΈΠΊΠ°Π΅Ρ‚.Если ΠΎΠ΄Π½ΠΎ ΠΈΠ· устройств Π² ΠΏΠ°Ρ€Π°Π»Π»Π΅Π»ΡŒΠ½ΠΎΠΉ Ρ†Π΅ΠΏΠΈ Π²Ρ‹Ρ…ΠΎΠ΄ΠΈΡ‚ ΠΈΠ· строя, Ρ‚ΠΎΠΊ всС Ρ€Π°Π²Π½ΠΎ ΠΏΡ€ΠΎΠ΄ΠΎΠ»ΠΆΠ°Π΅Ρ‚ Ρ‚Π΅Ρ‡ΡŒ Ρ‡Π΅Ρ€Π΅Π· Π΄Ρ€ΡƒΠ³ΠΈΠ΅ устройства Π² Ρ†Π΅ΠΏΠΈ.

ΠŸΡ€ΠΈΠ½Ρ†ΠΈΠΏΠΈΠ°Π»ΡŒΠ½Π°Ρ матСматичСская Π·Π°Π²ΠΈΡΠΈΠΌΠΎΡΡ‚ΡŒ, ΡƒΠΏΡ€Π°Π²Π»ΡΡŽΡ‰Π°Ρ ΠΏΡ€ΠΎΡ‚Π΅ΠΊΠ°Π½ΠΈΠ΅ΠΌ элСктричСского Ρ‚ΠΎΠΊΠ° Π² Ρ†Π΅ΠΏΠΈ, Π±Ρ‹Π»Π° ΠΎΠ±Π½Π°Ρ€ΡƒΠΆΠ΅Π½Π° Омом Π² 1827 Π³ΠΎΠ΄Ρƒ. Π—Π°ΠΊΠΎΠ½ Ома гласит, Ρ‡Ρ‚ΠΎ Π²Π΅Π»ΠΈΡ‡ΠΈΠ½Π° Ρ‚ΠΎΠΊΠ° (i) Π² Ρ†Π΅ΠΏΠΈ Π½Π°ΠΏΡ€ΡΠΌΡƒΡŽ связана с Ρ€Π°Π·Π½ΠΎΡΡ‚ΡŒΡŽ ΠΏΠΎΡ‚Π΅Π½Ρ†ΠΈΠ°Π»ΠΎΠ² (V) ΠΈ ΠΎΠ±Ρ€Π°Ρ‚Π½ΠΎ ΠΏΡ€ΠΎΠΏΠΎΡ€Ρ†ΠΈΠΎΠ½Π°Π»ΡŒΠ½Π° сопротивлСниС (r) Π² Ρ†Π΅ΠΏΠΈ. Π”Ρ€ΡƒΠ³ΠΈΠΌΠΈ словами, i = V / r. Π—Π°ΠΊΠΎΠ½ Ома гласит, Ρ‡Ρ‚ΠΎ ΡƒΠ²Π΅Π»ΠΈΡ‡Π΅Π½ΠΈΠ΅ разности ΠΏΠΎΡ‚Π΅Π½Ρ†ΠΈΠ°Π»ΠΎΠ²

ΠΈΠ»ΠΈ ΡƒΠΌΠ΅Π½ΡŒΡˆΠ΅Π½ΠΈΠ΅ сопротивлСния ΠΏΡ€ΠΈΠ²ΠΎΠ΄ΠΈΡ‚ ΠΊ ΡƒΠ²Π΅Π»ΠΈΡ‡Π΅Π½ΠΈΡŽ Ρ‚ΠΎΠΊΠ°.И Π½Π°ΠΎΠ±ΠΎΡ€ΠΎΡ‚, ΡƒΠΌΠ΅Π½ΡŒΡˆΠ΅Π½ΠΈΠ΅ разности ΠΏΠΎΡ‚Π΅Π½Ρ†ΠΈΠ°Π»ΠΎΠ² ΠΈΠ»ΠΈ ΡƒΠ²Π΅Π»ΠΈΡ‡Π΅Π½ΠΈΠ΅ сопротивлСния ΠΏΡ€ΠΈΠ²ΠΎΠ΄ΠΈΡ‚ ΠΊ ΡƒΠΌΠ΅Π½ΡŒΡˆΠ΅Π½ΠΈΡŽ Ρ‚ΠΎΠΊΠ°. Π§Π΅ΠΌ слоТнСС становится элСктричСская Ρ†Π΅ΠΏΡŒ, Ρ‚Π΅ΠΌ Ρ‚Ρ€ΡƒΠ΄Π½Π΅Π΅ становится ΠΏΡ€ΠΈΠΌΠ΅Π½ΠΈΡ‚ΡŒ Π·Π°ΠΊΠΎΠ½ Ома.

ΠŸΠΎΡ‚ΠΎΠΊ Ρ‚ΠΎΠΊΠ° ΠΈ ΠΏΠΎΡ‚ΠΎΠΊ элСктронов

ΠžΠ±Π»Π°ΡΡ‚ΡŒ элСктротСхники ΠΎΠ±Ρ€Π΅ΠΌΠ΅Π½Π΅Π½Π° странной ΠΏΡ€ΠΎΠ±Π»Π΅ΠΌΠΎΠΉ, которая Π²ΠΎΠ·Π½ΠΈΠΊΠ»Π° Π±ΠΎΠ»Π΅Π΅ 200 Π»Π΅Ρ‚ Π½Π°Π·Π°Π΄. Когда ΡƒΡ‡Π΅Π½Ρ‹Π΅ Π²ΠΏΠ΅Ρ€Π²Ρ‹Π΅ ΠΈΠ·ΡƒΡ‡ΠΈΠ»ΠΈ ΠΏΠΎΡ‚ΠΎΠΊ элСктричСского Ρ‚ΠΎΠΊΠ° ΠΈΠ· ΠΎΠ΄Π½ΠΎΠ³ΠΎ мСста Π² Π΄Ρ€ΡƒΠ³ΠΎΠ΅, ΠΎΠ½ΠΈ считали, Ρ‡Ρ‚ΠΎ этот ΠΏΠΎΡ‚ΠΎΠΊ создаСтся Π΄Π²ΠΈΠΆΠ΅Π½ΠΈΠ΅ΠΌ ΠΊΡ€ΠΎΡˆΠ΅Ρ‡Π½Ρ‹Ρ… частиц.ΠŸΠΎΡΠΊΠΎΠ»ΡŒΠΊΡƒ элСктрон Π΅Ρ‰Π΅ Π½Π΅ Π±Ρ‹Π» ΠΎΠ±Π½Π°Ρ€ΡƒΠΆΠ΅Π½, ΠΎΠ½ΠΈ ΠΏΡ€Π΅Π΄ΠΏΠΎΠ»ΠΎΠΆΠΈΠ»ΠΈ, Ρ‡Ρ‚ΠΎ эти частицы нСсут ΠΏΠΎΠ»ΠΎΠΆΠΈΡ‚Π΅Π»ΡŒΠ½Ρ‹ΠΉ заряд.

БСгодня ΠΌΡ‹ Π·Π½Π°Π΅ΠΌ ΠΈΠ½Π°Ρ‡Π΅. ЭлСктричСский Ρ‚ΠΎΠΊ — это ΠΏΠΎΡ‚ΠΎΠΊ ΠΎΡ‚Ρ€ΠΈΡ†Π°Ρ‚Π΅Π»ΡŒΠ½ΠΎ заряТСнных частиц: элСктронов. Но ΠΎΠ±Ρ‹Ρ‡Π°ΠΉ ΠΎΡ‚ΠΎΠ±Ρ€Π°ΠΆΠ°Ρ‚ΡŒ элСктричСский Ρ‚ΠΎΠΊ ΠΊΠ°ΠΊ ΠΏΠΎΠ»ΠΎΠΆΠΈΡ‚Π΅Π»ΡŒΠ½Ρ‹ΠΉ сущСствуСт ΡƒΠΆΠ΅ Π΄Π°Π²Π½ΠΎ ΠΈ Π΄ΠΎ сих ΠΏΠΎΡ€ ΡˆΠΈΡ€ΠΎΠΊΠΎ ΠΈΡΠΏΠΎΠ»ΡŒΠ·ΡƒΠ΅Ρ‚ΡΡ. По этой ΠΏΡ€ΠΈΡ‡ΠΈΠ½Π΅ Π½Π΅Ρ€Π΅Π΄ΠΊΠΎ ΠΌΠΎΠΆΠ½ΠΎ ΡƒΠ²ΠΈΠ΄Π΅Ρ‚ΡŒ элСктричСский Ρ‚ΠΎΠΊ Π² Π²ΠΈΠ΄Π΅ ΠΏΠΎΡ‚ΠΎΠΊΠ° ΠΏΠΎΠ»ΠΎΠΆΠΈΡ‚Π΅Π»ΡŒΠ½Ρ‹Ρ… зарядов, хотя ΠΌΡ‹ ΡƒΠΆΠ΅ Π΄Π°Π²Π½ΠΎ Π·Π½Π°Π΅ΠΌ это Π»ΡƒΡ‡ΡˆΠ΅.

Π’ΠΈΠΏΠΎΠΌ элСктричСского Ρ‚ΠΎΠΊΠ°, описанного Π΄ΠΎ сих ΠΏΠΎΡ€, являСтся постоянный Ρ‚ΠΎΠΊ (DC).ΠŸΠΎΡΡ‚ΠΎΡΠ½Π½Ρ‹ΠΉ Ρ‚ΠΎΠΊ всСгда ΠΏΡ€Π΅Π΄ΠΏΠΎΠ»Π°Π³Π°Π΅Ρ‚ Π΄Π²ΠΈΠΆΠ΅Π½ΠΈΠ΅ элСктронов ΠΈΠ· области с высоким ΠΎΡ‚Ρ€ΠΈΡ†Π°Ρ‚Π΅Π»ΡŒΠ½Ρ‹ΠΌ зарядом Π² ΠΎΠ±Π»Π°ΡΡ‚ΡŒ с Π±ΠΎΠ»Π΅Π΅ Π½ΠΈΠ·ΠΊΠΈΠΌ ΠΎΡ‚Ρ€ΠΈΡ†Π°Ρ‚Π΅Π»ΡŒΠ½Ρ‹ΠΌ зарядом. ЭлСктричСский Ρ‚ΠΎΠΊ, ΠΏΡ€ΠΎΠΈΠ·Π²ΠΎΠ΄ΠΈΠΌΡ‹ΠΉ батарСями, — это постоянный Ρ‚ΠΎΠΊ.

Π˜Π½Ρ‚Π΅Ρ€Π΅ΡΠ½ΠΎ, Ρ‡Ρ‚ΠΎ ΠΏΠΎΠ΄Π°Π²Π»ΡΡŽΡ‰Π΅Π΅ Π±ΠΎΠ»ΡŒΡˆΠΈΠ½ΡΡ‚Π²ΠΎ элСктричСского Ρ‚ΠΎΠΊΠ°, ΠΈΡΠΏΠΎΠ»ΡŒΠ·ΡƒΠ΅ΠΌΠΎΠ³ΠΎ Π² практичСских цСлях, — это ΠΏΠ΅Ρ€Π΅ΠΌΠ΅Π½Π½Ρ‹ΠΉ Ρ‚ΠΎΠΊ (AC). ΠŸΠ΅Ρ€Π΅ΠΌΠ΅Π½Π½Ρ‹ΠΉ Ρ‚ΠΎΠΊ — это Ρ‚ΠΎΠΊ, ΠΊΠΎΡ‚ΠΎΡ€Ρ‹ΠΉ ΠΎΡ‡Π΅Π½ΡŒ быстро мСняСт Π½Π°ΠΏΡ€Π°Π²Π»Π΅Π½ΠΈΠ΅ своСго протСкания. Π’ Π‘Π΅Π²Π΅Ρ€Π½ΠΎΠΉ АмСрикС, Π½Π°ΠΏΡ€ΠΈΠΌΠ΅Ρ€, коммСрчСскиС Π»ΠΈΠ½ΠΈΠΈ элСктропСрСдач Ρ€Π°Π±ΠΎΡ‚Π°ΡŽΡ‚ с частотой 60 Π³Π΅Ρ€Ρ†.(Π“Π΅Ρ€Ρ† — это Π΅Π΄ΠΈΠ½ΠΈΡ†Π° измСрСния частоты.) Π’ Π»ΠΈΠ½ΠΈΠΈ с частотой 60 Π“Ρ† Ρ‚ΠΎΠΊ мСняСт своС Π½Π°ΠΏΡ€Π°Π²Π»Π΅Π½ΠΈΠ΅ 60 Ρ€Π°Π· ΠΊΠ°ΠΆΠ΄ΡƒΡŽ сСкунду.

Π”Ρ€ΡƒΠ³ΠΈΠ΅ Π²ΠΈΠ΄Ρ‹ ΠΏΠ΅Ρ€Π΅ΠΌΠ΅Π½Π½ΠΎΠ³ΠΎ Ρ‚ΠΎΠΊΠ° Ρ‚Π°ΠΊΠΆΠ΅ ΡˆΠΈΡ€ΠΎΠΊΠΎ ΠΈΡΠΏΠΎΠ»ΡŒΠ·ΡƒΡŽΡ‚ΡΡ. Π—Π° ΠΏΡ€Π΅Π΄Π΅Π»Π°ΠΌΠΈ Π‘Π΅Π²Π΅Ρ€Π½ΠΎΠΉ АмСрики Ρ‡Π°Ρ‰Π΅ встрСчаСтся линия элСктропСрСдачи Π½Π° 50 Π³Π΅Ρ€Ρ†. А Π² самолСтах ΠΏΠ΅Ρ€Π΅ΠΌΠ΅Π½Π½Ρ‹ΠΉ Ρ‚ΠΎΠΊ ΠΎΠ±Ρ‹Ρ‡Π½ΠΎ составляСт 400 Π³Π΅Ρ€Ρ†.

[ Π‘ΠΌ. Π’Π°ΠΊΠΆΠ΅ ЭлСктричСство; Π­Π»Π΅ΠΊΡ‚Ρ€ΠΎΠ΄Π²ΠΈΠ³Π°Ρ‚Π΅Π»ΡŒ ]

Π§Ρ‚ΠΎ Ρ‚Π°ΠΊΠΎΠ΅ элСктричСский Ρ‚ΠΎΠΊ Β»Π­Π»Π΅ΠΊΡ‚Ρ€ΠΎΠ½ΠΈΠΊΠ°

ЭлСктричСский Ρ‚ΠΎΠΊ Π²ΠΎΠ·Π½ΠΈΠΊΠ°Π΅Ρ‚ ΠΏΡ€ΠΈ Π΄Π²ΠΈΠΆΠ΅Π½ΠΈΠΈ элСктричСских зарядов — это ΠΌΠΎΠ³ΡƒΡ‚ Π±Ρ‹Ρ‚ΡŒ ΠΎΡ‚Ρ€ΠΈΡ†Π°Ρ‚Π΅Π»ΡŒΠ½ΠΎ заряТСнныС элСктроны ΠΈΠ»ΠΈ ΠΏΠΎΠ»ΠΎΠΆΠΈΡ‚Π΅Π»ΡŒΠ½Ρ‹Π΅ носитСли заряда — ΠΏΠΎΠ»ΠΎΠΆΠΈΡ‚Π΅Π»ΡŒΠ½Ρ‹Π΅ ΠΈΠΎΠ½Ρ‹.


Π£Ρ‡Π΅Π±Π½ΠΎΠ΅ пособиС ΠΏΠΎ элСктричСскому Ρ‚ΠΎΠΊΡƒ Π’ΠΊΠ»ΡŽΡ‡Π°Π΅Ρ‚:
Π§Ρ‚ΠΎ Ρ‚Π°ΠΊΠΎΠ΅ элСктричСский Ρ‚ΠΎΠΊ Π•Π΄ΠΈΠ½ΠΈΡ†Π° измСрСния Ρ‚ΠΎΠΊΠ° — АмпСр ΠŸΠ•Π Π•ΠœΠ•ΠΠΠ«Π™ ВОК


ЭлСктричСский Ρ‚ΠΎΠΊ — ΠΎΠ΄Π½ΠΎ ΠΈΠ· самых основных понятий, ΡΡƒΡ‰Π΅ΡΡ‚Π²ΡƒΡŽΡ‰ΠΈΡ… Π² элСктротСхникС ΠΈ элСктроникС. ЭлСктричСский Ρ‚ΠΎΠΊ Π»Π΅ΠΆΠΈΡ‚ Π² основС Π½Π°ΡƒΠΊΠΈ ΠΎΠ± элСктричСствС.

Π‘ΡƒΠ΄ΡŒ Ρ‚ΠΎ элСктричСский Π½Π°Π³Ρ€Π΅Π²Π°Ρ‚Π΅Π»ΡŒ, большая ΡΠ»Π΅ΠΊΡ‚Ρ€ΠΎΡΠ΅Ρ‚ΡŒ, ΠΌΠΎΠ±ΠΈΠ»ΡŒΠ½Ρ‹ΠΉ Ρ‚Π΅Π»Π΅Ρ„ΠΎΠ½, ΠΊΠΎΠΌΠΏΡŒΡŽΡ‚Π΅Ρ€, ΡƒΠ΄Π°Π»Π΅Π½Π½Ρ‹ΠΉ сСнсорный ΡƒΠ·Π΅Π» ΠΈΠ»ΠΈ Ρ‡Ρ‚ΠΎ-Ρ‚ΠΎ Π΅Ρ‰Π΅, понятиС элСктричСского Ρ‚ΠΎΠΊΠ° являСтся Ρ†Π΅Π½Ρ‚Ρ€Π°Π»ΡŒΠ½Ρ‹ΠΌ для Π΅Π³ΠΎ Ρ€Π°Π±ΠΎΡ‚Ρ‹.

Однако Ρ‚ΠΎΠΊ ΠΊΠ°ΠΊ Ρ‚Π°ΠΊΠΎΠ²ΠΎΠΉ ΠΎΠ±Ρ‹Ρ‡Π½ΠΎ нСльзя ΡƒΠ²ΠΈΠ΄Π΅Ρ‚ΡŒ, хотя Π΅Π³ΠΎ эффСкты ΠΌΠΎΠΆΠ½ΠΎ ΡƒΠ²ΠΈΠ΄Π΅Ρ‚ΡŒ, ΡƒΡΠ»Ρ‹ΡˆΠ°Ρ‚ΡŒ ΠΈ ΠΏΠΎΡ‡ΡƒΠ²ΡΡ‚Π²ΠΎΠ²Π°Ρ‚ΡŒ всС врСмя, ΠΈ Π² Ρ€Π΅Π·ΡƒΠ»ΡŒΡ‚Π°Ρ‚Π΅ ΠΈΠ½ΠΎΠ³Π΄Π° Ρ‚Ρ€ΡƒΠ΄Π½ΠΎ ΠΏΠΎΠ»ΡƒΡ‡ΠΈΡ‚ΡŒ прСдставлСниС ΠΎ Ρ‚ΠΎΠΌ, Ρ‡Ρ‚ΠΎ это Ρ‚Π°ΠΊΠΎΠ΅ Π½Π° самом Π΄Π΅Π»Π΅.

Π£Π΄Π°Ρ€ ΠΌΠΎΠ»Π½ΠΈΠΈ — Π²ΠΏΠ΅Ρ‡Π°Ρ‚Π»ΡΡŽΡ‰Π΅Π΅ Π·Ρ€Π΅Π»ΠΈΡ‰Π΅ элСктричСского Ρ‚ΠΎΠΊΠ°
Ѐотография сдСлана с Π²Π΅Ρ€ΡˆΠΈΠ½Ρ‹ башСн ΠŸΠ΅Ρ‚Ρ€ΠΎΠ½Π°Ρ Π² ΠšΡƒΠ°Π»Π°-Π›ΡƒΠΌΠΏΡƒΡ€Π΅ Малайзия

ΠžΠΏΡ€Π΅Π΄Π΅Π»Π΅Π½ΠΈΠ΅ элСктричСского Ρ‚ΠΎΠΊΠ°

ΠžΠΏΡ€Π΅Π΄Π΅Π»Π΅Π½ΠΈΠ΅ элСктричСского Ρ‚ΠΎΠΊΠ°:

ЭлСктричСский Ρ‚ΠΎΠΊ — это ΠΏΠΎΡ‚ΠΎΠΊ элСктричСского заряда Π² Ρ†Π΅ΠΏΠΈ.Π‘ΠΎΠ»Π΅Π΅ ΠΊΠΎΠ½ΠΊΡ€Π΅Ρ‚Π½ΠΎ, элСктричСский Ρ‚ΠΎΠΊ — это ΡΠΊΠΎΡ€ΠΎΡΡ‚ΡŒ прохоТдСния заряда Ρ‡Π΅Ρ€Π΅Π· Π·Π°Π΄Π°Π½Π½ΡƒΡŽ Ρ‚ΠΎΡ‡ΠΊΡƒ Π² элСктричСской Ρ†Π΅ΠΏΠΈ. Заряд ΠΌΠΎΠΆΠ΅Ρ‚ Π±Ρ‹Ρ‚ΡŒ ΠΎΡ‚Ρ€ΠΈΡ†Π°Ρ‚Π΅Π»ΡŒΠ½ΠΎ заряТСнными элСктронами ΠΈΠ»ΠΈ ΠΏΠΎΠ»ΠΎΠΆΠΈΡ‚Π΅Π»ΡŒΠ½Ρ‹ΠΌΠΈ носитСлями заряда, Π²ΠΊΠ»ΡŽΡ‡Π°Ρ ΠΏΡ€ΠΎΡ‚ΠΎΠ½Ρ‹, ΠΏΠΎΠ»ΠΎΠΆΠΈΡ‚Π΅Π»ΡŒΠ½Ρ‹Π΅ ΠΈΠΎΠ½Ρ‹ ΠΈΠ»ΠΈ Π΄Ρ‹Ρ€ΠΊΠΈ.

Π’Π΅Π»ΠΈΡ‡ΠΈΠ½Π° элСктричСского Ρ‚ΠΎΠΊΠ° измСряСтся Π² ΠΊΡƒΠ»ΠΎΠ½Π°Ρ… Π² сСкунду, ΠΎΠ±Ρ‹Ρ‡Π½ΠΎ Π΅Π΄ΠΈΠ½ΠΈΡ†Π΅ΠΉ измСрСния являСтся Π°ΠΌΠΏΠ΅Ρ€ ΠΈΠ»ΠΈ Π°ΠΌΠΏΠ΅Ρ€, ΠΎΠ±ΠΎΠ·Π½Π°Ρ‡Π°Π΅ΠΌΡ‹ΠΉ Π±ΡƒΠΊΠ²ΠΎΠΉ «А».

АмпСр ΠΈΠ»ΠΈ ΡƒΡΠΈΠ»ΠΈΡ‚Π΅Π»ΡŒ ΡˆΠΈΡ€ΠΎΠΊΠΎ ΠΈΡΠΏΠΎΠ»ΡŒΠ·ΡƒΠ΅Ρ‚ΡΡ Π² элСктричСских ΠΈ элСктронных тСхнологиях вмСстС с умноТитСлями, Ρ‚Π°ΠΊΠΈΠΌΠΈ ΠΊΠ°ΠΊ ΠΌΠΈΠ»Π»ΠΈΠ°ΠΌΠΏΠ΅Ρ€ (0.001A), ΠΌΠΈΠΊΡ€ΠΎΠ°ΠΌΠΏΠ΅Ρ€ (0,000001A) ΠΈ Ρ‚. Π”.

Π’ΠΎΠΊ Π² Ρ†Π΅ΠΏΠΈ ΠΎΠ±Ρ‹Ρ‡Π½ΠΎ обозначаСтся Π±ΡƒΠΊΠ²ΠΎΠΉ Β«IΒ», ΠΈ эта Π±ΡƒΠΊΠ²Π° ΠΈΡΠΏΠΎΠ»ΡŒΠ·ΡƒΠ΅Ρ‚ΡΡ Π² уравнСниях, Ρ‚Π°ΠΊΠΈΡ… ΠΊΠ°ΠΊ Π·Π°ΠΊΠΎΠ½ Ома, Π³Π΄Π΅ V = Iβ‹…R.

Π§Ρ‚ΠΎ Ρ‚Π°ΠΊΠΎΠ΅ элСктричСский Ρ‚ΠΎΠΊ: основы

Основная концСпция Ρ‚ΠΎΠΊΠ° состоит Π² Ρ‚ΠΎΠΌ, Ρ‡Ρ‚ΠΎ это Π΄Π²ΠΈΠΆΠ΅Π½ΠΈΠ΅ элСктронов Π²Π½ΡƒΡ‚Ρ€ΠΈ вСщСства. Π­Π»Π΅ΠΊΡ‚Ρ€ΠΎΠ½Ρ‹ — это ΠΌΠ΅Π»ΡŒΡ‡Π°ΠΉΡˆΠΈΠ΅ частицы, ΠΊΠΎΡ‚ΠΎΡ€Ρ‹Π΅ ΡΡƒΡ‰Π΅ΡΡ‚Π²ΡƒΡŽΡ‚ ΠΊΠ°ΠΊ Ρ‡Π°ΡΡ‚ΡŒ молСкулярной структуры ΠΌΠ°Ρ‚Π΅Ρ€ΠΈΠ°Π»ΠΎΠ². Иногда эти элСктроны ΠΏΠ»ΠΎΡ‚Π½ΠΎ ΡƒΠ΄Π΅Ρ€ΠΆΠΈΠ²Π°ΡŽΡ‚ΡΡ Π²Π½ΡƒΡ‚Ρ€ΠΈ ΠΌΠΎΠ»Π΅ΠΊΡƒΠ», Π° ΠΈΠ½ΠΎΠ³Π΄Π° ΠΎΠ½ΠΈ ΡƒΠ΄Π΅Ρ€ΠΆΠΈΠ²Π°ΡŽΡ‚ΡΡ свободно, ΠΈ ΠΎΠ½ΠΈ ΠΌΠΎΠ³ΡƒΡ‚ ΠΎΡ‚Π½ΠΎΡΠΈΡ‚Π΅Π»ΡŒΠ½ΠΎ свободно ΠΏΠ΅Ρ€Π΅ΠΌΠ΅Ρ‰Π°Ρ‚ΡŒΡΡ ΠΏΠΎ структурС.

Одно ΠΎΡ‡Π΅Π½ΡŒ Π²Π°ΠΆΠ½ΠΎΠ΅ Π·Π°ΠΌΠ΅Ρ‡Π°Π½ΠΈΠ΅ ΠΎΡ‚Π½ΠΎΡΠΈΡ‚Π΅Π»ΡŒΠ½ΠΎ элСктронов — это Ρ‚ΠΎ, Ρ‡Ρ‚ΠΎ ΠΎΠ½ΠΈ заряТСнныС частицы — ΠΎΠ½ΠΈ нСсут ΠΎΡ‚Ρ€ΠΈΡ†Π°Ρ‚Π΅Π»ΡŒΠ½Ρ‹ΠΉ заряд. Если ΠΎΠ½ΠΈ ΠΏΠ΅Ρ€Π΅ΠΌΠ΅Ρ‰Π°ΡŽΡ‚ΡΡ, Ρ‚ΠΎ пСрСмСщаСтся количСство заряда, ΠΈ это называСтся Ρ‚ΠΎΠΊΠΎΠΌ.

Π’Π°ΠΊΠΆΠ΅ стоит ΠΎΡ‚ΠΌΠ΅Ρ‚ΠΈΡ‚ΡŒ, Ρ‡Ρ‚ΠΎ количСство элСктронов, ΠΊΠΎΡ‚ΠΎΡ€Ρ‹Π΅ ΠΌΠΎΠ³ΡƒΡ‚ Π΄Π²ΠΈΠ³Π°Ρ‚ΡŒΡΡ, опрСдСляСт ΡΠΏΠΎΡΠΎΠ±Π½ΠΎΡΡ‚ΡŒ ΠΊΠΎΠ½ΠΊΡ€Π΅Ρ‚Π½ΠΎΠ³ΠΎ вСщСства ΠΏΡ€ΠΎΠ²ΠΎΠ΄ΠΈΡ‚ΡŒ элСктричСство. НСкоторыС ΠΌΠ°Ρ‚Π΅Ρ€ΠΈΠ°Π»Ρ‹ ΠΏΠΎΠ·Π²ΠΎΠ»ΡΡŽΡ‚ Ρ‚ΠΎΠΊΡƒ Π΄Π²ΠΈΠ³Π°Ρ‚ΡŒΡΡ Π»ΡƒΡ‡ΡˆΠ΅, Ρ‡Π΅ΠΌ Π΄Ρ€ΡƒΠ³ΠΈΠ΅.

Π”Π²ΠΈΠΆΠ΅Π½ΠΈΠ΅ свободных элСктронов ΠΎΠ±Ρ‹Ρ‡Π½ΠΎ ΠΎΡ‡Π΅Π½ΡŒ случайноС — ΠΎΠ½ΠΎ случайноС — ΡΡ‚ΠΎΠ»ΡŒΠΊΠΎ элСктронов двиТСтся ΠΊΠ°ΠΊ Π² ΠΎΠ΄Π½ΠΎΠΌ Π½Π°ΠΏΡ€Π°Π²Π»Π΅Π½ΠΈΠΈ, Ρ‚Π°ΠΊ ΠΈ Π² Π΄Ρ€ΡƒΠ³ΠΎΠΌ, ΠΈ Π² Ρ€Π΅Π·ΡƒΠ»ΡŒΡ‚Π°Ρ‚Π΅ отсутствуСт ΠΎΠ±Ρ‰Π΅Π΅ Π΄Π²ΠΈΠΆΠ΅Π½ΠΈΠ΅ заряда.

Π‘Π»ΡƒΡ‡Π°ΠΉΠ½ΠΎΠ΅ Π΄Π²ΠΈΠΆΠ΅Π½ΠΈΠ΅ элСктронов Π² ΠΏΡ€ΠΎΠ²ΠΎΠ΄Π½ΠΈΠΊΠ΅ со свободными элСктронами

Если Π½Π° элСктроны дСйствуСт сила, ΠΏΠ΅Ρ€Π΅ΠΌΠ΅Ρ‰Π°ΡŽΡ‰Π°Ρ ΠΈΡ… Π² ΠΎΠΏΡ€Π΅Π΄Π΅Π»Π΅Π½Π½ΠΎΠΌ Π½Π°ΠΏΡ€Π°Π²Π»Π΅Π½ΠΈΠΈ, Ρ‚ΠΎ всС ΠΎΠ½ΠΈ Π±ΡƒΠ΄ΡƒΡ‚ Π΄Ρ€Π΅ΠΉΡ„ΠΎΠ²Π°Ρ‚ΡŒ Π² ΠΎΠ΄Π½ΠΎΠΌ ΠΈ Ρ‚ΠΎΠΌ ΠΆΠ΅ Π½Π°ΠΏΡ€Π°Π²Π»Π΅Π½ΠΈΠΈ, хотя ΠΈ Π² Π½Π΅ΠΊΠΎΡ‚ΠΎΡ€ΠΎΠΉ стСпСни случайным ΠΎΠ±Ρ€Π°Π·ΠΎΠΌ, Π½ΠΎ Π² Ρ†Π΅Π»ΠΎΠΌ Π΄Π²ΠΈΠΆΠ΅Π½ΠΈΠ΅ происходит Π² ΠΎΠ΄Π½ΠΎΠΌ Π½Π°ΠΏΡ€Π°Π²Π»Π΅Π½ΠΈΠΈ. Одно Π½Π°ΠΏΡ€Π°Π²Π»Π΅Π½ΠΈΠ΅.

Π‘ΠΈΠ»Π°, Π΄Π΅ΠΉΡΡ‚Π²ΡƒΡŽΡ‰Π°Ρ Π½Π° элСктроны, называСтся элСктродвиТущСй силой ΠΈΠ»ΠΈ Π­Π”Π‘, Π° Π΅Π΅ Π²Π΅Π»ΠΈΡ‡ΠΈΠ½Π° — это напряТСниС, измСряСмоС Π² Π²ΠΎΠ»ΡŒΡ‚Π°Ρ….

Π­Π»Π΅ΠΊΡ‚Ρ€ΠΎΠ½Π½Ρ‹ΠΉ ΠΏΠΎΡ‚ΠΎΠΊ ΠΏΠΎΠ΄ дСйствиСм ΠΏΡ€ΠΈΠ»ΠΎΠΆΠ΅Π½Π½ΠΎΠΉ элСктродвиТущСй силы

Π§Ρ‚ΠΎΠ±Ρ‹ Π½Π΅ΠΌΠ½ΠΎΠ³ΠΎ Π»ΡƒΡ‡ΡˆΠ΅ ΠΏΠΎΠ½ΡΡ‚ΡŒ, Ρ‡Ρ‚ΠΎ Ρ‚Π°ΠΊΠΎΠ΅ Ρ‚ΠΎΠΊ ΠΈ ΠΊΠ°ΠΊ ΠΎΠ½ дСйствуСт Π² ΠΏΡ€ΠΎΠ²ΠΎΠ΄Π½ΠΈΠΊΠ΅, Π΅Π³ΠΎ ΠΌΠΎΠΆΠ½ΠΎ ΡΡ€Π°Π²Π½ΠΈΡ‚ΡŒ с ΠΏΠΎΡ‚ΠΎΠΊΠΎΠΌ Π²ΠΎΠ΄Ρ‹ Π² Ρ‚Ρ€ΡƒΠ±Π΅.Π£ этого сравнСния Π΅ΡΡ‚ΡŒ ограничСния, Π½ΠΎ ΠΎΠ½ΠΎ слуТит ΠΎΡ‡Π΅Π½ΡŒ простой ΠΈΠ»Π»ΡŽΡΡ‚Ρ€Π°Ρ†ΠΈΠ΅ΠΉ Ρ‚ΠΎΠΊΠ° ΠΈ протСкания Ρ‚ΠΎΠΊΠ°.

Π’ΠΎΠΊ ΠΌΠΎΠΆΠ½ΠΎ Ρ€Π°ΡΡΠΌΠ°Ρ‚Ρ€ΠΈΠ²Π°Ρ‚ΡŒ ΠΊΠ°ΠΊ Π²ΠΎΠ΄Ρƒ, Ρ‚Π΅ΠΊΡƒΡ‰ΡƒΡŽ ΠΏΠΎ Ρ‚Ρ€ΡƒΠ±Π΅. Когда Π΄Π°Π²Π»Π΅Π½ΠΈΠ΅ оказываСтся Π½Π° ΠΎΠ΄ΠΈΠ½ ΠΊΠΎΠ½Π΅Ρ†, Π²ΠΎΠ΄Π° двиТСтся Π² ΠΎΠ΄Π½ΠΎΠΌ Π½Π°ΠΏΡ€Π°Π²Π»Π΅Π½ΠΈΠΈ ΠΈ Ρ‚Π΅Ρ‡Π΅Ρ‚ ΠΏΠΎ Ρ‚Ρ€ΡƒΠ±Π΅. ΠšΠΎΠ»ΠΈΡ‡Π΅ΡΡ‚Π²ΠΎ Π²ΠΎΠ΄Ρ‹ ΠΏΡ€ΠΎΠΏΠΎΡ€Ρ†ΠΈΠΎΠ½Π°Π»ΡŒΠ½ΠΎ давлСнию Π½Π° ΠΊΠΎΠ½Ρ†Π΅. Π”Π°Π²Π»Π΅Π½ΠΈΠ΅ ΠΈΠ»ΠΈ силу, ΠΏΡ€ΠΈΠ»ΠΎΠΆΠ΅Π½Π½ΡƒΡŽ ΠΊ ΠΊΠΎΠ½Ρ†Ρƒ, ΠΌΠΎΠΆΠ½ΠΎ ΡΡ€Π°Π²Π½ΠΈΡ‚ΡŒ с элСктродвиТущСй силой.

Когда ΠΊ Ρ‚Ρ€ΡƒΠ±Π΅ прикладываСтся Π΄Π°Π²Π»Π΅Π½ΠΈΠ΅ ΠΈΠ»ΠΈ Π²ΠΎΠ΄Π° Ρ‚Π΅Ρ‡Π΅Ρ‚ Π² Ρ€Π΅Π·ΡƒΠ»ΡŒΡ‚Π°Ρ‚Π΅ открытия ΠΊΡ€Π°Π½Π°, Π²ΠΎΠ΄Π° Ρ‚Π΅Ρ‡Π΅Ρ‚ практичСски ΠΌΠ³Π½ΠΎΠ²Π΅Π½Π½ΠΎ.Π’ΠΎ ΠΆΠ΅ самоС ΠΈ с элСктричСским Ρ‚ΠΎΠΊΠΎΠΌ.

Π§Ρ‚ΠΎΠ±Ρ‹ ΠΏΠΎΠ»ΡƒΡ‡ΠΈΡ‚ΡŒ прСдставлСниС ΠΎ ΠΏΠΎΡ‚ΠΎΠΊΠ΅ элСктронов, трСбуСтся 6,24 ΠΌΠΈΠ»Π»ΠΈΠ°Ρ€Π΄Π° ΠΌΠΈΠ»Π»ΠΈΠ°Ρ€Π΄ΠΎΠ² элСктронов Π² сСкунду для Ρ‚ΠΎΠΊΠ° Π² ΠΎΠ΄ΠΈΠ½ Π°ΠΌΠΏΠ΅Ρ€.

ΠžΠ±Ρ‹Ρ‡Π½Ρ‹ΠΉ Ρ‚ΠΎΠΊ ΠΈ ΠΏΠΎΡ‚ΠΎΠΊ элСктронов

Часто Π±Ρ‹Π²Π°Π΅Ρ‚ ΠΌΠ½ΠΎΠ³ΠΎ нСдопонимания ΠΎΡ‚Π½ΠΎΡΠΈΡ‚Π΅Π»ΡŒΠ½ΠΎ ΠΎΠ±Ρ‹Ρ‡Π½ΠΎΠ³ΠΎ ΠΏΠΎΡ‚ΠΎΠΊΠ° Ρ‚ΠΎΠΊΠ° ΠΈ ΠΏΠΎΡ‚ΠΎΠΊΠ° элСктронов. Π‘Π½Π°Ρ‡Π°Π»Π° это ΠΌΠΎΠΆΠ΅Ρ‚ Π½Π΅ΠΌΠ½ΠΎΠ³ΠΎ ΡΠ±ΠΈΠ²Π°Ρ‚ΡŒ с Ρ‚ΠΎΠ»ΠΊΡƒ, Π½ΠΎ Π½Π° самом Π΄Π΅Π»Π΅ всС довольно просто.

Частицы, пСрСносящиС заряд ΠΏΠΎ ΠΏΡ€ΠΎΠ²ΠΎΠ΄Π½ΠΈΠΊΠ°ΠΌ, ΡΠ²Π»ΡΡŽΡ‚ΡΡ свободными элСктронами.НаправлСниС элСктричСского поля Π² Ρ†Π΅ΠΏΠΈ ΠΏΠΎ ΠΎΠΏΡ€Π΅Π΄Π΅Π»Π΅Π½ΠΈΡŽ являСтся Π½Π°ΠΏΡ€Π°Π²Π»Π΅Π½ΠΈΠ΅ΠΌ, Π² ΠΊΠΎΡ‚ΠΎΡ€ΠΎΠΌ ΠΏΡ€ΠΎΡ‚Π°Π»ΠΊΠΈΠ²Π°ΡŽΡ‚ΡΡ ΠΏΠΎΠ»ΠΎΠΆΠΈΡ‚Π΅Π»ΡŒΠ½Ρ‹Π΅ ΠΈΡΠΏΡ‹Ρ‚Π°Ρ‚Π΅Π»ΡŒΠ½Ρ‹Π΅ заряды. Π’Π°ΠΊΠΈΠΌ ΠΎΠ±Ρ€Π°Π·ΠΎΠΌ, эти ΠΎΡ‚Ρ€ΠΈΡ†Π°Ρ‚Π΅Π»ΡŒΠ½ΠΎ заряТСнныС элСктроны двиТутся Π² Π½Π°ΠΏΡ€Π°Π²Π»Π΅Π½ΠΈΠΈ, ΠΏΡ€ΠΎΡ‚ΠΈΠ²ΠΎΠΏΠΎΠ»ΠΎΠΆΠ½ΠΎΠΌ элСктричСскому полю.

Π­Π»Π΅ΠΊΡ‚Ρ€ΠΎΠ½Π½Ρ‹ΠΉ ΠΈ ΠΎΠ±Ρ‹Ρ‡Π½Ρ‹ΠΉ Ρ‚ΠΎΠΊ.

Π­Ρ‚ΠΎ ΠΏΡ€ΠΎΠΈΠ·ΠΎΡˆΠ»ΠΎ ΠΏΠΎΡ‚ΠΎΠΌΡƒ, Ρ‡Ρ‚ΠΎ ΠΏΠ΅Ρ€Π²ΠΎΠ½Π°Ρ‡Π°Π»ΡŒΠ½Ρ‹Π΅ исслСдования статичСских ΠΈ динамичСских элСктричСских Ρ‚ΠΎΠΊΠΎΠ² Π±Ρ‹Π»ΠΈ основаны Π½Π° Ρ‚ΠΎΠΌ, Ρ‡Ρ‚ΠΎ ΠΌΡ‹ Ρ‚Π΅ΠΏΠ΅Ρ€ΡŒ Π½Π°Π·Π²Π°Π»ΠΈ Π±Ρ‹ ΠΏΠΎΠ»ΠΎΠΆΠΈΡ‚Π΅Π»ΡŒΠ½Ρ‹ΠΌΠΈ носитСлями заряда. Π­Ρ‚ΠΎ ΠΎΠ·Π½Π°Ρ‡Π°Π»ΠΎ, Ρ‡Ρ‚ΠΎ Ρ‚ΠΎΠ³Π΄Π° Ρ€Π°Π½Π½Π΅Π΅ соглашСниС ΠΎ Π½Π°ΠΏΡ€Π°Π²Π»Π΅Π½ΠΈΠΈ элСктричСского Ρ‚ΠΎΠΊΠ° Π±Ρ‹Π»ΠΎ установлСно ΠΊΠ°ΠΊ Π½Π°ΠΏΡ€Π°Π²Π»Π΅Π½ΠΈΠ΅, Π² ΠΊΠΎΡ‚ΠΎΡ€ΠΎΠΌ Π±ΡƒΠ΄ΡƒΡ‚ Π΄Π²ΠΈΠ³Π°Ρ‚ΡŒΡΡ ΠΏΠΎΠ»ΠΎΠΆΠΈΡ‚Π΅Π»ΡŒΠ½Ρ‹Π΅ заряды.Π­Ρ‚ΠΎ соглашСниС ΡΠΎΡ…Ρ€Π°Π½ΠΈΠ»ΠΎΡΡŒ ΠΈ ΠΈΡΠΏΠΎΠ»ΡŒΠ·ΡƒΠ΅Ρ‚ΡΡ Π΄ΠΎ сих ΠΏΠΎΡ€.

Π˜Ρ‚ΠΎΠ³ΠΎ:

  • ΠžΠ±Ρ‹Ρ‡Π½Ρ‹ΠΉ Ρ‚ΠΎΠΊ: ΠžΠ±Ρ‹Ρ‡Π½Ρ‹ΠΉ Ρ‚ΠΎΠΊ ΠΈΠ΄Π΅Ρ‚ ΠΎΡ‚ ΠΏΠΎΠ»ΠΎΠΆΠΈΡ‚Π΅Π»ΡŒΠ½ΠΎΠ³ΠΎ ΠΊ ΠΎΡ‚Ρ€ΠΈΡ†Π°Ρ‚Π΅Π»ΡŒΠ½ΠΎΠΌΡƒ Π²Ρ‹Π²ΠΎΠ΄Ρƒ ΠΈ ΡƒΠΊΠ°Π·Ρ‹Π²Π°Π΅Ρ‚ Π½Π°ΠΏΡ€Π°Π²Π»Π΅Π½ΠΈΠ΅, Π² ΠΊΠΎΡ‚ΠΎΡ€ΠΎΠΌ Π±ΡƒΠ΄ΡƒΡ‚ ΠΏΡ€ΠΎΡ‚Π΅ΠΊΠ°Ρ‚ΡŒ ΠΏΠΎΠ»ΠΎΠΆΠΈΡ‚Π΅Π»ΡŒΠ½Ρ‹Π΅ заряды.
  • Π­Π»Π΅ΠΊΡ‚Ρ€ΠΎΠ½Π½Ρ‹ΠΉ ΠΏΠΎΡ‚ΠΎΠΊ: Π­Π»Π΅ΠΊΡ‚Ρ€ΠΎΠ½Π½Ρ‹ΠΉ ΠΏΠΎΡ‚ΠΎΠΊ ΠΈΠ΄Π΅Ρ‚ ΠΎΡ‚ ΠΎΡ‚Ρ€ΠΈΡ†Π°Ρ‚Π΅Π»ΡŒΠ½ΠΎΠ³ΠΎ полюса ΠΊ ΠΏΠΎΠ»ΠΎΠΆΠΈΡ‚Π΅Π»ΡŒΠ½ΠΎΠΌΡƒ. Π­Π»Π΅ΠΊΡ‚Ρ€ΠΎΠ½Ρ‹ заряТСны ΠΎΡ‚Ρ€ΠΈΡ†Π°Ρ‚Π΅Π»ΡŒΠ½ΠΎ ΠΈ поэтому ΠΏΡ€ΠΈΡ‚ΡΠ³ΠΈΠ²Π°ΡŽΡ‚ΡΡ ΠΊ ΠΏΠΎΠ»ΠΎΠΆΠΈΡ‚Π΅Π»ΡŒΠ½ΠΎΠΌΡƒ ΠΏΠΎΠ»ΡŽΡΡƒ, Ρ‚Π°ΠΊ ΠΊΠ°ΠΊ ΠΏΡ€ΠΈΡ‚ΡΠ³ΠΈΠ²Π°ΡŽΡ‚ΡΡ Ρ€Π°Π·Π½Ρ‹Π΅ заряды.

Π­Ρ‚ΠΎ соглашСниС, ΠΊΠΎΡ‚ΠΎΡ€ΠΎΠ΅ ΠΈΡΠΏΠΎΠ»ΡŒΠ·ΡƒΠ΅Ρ‚ΡΡ Π²ΠΎ всСм ΠΌΠΈΡ€Π΅ ΠΏΠΎ сСй дСнь, Π΄Π°ΠΆΠ΅ Ссли ΠΎΠ½ΠΎ ΠΌΠΎΠΆΠ΅Ρ‚ ΠΏΠΎΠΊΠ°Π·Π°Ρ‚ΡŒΡΡ Π½Π΅ΠΌΠ½ΠΎΠ³ΠΎ странным ΠΈ ΡƒΡΡ‚Π°Ρ€Π΅Π²ΡˆΠΈΠΌ.

Π‘ΠΊΠΎΡ€ΠΎΡΡ‚ΡŒ двиТСния элСктрона ΠΈΠ»ΠΈ заряда

Π‘ΠΊΠΎΡ€ΠΎΡΡ‚ΡŒ ΠΏΠ΅Ρ€Π΅Π΄Π°Ρ‡ΠΈ элСктричСского Ρ‚ΠΎΠΊΠ° сильно отличаСтся ΠΎΡ‚ скорости Ρ€Π΅Π°Π»ΡŒΠ½ΠΎΠ³ΠΎ двиТСния элСктронов. Π‘Π°ΠΌ элСктрон ΠΏΠΎΠ΄ΠΏΡ€Ρ‹Π³ΠΈΠ²Π°Π΅Ρ‚ Π² ΠΏΡ€ΠΎΠ²ΠΎΠ΄Π½ΠΈΠΊΠ΅ ΠΈ, Π²ΠΎΠ·ΠΌΠΎΠΆΠ½ΠΎ, двиТСтся ΠΏΠΎ ΠΏΡ€ΠΎΠ²ΠΎΠ΄Π½ΠΈΠΊΡƒ Ρ‚ΠΎΠ»ΡŒΠΊΠΎ со ΡΠΊΠΎΡ€ΠΎΡΡ‚ΡŒΡŽ нСсколько ΠΌΠΈΠ»Π»ΠΈΠΌΠ΅Ρ‚Ρ€ΠΎΠ² Π² сСкунду. Π­Ρ‚ΠΎ ΠΎΠ·Π½Π°Ρ‡Π°Π΅Ρ‚, Ρ‡Ρ‚ΠΎ Π² случаС ΠΏΠ΅Ρ€Π΅ΠΌΠ΅Π½Π½ΠΎΠ³ΠΎ Ρ‚ΠΎΠΊΠ°, ΠΊΠΎΠ³Π΄Π° Ρ‚ΠΎΠΊ мСняСт Π½Π°ΠΏΡ€Π°Π²Π»Π΅Π½ΠΈΠ΅ 50 ΠΈΠ»ΠΈ 60 Ρ€Π°Π· Π² сСкунду, большая Ρ‡Π°ΡΡ‚ΡŒ элСктронов Π½ΠΈΠΊΠΎΠ³Π΄Π° Π½Π΅ Π²Ρ‹Ρ…ΠΎΠ΄ΠΈΡ‚ ΠΈΠ· ΠΏΡ€ΠΎΠ²ΠΎΠ΄Π°.

Π’ΠΎΠ·ΡŒΠΌΠ΅ΠΌ Π΄Ρ€ΡƒΠ³ΠΎΠΉ ΠΏΡ€ΠΈΠΌΠ΅Ρ€. Π’ ΠΏΠΎΡ‡Ρ‚ΠΈ ΠΏΠΎΠ»Π½ΠΎΠΌ Π²Π°ΠΊΡƒΡƒΠΌΠ΅ Π²Π½ΡƒΡ‚Ρ€ΠΈ элСктронно-Π»ΡƒΡ‡Π΅Π²ΠΎΠΉ Ρ‚Ρ€ΡƒΠ±ΠΊΠΈ элСктроны двиТутся ΠΏΠΎΡ‡Ρ‚ΠΈ ΠΏΠΎ прямым линиям со ΡΠΊΠΎΡ€ΠΎΡΡ‚ΡŒΡŽ ΠΏΡ€ΠΈΠΌΠ΅Ρ€Π½ΠΎ Π² ΠΎΠ΄Π½Ρƒ Π΄Π΅ΡΡΡ‚ΡƒΡŽ скорости свСта.

ВлияниС Ρ‚ΠΎΠΊΠ°

Когда элСктричСский Ρ‚ΠΎΠΊ Ρ‚Π΅Ρ‡Π΅Ρ‚ ΠΏΠΎ ΠΏΡ€ΠΎΠ²ΠΎΠ΄Π½ΠΈΠΊΡƒ, Π΅ΡΡ‚ΡŒ нСсколько ΠΏΡ€ΠΈΠ·Π½Π°ΠΊΠΎΠ², ΡƒΠΊΠ°Π·Ρ‹Π²Π°ΡŽΡ‰ΠΈΡ… Π½Π° Ρ‚ΠΎ, Ρ‡Ρ‚ΠΎ Ρ‚ΠΎΠΊ Ρ‚Π΅Ρ‡Π΅Ρ‚.

  • Π’Π΅ΠΏΠ»ΠΎ рассСиваСтся: Π’ΠΎΠ·ΠΌΠΎΠΆΠ½ΠΎ, Π½Π°ΠΈΠ±ΠΎΠ»Π΅Π΅ ΠΎΡ‡Π΅Π²ΠΈΠ΄Π½Ρ‹ΠΌ являСтся Ρ‚ΠΎ, Ρ‡Ρ‚ΠΎ Ρ‚Π΅ΠΏΠ»ΠΎ выдСляСтся. Если Ρ‚ΠΎΠΊ нСбольшой, Ρ‚ΠΎ количСство выдСляСмого Ρ‚Π΅ΠΏΠ»Π°, вСроятно, Π±ΡƒΠ΄Π΅Ρ‚ ΠΎΡ‡Π΅Π½ΡŒ нСбольшим ΠΈ Π΅Π³ΠΎ ΠΌΠΎΠΆΠ½ΠΎ Π½Π΅ Π·Π°ΠΌΠ΅Ρ‚ΠΈΡ‚ΡŒ.Однако, Ссли Ρ‚ΠΎΠΊ большС, Π²ΠΎΠ·ΠΌΠΎΠΆΠ½ΠΎ, выдСляСтся Π·Π°ΠΌΠ΅Ρ‚Π½ΠΎΠ΅ количСство Ρ‚Π΅ΠΏΠ»Π°. ЭлСктричСский огонь — яркий ΠΏΡ€ΠΈΠΌΠ΅Ρ€ Ρ‚ΠΎΠ³ΠΎ, ΠΊΠ°ΠΊ Ρ‚ΠΎΠΊ Π²Ρ‹Π·Ρ‹Π²Π°Π΅Ρ‚ Π²Ρ‹Π΄Π΅Π»Π΅Π½ΠΈΠ΅ Ρ‚Π΅ΠΏΠ»Π°. ЀактичСскоС количСство Ρ‚Π΅ΠΏΠ»Π° зависит Π½Π΅ Ρ‚ΠΎΠ»ΡŒΠΊΠΎ ΠΎΡ‚ Ρ‚ΠΎΠΊΠ°, Π½ΠΎ Ρ‚Π°ΠΊΠΆΠ΅ ΠΎΡ‚ напряТСния ΠΈ сопротивлСния ΠΏΡ€ΠΎΠ²ΠΎΠ΄Π½ΠΈΠΊΠ°.
  • ΠœΠ°Π³Π½ΠΈΡ‚Π½Ρ‹ΠΉ эффСкт: Π•Ρ‰Π΅ ΠΎΠ΄ΠΈΠ½ эффСкт, ΠΊΠΎΡ‚ΠΎΡ€Ρ‹ΠΉ ΠΌΠΎΠΆΠ½ΠΎ Π·Π°ΠΌΠ΅Ρ‚ΠΈΡ‚ΡŒ, — это созданиС ΠΌΠ°Π³Π½ΠΈΡ‚Π½ΠΎΠ³ΠΎ поля Π²ΠΎΠΊΡ€ΡƒΠ³ ΠΏΡ€ΠΎΠ²ΠΎΠ΄Π½ΠΈΠΊΠ°. Если Π² ΠΏΡ€ΠΎΠ²ΠΎΠ΄Π½ΠΈΠΊΠ΅ Ρ‚Π΅Ρ‡Π΅Ρ‚ Ρ‚ΠΎΠΊ, это ΠΌΠΎΠΆΠ½ΠΎ ΠΎΠ±Π½Π°Ρ€ΡƒΠΆΠΈΡ‚ΡŒ.ΠŸΠΎΠΌΠ΅ΡΡ‚ΠΈΠ² компас рядом с ΠΏΡ€ΠΎΠ²ΠΎΠ΄ΠΎΠΌ, ΠΏΠΎ ΠΊΠΎΡ‚ΠΎΡ€ΠΎΠΌΡƒ ΠΏΡ€ΠΎΡ…ΠΎΠ΄ΠΈΡ‚ достаточно большой постоянный Ρ‚ΠΎΠΊ, ΠΌΠΎΠΆΠ½ΠΎ ΡƒΠ²ΠΈΠ΄Π΅Ρ‚ΡŒ, Ρ‡Ρ‚ΠΎ стрСлка компаса отклоняСтся. ΠžΠ±Ρ€Π°Ρ‚ΠΈΡ‚Π΅ Π²Π½ΠΈΠΌΠ°Π½ΠΈΠ΅, Ρ‡Ρ‚ΠΎ это Π½Π΅ Π±ΡƒΠ΄Π΅Ρ‚ Ρ€Π°Π±ΠΎΡ‚Π°Ρ‚ΡŒ с ΡΠ΅Ρ‚ΡŒΡŽ, ΠΏΠΎΡ‚ΠΎΠΌΡƒ Ρ‡Ρ‚ΠΎ ΠΏΠΎΠ»Π΅ слишком быстро мСняСтся, ΠΈ ΠΈΠ³Π»Π° Π½Π΅ ΠΌΠΎΠΆΠ΅Ρ‚ Ρ€Π΅Π°Π³ΠΈΡ€ΠΎΠ²Π°Ρ‚ΡŒ, Π° Π΄Π²Π° ΠΏΡ€ΠΎΠ²ΠΎΠ΄Π° (ΠΏΠΎΠ΄ напряТСниСм ΠΈ Π½Π΅ΠΉΡ‚Ρ€Π°Π»ΡŒ), располоТСнныС Π±Π»ΠΈΠ·ΠΊΠΎ Π΄Ρ€ΡƒΠ³ ΠΊ Π΄Ρ€ΡƒΠ³Ρƒ Π² ΠΎΠ΄Π½ΠΎΠΌ ΠΊΠ°Π±Π΅Π»Π΅, Π½Π΅ΠΉΡ‚Ρ€Π°Π»ΠΈΠ·ΡƒΡŽΡ‚ ΠΏΠΎΠ»Π΅.

    ΠœΠ°Π³Π½ΠΈΡ‚Π½ΠΎΠ΅ ΠΏΠΎΠ»Π΅, создаваСмоС Ρ‚ΠΎΠΊΠΎΠΌ, Π½Π°Ρ…ΠΎΠ΄ΠΈΡ‚ Ρ…ΠΎΡ€ΠΎΡˆΠ΅Π΅ ΠΏΡ€ΠΈΠΌΠ΅Π½Π΅Π½ΠΈΠ΅ Π²ΠΎ ΠΌΠ½ΠΎΠ³ΠΈΡ… областях. Намотав ΠΏΡ€ΠΎΠ²ΠΎΠ΄ Π² ΠΊΠ°Ρ‚ΡƒΡˆΠΊΡƒ, ΠΌΠΎΠΆΠ½ΠΎ ΡƒΡΠΈΠ»ΠΈΡ‚ΡŒ эффСкт ΠΈ ΡΠΎΠ·Π΄Π°Ρ‚ΡŒ элСктромагнит.Π Π΅Π»Π΅ ΠΈ мноТСство Π΄Ρ€ΡƒΠ³ΠΈΡ… ΠΏΡ€Π΅Π΄ΠΌΠ΅Ρ‚ΠΎΠ² ΠΈΡΠΏΠΎΠ»ΡŒΠ·ΡƒΡŽΡ‚ этот эффСкт. Π“Ρ€ΠΎΠΌΠΊΠΎΠ³ΠΎΠ²ΠΎΡ€ΠΈΡ‚Π΅Π»ΠΈ Ρ‚Π°ΠΊΠΆΠ΅ ΠΈΡΠΏΠΎΠ»ΡŒΠ·ΡƒΡŽΡ‚ ΠΏΠ΅Ρ€Π΅ΠΌΠ΅Π½Π½Ρ‹ΠΉ Ρ‚ΠΎΠΊ Π² ΠΊΠ°Ρ‚ΡƒΡˆΠΊΠ΅, Ρ‡Ρ‚ΠΎΠ±Ρ‹ Π²Ρ‹Π·Π²Π°Ρ‚ΡŒ колСбания Π² Π΄ΠΈΠ°Ρ„Ρ€Π°Π³ΠΌΠ΅, ΠΊΠΎΡ‚ΠΎΡ€Ρ‹Π΅ ΠΏΠΎΠ·Π²ΠΎΠ»ΡΡŽΡ‚ элСктронным Ρ‚ΠΎΠΊΠ°ΠΌ ΠΏΡ€Π΅ΠΎΠ±Ρ€Π°Π·ΠΎΠ²Ρ‹Π²Π°Ρ‚ΡŒΡΡ Π² Π·Π²ΡƒΠΊΠΈ.

Как ΠΈΠ·ΠΌΠ΅Ρ€ΠΈΡ‚ΡŒ Ρ‚ΠΎΠΊ

Одним ΠΈΠ· Π²Π°ΠΆΠ½Ρ‹Ρ… аспСктов Ρ‚ΠΎΠΊΠ° являСтся Π·Π½Π°Π½ΠΈΠ΅ Π²Π΅Π»ΠΈΡ‡ΠΈΠ½Ρ‹ Ρ‚ΠΎΠΊΠ°, ΠΊΠΎΡ‚ΠΎΡ€Ρ‹ΠΉ ΠΌΠΎΠΆΠ΅Ρ‚ ΠΏΡ€ΠΎΡ‚Π΅ΠΊΠ°Ρ‚ΡŒ Π² ΠΏΡ€ΠΎΠ²ΠΎΠ΄Π½ΠΈΠΊΠ΅. ΠŸΠΎΡΠΊΠΎΠ»ΡŒΠΊΡƒ элСктричСский Ρ‚ΠΎΠΊ являСтся Ρ‚Π°ΠΊΠΈΠΌ ΠΊΠ»ΡŽΡ‡Π΅Π²Ρ‹ΠΌ Ρ„Π°ΠΊΡ‚ΠΎΡ€ΠΎΠΌ Π² элСктричСских ΠΈ элСктронных схСмах, ΠΎΡ‡Π΅Π½ΡŒ Π²Π°ΠΆΠ½ΠΎ Π·Π½Π°Ρ‚ΡŒ, ΠΊΠ°ΠΊΠΎΠΉ Ρ‚ΠΎΠΊ Ρ‚Π΅Ρ‡Π΅Ρ‚.

Π•ΡΡ‚ΡŒ ΠΌΠ½ΠΎΠ³ΠΎ Ρ€Π°Π·Π½Ρ‹Ρ… способов измСрСния Ρ‚ΠΎΠΊΠ°. Один ΠΈΠ· самых простых — ΠΈΡΠΏΠΎΠ»ΡŒΠ·ΠΎΠ²Π°Ρ‚ΡŒ ΠΌΡƒΠ»ΡŒΡ‚ΠΈΠΌΠ΅Ρ‚Ρ€.

Как ΠΈΠ·ΠΌΠ΅Ρ€ΠΈΡ‚ΡŒ Ρ‚ΠΎΠΊ с ΠΏΠΎΠΌΠΎΡ‰ΡŒΡŽ Ρ†ΠΈΡ„Ρ€ΠΎΠ²ΠΎΠ³ΠΎ ΠΌΡƒΠ»ΡŒΡ‚ΠΈΠΌΠ΅Ρ‚Ρ€Π°:

Π˜ΡΠΏΠΎΠ»ΡŒΠ·ΡƒΡ Ρ†ΠΈΡ„Ρ€ΠΎΠ²ΠΎΠΉ ΠΌΡƒΠ»ΡŒΡ‚ΠΈΠΌΠ΅Ρ‚Ρ€, Ρ†ΠΈΡ„Ρ€ΠΎΠ²ΠΎΠΉ ΠΌΡƒΠ»ΡŒΡ‚ΠΈΠΌΠ΅Ρ‚Ρ€, Π»Π΅Π³ΠΊΠΎ ΠΈΠ·ΠΌΠ΅Ρ€ΠΈΡ‚ΡŒ Ρ‚ΠΎΠΊ, помСстив Ρ†ΠΈΡ„Ρ€ΠΎΠ²ΠΎΠΉ ΠΌΡƒΠ»ΡŒΡ‚ΠΈΠΌΠ΅Ρ‚Ρ€ Π² Ρ†Π΅ΠΏΡŒ, ΠΏΠΎ ΠΊΠΎΡ‚ΠΎΡ€ΠΎΠΉ ΠΏΡ€ΠΎΡ…ΠΎΠ΄ΠΈΡ‚ Ρ‚ΠΎΠΊ. Π¦ΠΈΡ„Ρ€ΠΎΠ²ΠΎΠΉ ΠΌΡƒΠ»ΡŒΡ‚ΠΈΠΌΠ΅Ρ‚Ρ€ даст Ρ‚ΠΎΡ‡Π½Ρ‹Π΅ показания Ρ‚ΠΎΠΊΠ°, ΠΏΡ€ΠΎΡ‚Π΅ΠΊΠ°ΡŽΡ‰Π΅Π³ΠΎ Π² Ρ†Π΅ΠΏΠΈ

.

Π£Π·Π½Π°ΠΉΡ‚Π΅, , ΠΊΠ°ΠΊ ΠΈΠ·ΠΌΠ΅Ρ€ΠΈΡ‚ΡŒ Ρ‚ΠΎΠΊ с ΠΏΠΎΠΌΠΎΡ‰ΡŒΡŽ Ρ†ΠΈΡ„Ρ€ΠΎΠ²ΠΎΠ³ΠΎ ΠΌΡƒΠ»ΡŒΡ‚ΠΈΠΌΠ΅Ρ‚Ρ€Π°.

Π₯отя ΡΡƒΡ‰Π΅ΡΡ‚Π²ΡƒΡŽΡ‚ ΠΈ Π΄Ρ€ΡƒΠ³ΠΈΠ΅ ΠΌΠ΅Ρ‚ΠΎΠ΄Ρ‹ измСрСния Ρ‚ΠΎΠΊΠ°, это Π½Π°ΠΈΠ±ΠΎΠ»Π΅Π΅ распространСнный.

Π’ΠΎΠΊ — ΠΎΠ΄ΠΈΠ½ ΠΈΠ· самых Π²Π°ΠΆΠ½Ρ‹Ρ… ΠΈ Ρ„ΡƒΠ½Π΄Π°ΠΌΠ΅Π½Ρ‚Π°Π»ΡŒΠ½Ρ‹Ρ… элСмСнтов Π² элСктричСских ΠΈ элСктронных тСхнологиях. Π’ΠΎΠΊ, ΠΏΡ€ΠΎΡ‚Π΅ΠΊΠ°ΡŽΡ‰ΠΈΠΉ Π² Ρ†Π΅ΠΏΠΈ, ΠΌΠΎΠΆΠ½ΠΎ ΠΈΡΠΏΠΎΠ»ΡŒΠ·ΠΎΠ²Π°Ρ‚ΡŒ Ρ€Π°Π·Π»ΠΈΡ‡Π½Ρ‹ΠΌΠΈ способами: ΠΎΡ‚ гСнСрирования Ρ‚Π΅ΠΏΠ»Π° Π΄ΠΎ ΠΏΠ΅Ρ€Π΅ΠΊΠ»ΡŽΡ‡Π΅Π½ΠΈΡ схСм ΠΈΠ»ΠΈ сохранСния ΠΈΠ½Ρ„ΠΎΡ€ΠΌΠ°Ρ†ΠΈΠΈ Π² ΠΈΠ½Ρ‚Π΅Π³Ρ€Π°Π»ΡŒΠ½ΠΎΠΉ схСмС.

Π”Ρ€ΡƒΠ³ΠΈΠ΅ основныС ΠΊΠΎΠ½Ρ†Π΅ΠΏΡ†ΠΈΠΈ элСктроники:
НапряТСниС Π’Π΅ΠΊΡƒΡ‰ΠΈΠΉ ΠœΠΎΡ‰Π½ΠΎΡΡ‚ΡŒ Π‘ΠΎΠΏΡ€ΠΎΡ‚ΠΈΠ²Π»Π΅Π½ΠΈΠ΅ Π•ΠΌΠΊΠΎΡΡ‚ΡŒ Π˜Π½Π΄ΡƒΠΊΡ‚ΠΈΠ²Π½ΠΎΡΡ‚ΡŒ ВрансформСры Π”Π΅Ρ†ΠΈΠ±Π΅Π», Π΄Π‘ Π—Π°ΠΊΠΎΠ½Ρ‹ ΠšΠΈΡ€Ρ…Π³ΠΎΡ„Π° Q, Π΄ΠΎΠ±Ρ€ΠΎΡ‚Π½ΠΎΡΡ‚ΡŒ Радиочастотный ΡˆΡƒΠΌ
Π’Π΅Ρ€Π½ΡƒΡ‚ΡŒΡΡ Π² мСню Β«ΠžΡΠ½ΠΎΠ²Π½Ρ‹Π΅ понятия элСктроники».. .

Π§Ρ‚ΠΎ Ρ‚Π°ΠΊΠΎΠ΅ элСктричСский Ρ‚ΠΎΠΊ?

Когда Π²Ρ‹ ΠΏΠΎΠ΄ΠΊΠ»ΡŽΡ‡Π°Π΅Ρ‚Π΅ ΠΏΡ€ΠΎΠ²ΠΎΠ΄Π° ΠΊ ΠΊΠ»Π΅ΠΌΠΌΠ°ΠΌ аккумулятора, заряд ΠΌΠΎΠΆΠ΅Ρ‚ Ρ‚Π΅Ρ‡ΡŒ, ΠΈ образуСтся ΠΏΠΎΠ»Π½Ρ‹ΠΉ ΠΏΡƒΡ‚ΡŒ, Π½Π°Π·Ρ‹Π²Π°Π΅ΠΌΡ‹ΠΉ Ρ†Π΅ΠΏΡŒΡŽ. Π­Ρ‚ΠΎΡ‚ ΠΏΠΎΡ‚ΠΎΠΊ заряда называСтся Ρ‚ΠΎΠΊΠΎΠΌ . Π‘ΠΈΠΌΠ²ΠΎΠ»ΠΎΠΌ Ρ‚ΠΎΠΊΠ° являСтся Π±ΡƒΠΊΠ²Π° I, Π° Π΅Π΄ΠΈΠ½ΠΈΡ†Π΅ΠΉ измСрСния являСтся Π°ΠΌΠΏΠ΅Ρ€. Π§Π΅ΠΌ Π²Ρ‹ΡˆΠ΅ Ρ‚ΠΎΠΊ, ΠΏΡ€ΠΎΡ‚Π΅ΠΊΠ°ΡŽΡ‰ΠΈΠΉ ΠΏΠΎ ΠΏΡ€ΠΎΠ²ΠΎΠ΄Ρƒ, Ρ‚Π΅ΠΌ большС заряда ΠΏΡ€ΠΎΡ…ΠΎΠ΄ΠΈΡ‚.

Как заряд ΠΏΡ€ΠΎΡ…ΠΎΠ΄ΠΈΡ‚ Ρ‡Π΅Ρ€Π΅Π· ΠΊΠΎΠ½Ρ‚ΡƒΡ€?

Когда ΠΌΠ΅Π΄Π½Ρ‹ΠΉ ΠΏΡ€ΠΎΠ²ΠΎΠ΄ присоСдиняСтся ΠΊ ΠΊΠ»Π΅ΠΌΠΌΠ°ΠΌ аккумуляторной Π±Π°Ρ‚Π°Ρ€Π΅ΠΈ, ΠΈΠ΄Π΅Ρ‚ заряд.Если Π½Π°Π΄Π΅Ρ‚ΡŒ Π½Π° ΠΊΠ»Π΅ΠΌΠΌΡ‹ Ρ€Π΅Π·ΠΈΠ½ΠΊΡƒ, заряд Π½Π΅ сдвинСтся. ΠŸΠΎΡ‡Π΅ΠΌΡƒ это? ΠŸΡ€ΠΈΡ‡ΠΈΠ½Π° кроСтся Π² свойствах ΠΌΠ΅Π΄ΠΈ, которая являСтся Ρ…ΠΎΡ€ΠΎΡˆΠΈΠΌ ΠΏΡ€ΠΎΠ²ΠΎΠ΄Π½ΠΈΠΊΠΎΠΌ заряда. ΠœΠ°Ρ‚Π΅Ρ€ΠΈΠ°Π», ΠΊΠΎΡ‚ΠΎΡ€Ρ‹ΠΉ являСтся Ρ…ΠΎΡ€ΠΎΡˆΠΈΠΌΠΈ ΠΏΡ€ΠΎΠ²ΠΎΠ΄Π½ΠΈΠΊΠ°ΠΌΠΈ, ΠΈΠΌΠ΅Π΅Ρ‚ слабо ΡƒΠ΄Π΅Ρ€ΠΆΠΈΠ²Π°Π΅ΠΌΡ‹Π΅ элСктроны. Π”Π²ΠΈΠΆΠ΅Π½ΠΈΠ΅ элСктронов происходит ΠΎΡ‡Π΅Π½ΡŒ быстро, пСрСходя ΠΎΡ‚ Π°Ρ‚ΠΎΠΌΠ° ΠΊ Π°Ρ‚ΠΎΠΌΡƒ ΠΏΠΎ ΠΏΡ€ΠΎΠ²ΠΎΠ΄Ρƒ. Для пСрСмСщСния заряда ΠΌΠ΅ΠΆΠ΄Ρƒ двумя Ρ‚ΠΎΡ‡ΠΊΠ°ΠΌΠΈ Π΄ΠΎΠ»ΠΆΠ½Π° Π±Ρ‹Ρ‚ΡŒ Ρ€Π°Π·Π½ΠΎΡΡ‚ΡŒ ΠΏΠΎΡ‚Π΅Π½Ρ†ΠΈΠ°Π»ΠΎΠ² .

БущСствуСт связь ΠΌΠ΅ΠΆΠ΄Ρƒ элСктричСским ΠΏΠΎΡ‚Π΅Π½Ρ†ΠΈΠ°Π»ΠΎΠΌ ΠΈ элСктричСским Ρ‚ΠΎΠΊΠΎΠΌ.Π­Ρ‚Π° взаимосвязь Π±Ρ‹Π»Π° Π²ΠΏΠ΅Ρ€Π²Ρ‹Π΅ ΡΠΊΡΠΏΠ΅Ρ€ΠΈΠΌΠ΅Π½Ρ‚Π°Π»ΡŒΠ½ΠΎ продСмонстрирована ΡƒΡ‡Π΅Π½Ρ‹ΠΌ ΠΏΠΎ ΠΈΠΌΠ΅Π½ΠΈ Π”ΠΆΠΎΡ€Π΄ΠΆ Π‘Π°ΠΉΠΌΠΎΠ½ Ом.

Π’ Ρ‡Π΅ΠΌ Ρ€Π°Π·Π½ΠΈΡ†Π° ΠΌΠ΅ΠΆΠ΄Ρƒ элСктричСским Ρ‚ΠΎΠΊΠΎΠΌ ΠΈ ΠΏΠΎΡ‚ΠΎΠΊΠΎΠΌ элСктронов? ΠŸΠΎΡ‡Π΅ΠΌΡƒ ΠΎΠ½ΠΈ Ρ€Π°Π·ΠΎΡˆΠ»ΠΈΡΡŒ?

По этому ΠΏΠΎΠ²ΠΎΠ΄Ρƒ Π΄ΠΎ сих ΠΏΠΎΡ€ сущСствуСт большая ΠΏΡƒΡ‚Π°Π½ΠΈΡ†Π°. Π­Ρ‚ΠΎ происходит с Ρ‚ΠΎΠ³ΠΎ ΠΌΠΎΠΌΠ΅Π½Ρ‚Π°, ΠΊΠΎΠ³Π΄Π° Π‘Π΅Π½Π΄ΠΆΠ°ΠΌΠΈΠ½ Π€Ρ€Π°Π½ΠΊΠ»ΠΈΠ½ Π²ΠΏΠ΅Ρ€Π²Ρ‹Π΅ ΠΎΡ‚ΠΊΡ€Ρ‹Π» элСктричСство, ΠΎΠ½ Π΄ΡƒΠΌΠ°Π», Ρ‡Ρ‚ΠΎ ΠΏΠΎ ΠΏΡ€ΠΎΠ²ΠΎΠ΄Ρƒ двиТутся ΠΏΠΎΠ»ΠΎΠΆΠΈΡ‚Π΅Π»ΡŒΠ½Ρ‹Π΅ заряды. Π’Π°ΠΊΠΈΠΌ ΠΎΠ±Ρ€Π°Π·ΠΎΠΌ, Current Π±Ρ‹Π» ΠΎΠΏΡ€Π΅Π΄Π΅Π»Π΅Π½ ΠΊΠ°ΠΊ ΠΏΠΎΡ‚ΠΎΠΊ ΠΏΠΎΠ»ΠΎΠΆΠΈΡ‚Π΅Π»ΡŒΠ½ΠΎΠ³ΠΎ заряда, ΠΊΠΎΡ‚ΠΎΡ€Ρ‹ΠΉ двигался Π² мСталличСских ΠΏΡ€ΠΎΠ²ΠΎΠ΄Π°Ρ…, с ΠΊΠΎΡ‚ΠΎΡ€Ρ‹ΠΌΠΈ ΠΎΠ½ ΠΈΠ³Ρ€Π°Π» (это часто Π½Π°Π·Ρ‹Π²Π°ΡŽΡ‚ Ρ‚ΠΎΠΊΠΎΠΌ Convention). Π’Π΅ΠΏΠ΅Ρ€ΡŒ ΠΌΡ‹ Π·Π½Π°Π΅ΠΌ, Ρ‡Ρ‚ΠΎ Π€Ρ€Π°Π½ΠΊΠ»ΠΈΠ½ ошибался — ΠΎΡ‚Ρ€ΠΈΡ†Π°Ρ‚Π΅Π»ΡŒΠ½Ρ‹Π΅ (элСктроны) заряды двигались Π² Π½Π°ΠΏΡ€Π°Π²Π»Π΅Π½ΠΈΠΈ , ΠΏΡ€ΠΎΡ‚ΠΈΠ²ΠΎΠΏΠΎΠ»ΠΎΠΆΠ½ΠΎΠΌ Π½Π°ΠΏΡ€Π°Π²Π»Π΅Π½ΠΈΡŽ Π΅Π³ΠΎ ΠΏΠΎΠ»ΠΎΠΆΠΈΡ‚Π΅Π»ΡŒΠ½ΠΎΠ³ΠΎ заряда Π² ΠΏΡ€ΠΎΠ²ΠΎΠ΄Π΅. Π’Π°ΠΆΠ½ΠΎ ΠΎΡ‚ΠΌΠ΅Ρ‚ΠΈΡ‚ΡŒ, Ρ‡Ρ‚ΠΎ ΠΏΠΎΠ»ΠΎΠΆΠΈΡ‚Π΅Π»ΡŒΠ½Ρ‹Π΅ заряды Π½Π΅ двиТутся Π² ΠΌΠ΅Π΄Π½ΠΎΠΉ ΠΏΡ€ΠΎΠ²ΠΎΠ»ΠΎΠΊΠ΅, Π½ΠΎ Π² Π΄Ρ€ΡƒΠ³ΠΈΡ… случаях, Π½Π°ΠΏΡ€ΠΈΠΌΠ΅Ρ€, Π² растворах, ΠΏΠ΅Ρ€Π΅ΠΌΠ΅Ρ‰Π°ΡŽΡ‚ΡΡ ΠΊΠ°ΠΊ ΠΎΡ‚Ρ€ΠΈΡ†Π°Ρ‚Π΅Π»ΡŒΠ½Ρ‹Π΅, Ρ‚Π°ΠΊ ΠΈ ΠΏΠΎΠ»ΠΎΠΆΠΈΡ‚Π΅Π»ΡŒΠ½Ρ‹Π΅ заряды.

Π§Ρ‚ΠΎ Ρ‚Π°ΠΊΠΎΠ΅ Π·Π°ΠΊΠΎΠ½ Ома?

Π—Π°ΠΊΠΎΠ½

Ома гласит, Ρ‡Ρ‚ΠΎ Π² элСктричСской Ρ†Π΅ΠΏΠΈ Ρ‚ΠΎΠΊ, проходящий Ρ‡Π΅Ρ€Π΅Π· рСзистор ΠΌΠ΅ΠΆΠ΄Ρƒ двумя Ρ‚ΠΎΡ‡ΠΊΠ°ΠΌΠΈ, связан с Ρ€Π°Π·Π½ΠΎΡΡ‚ΡŒΡŽ напряТСний ΠΌΠ΅ΠΆΠ΄Ρƒ двумя Ρ‚ΠΎΡ‡ΠΊΠ°ΠΌΠΈ ΠΈ ΠΎΠ±Ρ€Π°Ρ‚Π½ΠΎ ΠΏΡ€ΠΎΠΏΠΎΡ€Ρ†ΠΈΠΎΠ½Π°Π»Π΅Π½ элСктричСскому ΡΠΎΠΏΡ€ΠΎΡ‚ΠΈΠ²Π»Π΅Π½ΠΈΡŽ ΠΌΠ΅ΠΆΠ΄Ρƒ двумя Ρ‚ΠΎΡ‡ΠΊΠ°ΠΌΠΈ.Π­Ρ‚Π° связь ΠΏΠΎΠΊΠ°Π·Π°Π½Π° Π² ΡΠ»Π΅Π΄ΡƒΡŽΡ‰Π΅ΠΌ письмС

.

Π“Π΄Π΅ I — Ρ‚ΠΎΠΊ Π² Π°ΠΌΠΏΠ΅Ρ€Π°Ρ…, V — Ρ€Π°Π·Π½ΠΎΡΡ‚ΡŒ ΠΏΠΎΡ‚Π΅Π½Ρ†ΠΈΠ°Π»ΠΎΠ² Π² Π²ΠΎΠ»ΡŒΡ‚Π°Ρ…, Π° R — постоянная, измСряСмая Π² ΠΎΠΌΠ°Ρ…, называСмая сопротивлСниСм.

Π’ΠΎΠΊ прямо ΠΏΡ€ΠΎΠΏΠΎΡ€Ρ†ΠΈΠΎΠ½Π°Π»Π΅Π½ ΠΏΠΎΡ‚Π΅Ρ€Π΅ напряТСния Ρ‡Π΅Ρ€Π΅Π· рСзистор. Π’ΠΎ Π΅ΡΡ‚ΡŒ, Ссли удваиваСтся Ρ‚ΠΎΠΊ, Ρ‚ΠΎ увСличиваСтся ΠΈ напряТСниС. Π§Ρ‚ΠΎΠ±Ρ‹ Ρ‚ΠΎΠΊ ΠΏΡ€ΠΎΡ‚Π΅ΠΊΠ°Π» Ρ‡Π΅Ρ€Π΅Π· сопротивлСниС, Π½Π° этом сопротивлСнии Π΄ΠΎΠ»ΠΆΠ½ΠΎ Π±Ρ‹Ρ‚ΡŒ напряТСниС. Π—Π°ΠΊΠΎΠ½ Ома ΠΏΠΎΠΊΠ°Π·Ρ‹Π²Π°Π΅Ρ‚ взаимосвязь ΠΌΠ΅ΠΆΠ΄Ρƒ напряТСниСм (V), Ρ‚ΠΎΠΊΠΎΠΌ (I) ΠΈ сопротивлСниСм (R).Π­Ρ‚ΠΎ ΠΌΠΎΠΆΠ½ΠΎ Π·Π°ΠΏΠΈΡΠ°Ρ‚ΡŒ трСмя способами:

НаправлСниС Ρ‚ΠΎΠΊΠ°

Π­Π»Π΅ΠΊΡ‚Ρ€ΠΎΠ½Ρ‹, двиТущиСся ΠΏΠΎ ΠΏΡ€ΠΎΠ²ΠΎΠ»ΠΎΠΊΠ΅, ΠΌΠΎΠ³ΡƒΡ‚ Π½Π΅ΠΏΡ€Π΅Ρ€Ρ‹Π²Π½ΠΎ Π΄Π²ΠΈΠ³Π°Ρ‚ΡŒΡΡ ΠΏΠΎ ΠΏΡ€ΠΎΠ²ΠΎΠ»ΠΎΠΊΠ΅ Π² ΠΎΠ΄Π½ΠΎΠΌ ΠΈ Ρ‚ΠΎΠΌ ΠΆΠ΅ Π½Π°ΠΏΡ€Π°Π²Π»Π΅Π½ΠΈΠΈ. Π­Ρ‚ΠΎ называСтся постоянным Ρ‚ΠΎΠΊΠΎΠΌ. ЭлСктричСство ΠΎΡ‚ сухих элСмСнтов ΠΈΠ»ΠΈ Π±Π°Ρ‚Π°Ρ€Π΅ΠΉ являСтся ΠΏΡ€ΠΈΠΌΠ΅Ρ€ΠΎΠΌ постоянного Ρ‚ΠΎΠΊΠ°. Π­Π»Π΅ΠΊΡ‚Ρ€ΠΎΠ½Ρ‹ Ρ‚Π°ΠΊΠΆΠ΅ ΠΌΠΎΠ³ΡƒΡ‚ рСгулярно ΠΌΠ΅Π½ΡΡ‚ΡŒ ΠΈΠ»ΠΈ ΠΌΠ΅Π½ΡΡ‚ΡŒ своС Π½Π°ΠΏΡ€Π°Π²Π»Π΅Π½ΠΈΠ΅. Π­Ρ‚ΠΎ называСтся ΠΏΠ΅Ρ€Π΅ΠΌΠ΅Π½Π½Ρ‹ΠΌ Ρ‚ΠΎΠΊΠΎΠΌ. ЭлСктроэнСргия Π² вашСм Π΄ΠΎΠΌΠ΅ — ΠΏΠ΅Ρ€Π΅ΠΌΠ΅Π½Π½Ρ‹ΠΉ Ρ‚ΠΎΠΊ. Π’ Π‘ΠΎΠ΅Π΄ΠΈΠ½Π΅Π½Π½Ρ‹Ρ… Π¨Ρ‚Π°Ρ‚Π°Ρ… Ρ‚ΠΎΠΊ мСняСт Π½Π°ΠΏΡ€Π°Π²Π»Π΅Π½ΠΈΠ΅ 120 Ρ€Π°Π· Π² сСкунду.

Π’ Ρ‡Π΅ΠΌ Ρ€Π°Π·Π½ΠΈΡ†Π° ΠΌΠ΅ΠΆΠ΄Ρƒ прСимущСствами ΠΈ нСдостатками постоянного ΠΈ ΠΏΠ΅Ρ€Π΅ΠΌΠ΅Π½Π½ΠΎΠ³ΠΎ Ρ‚ΠΎΠΊΠ°?

ΠŸΠΎΡΡ‚ΠΎΡΠ½Π½Ρ‹ΠΉ Ρ‚ΠΎΠΊ

ΠŸΠΎΡΡ‚ΠΎΡΠ½Π½Ρ‹ΠΉ Ρ‚ΠΎΠΊ вырабатываСтся Ρ‚Π°ΠΊΠΈΠΌΠΈ источниками, ΠΊΠ°ΠΊ Π±Π°Ρ‚Π°Ρ€Π΅ΠΈ, Ρ‚Π΅Ρ€ΠΌΠΎΠΏΠ°Ρ€Ρ‹, солнСчныС элСмСнты ΠΈ элСктричСскиС ΠΌΠ°ΡˆΠΈΠ½Ρ‹ ΠΊΠΎΠΌΠΌΡƒΡ‚Π°Ρ‚ΠΎΡ€Π½ΠΎΠ³ΠΎ Ρ‚ΠΈΠΏΠ°. ΠŸΠΎΡΡ‚ΠΎΡΠ½Π½Ρ‹ΠΉ Ρ‚ΠΎΠΊ ΠΌΠΎΠΆΠ΅Ρ‚ Ρ‚Π΅Ρ‡ΡŒ Π½Π΅ Ρ‚ΠΎΠ»ΡŒΠΊΠΎ ΠΏΠΎ ΠΏΡ€ΠΎΠ²ΠΎΠ΄Π°ΠΌ, Π½ΠΎ ΠΈ ΠΏΠΎ ΠΏΠΎΠ»ΡƒΠΏΡ€ΠΎΠ²ΠΎΠ΄Π½ΠΈΠΊΠ°ΠΌ. ΠŸΠΎΡΡ‚ΠΎΡΠ½Π½Ρ‹ΠΉ Ρ‚ΠΎΠΊ ΠΌΠΎΠΆΠ½ΠΎ ΠΏΠΎΠ»ΡƒΡ‡ΠΈΡ‚ΡŒ ΠΈΠ· ΠΏΠ΅Ρ€Π΅ΠΌΠ΅Π½Π½ΠΎΠ³ΠΎ Ρ‚ΠΎΠΊΠ° с ΠΏΠΎΠΌΠΎΡ‰ΡŒΡŽ выпрямитСля .

ΠŸΠ΅Ρ€Π²Π°Ρ коммСрчСская ΠΏΠ΅Ρ€Π΅Π΄Π°Ρ‡Π° элСктроэнСргии, разработанная Вомасом Эдисоном, использовала постоянный Ρ‚ΠΎΠΊ.

DC ΠΎΠ±Ρ‹Ρ‡Π½ΠΎ ΠΈΡΠΏΠΎΠ»ΡŒΠ·ΡƒΠ΅Ρ‚ΡΡ Π²ΠΎ ΠΌΠ½ΠΎΠ³ΠΈΡ… Π½ΠΈΠ·ΠΊΠΎΠ²ΠΎΠ»ΡŒΡ‚Π½Ρ‹Ρ… устройствах, особСнно Ρ‚Π°ΠΌ, Π³Π΄Π΅ ΠΎΠ½ΠΈ ΠΏΠΈΡ‚Π°ΡŽΡ‚ΡΡ ΠΎΡ‚ Π±Π°Ρ‚Π°Ρ€Π΅ΠΉ. Π’ Π±ΠΎΠ»ΡŒΡˆΠΈΠ½ΡΡ‚Π²Π΅ Π°Π²Ρ‚ΠΎΠΌΠΎΠ±ΠΈΠ»ΡŒΠ½Ρ‹Ρ… ΠΏΡ€ΠΈΠ»ΠΎΠΆΠ΅Π½ΠΈΠΉ ΠΈΡΠΏΠΎΠ»ΡŒΠ·ΡƒΠ΅Ρ‚ΡΡ постоянный Ρ‚ΠΎΠΊ, хотя Π³Π΅Π½Π΅Ρ€Π°Ρ‚ΠΎΡ€ ΠΏΠ΅Ρ€Π΅ΠΌΠ΅Π½Π½ΠΎΠ³ΠΎ Ρ‚ΠΎΠΊΠ° являСтся устройством ΠΏΠ΅Ρ€Π΅ΠΌΠ΅Π½Π½ΠΎΠ³ΠΎ Ρ‚ΠΎΠΊΠ°, ΠΊΠΎΡ‚ΠΎΡ€ΠΎΠ΅ ΠΈΡΠΏΠΎΠ»ΡŒΠ·ΡƒΠ΅Ρ‚ΡΡ для Π²Ρ‹Ρ€Π°Π±ΠΎΡ‚ΠΊΠΈ постоянного Ρ‚ΠΎΠΊΠ°. Π‘ΠΎΠ»ΡŒΡˆΠΈΠ½ΡΡ‚Π²Ρƒ элСктронных устройств трСбуСтся постоянный Ρ‚ΠΎΠΊ.

ΠŸΠ΅Ρ€Π΅ΠΌΠ΅Π½Π½Ρ‹ΠΉ Ρ‚ΠΎΠΊ

ΠŸΠ΅Ρ€Π΅ΠΌΠ΅Π½Π½Ρ‹ΠΉ Ρ‚ΠΎΠΊ — это способ ΠΏΠΎΠ΄Π°Ρ‡ΠΈ элСктроэнСргии Π² Π΄ΠΎΠΌΠ° ΠΈ Π½Π° прСдприятия.ΠŸΠ΅Ρ€Π²ΠΎΠ΅ зарСгистрированноС ΠΏΡ€ΠΈΠΌΠ΅Π½Π΅Π½ΠΈΠ΅ ΠΏΠ΅Ρ€Π΅ΠΌΠ΅Π½Π½ΠΎΠ³ΠΎ Ρ‚ΠΎΠΊΠ° Π±Ρ‹Π»ΠΎ сдСлано Π“ΠΈΠΉΠΎΠΌΠΎΠΌ Π”ΡŽΡˆΠ΅Π½Π½ΠΎΠΌ, ΠΊΠΎΡ‚ΠΎΡ€Ρ‹ΠΉ ΠΈΠ·ΠΎΠ±Ρ€Π΅Π» ΡΠ»Π΅ΠΊΡ‚Ρ€ΠΎΡ‚Π΅Ρ€Π°ΠΏΠΈΡŽ. Он ΠΏΡ€ΠΈΡˆΠ΅Π» ΠΊ Π²Ρ‹Π²ΠΎΠ΄Ρƒ, Ρ‡Ρ‚ΠΎ ΠΏΠ΅Ρ€Π΅ΠΌΠ΅Π½Π½Ρ‹ΠΉ Ρ‚ΠΎΠΊ прСвосходит постоянный Ρ‚ΠΎΠΊ для элСктротСплового запуска ΠΌΡ‹ΡˆΠ΅Ρ‡Π½Ρ‹Ρ… сокращСний. Π‘ΠΈΠ»ΠΎΠ²ΠΎΠΉ трансформатор Π±Ρ‹Π» Π²ΠΏΠ΅Ρ€Π²Ρ‹Π΅ продСмонстрирован Π² Π›ΠΎΠ½Π΄ΠΎΠ½Π΅ Π² 1881 Π³ΠΎΠ΄Ρƒ Π›ΡŽΡΡŒΠ΅Π½ΠΎΠΌ Гауляром ΠΈ Π²Ρ‹Π·Π²Π°Π» интСрСс Ρƒ ВСстингауза. Π₯отя ΠΎΠ½ΠΈ ΠΏΠΎΠ΄Π°Π»ΠΈ заявку Π½Π° ΠΏΠ°Ρ‚Π΅Π½Ρ‚Ρ‹ Π½Π° свои Ρ‚Π΅Ρ…Π½ΠΎΠ»ΠΎΠ³ΠΈΠΈ, ΠΎΠ½ΠΈ Π±Ρ‹Π»ΠΈ ΠΎΡ‚ΠΌΠ΅Π½Π΅Π½Ρ‹, ΠΏΠΎΡ‚ΠΎΠΌΡƒ Ρ‡Ρ‚ΠΎ Никола ВСсла смог ΠΏΡ€ΠΎΠ΄Π΅ΠΌΠΎΠ½ΡΡ‚Ρ€ΠΈΡ€ΠΎΠ²Π°Ρ‚ΡŒ ΠΏΡ€Π΅Π΄Ρ‹Π΄ΡƒΡ‰ΠΈΠ΅ Ρ€Π°Π±ΠΎΡ‚Ρ‹ Π² этой области. Никола ВСсла Π½Π°ΠΈΠ±ΠΎΠ»Π΅Π΅ извСстСн Ρ€Π°Π·Ρ€Π°Π±ΠΎΡ‚ΠΊΠΎΠΉ соврСмСнной систСмы элСктроснабТСния ΠΏΠ΅Ρ€Π΅ΠΌΠ΅Π½Π½ΠΎΠ³ΠΎ Ρ‚ΠΎΠΊΠ°.

НапряТСниС ΠΏΠ΅Ρ€Π΅ΠΌΠ΅Π½Π½ΠΎΠ³ΠΎ Ρ‚ΠΎΠΊΠ° ΠΌΠΎΠΆΠ΅Ρ‚ Π±Ρ‹Ρ‚ΡŒ ΡƒΠ²Π΅Π»ΠΈΡ‡Π΅Π½ΠΎ ΠΈΠ»ΠΈ ΡƒΠΌΠ΅Π½ΡŒΡˆΠ΅Π½ΠΎ с ΠΏΠΎΠΌΠΎΡ‰ΡŒΡŽ трансформатора. ИспользованиС высокого напряТСния ΠΏΡ€ΠΈΠ²ΠΎΠ΄ΠΈΡ‚ ΠΊ Π·Π½Π°Ρ‡ΠΈΡ‚Π΅Π»ΡŒΠ½ΠΎ большСй эффСктивности ΠΏΠ΅Ρ€Π΅Π΄Π°Ρ‡ΠΈ энСргии. НСдостатком высокого напряТСния являСтся Π½Π΅ΠΎΠ±Ρ…ΠΎΠ΄ΠΈΠΌΠΎΡΡ‚ΡŒ большСй изоляции.

Как ΠΈΠ·ΠΌΠ΅Ρ€ΠΈΡ‚ΡŒ элСктричСский Ρ‚ΠΎΠΊ?

АмпСрмСтр — это ΠΏΡ€ΠΈΠ±ΠΎΡ€ для измСрСния Ρ‚ΠΎΠΊΠ°. (IMGE NEEDED (. АмпСрмСтр

АмпСрмСтр измСряСт количСство элСктронов, ΠΊΠΎΡ‚ΠΎΡ€Ρ‹Π΅ проходят Ρ‡Π΅Ρ€Π΅Π· ΠΈΠ·ΠΌΠ΅Ρ€ΠΈΡ‚Π΅Π»ΡŒ ΠΊΠ°ΠΆΠ΄ΡƒΡŽ сСкунду.Π’ΠΎΠΊ измСряСтся Π² Π΅Π΄ΠΈΠ½ΠΈΡ†Π°Ρ…, Π½Π°Π·Ρ‹Π²Π°Π΅ΠΌΡ‹Ρ… Π°ΠΌΠΏΠ΅Ρ€Π°ΠΌΠΈ. Π§Ρ‚ΠΎΠ±Ρ‹ ΠΈΡΠΏΠΎΠ»ΡŒΠ·ΠΎΠ²Π°Ρ‚ΡŒ Π°ΠΌΠΏΠ΅Ρ€ΠΌΠ΅Ρ‚Ρ€, Π²Ρ‹ ΠΏΠΎΠ΄ΠΊΠ»ΡŽΡ‡Π°Π΅Ρ‚Π΅ Π΅Π³ΠΎ ΠΏΠΎΡΠ»Π΅Π΄ΠΎΠ²Π°Ρ‚Π΅Π»ΡŒΠ½ΠΎ ΠΊ Π½Π°Π³Ρ€ΡƒΠ·ΠΊΠ΅, ΠΊΠΎΡ‚ΠΎΡ€ΡƒΡŽ Π²Ρ‹ ΠΈΡΠΏΠΎΠ»ΡŒΠ·ΡƒΠ΅Ρ‚Π΅. (Π‘ΠΌ. Π˜Π·ΠΎΠ±Ρ€Π°ΠΆΠ΅Π½ΠΈΠ΅ Π²Ρ‹ΡˆΠ΅.)

Для измСрСния напряТСния ΠΈΠ»ΠΈ элСктричСского ΠΏΠΎΡ‚Π΅Π½Ρ†ΠΈΠ°Π»Π° Π²Ρ‹ Π²ΠΊΠ»ΡŽΡ‡Π°Π΅Ρ‚Π΅ Π²ΠΎΠ»ΡŒΡ‚ΠΌΠ΅Ρ‚Ρ€ Π² Ρ†Π΅ΠΏΡŒ ΠΏΠ°Ρ€Π°Π»Π»Π΅Π»ΡŒΠ½ΠΎ. Π’ΠΎΠ»ΡŒΡ‚ΠΌΠ΅Ρ‚Ρ€ Π΄ΠΎΠ»ΠΆΠ΅Π½ Π±Ρ‹Ρ‚ΡŒ ΠΏΠΎΠ΄ΠΊΠ»ΡŽΡ‡Π΅Π½ ΠΏΠ°Ρ€Π°Π»Π»Π΅Π»ΡŒΠ½ΠΎ, Ρ‡Ρ‚ΠΎΠ±Ρ‹ ΠΈΠ·ΠΌΠ΅Ρ€ΠΈΡ‚ΡŒ напряТСниС устройства, ΠΏΠΎΡ‚ΠΎΠΌΡƒ Ρ‡Ρ‚ΠΎ ΠΎΠ±ΡŠΠ΅ΠΊΡ‚Ρ‹, ΠΏΠΎΠ΄ΠΊΠ»ΡŽΡ‡Π΅Π½Π½Ρ‹Π΅ ΠΏΠ°Ρ€Π°Π»Π»Π΅Π»ΡŒΠ½ΠΎ, ΠΈΡΠΏΡ‹Ρ‚Ρ‹Π²Π°ΡŽΡ‚ ΠΎΠ΄ΠΈΠ½Π°ΠΊΠΎΠ²ΡƒΡŽ Ρ€Π°Π·Π½ΠΎΡΡ‚ΡŒ ΠΏΠΎΡ‚Π΅Π½Ρ†ΠΈΠ°Π»ΠΎΠ². Π‘ΠΌ. Π˜Π·ΠΎΠ±Ρ€Π°ΠΆΠ΅Π½ΠΈΠ΅ Π½ΠΈΠΆΠ΅.

Бвязь ΠΌΠ΅ΠΆΠ΄Ρƒ элСктричСским Ρ‚ΠΎΠΊΠΎΠΌ ΠΈ ΠΌΠ°Π³Π½ΠΈΡ‚Π½Ρ‹ΠΌΠΈ полями

Бвязь ΠΌΠ΅ΠΆΠ΄Ρƒ элСктричСским Ρ‚ΠΎΠΊΠΎΠΌ, ΠΌΠ°Π³Π½ΠΈΡ‚Π½Ρ‹ΠΌΠΈ полями ΠΈ физичСскими силами Π±Ρ‹Π»Π° Π²ΠΏΠ΅Ρ€Π²Ρ‹Π΅ ΠΎΠ±Π½Π°Ρ€ΡƒΠΆΠ΅Π½Π° Гансом ΠšΡ€ΠΈΡΡ‚ΠΈΠ°Π½ΠΎΠΌ ΠžΡ€ΡΡ‚Π΅Π΄ΠΎΠΌ Π² 1820 Π³ΠΎΠ΄Ρƒ.Он наблюдал, ΠΊΠ°ΠΊ стрСлка компаса ΠΎΡ‚ΠΊΠ»ΠΎΠ½ΡΠ»Π°ΡΡŒ ΠΎΡ‚ направлСния Π½Π° сСвСр, ΠΊΠΎΠ³Π΄Π° Π² сосСднСм ΠΏΡ€ΠΎΠ²ΠΎΠ΄Π΅ ΠΏΡ€ΠΎΡ‚Π΅ΠΊΠ°Π» Ρ‚ΠΎΠΊ. Π­Ρ‚ΠΎ Π±Ρ‹Π»ΠΎ извСстно ΠΊΠ°ΠΊ ΠΊΠ°ΡΠ°Ρ‚Π΅Π»ΡŒΠ½Ρ‹ΠΉ Π³Π°Π»ΡŒΠ²Π°Π½ΠΎΠΌΠ΅Ρ‚Ρ€. Π’Π°Π½Π³Π΅Π½Ρ†ΠΈΠ°Π»ΡŒΠ½Ρ‹ΠΉ Π³Π°Π»ΡŒΠ²Π°Π½ΠΎΠΌΠ΅Ρ‚Ρ€ использовался для измСрСния Ρ‚ΠΎΠΊΠΎΠ² с использованиСм этого эффСкта. Π’ΠΎΠ·Π²Ρ€Π°Ρ‰Π°ΡŽΡ‰Π΅ΠΉ силой ΠΊ ΠΎΠ±Π½ΡƒΠ»Π΅Π½ΠΈΡŽ счСтчика Π² этой систСмС Π±Ρ‹Π»Π° сила ΠΌΠ°Π³Π½ΠΈΡ‚Π½ΠΎΠ³ΠΎ поля Π—Π΅ΠΌΠ»ΠΈ. Π’Π°ΠΊΠΈΠΌ ΠΎΠ±Ρ€Π°Π·ΠΎΠΌ, ΠΈΠ·ΠΌΠ΅Ρ€ΠΈΡ‚Π΅Π»ΡŒ ΠΌΠΎΠΆΠ½ΠΎ Π±Ρ‹Π»ΠΎ ΠΈΡΠΏΠΎΠ»ΡŒΠ·ΠΎΠ²Π°Ρ‚ΡŒ Ρ‚ΠΎΠ»ΡŒΠΊΠΎ Ρ‚ΠΎΠ³Π΄Π°, ΠΊΠΎΠ³Π΄Π° ΠΎΠ½ Π±Ρ‹Π» привязан ΠΊ ΠΌΠ°Π³Π½ΠΈΡ‚Π½ΠΎΠΌΡƒ полю Π—Π΅ΠΌΠ»ΠΈ. Π§ΡƒΠ²ΡΡ‚Π²ΠΈΡ‚Π΅Π»ΡŒΠ½ΠΎΡΡ‚ΡŒ ΠΏΡ€ΠΈΠ±ΠΎΡ€Π° стала Π²ΠΎΠ·ΠΌΠΎΠΆΠ½ΠΎΠΉ Π·Π° счСт увСличСния числа Π²ΠΈΡ‚ΠΊΠΎΠ² ΠΈΡΠΏΠΎΠ»ΡŒΠ·ΡƒΠ΅ΠΌΠΎΠ³ΠΎ ΠΏΡ€ΠΎΠ²ΠΎΠ΄Π°.


ΠŸΡ€ΠΎΠ²Π΅Ρ€ΡŒΡ‚Π΅ своС ПониманиС:

НапряТСниС ΠΈ Ρ‚ΠΎΠΊ | ΠžΡΠ½ΠΎΠ²Π½Ρ‹Π΅ понятия элСктричСства

Как ΡƒΠΏΠΎΠΌΠΈΠ½Π°Π»ΠΎΡΡŒ Ρ€Π°Π½Π΅Π΅, Π½Π°ΠΌ Π½ΡƒΠΆΠ½ΠΎ Π½Π΅Ρ‡Ρ‚ΠΎ большСС, Ρ‡Π΅ΠΌ просто Π½Π΅ΠΏΡ€Π΅Ρ€Ρ‹Π²Π½Ρ‹ΠΉ ΠΏΡƒΡ‚ΡŒ (Ρ‚.Π΅.Π΅., Ρ†Π΅ΠΏΡŒ) Π΄ΠΎ Ρ‚ΠΎΠ³ΠΎ, ΠΊΠ°ΠΊ Π²ΠΎΠ·Π½ΠΈΠΊΠ½Π΅Ρ‚ Π½Π΅ΠΏΡ€Π΅Ρ€Ρ‹Π²Π½Ρ‹ΠΉ ΠΏΠΎΡ‚ΠΎΠΊ заряда: Π½Π°ΠΌ Ρ‚Π°ΠΊΠΆΠ΅ Π½ΡƒΠΆΠ½Ρ‹ срСдства, Ρ‡Ρ‚ΠΎΠ±Ρ‹ ΠΏΡ€ΠΎΡ‚ΠΎΠ»ΠΊΠ½ΡƒΡ‚ΡŒ эти носитСли заряда ΠΏΠΎ Ρ†Π΅ΠΏΠΈ. Π’Π°ΠΊ ΠΆΠ΅, ΠΊΠ°ΠΊ ΠΌΡ€Π°ΠΌΠΎΡ€ Π² Ρ‚Ρ€ΡƒΠ±Π΅ ΠΈΠ»ΠΈ Π²ΠΎΠ΄Π° Π² Ρ‚Ρ€ΡƒΠ±Π΅, для инициирования ΠΏΠΎΡ‚ΠΎΠΊΠ° трСбуСтся нСкоторая сила воздСйствия. Π’ случаС элСктронов эта сила — это Ρ‚Π° ΠΆΠ΅ сила, которая дСйствуСт Π² статичСском элСктричСствС: сила, создаваСмая дисбалансом элСктричСского заряда. Если ΠΌΡ‹ возьмСм ΠΏΡ€ΠΈΠΌΠ΅Ρ€Ρ‹ воска ΠΈ ΡˆΠ΅Ρ€ΡΡ‚ΠΈ, ΠΊΠΎΡ‚ΠΎΡ€Ρ‹Π΅ Π±Ρ‹Π»ΠΈ Π½Π°Ρ‚Π΅Ρ€Ρ‚Ρ‹ Π΄Ρ€ΡƒΠ³ с Π΄Ρ€ΡƒΠ³ΠΎΠΌ, ΠΌΡ‹ ΠΎΠ±Π½Π°Ρ€ΡƒΠΆΠΈΠΌ, Ρ‡Ρ‚ΠΎ ΠΈΠ·Π±Ρ‹Ρ‚ΠΎΠΊ элСктронов Π² воскС (ΠΎΡ‚Ρ€ΠΈΡ†Π°Ρ‚Π΅Π»ΡŒΠ½Ρ‹ΠΉ заряд) ΠΈ Π΄Π΅Ρ„ΠΈΡ†ΠΈΡ‚ элСктронов Π² ΡˆΠ΅Ρ€ΡΡ‚ΠΈ (ΠΏΠΎΠ»ΠΎΠΆΠΈΡ‚Π΅Π»ΡŒΠ½Ρ‹ΠΉ заряд) ΡΠΎΠ·Π΄Π°ΡŽΡ‚ дисбаланс заряда ΠΌΠ΅ΠΆΠ΄Ρƒ Π½ΠΈΠΌΠΈ.Π­Ρ‚ΠΎΡ‚ дисбаланс проявляСтся ΠΊΠ°ΠΊ сила притяТСния ΠΌΠ΅ΠΆΠ΄Ρƒ двумя ΠΎΠ±ΡŠΠ΅ΠΊΡ‚Π°ΠΌΠΈ:

Если ΠΌΠ΅ΠΆΠ΄Ρƒ заряТСнным воском ΠΈ ΡˆΠ΅Ρ€ΡΡ‚ΡŒΡŽ ΠΏΠΎΠΌΠ΅ΡΡ‚ΠΈΡ‚ΡŒ проводящий ΠΏΡ€ΠΎΠ²ΠΎΠ΄, Ρ‡Π΅Ρ€Π΅Π· Π½Π΅Π³ΠΎ Π±ΡƒΠ΄ΡƒΡ‚ ΠΏΡ€ΠΎΡ‚Π΅ΠΊΠ°Ρ‚ΡŒ элСктроны, ΠΏΠΎΡΠΊΠΎΠ»ΡŒΠΊΡƒ Π½Π΅ΠΊΠΎΡ‚ΠΎΡ€Ρ‹Π΅ ΠΈΠ· ΠΈΠ·Π±Ρ‹Ρ‚ΠΎΡ‡Π½Ρ‹Ρ… элСктронов Π² воскС устрСмятся Ρ‡Π΅Ρ€Π΅Π· ΠΏΡ€ΠΎΠ²ΠΎΠ΄, Ρ‡Ρ‚ΠΎΠ±Ρ‹ Π²Π΅Ρ€Π½ΡƒΡ‚ΡŒΡΡ ΠΊ ΡˆΠ΅Ρ€ΡΡ‚ΠΈ, восполняя Ρ‚Π°ΠΌ нСдостаток элСктронов:

Дисбаланс элСктронов ΠΌΠ΅ΠΆΠ΄Ρƒ Π°Ρ‚ΠΎΠΌΠ°ΠΌΠΈ воска ΠΈ Π°Ρ‚ΠΎΠΌΠ°ΠΌΠΈ ΡˆΠ΅Ρ€ΡΡ‚ΠΈ создаСт силу ΠΌΠ΅ΠΆΠ΄Ρƒ двумя ΠΌΠ°Ρ‚Π΅Ρ€ΠΈΠ°Π»Π°ΠΌΠΈ. ΠŸΠΎΡΠΊΠΎΠ»ΡŒΠΊΡƒ элСктроны Π½Π΅ ΠΌΠΎΠ³ΡƒΡ‚ ΠΏΠ΅Ρ€Π΅Ρ‚Π΅ΠΊΠ°Ρ‚ΡŒ ΠΎΡ‚ воска ΠΊ ΡˆΠ΅Ρ€ΡΡ‚ΠΈ, всС, Ρ‡Ρ‚ΠΎ ΠΌΠΎΠΆΠ΅Ρ‚ ΡΠ΄Π΅Π»Π°Ρ‚ΡŒ эта сила, — это ΠΏΡ€ΠΈΡ‚ΡΠ³ΠΈΠ²Π°Ρ‚ΡŒ Π΄Π²Π° ΠΎΠ±ΡŠΠ΅ΠΊΡ‚Π° вмСстС.Однако Ρ‚Π΅ΠΏΠ΅Ρ€ΡŒ, ΠΊΠΎΠ³Π΄Π° ΠΏΡ€ΠΎΠ²ΠΎΠ΄Π½ΠΈΠΊ ΠΏΠ΅Ρ€Π΅ΠΊΡ€Ρ‹Π²Π°Π΅Ρ‚ ΠΈΠ·ΠΎΠ»ΠΈΡ€ΡƒΡŽΡ‰ΠΈΠΉ Π·Π°Π·ΠΎΡ€, сила заставит элСктроны Ρ‚Π΅Ρ‡ΡŒ Π² ΠΎΠ΄Π½ΠΎΡ€ΠΎΠ΄Π½ΠΎΠΌ Π½Π°ΠΏΡ€Π°Π²Π»Π΅Π½ΠΈΠΈ Ρ‡Π΅Ρ€Π΅Π· ΠΏΡ€ΠΎΠ²ΠΎΠ΄, хотя Π±Ρ‹ Π½Π° ΠΌΠ³Π½ΠΎΠ²Π΅Π½ΠΈΠ΅, ΠΏΠΎΠΊΠ° заряд Π² этой области Π½Π΅ нСйтрализуСтся ΠΈ сила ΠΌΠ΅ΠΆΠ΄Ρƒ воском ΠΈ ΡˆΠ΅Ρ€ΡΡ‚ΡŒΡŽ Π½Π΅ ΡƒΠΌΠ΅Π½ΡŒΡˆΠΈΡ‚ΡΡ. ЭлСктричСский заряд, ΠΎΠ±Ρ€Π°Π·ΠΎΠ²Π°Π½Π½Ρ‹ΠΉ ΠΌΠ΅ΠΆΠ΄Ρƒ этими двумя ΠΌΠ°Ρ‚Π΅Ρ€ΠΈΠ°Π»Π°ΠΌΠΈ ΠΏΡ€ΠΈ Ρ‚Ρ€Π΅Π½ΠΈΠΈ ΠΈΡ… Π΄Ρ€ΡƒΠ³ ΠΎ Π΄Ρ€ΡƒΠ³Π°, слуТит для хранСния ΠΎΠΏΡ€Π΅Π΄Π΅Π»Π΅Π½Π½ΠΎΠ³ΠΎ количСства энСргии. Π­Ρ‚Π° энСргия ΠΌΠ°Π»ΠΎ Ρ‡Π΅ΠΌ отличаСтся ΠΎΡ‚ энСргии, Π½Π°ΠΊΠΎΠΏΠ»Π΅Π½Π½ΠΎΠΉ Π² высоком Ρ€Π΅Π·Π΅Ρ€Π²ΡƒΠ°Ρ€Π΅ с Π²ΠΎΠ΄ΠΎΠΉ, ΠΊΠΎΡ‚ΠΎΡ€Ρ‹ΠΉ выкачиваСтся ΠΈΠ· ΠΏΡ€ΡƒΠ΄Π° Π½ΠΈΠΆΠ½Π΅Π³ΠΎ уровня:

ВлияниС силы тяТСсти Π½Π° Π²ΠΎΠ΄Ρƒ Π² Ρ€Π΅Π·Π΅Ρ€Π²ΡƒΠ°Ρ€Π΅ создаСт силу, которая пытаСтся снова ΠΎΠΏΡƒΡΡ‚ΠΈΡ‚ΡŒ Π²ΠΎΠ΄Ρƒ Π½Π° Π±ΠΎΠ»Π΅Π΅ Π½ΠΈΠ·ΠΊΠΈΠΉ ΡƒΡ€ΠΎΠ²Π΅Π½ΡŒ.Если подходящая Ρ‚Ρ€ΡƒΠ±Π° ΠΏΡ€ΠΎΠ»ΠΎΠΆΠ΅Π½Π° ΠΎΡ‚ Ρ€Π΅Π·Π΅Ρ€Π²ΡƒΠ°Ρ€Π° ΠΎΠ±Ρ€Π°Ρ‚Π½ΠΎ ΠΊ ΠΏΡ€ΡƒΠ΄Ρƒ, Π²ΠΎΠ΄Π° ΠΏΠΎΠ΄ дСйствиСм силы тяТСсти ΠΏΠΎΡ‚Π΅Ρ‡Π΅Ρ‚ Π²Π½ΠΈΠ· ΠΈΠ· Ρ€Π΅Π·Π΅Ρ€Π²ΡƒΠ°Ρ€Π° ΠΏΠΎ Ρ‚Ρ€ΡƒΠ±Π΅:

ВрСбуСтся энСргия, Ρ‡Ρ‚ΠΎΠ±Ρ‹ ΠΏΠ΅Ρ€Π΅ΠΊΠ°Ρ‡ΠΈΠ²Π°Ρ‚ΡŒ эту Π²ΠΎΠ΄Ρƒ ΠΈΠ· ΠΏΡ€ΡƒΠ΄Π° с Π½ΠΈΠ·ΠΊΠΈΠΌ ΡƒΡ€ΠΎΠ²Π½Π΅ΠΌ Π² Ρ€Π΅Π·Π΅Ρ€Π²ΡƒΠ°Ρ€ с высоким ΡƒΡ€ΠΎΠ²Π½Π΅ΠΌ, ΠΈ Π΄Π²ΠΈΠΆΠ΅Π½ΠΈΠ΅ Π²ΠΎΠ΄Ρ‹ ΠΏΠΎ Ρ‚Ρ€ΡƒΠ±ΠΎΠΏΡ€ΠΎΠ²ΠΎΠ΄Ρƒ ΠΎΠ±Ρ€Π°Ρ‚Π½ΠΎ ΠΊ исходному ΡƒΡ€ΠΎΠ²Π½ΡŽ прСдставляСт собой высвобоТдСниС энСргии, Π½Π°ΠΊΠΎΠΏΠ»Π΅Π½Π½ΠΎΠΉ ΠΎΡ‚ ΠΏΡ€Π΅Π΄Ρ‹Π΄ΡƒΡ‰Π΅ΠΉ ΠΎΡ‚ΠΊΠ°Ρ‡ΠΊΠΈ. Если Π²ΠΎΠ΄Π° пСрСкачиваСтся Π½Π° Π΅Ρ‰Π΅ Π±ΠΎΠ»Π΅Π΅ высокий ΡƒΡ€ΠΎΠ²Π΅Π½ΡŒ, для этого потрСбуСтся Π΅Ρ‰Π΅ большС энСргии, Ρ‚Π°ΠΊΠΈΠΌ ΠΎΠ±Ρ€Π°Π·ΠΎΠΌ, Π±ΡƒΠ΄Π΅Ρ‚ сохранСно большС энСргии, ΠΈ большС энСргии Π±ΡƒΠ΄Π΅Ρ‚ высвобоТдСно, Ссли Π²ΠΎΠ΄Π΅ ΠΏΠΎΠ·Π²ΠΎΠ»ΠΈΡ‚ΡŒ снова Ρ‚Π΅Ρ‡ΡŒ ΠΏΠΎ Ρ‚Ρ€ΡƒΠ±Π΅ ΠΎΠ±Ρ€Π°Ρ‚Π½ΠΎ Π²Π½ΠΈΠ·:

Π­Π»Π΅ΠΊΡ‚Ρ€ΠΎΠ½Ρ‹ ΠΌΠ°Π»ΠΎ Ρ‡Π΅ΠΌ ΠΎΡ‚Π»ΠΈΡ‡Π°ΡŽΡ‚ΡΡ.Если ΠΌΡ‹ ΠΏΡ€ΠΎΡ‚ΠΈΡ€Π°Π΅ΠΌ воск ΠΈ ΡˆΠ΅Ρ€ΡΡ‚ΡŒ вмСстС, ΠΌΡ‹ Β«ΠΎΡ‚ΠΊΠ°Ρ‡ΠΈΠ²Π°Π΅ΠΌΒ» элСктроны ΠΎΡ‚ ΠΈΡ… Π½ΠΎΡ€ΠΌΠ°Π»ΡŒΠ½Ρ‹Ρ… Β«ΡƒΡ€ΠΎΠ²Π½Π΅ΠΉΒ», создавая условия, ΠΏΡ€ΠΈ ΠΊΠΎΡ‚ΠΎΡ€Ρ‹Ρ… сущСствуСт сила ΠΌΠ΅ΠΆΠ΄Ρƒ ΠΏΠ°Ρ€Π°Ρ„ΠΈΠ½ΠΎΠΌ ΠΈ ΡˆΠ΅Ρ€ΡΡ‚ΡŒΡŽ, ΠΏΠΎΡΠΊΠΎΠ»ΡŒΠΊΡƒ элСктроны стрСмятся Π²ΠΎΡΡΡ‚Π°Π½ΠΎΠ²ΠΈΡ‚ΡŒ свои ΠΏΡ€Π΅ΠΆΠ½ΠΈΠ΅ полоТСния (ΠΈ Π±Π°Π»Π°Π½ΡΠΈΡ€ΠΎΠ²Π°Ρ‚ΡŒ Π² своих ΡΠΎΠΎΡ‚Π²Π΅Ρ‚ΡΡ‚Π²ΡƒΡŽΡ‰ΠΈΠ΅ Π°Ρ‚ΠΎΠΌΡ‹). Π‘ΠΈΠ»Π°, ΠΏΡ€ΠΈΡ‚ΡΠ³ΠΈΠ²Π°ΡŽΡ‰Π°Ρ элСктроны ΠΎΠ±Ρ€Π°Ρ‚Π½ΠΎ Π² ΠΈΡ… исходноС ΠΏΠΎΠ»ΠΎΠΆΠ΅Π½ΠΈΠ΅ Π²ΠΎΠΊΡ€ΡƒΠ³ ΠΏΠΎΠ»ΠΎΠΆΠΈΡ‚Π΅Π»ΡŒΠ½Ρ‹Ρ… ядСр ΠΈΡ… Π°Ρ‚ΠΎΠΌΠΎΠ², Π°Π½Π°Π»ΠΎΠ³ΠΈΡ‡Π½Π° силС Π³Ρ€Π°Π²ΠΈΡ‚Π°Ρ†ΠΈΠΈ, Π΄Π΅ΠΉΡΡ‚Π²ΡƒΡŽΡ‰Π΅ΠΉ Π½Π° Π²ΠΎΠ΄Ρƒ Π² Ρ€Π΅Π·Π΅Ρ€Π²ΡƒΠ°Ρ€Π΅, ΠΏΡ‹Ρ‚Π°ΡΡΡŒ Π²Π΅Ρ€Π½ΡƒΡ‚ΡŒ Π΅Π΅ ΠΊ ΠΏΡ€Π΅ΠΆΠ½Π΅ΠΌΡƒ ΡƒΡ€ΠΎΠ²Π½ΡŽ. Подобно Ρ‚ΠΎΠΌΡƒ, ΠΊΠ°ΠΊ ΠΏΠ΅Ρ€Π΅ΠΊΠ°Ρ‡ΠΊΠ° Π²ΠΎΠ΄Ρ‹ Π½Π° Π±ΠΎΠ»Π΅Π΅ высокий ΡƒΡ€ΠΎΠ²Π΅Π½ΡŒ ΠΏΡ€ΠΈΠ²ΠΎΠ΄ΠΈΡ‚ ΠΊ накоплСнию энСргии, Β«ΠΏΠ΅Ρ€Π΅ΠΊΠ°Ρ‡ΠΊΠ°Β» элСктронов для создания дисбаланса элСктричСского заряда ΠΏΡ€ΠΈΠ²ΠΎΠ΄ΠΈΡ‚ ΠΊ накоплСнию ΠΎΠΏΡ€Π΅Π΄Π΅Π»Π΅Π½Π½ΠΎΠ³ΠΎ количСства энСргии Π² этом дисбалансС.И Ρ‚ΠΎΡ‡Π½ΠΎ Ρ‚Π°ΠΊ ΠΆΠ΅, ΠΊΠ°ΠΊ обСспСчСниС ΠΏΡƒΡ‚ΠΈ для Π²ΠΎΠ΄Ρ‹ ΡΡ‚Π΅ΠΊΠ°Ρ‚ΡŒ ΠΎΠ±Ρ€Π°Ρ‚Π½ΠΎ с высоты Ρ€Π΅Π·Π΅Ρ€Π²ΡƒΠ°Ρ€Π° ΠΏΡ€ΠΈΠ²ΠΎΠ΄ΠΈΡ‚ ΠΊ Π²Ρ‹ΡΠ²ΠΎΠ±ΠΎΠΆΠ΄Π΅Π½ΠΈΡŽ этой Π½Π°ΠΊΠΎΠΏΠ»Π΅Π½Π½ΠΎΠΉ энСргии, прСдоставлСниС возмоТности элСктронам Ρ‚Π΅Ρ‡ΡŒ ΠΎΠ±Ρ€Π°Ρ‚Π½ΠΎ ΠΊ ΠΈΡ… ΠΏΠ΅Ρ€Π²ΠΎΠ½Π°Ρ‡Π°Π»ΡŒΠ½Ρ‹ΠΌ «уровням» ΠΏΡ€ΠΈΠ²ΠΎΠ΄ΠΈΡ‚ ΠΊ Π²Ρ‹ΡΠ²ΠΎΠ±ΠΎΠΆΠ΄Π΅Π½ΠΈΡŽ Π½Π°ΠΊΠΎΠΏΠ»Π΅Π½Π½ΠΎΠΉ энСргии. Когда носитСли заряда находятся Π² этом статичСском состоянии (Ρ‚ΠΎΡ‡Π½ΠΎ Ρ‚Π°ΠΊ ΠΆΠ΅, ΠΊΠ°ΠΊ Π²ΠΎΠ΄Π° Π½Π΅ΠΏΠΎΠ΄Π²ΠΈΠΆΠ½ΠΎ сидит высоко Π² Ρ€Π΅Π·Π΅Ρ€Π²ΡƒΠ°Ρ€Π΅), запасСнная Ρ‚Π°ΠΌ энСргия называСтся ΠΏΠΎΡ‚Π΅Π½Ρ†ΠΈΠ°Π»ΡŒΠ½ΠΎΠΉ энСргиСй , ΠΏΠΎΡ‚ΠΎΠΌΡƒ Ρ‡Ρ‚ΠΎ Ρƒ Π½Π΅Π΅ Π΅ΡΡ‚ΡŒ Π²ΠΎΠ·ΠΌΠΎΠΆΠ½ΠΎΡΡ‚ΡŒ (ΠΏΠΎΡ‚Π΅Π½Ρ†ΠΈΠ°Π») высвобоТдСния, которая Π½Π΅ Π±Ρ‹Π»Π° ΠΏΠΎΠ»Π½ΠΎΡΡ‚ΡŒΡŽ Ρ€Π΅Π°Π»ΠΈΠ·ΠΎΠ²Π°Π½Π°. всС ΠΆΠ΅.

ПониманиС ΠΊΠΎΠ½Ρ†Π΅ΠΏΡ†ΠΈΠΈ напряТСния

Когда носитСли заряда находятся Π² этом статичСском состоянии (Ρ‚ΠΎΡ‡Π½ΠΎ Ρ‚Π°ΠΊ ΠΆΠ΅, ΠΊΠ°ΠΊ Π²ΠΎΠ΄Π°, нСподвиТная, высоко Π² Ρ€Π΅Π·Π΅Ρ€Π²ΡƒΠ°Ρ€Π΅), энСргия, хранящаяся Ρ‚Π°ΠΌ, называСтся ΠΏΠΎΡ‚Π΅Π½Ρ†ΠΈΠ°Π»ΡŒΠ½ΠΎΠΉ энСргиСй, ΠΏΠΎΡ‚ΠΎΠΌΡƒ Ρ‡Ρ‚ΠΎ Ρƒ Π½Π΅Π΅ Π΅ΡΡ‚ΡŒ Π²ΠΎΠ·ΠΌΠΎΠΆΠ½ΠΎΡΡ‚ΡŒ (ΠΏΠΎΡ‚Π΅Π½Ρ†ΠΈΠ°Π») высвобоТдСния, которая Π΅Ρ‰Π΅ Π½Π΅ ΠΏΠΎΠ»Π½ΠΎΡΡ‚ΡŒΡŽ Ρ€Π΅Π°Π»ΠΈΠ·ΠΎΠ²Π°Π½Π°. . Когда Π²Ρ‹ Ρ‚Π΅Ρ€Π·Π°Π΅Ρ‚Π΅ ΠΎΠ±ΡƒΠ²ΡŒ с Ρ€Π΅Π·ΠΈΠ½ΠΎΠ²ΠΎΠΉ подошвой ΠΎ Ρ‚ΠΊΠ°Π½Π΅Π²Ρ‹ΠΉ ΠΊΠΎΠ²Π΅Ρ€ Π² сухой дСнь, Π²Ρ‹ создаСтС дисбаланс элСктричСского заряда ΠΌΠ΅ΠΆΠ΄Ρƒ Π²Π°ΠΌΠΈ ΠΈ ΠΊΠΎΠ²Ρ€ΠΎΠΌ. ΠŸΡ€ΠΈ Ρ†Π°Ρ€Π°ΠΏΠ°Π½ΠΈΠΈ Π½ΠΎΠ³Π°ΠΌΠΈ накапливаСтся энСргия Π² Π²ΠΈΠ΄Π΅ дисбаланса зарядов, вытСсняСмых ΠΈΠ· ΠΈΡ… ΠΏΠ΅Ρ€Π²ΠΎΠ½Π°Ρ‡Π°Π»ΡŒΠ½Ρ‹Ρ… мСст.Π­Ρ‚ΠΎΡ‚ заряд (статичСскоС элСктричСство) являСтся стационарным, ΠΈ Π²Ρ‹ Π²ΠΎΠΎΠ±Ρ‰Π΅ Π½Π΅ Π·Π°ΠΌΠ΅Ρ‚ΠΈΡ‚Π΅, Ρ‡Ρ‚ΠΎ энСргия накапливаСтся. Однако, ΠΊΠ°ΠΊ Ρ‚ΠΎΠ»ΡŒΠΊΠΎ Π²Ρ‹ ΠΏΠΎΠ»ΠΎΠΆΠΈΡ‚Π΅ Ρ€ΡƒΠΊΡƒ Π½Π° ΠΌΠ΅Ρ‚Π°Π»Π»ΠΈΡ‡Π΅ΡΠΊΡƒΡŽ Π΄Π²Π΅Ρ€Π½ΡƒΡŽ Ρ€ΡƒΡ‡ΠΊΡƒ (с большой ΠΏΠΎΠ΄Π²ΠΈΠΆΠ½ΠΎΡΡ‚ΡŒΡŽ элСктронов для Π½Π΅ΠΉΡ‚Ρ€Π°Π»ΠΈΠ·Π°Ρ†ΠΈΠΈ вашСго элСктричСского заряда), эта накоплСнная энСргия Π±ΡƒΠ΄Π΅Ρ‚ высвобоТдСна Π² Π²ΠΈΠ΄Π΅ Π²Π½Π΅Π·Π°ΠΏΠ½ΠΎΠ³ΠΎ ΠΏΠΎΡ‚ΠΎΠΊΠ° заряда Ρ‡Π΅Ρ€Π΅Π· Π²Π°ΡˆΡƒ Ρ€ΡƒΠΊΡƒ, ΠΈ Π²Ρ‹ Π±ΡƒΠ΄Π΅Ρ‚Π΅ Π²ΠΎΡΠΏΡ€ΠΈΠ½ΠΈΠΌΠ°Ρ‚ΡŒ Π΅Π΅ ΠΊΠ°ΠΊ ΠΏΠΎΡ€Π°ΠΆΠ΅Π½ΠΈΠ΅ элСктричСским Ρ‚ΠΎΠΊΠΎΠΌ! Π­Ρ‚Π° ΠΏΠΎΡ‚Π΅Π½Ρ†ΠΈΠ°Π»ΡŒΠ½Π°Ρ энСргия, хранящаяся Π² Π²ΠΈΠ΄Π΅ дисбаланса элСктричСского заряда ΠΈ способная ΠΏΡ€ΠΎΠ²ΠΎΡ†ΠΈΡ€ΠΎΠ²Π°Ρ‚ΡŒ ΠΏΡ€ΠΎΡ‚Π΅ΠΊΠ°Π½ΠΈΠ΅ носитСлСй заряда Ρ‡Π΅Ρ€Π΅Π· ΠΏΡ€ΠΎΠ²ΠΎΠ΄Π½ΠΈΠΊ, ΠΌΠΎΠΆΠ΅Ρ‚ Π±Ρ‹Ρ‚ΡŒ Π²Ρ‹Ρ€Π°ΠΆΠ΅Π½Π° Ρ‚Π΅Ρ€ΠΌΠΈΠ½ΠΎΠΌ, Π½Π°Π·Ρ‹Π²Π°Π΅ΠΌΡ‹ΠΌ напряТСниСм, ΠΊΠΎΡ‚ΠΎΡ€ΠΎΠ΅ тСхничСски являСтся ΠΌΠ΅Ρ€ΠΎΠΉ ΠΏΠΎΡ‚Π΅Π½Ρ†ΠΈΠ°Π»ΡŒΠ½ΠΎΠΉ энСргии Π½Π° Π΅Π΄ΠΈΠ½ΠΈΡ†Ρƒ заряда ΠΈΠ»ΠΈ Ρ‡Π΅Π³ΠΎ-Ρ‚ΠΎ, Ρ‡Ρ‚ΠΎ Ρ„ΠΈΠ·ΠΈΠΊ ΠΌΠΎΠ³ Π±Ρ‹ Π½Π°Π·Ρ‹Π²Π°ΡŽΡ‚ ΡƒΠ΄Π΅Π»ΡŒΠ½ΠΎΠΉ ΠΏΠΎΡ‚Π΅Π½Ρ†ΠΈΠ°Π»ΡŒΠ½ΠΎΠΉ энСргиСй.

ΠžΠΏΡ€Π΅Π΄Π΅Π»Π΅Π½ΠΈΠ΅ напряТСния

ΠžΠΏΡ€Π΅Π΄Π΅Π»ΡΠ΅ΠΌΠΎΠ΅ Π² контСкстС статичСского элСктричСства, напряТСниС — это ΠΌΠ΅Ρ€Π° Ρ€Π°Π±ΠΎΡ‚Ρ‹, Π½Π΅ΠΎΠ±Ρ…ΠΎΠ΄ΠΈΠΌΠΎΠΉ для пСрСмСщСния Π΅Π΄ΠΈΠ½ΠΈΡ‡Π½ΠΎΠ³ΠΎ заряда ΠΈΠ· ΠΎΠ΄Π½ΠΎΠ³ΠΎ мСста Π² Π΄Ρ€ΡƒΠ³ΠΎΠ΅, ΠΏΡ€ΠΎΡ‚ΠΈΠ² силы, которая пытаСтся ΡΠΎΡ…Ρ€Π°Π½ΠΈΡ‚ΡŒ баланс элСктричСских зарядов. Π’ контСкстС источников элСктроэнСргии напряТСниС — это количСство доступной ΠΏΠΎΡ‚Π΅Π½Ρ†ΠΈΠ°Π»ΡŒΠ½ΠΎΠΉ энСргии (Ρ€Π°Π±ΠΎΡ‚Π°, ΠΊΠΎΡ‚ΠΎΡ€ΡƒΡŽ Π½Π΅ΠΎΠ±Ρ…ΠΎΠ΄ΠΈΠΌΠΎ Π²Ρ‹ΠΏΠΎΠ»Π½ΠΈΡ‚ΡŒ) Π½Π° Π΅Π΄ΠΈΠ½ΠΈΡ†Ρƒ заряда для пСрСмСщСния зарядов Ρ‡Π΅Ρ€Π΅Π· ΠΏΡ€ΠΎΠ²ΠΎΠ΄Π½ΠΈΠΊ, ΠΏΠΎΡΠΊΠΎΠ»ΡŒΠΊΡƒ напряТСниС — это Π²Ρ‹Ρ€Π°ΠΆΠ΅Π½ΠΈΠ΅ ΠΏΠΎΡ‚Π΅Π½Ρ†ΠΈΠ°Π»ΡŒΠ½ΠΎΠΉ энСргии, ΠΏΡ€Π΅Π΄ΡΡ‚Π°Π²Π»ΡΡŽΡ‰Π΅Π΅ Π²ΠΎΠ·ΠΌΠΎΠΆΠ½ΠΎΡΡ‚ΡŒ ΠΈΠ»ΠΈ ΠΏΠΎΡ‚Π΅Π½Ρ†ΠΈΠ°Π» для выдСлСния энСргии ΠΊΠΎΠ³Π΄Π° заряд пСрСмСщаСтся с ΠΎΠ΄Π½ΠΎΠ³ΠΎ «уровня» Π½Π° Π΄Ρ€ΡƒΠ³ΠΎΠΉ, ΠΎΠ½ всСгда находится ΠΌΠ΅ΠΆΠ΄Ρƒ двумя Ρ‚ΠΎΡ‡ΠΊΠ°ΠΌΠΈ.Рассмотрим аналогию с Π²ΠΎΠ΄ΠΎΡ…Ρ€Π°Π½ΠΈΠ»ΠΈΡ‰Π΅ΠΌ:

Из-Π·Π° Ρ€Π°Π·Π½ΠΈΡ†Ρ‹ Π² высотС падСния сущСствуСт Π²Π΅Ρ€ΠΎΡΡ‚Π½ΠΎΡΡ‚ΡŒ Ρ‚ΠΎΠ³ΠΎ, Ρ‡Ρ‚ΠΎ Π³ΠΎΡ€Π°Π·Π΄ΠΎ большС энСргии Π±ΡƒΠ΄Π΅Ρ‚ Π²Ρ‹ΠΏΡƒΡ‰Π΅Π½ΠΎ ΠΈΠ· Ρ€Π΅Π·Π΅Ρ€Π²ΡƒΠ°Ρ€Π° Ρ‡Π΅Ρ€Π΅Π· Ρ‚Ρ€ΡƒΠ±ΠΎΠΏΡ€ΠΎΠ²ΠΎΠ΄ Π² Ρ‚ΠΎΡ‡ΠΊΡƒ 2, Ρ‡Π΅ΠΌ Π² Ρ‚ΠΎΡ‡ΠΊΡƒ 1. ΠŸΡ€ΠΈΠ½Ρ†ΠΈΠΏ ΠΌΠΎΠΆΠ½ΠΎ ΠΈΠ½Ρ‚ΡƒΠΈΡ‚ΠΈΠ²Π½ΠΎ ΠΏΠΎΠ½ΡΡ‚ΡŒ ΠΏΡ€ΠΈ ΠΏΠ°Π΄Π΅Π½ΠΈΠΈ камня: Ρ‡Ρ‚ΠΎ ΠΏΡ€ΠΈΠ²ΠΎΠ΄ΠΈΡ‚ ΠΊ Π±ΠΎΠ»Π΅Π΅ ΡΠΈΠ»ΡŒΠ½Ρ‹ΠΉ ΡƒΠ΄Π°Ρ€, камСнь ΡƒΠΏΠ°Π» с высоты ΠΎΠ΄Π½ΠΎΠ³ΠΎ Ρ„ΡƒΡ‚Π° ΠΈΠ»ΠΈ Ρ‚ΠΎΡ‚ ΠΆΠ΅ камСнь ΡƒΠΏΠ°Π» с высоты ΠΎΠ΄Π½ΠΎΠΉ ΠΌΠΈΠ»ΠΈ? ΠžΡ‡Π΅Π²ΠΈΠ΄Π½ΠΎ, Ρ‡Ρ‚ΠΎ ΠΏΠ°Π΄Π΅Π½ΠΈΠ΅ с большСй высоты ΠΏΡ€ΠΈΠ²ΠΎΠ΄ΠΈΡ‚ ΠΊ Π²Ρ‹ΡΠ²ΠΎΠ±ΠΎΠΆΠ΄Π΅Π½ΠΈΡŽ большСй энСргии (Π±ΠΎΠ»Π΅Π΅ ΡΠΈΠ»ΡŒΠ½ΠΎΠΌΡƒ ΡƒΠ΄Π°Ρ€Ρƒ).ΠœΡ‹ Π½Π΅ ΠΌΠΎΠΆΠ΅ΠΌ ΠΎΡ†Π΅Π½ΠΈΡ‚ΡŒ количСство Π½Π°ΠΊΠΎΠΏΠ»Π΅Π½Π½ΠΎΠΉ энСргии Π² Π²ΠΎΠ΄Π½ΠΎΠΌ Ρ€Π΅Π·Π΅Ρ€Π²ΡƒΠ°Ρ€Π΅, просто ΠΈΠ·ΠΌΠ΅Ρ€ΠΈΠ² объСм Π²ΠΎΠ΄Ρ‹, Ρ‚ΠΎΡ‡Π½ΠΎ Ρ‚Π°ΠΊ ΠΆΠ΅, ΠΊΠ°ΠΊ ΠΌΡ‹ ΠΌΠΎΠΆΠ΅ΠΌ ΠΏΡ€Π΅Π΄ΡΠΊΠ°Π·Π°Ρ‚ΡŒ ΡΠ΅Ρ€ΡŒΠ΅Π·Π½ΠΎΡΡ‚ΡŒ ΡƒΠ΄Π°Ρ€Π° ΠΏΠ°Π΄Π°ΡŽΡ‰Π΅ΠΉ ΠΏΠΎΡ€ΠΎΠ΄Ρ‹, просто зная вСс ΠΏΠΎΡ€ΠΎΠ΄Ρ‹: Π² ΠΎΠ±ΠΎΠΈΡ… случаях ΠΌΡ‹ Ρ‚Π°ΠΊΠΆΠ΅ Π΄ΠΎΠ»ΠΆΠ½Ρ‹ ΡƒΡ‡ΠΈΡ‚Ρ‹Π²Π°Ρ‚ΡŒ, ΠΊΠ°ΠΊ Π΄Π°Π»Π΅ΠΊΠΎ эти массы ΡƒΠΏΠ°Π΄ΡƒΡ‚ с ΠΈΡ… Π½Π°Ρ‡Π°Π»ΡŒΠ½ΠΎΠΉ высоты. ΠšΠΎΠ»ΠΈΡ‡Π΅ΡΡ‚Π²ΠΎ энСргии, высвобоТдаСмой ΠΏΡ€ΠΈ ΠΏΠ°Π΄Π΅Π½ΠΈΠΈ массы, зависит ΠΎΡ‚ расстояния ΠΌΠ΅ΠΆΠ΄Ρƒ Π΅Π³ΠΎ Π½Π°Ρ‡Π°Π»ΡŒΠ½ΠΎΠΉ ΠΈ ΠΊΠΎΠ½Π΅Ρ‡Π½ΠΎΠΉ Ρ‚ΠΎΡ‡ΠΊΠ°ΠΌΠΈ. Π’ΠΎΡ‡Π½ΠΎ Ρ‚Π°ΠΊ ΠΆΠ΅ ΠΏΠΎΡ‚Π΅Π½Ρ†ΠΈΠ°Π»ΡŒΠ½Π°Ρ энСргия, доступная для пСрСмСщСния носитСлСй заряда ΠΈΠ· ΠΎΠ΄Π½ΠΎΠΉ Ρ‚ΠΎΡ‡ΠΊΠΈ Π² Π΄Ρ€ΡƒΠ³ΡƒΡŽ, зависит ΠΎΡ‚ этих Π΄Π²ΡƒΡ… Ρ‚ΠΎΡ‡Π΅ΠΊ.Π‘Π»Π΅Π΄ΠΎΠ²Π°Ρ‚Π΅Π»ΡŒΠ½ΠΎ, напряТСниС всСгда выраТаСтся ΠΊΠ°ΠΊ Π²Π΅Π»ΠΈΡ‡ΠΈΠ½Π° ΠΌΠ΅ΠΆΠ΄Ρƒ двумя Ρ‚ΠΎΡ‡ΠΊΠ°ΠΌΠΈ. Достаточно интСрСсно, Ρ‡Ρ‚ΠΎ аналогия с массой, ΠΏΠΎΡ‚Π΅Π½Ρ†ΠΈΠ°Π»ΡŒΠ½ΠΎ Β«ΠΏΠ°Π΄Π°ΡŽΡ‰Π΅ΠΉΒ» с ΠΎΠ΄Π½ΠΎΠΉ высоты Π½Π° Π΄Ρ€ΡƒΠ³ΡƒΡŽ, являСтся Π½Π°ΡΡ‚ΠΎΠ»ΡŒΠΊΠΎ ΡƒΠ΄Π°Ρ‡Π½ΠΎΠΉ модСлью, Ρ‡Ρ‚ΠΎ напряТСниС ΠΌΠ΅ΠΆΠ΄Ρƒ двумя Ρ‚ΠΎΡ‡ΠΊΠ°ΠΌΠΈ ΠΈΠ½ΠΎΠ³Π΄Π° Π½Π°Π·Ρ‹Π²Π°ΡŽΡ‚ ΠΏΠ°Π΄Π΅Π½ΠΈΠ΅ΠΌ напряТСния .

Π“Π΅Π½Π΅Ρ€ΠΈΡ€ΡƒΡŽΡ‰Π΅Π΅ напряТСниС

НапряТСниС ΠΌΠΎΠΆΠ½ΠΎ Π³Π΅Π½Π΅Ρ€ΠΈΡ€ΠΎΠ²Π°Ρ‚ΡŒ Π΄Ρ€ΡƒΠ³ΠΈΠΌΠΈ способами, ΠΊΡ€ΠΎΠΌΠ΅ трСния ΠΌΠ°Ρ‚Π΅Ρ€ΠΈΠ°Π»ΠΎΠ² ΠΎΠΏΡ€Π΅Π΄Π΅Π»Π΅Π½Π½Ρ‹Ρ… Ρ‚ΠΈΠΏΠΎΠ² Π΄Ρ€ΡƒΠ³ ΠΎ Π΄Ρ€ΡƒΠ³Π°. Π₯имичСскиС Ρ€Π΅Π°ΠΊΡ†ΠΈΠΈ, лучистая энСргия ΠΈ влияниС ΠΌΠ°Π³Π½Π΅Ρ‚ΠΈΠ·ΠΌΠ° Π½Π° ΠΏΡ€ΠΎΠ²ΠΎΠ΄Π½ΠΈΠΊΠΈ — Π²ΠΎΡ‚ нСсколько способов создания напряТСния.Π‘ΠΎΠΎΡ‚Π²Π΅Ρ‚ΡΡ‚Π²ΡƒΡŽΡ‰ΠΈΠΌΠΈ ΠΏΡ€ΠΈΠΌΠ΅Ρ€Π°ΠΌΠΈ этих Ρ‚Ρ€Π΅Ρ… источников напряТСния ΡΠ²Π»ΡΡŽΡ‚ΡΡ Π±Π°Ρ‚Π°Ρ€Π΅ΠΈ, солнСчныС элСмСнты ΠΈ Π³Π΅Π½Π΅Ρ€Π°Ρ‚ΠΎΡ€Ρ‹ (Π½Π°ΠΏΡ€ΠΈΠΌΠ΅Ρ€, Β«Π³Π΅Π½Π΅Ρ€Π°Ρ‚ΠΎΡ€ ΠΏΠ΅Ρ€Π΅ΠΌΠ΅Π½Π½ΠΎΠ³ΠΎ Ρ‚ΠΎΠΊΠ°Β» ΠΏΠΎΠ΄ ΠΊΠ°ΠΏΠΎΡ‚ΠΎΠΌ вашСго автомобиля). На Π΄Π°Π½Π½Ρ‹ΠΉ ΠΌΠΎΠΌΠ΅Π½Ρ‚ ΠΌΡ‹ Π½Π΅ Π±ΡƒΠ΄Π΅ΠΌ Π²Π΄Π°Π²Π°Ρ‚ΡŒΡΡ Π² подробности Ρ‚ΠΎΠ³ΠΎ, ΠΊΠ°ΠΊ Ρ€Π°Π±ΠΎΡ‚Π°Π΅Ρ‚ ΠΊΠ°ΠΆΠ΄Ρ‹ΠΉ ΠΈΠ· этих источников напряТСния — Π±ΠΎΠ»Π΅Π΅ Π²Π°ΠΆΠ½ΠΎ Ρ‚ΠΎ, Ρ‡Ρ‚ΠΎ ΠΌΡ‹ ΠΏΠΎΠ½ΠΈΠΌΠ°Π΅ΠΌ, ΠΊΠ°ΠΊ источники напряТСния ΠΌΠΎΠ³ΡƒΡ‚ ΠΏΡ€ΠΈΠΌΠ΅Π½ΡΡ‚ΡŒΡΡ для создания ΠΏΠΎΡ‚ΠΎΠΊΠ° заряда Π² элСктричСской Ρ†Π΅ΠΏΠΈ. Π”Π°Π²Π°ΠΉΡ‚Π΅ возьмСм символ химичСской Π±Π°Ρ‚Π°Ρ€Π΅ΠΈ ΠΈ шаг Π·Π° шагом построим схСму:

Как Ρ€Π°Π±ΠΎΡ‚Π°ΡŽΡ‚ источники напряТСния?

Π›ΡŽΠ±ΠΎΠΉ источник напряТСния, Π²ΠΊΠ»ΡŽΡ‡Π°Ρ аккумуляторныС Π±Π°Ρ‚Π°Ρ€Π΅ΠΈ, ΠΈΠΌΠ΅Π΅Ρ‚ Π΄Π²Π΅ Ρ‚ΠΎΡ‡ΠΊΠΈ элСктричСского ΠΊΠΎΠ½Ρ‚Π°ΠΊΡ‚Π°.Π’ этом случаС Ρƒ нас Π΅ΡΡ‚ΡŒ Ρ‚ΠΎΡ‡ΠΊΠ° 1 ΠΈ Ρ‚ΠΎΡ‡ΠΊΠ° 2 Π½Π° ΠΏΡ€ΠΈΠ²Π΅Π΄Π΅Π½Π½ΠΎΠΉ Π²Ρ‹ΡˆΠ΅ Π΄ΠΈΠ°Π³Ρ€Π°ΠΌΠΌΠ΅. Π“ΠΎΡ€ΠΈΠ·ΠΎΠ½Ρ‚Π°Π»ΡŒΠ½Ρ‹Π΅ Π»ΠΈΠ½ΠΈΠΈ Ρ€Π°Π·Π½ΠΎΠΉ Π΄Π»ΠΈΠ½Ρ‹ ΡƒΠΊΠ°Π·Ρ‹Π²Π°ΡŽΡ‚ Π½Π° Ρ‚ΠΎ, Ρ‡Ρ‚ΠΎ это батарСя, ΠΈ Π΄ΠΎΠΏΠΎΠ»Π½ΠΈΡ‚Π΅Π»ΡŒΠ½ΠΎ ΡƒΠΊΠ°Π·Ρ‹Π²Π°ΡŽΡ‚ Π½Π°ΠΏΡ€Π°Π²Π»Π΅Π½ΠΈΠ΅, Π² ΠΊΠΎΡ‚ΠΎΡ€ΠΎΠΌ напряТСниС этой Π±Π°Ρ‚Π°Ρ€Π΅ΠΈ Π±ΡƒΠ΄Π΅Ρ‚ ΠΏΡ‹Ρ‚Π°Ρ‚ΡŒΡΡ ΠΏΡ€ΠΎΡ‚ΠΎΠ»ΠΊΠ½ΡƒΡ‚ΡŒ носитСли заряда ΠΏΠΎ Ρ†Π΅ΠΏΠΈ. Π’ΠΎΡ‚ Ρ„Π°ΠΊΡ‚, Ρ‡Ρ‚ΠΎ Π³ΠΎΡ€ΠΈΠ·ΠΎΠ½Ρ‚Π°Π»ΡŒΠ½Ρ‹Π΅ Π»ΠΈΠ½ΠΈΠΈ Π² символС Π±Π°Ρ‚Π°Ρ€Π΅ΠΈ каТутся Ρ€Π°Π·Π΄Π΅Π»Π΅Π½Π½Ρ‹ΠΌΠΈ (ΠΈ, Ρ‚Π°ΠΊΠΈΠΌ ΠΎΠ±Ρ€Π°Π·ΠΎΠΌ, Π½Π΅ ΠΌΠΎΠ³ΡƒΡ‚ ΡΠ»ΡƒΠΆΠΈΡ‚ΡŒ ΠΏΡƒΡ‚Π΅ΠΌ для ΠΏΠΎΡ‚ΠΎΠΊΠ° заряда), Π½Π΅ Π²Ρ‹Π·Ρ‹Π²Π°Π΅Ρ‚ бСспокойства: Π² Ρ€Π΅Π°Π»ΡŒΠ½ΠΎΠΉ ΠΆΠΈΠ·Π½ΠΈ эти Π³ΠΎΡ€ΠΈΠ·ΠΎΠ½Ρ‚Π°Π»ΡŒΠ½Ρ‹Π΅ Π»ΠΈΠ½ΠΈΠΈ ΠΏΡ€Π΅Π΄ΡΡ‚Π°Π²Π»ΡΡŽΡ‚ собой мСталличСскиС пластины, ΠΏΠΎΠ³Ρ€ΡƒΠΆΠ΅Π½Π½Ρ‹Π΅ Π² ΠΆΠΈΠ΄ΠΊΠΈΠΉ ΠΈΠ»ΠΈ ΠΏΠΎΠ»ΡƒΡ‚Π²Π΅Ρ€Π΄Ρ‹ΠΉ ΠΌΠ°Ρ‚Π΅Ρ€ΠΈΠ°Π». ΠΊΠΎΡ‚ΠΎΡ€Ρ‹ΠΉ Π½Π΅ Ρ‚ΠΎΠ»ΡŒΠΊΠΎ ΠΏΡ€ΠΎΠ²ΠΎΠ΄ΠΈΡ‚ заряды, Π½ΠΎ ΠΈ Π³Π΅Π½Π΅Ρ€ΠΈΡ€ΡƒΠ΅Ρ‚ напряТСниС, Ρ‡Ρ‚ΠΎΠ±Ρ‹ ΠΏΠΎΠ΄Ρ‚ΠΎΠ»ΠΊΠ½ΡƒΡ‚ΡŒ ΠΈΡ…, взаимодСйствуя с пластинами.ΠžΠ±Ρ€Π°Ρ‚ΠΈΡ‚Π΅ Π²Π½ΠΈΠΌΠ°Π½ΠΈΠ΅ Π½Π° малСнькиС Π·Π½Π°Ρ‡ΠΊΠΈ Β«+Β» ΠΈ Β«-Β» нСпосрСдствСнно слСва ΠΎΡ‚ символа Π±Π°Ρ‚Π°Ρ€Π΅ΠΈ. ΠžΡ‚Ρ€ΠΈΡ†Π°Ρ‚Π΅Π»ΡŒΠ½Ρ‹ΠΉ (-) ΠΊΠΎΠ½Π΅Ρ† Π±Π°Ρ‚Π°Ρ€Π΅ΠΈ всСгда являСтся ΠΊΠΎΠ½Ρ†ΠΎΠΌ с самым ΠΊΠΎΡ€ΠΎΡ‚ΠΊΠΈΠΌ Ρ‚ΠΈΡ€Π΅, Π° ΠΏΠΎΠ»ΠΎΠΆΠΈΡ‚Π΅Π»ΡŒΠ½Ρ‹ΠΉ (+) ΠΊΠΎΠ½Π΅Ρ† Π±Π°Ρ‚Π°Ρ€Π΅ΠΈ всСгда являСтся ΠΊΠΎΠ½Ρ†ΠΎΠΌ с самым Π΄Π»ΠΈΠ½Π½Ρ‹ΠΌ Ρ‚ΠΈΡ€Π΅. ΠŸΠΎΠ»ΠΎΠΆΠΈΡ‚Π΅Π»ΡŒΠ½Ρ‹ΠΉ ΠΊΠΎΠ½Π΅Ρ† Π±Π°Ρ‚Π°Ρ€Π΅ΠΈ — это ΠΊΠΎΠ½Π΅Ρ†, ΠΊΠΎΡ‚ΠΎΡ€Ρ‹ΠΉ пытаСтся Π²Ρ‹Ρ‚ΠΎΠ»ΠΊΠ½ΡƒΡ‚ΡŒ ΠΈΠ· Π½Π΅Π΅ носитСли заряда (ΠΏΠΎΠΌΠ½ΠΈΡ‚Π΅, Ρ‡Ρ‚ΠΎ ΠΏΠΎ Ρ‚Ρ€Π°Π΄ΠΈΡ†ΠΈΠΈ ΠΌΡ‹ Π΄ΡƒΠΌΠ°Π΅ΠΌ, Ρ‡Ρ‚ΠΎ носитСли заряда заряТСны ΠΏΠΎΠ»ΠΎΠΆΠΈΡ‚Π΅Π»ΡŒΠ½ΠΎ, хотя элСктроны заряТСны ΠΎΡ‚Ρ€ΠΈΡ†Π°Ρ‚Π΅Π»ΡŒΠ½ΠΎ). Π’ΠΎΡ‡Π½ΠΎ Ρ‚Π°ΠΊ ΠΆΠ΅ ΠΎΡ‚Ρ€ΠΈΡ†Π°Ρ‚Π΅Π»ΡŒΠ½Ρ‹ΠΉ ΠΊΠΎΠ½Π΅Ρ† — это ΠΊΠΎΠ½Π΅Ρ†, ΠΊΠΎΡ‚ΠΎΡ€Ρ‹ΠΉ пытаСтся ΠΏΡ€ΠΈΠ²Π»Π΅Ρ‡ΡŒ носитСли заряда.Когда Β«+Β» ΠΈ Β«-Β» ΠΊΠΎΠ½Ρ†Ρ‹ Π±Π°Ρ‚Π°Ρ€Π΅ΠΈ Π½ΠΈ ΠΊ Ρ‡Π΅ΠΌΡƒ Π½Π΅ ΠΏΠΎΠ΄ΠΊΠ»ΡŽΡ‡Π΅Π½Ρ‹, ΠΌΠ΅ΠΆΠ΄Ρƒ этими двумя Ρ‚ΠΎΡ‡ΠΊΠ°ΠΌΠΈ Π±ΡƒΠ΄Π΅Ρ‚ напряТСниС, Π½ΠΎ Π½Π΅ Π±ΡƒΠ΄Π΅Ρ‚ ΠΏΠΎΡ‚ΠΎΠΊΠ° заряда Ρ‡Π΅Ρ€Π΅Π· Π±Π°Ρ‚Π°Ρ€Π΅ΡŽ, ΠΏΠΎΡ‚ΠΎΠΌΡƒ Ρ‡Ρ‚ΠΎ Π½Π΅Ρ‚ Π½Π΅ΠΏΡ€Π΅Ρ€Ρ‹Π²Π½ΠΎΠ³ΠΎ ΠΏΡƒΡ‚ΠΈ, ΠΏΠΎ ΠΊΠΎΡ‚ΠΎΡ€ΠΎΠΌΡƒ ΠΌΠΎΠ³ΡƒΡ‚ ΠΏΠ΅Ρ€Π΅ΠΌΠ΅Ρ‰Π°Ρ‚ΡŒΡΡ носитСли заряда.

Π’ΠΎΡ‚ ΠΆΠ΅ ΠΏΡ€ΠΈΠ½Ρ†ΠΈΠΏ справСдлив ΠΈ для Π°Π½Π°Π»ΠΎΠ³ΠΈΠΈ с Ρ€Π΅Π·Π΅Ρ€Π²ΡƒΠ°Ρ€ΠΎΠΌ для Π²ΠΎΠ΄Ρ‹ ΠΈ насосом: Π±Π΅Π· Π²ΠΎΠ·Π²Ρ€Π°Ρ‚Π½ΠΎΠΉ Ρ‚Ρ€ΡƒΠ±Ρ‹ ΠΎΠ±Ρ€Π°Ρ‚Π½ΠΎ Π² ΠΏΡ€ΡƒΠ΄ накоплСнная энСргия Π² Ρ€Π΅Π·Π΅Ρ€Π²ΡƒΠ°Ρ€Π΅ Π½Π΅ ΠΌΠΎΠΆΠ΅Ρ‚ Π±Ρ‹Ρ‚ΡŒ Π²Ρ‹ΠΏΡƒΡ‰Π΅Π½Π° Π² Π²ΠΈΠ΄Π΅ ΠΏΠΎΡ‚ΠΎΠΊΠ° Π²ΠΎΠ΄Ρ‹. Когда Ρ€Π΅Π·Π΅Ρ€Π²ΡƒΠ°Ρ€ ΠΏΠΎΠ»Π½ΠΎΡΡ‚ΡŒΡŽ Π·Π°ΠΏΠΎΠ»Π½Π΅Π½, ΠΏΠΎΡ‚ΠΎΠΊ Π½Π΅ ΠΌΠΎΠΆΠ΅Ρ‚ Π²ΠΎΠ·Π½ΠΈΠΊΠ½ΡƒΡ‚ΡŒ, нСзависимо ΠΎΡ‚ Ρ‚ΠΎΠ³ΠΎ, ΠΊΠ°ΠΊΠΎΠ΅ Π΄Π°Π²Π»Π΅Π½ΠΈΠ΅ ΠΌΠΎΠΆΠ΅Ρ‚ ΡΠΎΠ·Π΄Π°Ρ‚ΡŒ насос.Π”ΠΎΠ»ΠΆΠ΅Π½ ΡΡƒΡ‰Π΅ΡΡ‚Π²ΠΎΠ²Π°Ρ‚ΡŒ ΠΏΠΎΠ»Π½Ρ‹ΠΉ ΠΏΡƒΡ‚ΡŒ (ΠΊΠΎΠ½Ρ‚ΡƒΡ€), ΠΏΠΎ ΠΊΠΎΡ‚ΠΎΡ€ΠΎΠΌΡƒ Π²ΠΎΠ΄Π° ΠΌΠΎΠΆΠ΅Ρ‚ Ρ‚Π΅Ρ‡ΡŒ ΠΈΠ· ΠΏΡ€ΡƒΠ΄Π° Π² Ρ€Π΅Π·Π΅Ρ€Π²ΡƒΠ°Ρ€ ΠΈ ΠΎΠ±Ρ€Π°Ρ‚Π½ΠΎ Π² ΠΏΡ€ΡƒΠ΄ для обСспСчСния Π½Π΅ΠΏΡ€Π΅Ρ€Ρ‹Π²Π½ΠΎΠ³ΠΎ ΠΏΠΎΡ‚ΠΎΠΊΠ°. ΠœΡ‹ ΠΌΠΎΠΆΠ΅ΠΌ ΠΎΠ±Π΅ΡΠΏΠ΅Ρ‡ΠΈΡ‚ΡŒ Ρ‚Π°ΠΊΠΎΠΉ ΠΏΡƒΡ‚ΡŒ для Π±Π°Ρ‚Π°Ρ€Π΅ΠΈ, соСдинив кусок ΠΏΡ€ΠΎΠ²ΠΎΠ΄Π° ΠΎΡ‚ ΠΎΠ΄Π½ΠΎΠ³ΠΎ ΠΊΠΎΠ½Ρ†Π° Π±Π°Ρ‚Π°Ρ€Π΅ΠΈ ΠΊ Π΄Ρ€ΡƒΠ³ΠΎΠΌΡƒ. Ѐормируя Ρ†Π΅ΠΏΡŒ с ΠΏΠ΅Ρ‚Π»Π΅ΠΉ ΠΈΠ· ΠΏΡ€ΠΎΠ²ΠΎΠ»ΠΎΠΊΠΈ, ΠΌΡ‹ ΠΈΠ½ΠΈΡ†ΠΈΠΈΡ€ΡƒΠ΅ΠΌ Π½Π΅ΠΏΡ€Π΅Ρ€Ρ‹Π²Π½Ρ‹ΠΉ ΠΏΠΎΡ‚ΠΎΠΊ заряда ΠΏΠΎ часовой стрСлкС:

ПониманиС ΠΊΠΎΠ½Ρ†Π΅ΠΏΡ†ΠΈΠΈ элСктричСского Ρ‚ΠΎΠΊΠ°

Пока батарСя ΠΏΡ€ΠΎΠ΄ΠΎΠ»ΠΆΠ°Π΅Ρ‚ Π²Ρ‹Ρ€Π°Π±Π°Ρ‚Ρ‹Π²Π°Ρ‚ΡŒ напряТСниС ΠΈ Π½Π΅ΠΏΡ€Π΅Ρ€Ρ‹Π²Π½ΠΎΡΡ‚ΡŒ элСктричСского ΠΏΡƒΡ‚ΠΈ Π½Π΅ Π½Π°Ρ€ΡƒΡˆΠ΅Π½Π°, носитСли заряда Π±ΡƒΠ΄ΡƒΡ‚ ΠΏΡ€ΠΎΠ΄ΠΎΠ»ΠΆΠ°Ρ‚ΡŒ Ρ‚Π΅Ρ‡ΡŒ Π² Ρ†Π΅ΠΏΠΈ.БлСдуя ΠΌΠ΅Ρ‚Π°Ρ„ΠΎΡ€Π΅ Π²ΠΎΠ΄Ρ‹, двиТущСйся ΠΏΠΎ Ρ‚Ρ€ΡƒΠ±Π΅, этот Π½Π΅ΠΏΡ€Π΅Ρ€Ρ‹Π²Π½Ρ‹ΠΉ, Ρ€Π°Π²Π½ΠΎΠΌΠ΅Ρ€Π½Ρ‹ΠΉ ΠΏΠΎΡ‚ΠΎΠΊ заряда Ρ‡Π΅Ρ€Π΅Π· Ρ†Π΅ΠΏΡŒ называСтся Ρ‚ΠΎΠΊΠΎΠΌ . Пока источник напряТСния ΠΏΡ€ΠΎΠ΄ΠΎΠ»ΠΆΠ°Π΅Ρ‚ Β«Ρ‚ΠΎΠ»ΠΊΠ°Ρ‚ΡŒΒ» Π² ΠΎΠ΄Π½ΠΎΠΌ Π½Π°ΠΏΡ€Π°Π²Π»Π΅Π½ΠΈΠΈ, носитСли заряда Π±ΡƒΠ΄ΡƒΡ‚ ΠΏΡ€ΠΎΠ΄ΠΎΠ»ΠΆΠ°Ρ‚ΡŒ Π΄Π²ΠΈΠ³Π°Ρ‚ΡŒΡΡ Π² Ρ‚ΠΎΠΌ ΠΆΠ΅ Π½Π°ΠΏΡ€Π°Π²Π»Π΅Π½ΠΈΠΈ Π² Ρ†Π΅ΠΏΠΈ. Π­Ρ‚ΠΎΡ‚ ΠΎΠ΄Π½ΠΎΠ½Π°ΠΏΡ€Π°Π²Π»Π΅Π½Π½Ρ‹ΠΉ ΠΏΠΎΡ‚ΠΎΠΊ Ρ‚ΠΎΠΊΠ° называСтся постоянным Ρ‚ΠΎΠΊΠΎΠΌ, ΠΈΠ»ΠΈ постоянным Ρ‚ΠΎΠΊΠΎΠΌ. Π’ΠΎ Π²Ρ‚ΠΎΡ€ΠΎΠΌ Ρ‚ΠΎΠΌΠ΅ этой сСрии ΠΊΠ½ΠΈΠ³ ΠΈΡΡΠ»Π΅Π΄ΡƒΡŽΡ‚ΡΡ элСктричСскиС Ρ†Π΅ΠΏΠΈ, Π² ΠΊΠΎΡ‚ΠΎΡ€Ρ‹Ρ… Π½Π°ΠΏΡ€Π°Π²Π»Π΅Π½ΠΈΠ΅ Ρ‚ΠΎΠΊΠ° ΠΏΠ΅Ρ€Π΅ΠΊΠ»ΡŽΡ‡Π°Π΅Ρ‚ΡΡ Π²Π·Π°Π΄ ΠΈ Π²ΠΏΠ΅Ρ€Π΅Π΄: ΠΏΠ΅Ρ€Π΅ΠΌΠ΅Π½Π½ΠΎΠ³ΠΎ Ρ‚ΠΎΠΊΠ°, ΠΈΠ»ΠΈ ΠΏΠ΅Ρ€Π΅ΠΌΠ΅Π½Π½ΠΎΠ³ΠΎ Ρ‚ΠΎΠΊΠ°.Но ΠΏΠΎΠΊΠ° ΠΌΡ‹ просто займСмся цСпями постоянного Ρ‚ΠΎΠΊΠ°. ΠŸΠΎΡΠΊΠΎΠ»ΡŒΠΊΡƒ элСктричСский Ρ‚ΠΎΠΊ состоит ΠΈΠ· ΠΎΡ‚Π΄Π΅Π»ΡŒΠ½Ρ‹Ρ… носитСлСй заряда, ΠΏΡ€ΠΎΡ‚Π΅ΠΊΠ°ΡŽΡ‰ΠΈΡ… Π² унисон Ρ‡Π΅Ρ€Π΅Π· ΠΏΡ€ΠΎΠ²ΠΎΠ΄Π½ΠΈΠΊ, ΠΏΠ΅Ρ€Π΅ΠΌΠ΅Ρ‰Π°ΡΡΡŒ ΠΈ толкая носитСли заряда Π²ΠΏΠ΅Ρ€Π΅Π΄ΠΈ, Ρ‚ΠΎΡ‡Π½ΠΎ Ρ‚Π°ΠΊ ΠΆΠ΅, ΠΊΠ°ΠΊ ΡˆΠ°Ρ€ΠΈΠΊΠΈ Ρ‡Π΅Ρ€Π΅Π· Ρ‚Ρ€ΡƒΠ±Ρƒ ΠΈΠ»ΠΈ Π²ΠΎΠ΄Π° Ρ‡Π΅Ρ€Π΅Π· Ρ‚Ρ€ΡƒΠ±Ρƒ, Π²Π΅Π»ΠΈΡ‡ΠΈΠ½Π° ΠΏΠΎΡ‚ΠΎΠΊΠ° Π² ΠΎΠ΄Π½ΠΎΠΉ Ρ†Π΅ΠΏΠΈ Π±ΡƒΠ΄Π΅Ρ‚ ΠΎΠ΄ΠΈΠ½Π°ΠΊΠΎΠ²ΠΎΠΉ. Π² любой Ρ‚ΠΎΡ‡ΠΊΠ΅. Если Π±Ρ‹ ΠΌΡ‹ отслСТивали ΠΏΠΎΠΏΠ΅Ρ€Π΅Ρ‡Π½ΠΎΠ΅ сСчСниС ΠΏΡ€ΠΎΠ²ΠΎΠ΄Π° Π² ΠΎΠ΄Π½ΠΎΠΉ Ρ†Π΅ΠΏΠΈ, считая ΠΏΡ€ΠΎΡ‚Π΅ΠΊΠ°ΡŽΡ‰ΠΈΠ΅ носитСли заряда, ΠΌΡ‹ Π±Ρ‹ Π·Π°ΠΌΠ΅Ρ‚ΠΈΠ»ΠΈ Ρ‚ΠΎΡ‡Π½ΠΎ Ρ‚Π°ΠΊΠΎΠ΅ ΠΆΠ΅ количСство Π² Π΅Π΄ΠΈΠ½ΠΈΡ†Ρƒ Π²Ρ€Π΅ΠΌΠ΅Π½ΠΈ, Ρ‡Ρ‚ΠΎ ΠΈ Π² любой Π΄Ρ€ΡƒΠ³ΠΎΠΉ части Ρ†Π΅ΠΏΠΈ, нСзависимо ΠΎΡ‚ Π΄Π»ΠΈΠ½Ρ‹ ΠΏΡ€ΠΎΠ²ΠΎΠ΄Π½ΠΈΠΊΠ° ΠΈΠ»ΠΈ ΠΏΡ€ΠΎΠ²ΠΎΠ΄Π½ΠΈΠΊΠ°. Π΄ΠΈΠ°ΠΌΠ΅Ρ‚Ρ€.Если ΠΌΡ‹ Π½Π°Ρ€ΡƒΡˆΠΈΠΌ Π½Π΅ΠΏΡ€Π΅Ρ€Ρ‹Π²Π½ΠΎΡΡ‚ΡŒ Ρ†Π΅ΠΏΠΈ Π² любой Ρ‚ΠΎΡ‡ΠΊΠ΅ , элСктричСский Ρ‚ΠΎΠΊ прСкратится Π²ΠΎ всСй ΠΏΠ΅Ρ‚Π»Π΅, ΠΈ ΠΏΠΎΠ»Π½ΠΎΠ΅ напряТСниС, создаваСмоС Π±Π°Ρ‚Π°Ρ€Π΅Π΅ΠΉ, Π±ΡƒΠ΄Π΅Ρ‚ ΠΏΡ€ΠΎΡΠ²Π»ΡΡ‚ΡŒΡΡ Ρ‡Π΅Ρ€Π΅Π· Ρ€Π°Π·Ρ€Ρ‹Π² ΠΌΠ΅ΠΆΠ΄Ρƒ ΠΊΠΎΠ½Ρ†Π°ΠΌΠΈ ΠΏΡ€ΠΎΠ²ΠΎΠ΄ΠΎΠ², ΠΊΠΎΡ‚ΠΎΡ€Ρ‹Π΅ Ρ€Π°Π½ΡŒΡˆΠ΅ Π±Ρ‹Π»ΠΈ соСдинСны:

Π§Ρ‚ΠΎ Ρ‚Π°ΠΊΠΎΠ΅ ΠΏΠΎΠ»ΡΡ€Π½ΠΎΡΡ‚ΡŒ падСния напряТСния?

ΠžΠ±Ρ€Π°Ρ‚ΠΈΡ‚Π΅ Π²Π½ΠΈΠΌΠ°Π½ΠΈΠ΅ Π½Π° Π·Π½Π°ΠΊΠΈ Β«+Β» ΠΈ Β«-Β», нарисованныС Π½Π° ΠΊΠΎΠ½Ρ†Π°Ρ… Ρ€Π°Π·Ρ€Ρ‹Π²Π° Ρ†Π΅ΠΏΠΈ, ΠΈ Ρ‚ΠΎ, ΠΊΠ°ΠΊ ΠΎΠ½ΠΈ ΡΠΎΠΎΡ‚Π²Π΅Ρ‚ΡΡ‚Π²ΡƒΡŽΡ‚ Π·Π½Π°ΠΊΠ°ΠΌ Β«+Β» ΠΈ Β«-Β» рядом с Π²Ρ‹Π²ΠΎΠ΄Π°ΠΌΠΈ аккумулятора. Π­Ρ‚ΠΈ ΠΌΠ°Ρ€ΠΊΠ΅Ρ€Ρ‹ ΡƒΠΊΠ°Π·Ρ‹Π²Π°ΡŽΡ‚ Π½Π°ΠΏΡ€Π°Π²Π»Π΅Π½ΠΈΠ΅, Π² ΠΊΠΎΡ‚ΠΎΡ€ΠΎΠΌ напряТСниС пытаСтся ΠΏΠΎΠ΄Ρ‚ΠΎΠ»ΠΊΠ½ΡƒΡ‚ΡŒ Ρ‚ΠΎΠΊ, это Π½Π°ΠΏΡ€Π°Π²Π»Π΅Π½ΠΈΠ΅ ΠΏΠΎΡ‚Π΅Π½Ρ†ΠΈΠ°Π»Π°, ΠΎΠ±Ρ‹Ρ‡Π½ΠΎ Π½Π°Π·Ρ‹Π²Π°Π΅ΠΌΠΎΠ΅ ΠΏΠΎΠ»ΡΡ€Π½ΠΎΡΡ‚ΡŒΡŽ .ΠŸΠΎΠΌΠ½ΠΈΡ‚Π΅, Ρ‡Ρ‚ΠΎ напряТСниС всСгда ΠΎΡ‚Π½ΠΎΡΠΈΡ‚Π΅Π»ΡŒΠ½ΠΎ ΠΌΠ΅ΠΆΠ΄Ρƒ двумя Ρ‚ΠΎΡ‡ΠΊΠ°ΠΌΠΈ. По этой ΠΏΡ€ΠΈΡ‡ΠΈΠ½Π΅ ΠΏΠΎΠ»ΡΡ€Π½ΠΎΡΡ‚ΡŒ падСния напряТСния Ρ‚Π°ΠΊΠΆΠ΅ являСтся ΠΎΡ‚Π½ΠΎΡΠΈΡ‚Π΅Π»ΡŒΠ½ΠΎΠΉ ΠΌΠ΅ΠΆΠ΄Ρƒ двумя Ρ‚ΠΎΡ‡ΠΊΠ°ΠΌΠΈ: Π±ΡƒΠ΄Π΅Ρ‚ Π»ΠΈ Ρ‚ΠΎΡ‡ΠΊΠ° Π² Ρ†Π΅ΠΏΠΈ ΠΏΠΎΠΌΠ΅Ρ‡Π΅Π½Π° Π·Π½Π°ΠΊΠΎΠΌ Β«+Β» ΠΈΠ»ΠΈ Β«-Β», зависит ΠΎΡ‚ Π΄Ρ€ΡƒΠ³ΠΎΠΉ Ρ‚ΠΎΡ‡ΠΊΠΈ, ΠΊ ΠΊΠΎΡ‚ΠΎΡ€ΠΎΠΉ ΠΎΠ½Π° относится. ВзглянитС Π½Π° ΡΠ»Π΅Π΄ΡƒΡŽΡ‰ΡƒΡŽ схСму, Π³Π΄Π΅ ΠΊΠ°ΠΆΠ΄Ρ‹ΠΉ ΡƒΠ³ΠΎΠ» ΠΏΠ΅Ρ‚Π»ΠΈ ΠΎΡ‚ΠΌΠ΅Ρ‡Π΅Π½ Π½ΠΎΠΌΠ΅Ρ€ΠΎΠΌ для справки:

ΠŸΡ€ΠΈ Π½Π°Ρ€ΡƒΡˆΠ΅Π½ΠΈΠΈ цСлостности Ρ†Π΅ΠΏΠΈ ΠΌΠ΅ΠΆΠ΄Ρƒ Ρ‚ΠΎΡ‡ΠΊΠ°ΠΌΠΈ 2 ΠΈ 3 ΠΏΠΎΠ»ΡΡ€Π½ΠΎΡΡ‚ΡŒ падСния напряТСния ΠΌΠ΅ΠΆΠ΄Ρƒ Ρ‚ΠΎΡ‡ΠΊΠ°ΠΌΠΈ 2 ΠΈ 3 Π±ΡƒΠ΄Π΅Ρ‚ Β«+Β» для Ρ‚ΠΎΡ‡ΠΊΠΈ 2 ΠΈ Β«-Β» для Ρ‚ΠΎΡ‡ΠΊΠΈ 3.ΠŸΠΎΠ»ΡΡ€Π½ΠΎΡΡ‚ΡŒ Π±Π°Ρ‚Π°Ρ€Π΅ΠΈ (1 Β«+Β» ΠΈ 4 Β«-Β») пытаСтся ΠΏΡ€ΠΎΡ‚ΠΎΠ»ΠΊΠ½ΡƒΡ‚ΡŒ Ρ‚ΠΎΠΊ Ρ‡Π΅Ρ€Π΅Π· ΠΏΠ΅Ρ‚Π»ΡŽ ΠΏΠΎ часовой стрСлкС ΠΎΡ‚ 1 Π΄ΠΎ 2, Π΄ΠΎ 3 Π΄ΠΎ 4 ΠΈ снова ΠΎΠ±Ρ€Π°Ρ‚Π½ΠΎ ΠΊ 1. Π’Π΅ΠΏΠ΅Ρ€ΡŒ Π΄Π°Π²Π°ΠΉΡ‚Π΅ посмотрим, Ρ‡Ρ‚ΠΎ ΠΏΡ€ΠΎΠΈΠ·ΠΎΠΉΠ΄Π΅Ρ‚, Ссли ΠΌΡ‹ снова соСдиним Ρ‚ΠΎΡ‡ΠΊΠΈ 2 ΠΈ 3 вмСстС, Π½ΠΎ сдСлаСм Ρ€Π°Π·Ρ€Ρ‹Π² Ρ†Π΅ΠΏΠΈ ΠΌΠ΅ΠΆΠ΄Ρƒ Ρ‚ΠΎΡ‡ΠΊΠ°ΠΌΠΈ 3 ΠΈ 4:

ΠŸΡ€ΠΈ Ρ€Π°Π·Ρ€Ρ‹Π²Π΅ ΠΌΠ΅ΠΆΠ΄Ρƒ 3 ΠΈ 4 ΠΏΠΎΠ»ΡΡ€Π½ΠΎΡΡ‚ΡŒ падСния напряТСния ΠΌΠ΅ΠΆΠ΄Ρƒ этими двумя Ρ‚ΠΎΡ‡ΠΊΠ°ΠΌΠΈ Π±ΡƒΠ΄Π΅Ρ‚ Β«-Β» для 4 ΠΈ Β«+Β» для 3. ΠžΠ±Ρ€Π°Ρ‚ΠΈΡ‚Π΅ особоС Π²Π½ΠΈΠΌΠ°Π½ΠΈΠ΅ Π½Π° Ρ‚ΠΎΡ‚ Ρ„Π°ΠΊΡ‚, Ρ‡Ρ‚ΠΎ Β«Π·Π½Π°ΠΊΒ» Ρ‚ΠΎΡ‡ΠΊΠΈ 3 ΠΏΡ€ΠΎΡ‚ΠΈΠ²ΠΎΠΏΠΎΠ»ΠΎΠΆΠ΅Π½ Π·Π½Π°ΠΊΡƒ Π² ΠŸΠ΅Ρ€Π²Ρ‹ΠΉ ΠΏΡ€ΠΈΠΌΠ΅Ρ€, Π³Π΄Π΅ Ρ€Π°Π·Ρ€Ρ‹Π² Π±Ρ‹Π» ΠΌΠ΅ΠΆΠ΄Ρƒ Ρ‚ΠΎΡ‡ΠΊΠ°ΠΌΠΈ 2 ΠΈ 3 (Π³Π΄Π΅ Ρ‚ΠΎΡ‡ΠΊΠ° 3 Π±Ρ‹Π»Π° ΠΏΠΎΠΌΠ΅Ρ‡Π΅Π½Π° Β«-Β»).ΠœΡ‹ Π½Π΅ ΠΌΠΎΠΆΠ΅ΠΌ ΡΠΊΠ°Π·Π°Ρ‚ΡŒ, Ρ‡Ρ‚ΠΎ Ρ‚ΠΎΡ‡ΠΊΠ° 3 Π² этой Ρ†Π΅ΠΏΠΈ всСгда Π±ΡƒΠ΄Π΅Ρ‚ Π»ΠΈΠ±ΠΎ Β«+Β», Π»ΠΈΠ±ΠΎ Β«-Β», ΠΏΠΎΡ‚ΠΎΠΌΡƒ Ρ‡Ρ‚ΠΎ ΠΏΠΎΠ»ΡΡ€Π½ΠΎΡΡ‚ΡŒ, ΠΊΠ°ΠΊ ΠΈ само напряТСниС, Π½Π΅ зависит ΠΎΡ‚ ΠΎΠ΄Π½ΠΎΠΉ Ρ‚ΠΎΡ‡ΠΊΠΈ, Π° всСгда ΠΎΡ‚Π½ΠΎΡΠΈΡ‚Π΅Π»ΡŒΠ½Π° ΠΌΠ΅ΠΆΠ΄Ρƒ двумя Ρ‚ΠΎΡ‡ΠΊΠ°ΠΌΠΈ!

ΠžΠ‘Π—ΠžΠ :

  • НоситСли заряда ΠΌΠΎΠ³ΡƒΡ‚ Π΄Π²ΠΈΠ³Π°Ρ‚ΡŒΡΡ Ρ‡Π΅Ρ€Π΅Π· ΠΏΡ€ΠΎΠ²ΠΎΠ΄Π½ΠΈΠΊ с ΠΏΠΎΠΌΠΎΡ‰ΡŒΡŽ Ρ‚ΠΎΠΉ ΠΆΠ΅ силы, которая проявляСтся Π² статичСском элСктричСствС.
  • НапряТСниС — это ΠΌΠ΅Ρ€Π° ΡƒΠ΄Π΅Π»ΡŒΠ½ΠΎΠΉ ΠΏΠΎΡ‚Π΅Π½Ρ†ΠΈΠ°Π»ΡŒΠ½ΠΎΠΉ энСргии (ΠΏΠΎΡ‚Π΅Π½Ρ†ΠΈΠ°Π»ΡŒΠ½ΠΎΠΉ энСргии Π½Π° Π΅Π΄ΠΈΠ½ΠΈΡ†Ρƒ заряда) ΠΌΠ΅ΠΆΠ΄Ρƒ двумя Ρ‚ΠΎΡ‡ΠΊΠ°ΠΌΠΈ.Π‘ Ρ‚ΠΎΡ‡ΠΊΠΈ зрСния нСпрофСссионала, это ΠΌΠ΅Ρ€Π° Β«Ρ‚ΠΎΠ»Ρ‡ΠΊΠ°Β», ΠΏΠΎΠ·Π²ΠΎΠ»ΡΡŽΡ‰Π°Ρ ΠΌΠΎΡ‚ΠΈΠ²ΠΈΡ€ΠΎΠ²Π°Ρ‚ΡŒ ΠΎΠ±Π²ΠΈΠ½Π΅Π½ΠΈΠ΅.
  • НапряТСниС, ΠΊΠ°ΠΊ Π²Ρ‹Ρ€Π°ΠΆΠ΅Π½ΠΈΠ΅ ΠΏΠΎΡ‚Π΅Π½Ρ†ΠΈΠ°Π»ΡŒΠ½ΠΎΠΉ энСргии, всСгда ΠΎΡ‚Π½ΠΎΡΠΈΡ‚Π΅Π»ΡŒΠ½ΠΎ ΠΌΠ΅ΠΆΠ΄Ρƒ двумя мСстополоТСниями ΠΈΠ»ΠΈ Ρ‚ΠΎΡ‡ΠΊΠ°ΠΌΠΈ. Иногда это Π½Π°Π·Ρ‹Π²Π°ΡŽΡ‚ Β«ΠΏΠ°Π΄Π΅Π½ΠΈΠ΅ΠΌ напряТСния».
  • Когда источник напряТСния ΠΏΠΎΠ΄ΠΊΠ»ΡŽΡ‡Π΅Π½ ΠΊ Ρ†Π΅ΠΏΠΈ, напряТСниС Π²Ρ‹Π·Ρ‹Π²Π°Π΅Ρ‚ Ρ€Π°Π²Π½ΠΎΠΌΠ΅Ρ€Π½Ρ‹ΠΉ ΠΏΠΎΡ‚ΠΎΠΊ носитСлСй заряда Ρ‡Π΅Ρ€Π΅Π· эту Ρ†Π΅ΠΏΡŒ, Π½Π°Π·Ρ‹Π²Π°Π΅ΠΌΡ‹ΠΉ Ρ‚ΠΎΠΊΠΎΠΌ .

alexxlab

Π”ΠΎΠ±Π°Π²ΠΈΡ‚ΡŒ ΠΊΠΎΠΌΠΌΠ΅Π½Ρ‚Π°Ρ€ΠΈΠΉ

Π’Π°Ρˆ адрСс email Π½Π΅ Π±ΡƒΠ΄Π΅Ρ‚ ΠΎΠΏΡƒΠ±Π»ΠΈΠΊΠΎΠ²Π°Π½. ΠžΠ±ΡΠ·Π°Ρ‚Π΅Π»ΡŒΠ½Ρ‹Π΅ поля ΠΏΠΎΠΌΠ΅Ρ‡Π΅Π½Ρ‹ *